Вы находитесь на странице: 1из 256

Republic of the Philippines JOSE MAX and VILMI QUIPIT, BUGHAW CIELO, CRISANTO, ANNA,

SUPREME COURT DANIEL and FRANCISCO, all surnamed BIBAL, minors, represented
Manila by their parents FRANCISCO, JR. and MILAGROS BIBAL, and THE
PHILIPPINE ECOLOGICAL NETWORK, INC., petitioners,
EN BANC vs.
THE HONORABLE FULGENCIO S. FACTORAN, JR., in his capacity as
the Secretary of the Department of Environment and Natural
Resources, and THE HONORABLE ERIBERTO U. ROSARIO, Presiding
Judge of the RTC, Makati, Branch 66, respondents.
G.R. No. 101083 July 30, 1993
Oposa Law Office for petitioners.
JUAN ANTONIO, ANNA ROSARIO and JOSE ALFONSO, all surnamed
OPOSA, minors, and represented by their parents ANTONIO and
The Solicitor General for respondents.
RIZALINA OPOSA, ROBERTA NICOLE SADIUA, minor, represented
by her parents CALVIN and ROBERTA SADIUA, CARLO, AMANDA
SALUD and PATRISHA, all surnamed FLORES, minors and
represented by their parents ENRICO and NIDA FLORES, GIANINA
DITA R. FORTUN, minor, represented by her parents SIGRID and DAVIDE, JR., J.:
DOLORES FORTUN, GEORGE II and MA. CONCEPCION, all surnamed
MISA, minors and represented by their parents GEORGE and MYRA In a broader sense, this petition bears upon the right of Filipinos to a
MISA, BENJAMIN ALAN V. PESIGAN, minor, represented by his balanced and healthful ecology which the petitioners dramatically associate
parents ANTONIO and ALICE PESIGAN, JOVIE MARIE ALFARO, with the twin concepts of "inter-generational responsibility" and "inter-
minor, represented by her parents JOSE and MARIA VIOLETA generational justice." Specifically, it touches on the issue of whether the
ALFARO, MARIA CONCEPCION T. CASTRO, minor, represented by said petitioners have a cause of action to "prevent the misappropriation or
her parents FREDENIL and JANE CASTRO, JOHANNA impairment" of Philippine rainforests and "arrest the unabated hemorrhage
DESAMPARADO, of the country's vital life support systems and continued rape of Mother
minor, represented by her parents JOSE and ANGELA Earth."
DESAMPRADO, CARLO JOAQUIN T. NARVASA, minor, represented
by his parents GREGORIO II and CRISTINE CHARITY NARVASA,
The controversy has its genesis in Civil Case No. 90-77 which was filed
MA. MARGARITA, JESUS IGNACIO, MA. ANGELA and MARIE
before Branch 66 (Makati, Metro Manila) of the Regional Trial Court (RTC),
GABRIELLE, all surnamed SAENZ, minors, represented by their
National Capital Judicial Region. The principal plaintiffs therein, now the
parents ROBERTO and AURORA SAENZ, KRISTINE, MARY ELLEN,
principal petitioners, are all minors duly represented and joined by their
MAY, GOLDA MARTHE and DAVID IAN, all surnamed KING, minors,
respective parents. Impleaded as an additional plaintiff is the Philippine
represented by their parents MARIO and HAYDEE KING, DAVID,
Ecological Network, Inc. (PENI), a domestic, non-stock and non-profit
FRANCISCO and THERESE VICTORIA, all surnamed ENDRIGA,
corporation organized for the purpose of, inter alia, engaging in concerted
minors, represented by their parents BALTAZAR and TERESITA
action geared for the protection of our environment and natural resources.
ENDRIGA, JOSE MA. and REGINA MA., all surnamed ABAYA, minors,
The original defendant was the Honorable Fulgencio S. Factoran, Jr., then
represented by their parents ANTONIO and MARICA ABAYA,
Secretary of the Department of Environment and Natural Resources
MARILIN, MARIO, JR. and MARIETTE, all surnamed CARDAMA,
(DENR). His substitution in this petition by the new Secretary, the
minors, represented by their parents MARIO and LINA CARDAMA,
Honorable Angel C. Alcala, was subsequently ordered upon proper motion
CLARISSA, ANN MARIE, NAGEL, and IMEE LYN, all surnamed
by the petitioners.1 The complaint2 was instituted as a taxpayers' class
OPOSA, minors and represented by their parents RICARDO and
suit3 and alleges that the plaintiffs "are all citizens of the Republic of the
MARISSA OPOSA, PHILIP JOSEPH, STEPHEN JOHN and ISAIAH
Philippines, taxpayers, and entitled to the full benefit, use and enjoyment
JAMES, all surnamed QUIPIT, minors, represented by their parents
of the natural resource treasure that is the country's virgin tropical
forests." The same was filed for themselves and others who are equally consequential destruction of corals and other aquatic life leading to a
concerned about the preservation of said resource but are "so numerous critical reduction in marine resource productivity, (g) recurrent spells of
that it is impracticable to bring them all before the Court." The minors drought as is presently experienced by the entire country, (h) increasing
further asseverate that they "represent their generation as well as velocity of typhoon winds which result from the absence of windbreakers,
generations yet unborn."4 Consequently, it is prayed for that judgment be (i) the floodings of lowlands and agricultural plains arising from the
rendered: absence of the absorbent mechanism of forests, (j) the siltation and
shortening of the lifespan of multi-billion peso dams constructed and
. . . ordering defendant, his agents, representatives and operated for the purpose of supplying water for domestic uses, irrigation
other persons acting in his behalf to — and the generation of electric power, and (k) the reduction of the earth's
capacity to process carbon dioxide gases which has led to perplexing and
catastrophic climatic changes such as the phenomenon of global warming,
(1) Cancel all existing timber license agreements in the
otherwise known as the "greenhouse effect."
country;

Plaintiffs further assert that the adverse and detrimental consequences of


(2) Cease and desist from receiving, accepting, processing,
continued and deforestation are so capable of unquestionable
renewing or approving new timber license agreements.
demonstration that the same may be submitted as a matter of judicial
notice. This notwithstanding, they expressed their intention to present
and granting the plaintiffs ". . . such other reliefs just and equitable under expert witnesses as well as documentary, photographic and film evidence
the premises."5 in the course of the trial.

The complaint starts off with the general averments that the Philippine As their cause of action, they specifically allege that:
archipelago of 7,100 islands has a land area of thirty million (30,000,000)
hectares and is endowed with rich, lush and verdant rainforests in which
CAUSE OF ACTION
varied, rare and unique species of flora and fauna may be found; these
rainforests contain a genetic, biological and chemical pool which is
irreplaceable; they are also the habitat of indigenous Philippine cultures 7. Plaintiffs replead by reference the foregoing allegations.
which have existed, endured and flourished since time immemorial;
scientific evidence reveals that in order to maintain a balanced and 8. Twenty-five (25) years ago, the Philippines had some
healthful ecology, the country's land area should be utilized on the basis of sixteen (16) million hectares of rainforests constituting
a ratio of fifty-four per cent (54%) for forest cover and forty-six per cent roughly 53% of the country's land mass.
(46%) for agricultural, residential, industrial, commercial and other uses;
the distortion and disturbance of this balance as a consequence of 9. Satellite images taken in 1987 reveal that there
deforestation have resulted in a host of environmental tragedies, such as remained no more than 1.2 million hectares of said
(a) water shortages resulting from drying up of the water table, otherwise rainforests or four per cent (4.0%) of the country's land
known as the "aquifer," as well as of rivers, brooks and streams, (b) area.
salinization of the water table as a result of the intrusion therein of salt
water, incontrovertible examples of which may be found in the island of
10. More recent surveys reveal that a mere 850,000
Cebu and the Municipality of Bacoor, Cavite, (c) massive erosion and the
hectares of virgin old-growth rainforests are left, barely
consequential loss of soil fertility and agricultural productivity, with the
2.8% of the entire land mass of the Philippine archipelago
volume of soil eroded estimated at one billion (1,000,000,000) cubic
and about 3.0 million hectares of immature and
meters per annum — approximately the size of the entire island of
uneconomical secondary growth forests.
Catanduanes, (d) the endangering and extinction of the country's unique,
rare and varied flora and fauna, (e) the disturbance and dislocation of
cultural communities, including the disappearance of the Filipino's 11. Public records reveal that the defendant's,
indigenous cultures, (f) the siltation of rivers and seabeds and predecessors have granted timber license agreements
('TLA's') to various corporations to cut the aggregate area A copy of the plaintiffs' letter dated March 1, 1990 is
of 3.89 million hectares for commercial logging purposes. hereto attached as Annex "B".

A copy of the TLA holders and the corresponding areas 17. Defendant, however, fails and refuses to cancel the
covered is hereto attached as Annex "A". existing TLA's to the continuing serious damage and
extreme prejudice of plaintiffs.
12. At the present rate of deforestation, i.e. about 200,000
hectares per annum or 25 hectares per hour — nighttime, 18. The continued failure and refusal by defendant to
Saturdays, Sundays and holidays included — the cancel the TLA's is an act violative of the rights of
Philippines will be bereft of forest resources after the end plaintiffs, especially plaintiff minors who may be left with a
of this ensuing decade, if not earlier. country that is desertified (sic), bare, barren and devoid of
the wonderful flora, fauna and indigenous cultures which
13. The adverse effects, disastrous consequences, serious the Philippines had been abundantly blessed with.
injury and irreparable damage of this continued trend of
deforestation to the plaintiff minor's generation and to 19. Defendant's refusal to cancel the aforementioned TLA's
generations yet unborn are evident and incontrovertible. is manifestly contrary to the public policy enunciated in the
As a matter of fact, the environmental damages Philippine Environmental Policy which, in pertinent part,
enumerated in paragraph 6 hereof are already being felt, states that it is the policy of the State —
experienced and suffered by the generation of plaintiff
adults. (a) to create, develop, maintain and improve conditions
under which man and nature can thrive in productive and
14. The continued allowance by defendant of TLA holders enjoyable harmony with each other;
to cut and deforest the remaining forest stands will work
great damage and irreparable injury to plaintiffs — (b) to fulfill the social, economic and other requirements of
especially plaintiff minors and their successors — who may present and future generations of Filipinos and;
never see, use, benefit from and enjoy this rare and
unique natural resource treasure.
(c) to ensure the attainment of an environmental quality
that is conductive to a life of dignity and well-being. (P.D.
This act of defendant constitutes a misappropriation and/or 1151, 6 June 1977)
impairment of the natural resource property he holds in
trust for the benefit of plaintiff minors and succeeding
20. Furthermore, defendant's continued refusal to cancel
generations.
the aforementioned TLA's is contradictory to the
Constitutional policy of the State to —
15. Plaintiffs have a clear and constitutional right to a
balanced and healthful ecology and are entitled to
a. effect "a more equitable distribution of opportunities,
protection by the State in its capacity as the parens
income and wealth" and "make full and efficient use of
patriae.
natural resources (sic)." (Section 1, Article XII of the
Constitution);
16. Plaintiff have exhausted all administrative remedies
with the defendant's office. On March 2, 1990, plaintiffs
b. "protect the nation's marine wealth." (Section 2, ibid);
served upon defendant a final demand to cancel all logging
permits in the country.
c. "conserve and promote the nation's cultural heritage
and resources (sic)" (Section 14, Article XIV, id.);
d. "protect and advance the right of the people to a Petitioners contend that the complaint clearly and unmistakably states a
balanced and healthful ecology in accord with the rhythm cause of action as it contains sufficient allegations concerning their right to
and harmony of nature." (Section 16, Article II, id.) a sound environment based on Articles 19, 20 and 21 of the Civil Code
(Human Relations), Section 4 of Executive Order (E.O.) No. 192 creating
21. Finally, defendant's act is contrary to the highest law the DENR, Section 3 of Presidential Decree (P.D.) No. 1151 (Philippine
of humankind — the natural law — and violative of Environmental Policy), Section 16, Article II of the 1987 Constitution
plaintiffs' right to self-preservation and perpetuation. recognizing the right of the people to a balanced and healthful ecology, the
concept of generational genocide in Criminal Law and the concept of man's
inalienable right to self-preservation and self-perpetuation embodied in
22. There is no other plain, speedy and adequate remedy
natural law. Petitioners likewise rely on the respondent's correlative
in law other than the instant action to arrest the unabated
obligation per Section 4 of E.O. No. 192, to safeguard the people's right to
hemorrhage of the country's vital life support systems and
a healthful environment.
continued rape of Mother Earth. 6

It is further claimed that the issue of the respondent Secretary's alleged


On 22 June 1990, the original defendant, Secretary Factoran, Jr., filed a
grave abuse of discretion in granting Timber License Agreements (TLAs) to
Motion to Dismiss the complaint based on two (2) grounds, namely: (1)
cover more areas for logging than what is available involves a judicial
the plaintiffs have no cause of action against him and (2) the issue raised
question.
by the plaintiffs is a political question which properly pertains to the
legislative or executive branches of Government. In their 12 July 1990
Opposition to the Motion, the petitioners maintain that (1) the complaint Anent the invocation by the respondent Judge of the Constitution's non-
shows a clear and unmistakable cause of action, (2) the motion is dilatory impairment clause, petitioners maintain that the same does not apply in
and (3) the action presents a justiciable question as it involves the this case because TLAs are not contracts. They likewise submit that even if
defendant's abuse of discretion. TLAs may be considered protected by the said clause, it is well settled that
they may still be revoked by the State when the public interest so
requires.
On 18 July 1991, respondent Judge issued an order granting the
aforementioned motion to dismiss.7 In the said order, not only was the
defendant's claim — that the complaint states no cause of action against On the other hand, the respondents aver that the petitioners failed to
him and that it raises a political question — sustained, the respondent allege in their complaint a specific legal right violated by the respondent
Judge further ruled that the granting of the relief prayed for would result Secretary for which any relief is provided by law. They see nothing in the
in the impairment of contracts which is prohibited by the fundamental law complaint but vague and nebulous allegations concerning an
of the land. "environmental right" which supposedly entitles the petitioners to the
"protection by the state in its capacity as parens patriae." Such
allegations, according to them, do not reveal a valid cause of action. They
Plaintiffs thus filed the instant special civil action for certiorari under Rule
then reiterate the theory that the question of whether logging should be
65 of the Revised Rules of Court and ask this Court to rescind and set
permitted in the country is a political question which should be properly
aside the dismissal order on the ground that the respondent Judge gravely
addressed to the executive or legislative branches of Government. They
abused his discretion in dismissing the action. Again, the parents of the
therefore assert that the petitioners' resources is not to file an action to
plaintiffs-minors not only represent their children, but have also joined the
court, but to lobby before Congress for the passage of a bill that would ban
latter in this case.8
logging totally.

On 14 May 1992, We resolved to give due course to the petition and


As to the matter of the cancellation of the TLAs, respondents submit that
required the parties to submit their respective Memoranda after the Office
the same cannot be done by the State without due process of law. Once
of the Solicitor General (OSG) filed a Comment in behalf of the
issued, a TLA remains effective for a certain period of time — usually for
respondents and the petitioners filed a reply thereto.
twenty-five (25) years. During its effectivity, the same can neither be
revised nor cancelled unless the holder has been found, after due notice
and hearing, to have violated the terms of the agreement or other forestry After a careful perusal of the complaint in question and a meticulous
laws and regulations. Petitioners' proposition to have all the TLAs consideration and evaluation of the issues raised and arguments adduced
indiscriminately cancelled without the requisite hearing would be violative by the parties, We do not hesitate to find for the petitioners and rule
of the requirements of due process. against the respondent Judge's challenged order for having been issued
with grave abuse of discretion amounting to lack of jurisdiction. The
Before going any further, We must first focus on some procedural matters. pertinent portions of the said order reads as follows:
Petitioners instituted Civil Case No. 90-777 as a class suit. The original
defendant and the present respondents did not take issue with this matter. xxx xxx xxx
Nevertheless, We hereby rule that the said civil case is indeed a class suit.
The subject matter of the complaint is of common and general interest not After a careful and circumspect evaluation of the
just to several, but to all citizens of the Philippines. Consequently, since Complaint, the Court cannot help but agree with the
the parties are so numerous, it, becomes impracticable, if not totally defendant. For although we believe that plaintiffs have but
impossible, to bring all of them before the court. We likewise declare that the noblest of all intentions, it (sic) fell short of alleging,
the plaintiffs therein are numerous and representative enough to ensure with sufficient definiteness, a specific legal right they are
the full protection of all concerned interests. Hence, all the requisites for seeking to enforce and protect, or a specific legal wrong
the filing of a valid class suit under Section 12, Rule 3 of the Revised Rules they are seeking to prevent and redress (Sec. 1, Rule 2,
of Court are present both in the said civil case and in the instant petition, RRC). Furthermore, the Court notes that the Complaint is
the latter being but an incident to the former. replete with vague assumptions and vague conclusions
based on unverified data. In fine, plaintiffs fail to state a
This case, however, has a special and novel element. Petitioners minors cause of action in its Complaint against the herein
assert that they represent their generation as well as generations yet defendant.
unborn. We find no difficulty in ruling that they can, for themselves, for
others of their generation and for the succeeding generations, file a class Furthermore, the Court firmly believes that the matter
suit. Their personality to sue in behalf of the succeeding generations can before it, being impressed with political color and involving
only be based on the concept of intergenerational responsibility insofar as a matter of public policy, may not be taken cognizance of
the right to a balanced and healthful ecology is concerned. Such a right, as by this Court without doing violence to the sacred principle
hereinafter expounded, considers of "Separation of Powers" of the three (3) co-equal
the "rhythm and harmony of nature." Nature means the created world in branches of the Government.
its entirety.9 Such rhythm and harmony indispensably include, inter alia,
the judicious disposition, utilization, management, renewal and
The Court is likewise of the impression that it cannot, no
conservation of the country's forest, mineral, land, waters, fisheries,
matter how we stretch our jurisdiction, grant the reliefs
wildlife, off-shore areas and other natural resources to the end that their
prayed for by the plaintiffs, i.e., to cancel all existing
exploration, development and utilization be equitably accessible to the
timber license agreements in the country and to cease and
present as well as future generations. 10Needless to say, every generation
desist from receiving, accepting, processing, renewing or
has a responsibility to the next to preserve that rhythm and harmony for
approving new timber license agreements. For to do
the full enjoyment of a balanced and healthful ecology. Put a little
otherwise would amount to "impairment of contracts"
differently, the minors' assertion of their right to a sound environment
abhored (sic) by the fundamental law. 11
constitutes, at the same time, the performance of their obligation to
ensure the protection of that right for the generations to come.
We do not agree with the trial court's conclusions that the plaintiffs failed
to allege with sufficient definiteness a specific legal right involved or a
The locus standi of the petitioners having thus been addressed, We shall
specific legal wrong committed, and that the complaint is replete with
now proceed to the merits of the petition.
vague assumptions and conclusions based on unverified data. A reading of
the complaint itself belies these conclusions.
The complaint focuses on one specific fundamental legal right — the right Does this section mandate the State to
to a balanced and healthful ecology which, for the first time in our nation's provide sanctions against all forms of
constitutional history, is solemnly incorporated in the fundamental law. pollution — air, water and noise pollution?
Section 16, Article II of the 1987 Constitution explicitly provides:
MR. AZCUNA:
Sec. 16. The State shall protect and advance the right of
the people to a balanced and healthful ecology in accord Yes, Madam President. The right to
with the rhythm and harmony of nature. healthful (sic) environment necessarily
carries with it the correlative duty of not
This right unites with the right to health which is provided impairing the same and, therefore,
for in the preceding section of the same article: sanctions may be provided for impairment
of environmental balance. 12
Sec. 15. The State shall protect and promote the right to
health of the people and instill health consciousness The said right implies, among many other things, the judicious
among them. management and conservation of the country's forests.

While the right to a balanced and healthful ecology is to be found under Without such forests, the ecological or environmental balance
the Declaration of Principles and State Policies and not under the Bill of would be irreversiby disrupted.
Rights, it does not follow that it is less important than any of the civil and
political rights enumerated in the latter. Such a right belongs to a different Conformably with the enunciated right to a balanced and healthful ecology
category of rights altogether for it concerns nothing less than self- and the right to health, as well as the other related provisions of the
preservation and self-perpetuation — aptly and fittingly stressed by the Constitution concerning the conservation, development and utilization of
petitioners — the advancement of which may even be said to predate all the country's natural resources, 13 then President Corazon C. Aquino
governments and constitutions. As a matter of fact, these basic rights promulgated on 10 June 1987 E.O. No. 192, 14 Section 4 of which
need not even be written in the Constitution for they are assumed to exist expressly mandates that the Department of Environment and Natural
from the inception of humankind. If they are now explicitly mentioned in Resources "shall be the primary government agency responsible for the
the fundamental charter, it is because of the well-founded fear of its conservation, management, development and proper use of the country's
framers that unless the rights to a balanced and healthful ecology and to environment and natural resources, specifically forest and grazing lands,
health are mandated as state policies by the Constitution itself, thereby mineral, resources, including those in reservation and watershed areas,
highlighting their continuing importance and imposing upon the state a and lands of the public domain, as well as the licensing and regulation of
solemn obligation to preserve the first and protect and advance the all natural resources as may be provided for by law in order to ensure
second, the day would not be too far when all else would be lost not only equitable sharing of the benefits derived therefrom for the welfare of the
for the present generation, but also for those to come — generations present and future generations of Filipinos." Section 3 thereof makes the
which stand to inherit nothing but parched earth incapable of sustaining following statement of policy:
life.
Sec. 3. Declaration of Policy. — It is hereby declared the
The right to a balanced and healthful ecology carries with it the correlative policy of the State to ensure the sustainable use,
duty to refrain from impairing the environment. During the debates on this development, management, renewal, and conservation of
right in one of the plenary sessions of the 1986 Constitutional Commission, the country's forest, mineral, land, off-shore areas and
the following exchange transpired between Commissioner Wilfrido other natural resources, including the protection and
Villacorta and Commissioner Adolfo Azcuna who sponsored the section in enhancement of the quality of the environment, and
question: equitable access of the different segments of the
population to the development and the use of the country's
MR. VILLACORTA: natural resources, not only for the present generation but
for future generations as well. It is also the policy of the Both E.O. NO. 192 and the Administrative Code of 1987 have set the
state to recognize and apply a true value system including objectives which will serve as the bases for policy formulation, and have
social and environmental cost implications relative to their defined the powers and functions of the DENR.
utilization, development and conservation of our natural
resources. It may, however, be recalled that even before the ratification of the 1987
Constitution, specific statutes already paid special attention to the
This policy declaration is substantially re-stated it Title XIV, Book IV of the "environmental right" of the present and future generations. On 6 June
Administrative Code of 1987, 15 specifically in Section 1 thereof which 1977, P.D. No. 1151 (Philippine Environmental Policy) and P.D. No. 1152
reads: (Philippine Environment Code) were issued. The former "declared a
continuing policy of the State (a) to create, develop, maintain and improve
Sec. 1. Declaration of Policy. — (1) The State shall ensure, conditions under which man and nature can thrive in productive and
for the benefit of the Filipino people, the full exploration enjoyable harmony with each other, (b) to fulfill the social, economic and
and development as well as the judicious disposition, other requirements of present and future generations of Filipinos, and (c)
utilization, management, renewal and conservation of the to insure the attainment of an environmental quality that is conducive to a
country's forest, mineral, land, waters, fisheries, wildlife, life of dignity and well-being." 16 As its goal, it speaks of the
off-shore areas and other natural resources, consistent "responsibilities of each generation as trustee and guardian of the
with the necessity of maintaining a sound ecological environment for succeeding generations." 17 The latter statute, on the
balance and protecting and enhancing the quality of the other hand, gave flesh to the said policy.
environment and the objective of making the exploration,
development and utilization of such natural resources Thus, the right of the petitioners (and all those they represent) to a
equitably accessible to the different segments of the balanced and healthful ecology is as clear as the DENR's duty — under its
present as well as future generations. mandate and by virtue of its powers and functions under E.O. No. 192 and
the Administrative Code of 1987 — to protect and advance the said right.
(2) The State shall likewise recognize and apply a true
value system that takes into account social and A denial or violation of that right by the other who has the corelative duty
environmental cost implications relative to the utilization, or obligation to respect or protect the same gives rise to a cause of action.
development and conservation of our natural resources. Petitioners maintain that the granting of the TLAs, which they claim was
done with grave abuse of discretion, violated their right to a balanced and
The above provision stresses "the necessity of maintaining a sound healthful ecology; hence, the full protection thereof requires that no
ecological balance and protecting and enhancing the quality of the further TLAs should be renewed or granted.
environment." Section 2 of the same Title, on the other hand, specifically
speaks of the mandate of the DENR; however, it makes particular A cause of action is defined as:
reference to the fact of the agency's being subject to law and higher
authority. Said section provides: . . . an act or omission of one party in violation of the legal
right or rights of the other; and its essential elements are
Sec. 2. Mandate. — (1) The Department of Environment legal right of the plaintiff, correlative obligation of the
and Natural Resources shall be primarily responsible for defendant, and act or omission of the defendant in
the implementation of the foregoing policy. violation of said legal right. 18

(2) It shall, subject to law and higher authority, be in It is settled in this jurisdiction that in a motion to dismiss based on the
charge of carrying out the State's constitutional mandate ground that the complaint fails to state a cause of action, 19 the question
to control and supervise the exploration, development, submitted to the court for resolution involves the sufficiency of the facts
utilization, and conservation of the country's natural alleged in the complaint itself. No other matter should be considered;
resources. furthermore, the truth of falsity of the said allegations is beside the point
for the truth thereof is deemed hypothetically admitted. The only issue to the authority represents a broadening of judicial power to
be resolved in such a case is: admitting such alleged facts to be true, may enable the courts of justice to review what was before
the court render a valid judgment in accordance with the prayer in the forbidden territory, to wit, the discretion of the political
complaint? 20 In Militante vs. Edrosolano, 21 this Court laid down the rule departments of the government.
that the judiciary should "exercise the utmost care and circumspection in
passing upon a motion to dismiss on the ground of the absence thereof As worded, the new provision vests in the judiciary, and
[cause of action] lest, by its failure to manifest a correct appreciation of particularly the Supreme Court, the power to rule upon
the facts alleged and deemed hypothetically admitted, what the law grants even the wisdom of the decisions of the executive and the
or recognizes is effectively nullified. If that happens, there is a blot on the legislature and to declare their acts invalid for lack or
legal order. The law itself stands in disrepute." excess of jurisdiction because tainted with grave abuse of
discretion. The catch, of course, is the meaning of "grave
After careful examination of the petitioners' complaint, We find the abuse of discretion," which is a very elastic phrase that can
statements under the introductory affirmative allegations, as well as the expand or contract according to the disposition of the
specific averments under the sub-heading CAUSE OF ACTION, to be judiciary.
adequate enough to show, prima facie, the claimed violation of their
rights. On the basis thereof, they may thus be granted, wholly or partly, In Daza vs. Singson, 23
Mr. Justice Cruz, now speaking for this Court,
the reliefs prayed for. It bears stressing, however, that insofar as the noted:
cancellation of the TLAs is concerned, there is the need to implead, as
party defendants, the grantees thereof for they are indispensable parties.
In the case now before us, the jurisdictional objection
becomes even less tenable and decisive. The reason is
The foregoing considered, Civil Case No. 90-777 be said to raise a political that, even if we were to assume that the issue presented
question. Policy formulation or determination by the executive or before us was political in nature, we would still not be
legislative branches of Government is not squarely put in issue. What is precluded from revolving it under the expanded jurisdiction
principally involved is the enforcement of a right vis-a-vis policies already conferred upon us that now covers, in proper cases, even
formulated and expressed in legislation. It must, nonetheless, be the political question. Article VII, Section 1, of the
emphasized that the political question doctrine is no longer, the Constitution clearly provides: . . .
insurmountable obstacle to the exercise of judicial power or the
impenetrable shield that protects executive and legislative actions from
The last ground invoked by the trial court in dismissing the complaint is
judicial inquiry or review. The second paragraph of section 1, Article VIII
the non-impairment of contracts clause found in the Constitution. The
of the Constitution states that:
court a quo declared that:

Judicial power includes the duty of the courts of justice to


The Court is likewise of the impression that it cannot, no
settle actual controversies involving rights which are
matter how we stretch our jurisdiction, grant the reliefs
legally demandable and enforceable, and to determine
prayed for by the plaintiffs, i.e., to cancel all existing
whether or not there has been a grave abuse of discretion
timber license agreements in the country and to cease and
amounting to lack or excess of jurisdiction on the part of
desist from receiving, accepting, processing, renewing or
any branch or instrumentality of the Government.
approving new timber license agreements. For to do
otherwise would amount to "impairment of contracts"
Commenting on this provision in his book, Philippine Political Law, 22 Mr. abhored (sic) by the fundamental law. 24
Justice Isagani A. Cruz, a distinguished member of this Court, says:
We are not persuaded at all; on the contrary, We are amazed, if not
The first part of the authority represents the traditional shocked, by such a sweeping pronouncement. In the first place, the
concept of judicial power, involving the settlement of respondent Secretary did not, for obvious reasons, even invoke in his
conflicting rights as conferred as law. The second part of motion to dismiss the non-impairment clause. If he had done so, he would
have acted with utmost infidelity to the Government by providing undue gainsaid that they merely evidence a privilege granted by
and unwarranted benefits and advantages to the timber license holders the State to qualified entities, and do not vest in the latter
because he would have forever bound the Government to strictly respect a permanent or irrevocable right to the particular
the said licenses according to their terms and conditions regardless of concession area and the forest products therein. They may
changes in policy and the demands of public interest and welfare. He was be validly amended, modified, replaced or rescinded by the
aware that as correctly pointed out by the petitioners, into every timber Chief Executive when national interests so require. Thus,
license must be read Section 20 of the Forestry Reform Code (P.D. No. they are not deemed contracts within the purview of the
705) which provides: due process of law clause [See Sections 3(ee) and 20 of
Pres. Decree No. 705, as amended. Also, Tan v. Director of
. . . Provided, That when the national interest so requires, Forestry, G.R. No. L-24548, October 27, 1983, 125 SCRA
the President may amend, modify, replace or rescind any 302].
contract, concession, permit, licenses or any other form of
privilege granted herein . . . Since timber licenses are not contracts, the non-impairment clause, which
reads:
Needless to say, all licenses may thus be revoked or rescinded by
executive action. It is not a contract, property or a property right Sec. 10. No law impairing, the obligation of contracts shall
protested by the due process clause of the Constitution. In Tan vs. be passed. 27
Director of Forestry, 25 this Court held:
cannot be invoked.
. . . A timber license is an instrument by which the State
regulates the utilization and disposition of forest resources In the second place, even if it is to be assumed that the same are
to the end that public welfare is promoted. A timber contracts, the instant case does not involve a law or even an executive
license is not a contract within the purview of the due issuance declaring the cancellation or modification of existing timber
process clause; it is only a license or privilege, which can licenses. Hence, the non-impairment clause cannot as yet be invoked.
be validly withdrawn whenever dictated by public interest Nevertheless, granting further that a law has actually been passed
or public welfare as in this case. mandating cancellations or modifications, the same cannot still be
stigmatized as a violation of the non-impairment clause. This is because by
A license is merely a permit or privilege to do what its very nature and purpose, such as law could have only been passed in
otherwise would be unlawful, and is not a contract the exercise of the police power of the state for the purpose of advancing
between the authority, federal, state, or municipal, the right of the people to a balanced and healthful ecology, promoting
granting it and the person to whom it is granted; neither is their health and enhancing the general welfare. In Abe vs. Foster Wheeler
it property or a property right, nor does it create a vested Corp. 28 this Court stated:
right; nor is it taxation (37 C.J. 168). Thus, this Court held
that the granting of license does not create irrevocable The freedom of contract, under our system of government,
rights, neither is it property or property rights (People vs. is not meant to be absolute. The same is understood to be
Ong Tin, 54 O.G. 7576). subject to reasonable legislative regulation aimed at the
promotion of public health, moral, safety and welfare. In
We reiterated this pronouncement in Felipe Ysmael, Jr. & Co., Inc. vs. other words, the constitutional guaranty of non-
Deputy Executive Secretary: 26 impairment of obligations of contract is limited by the
exercise of the police power of the State, in the interest of
. . . Timber licenses, permits and license agreements are public health, safety, moral and general welfare.
the principal instruments by which the State regulates the
utilization and disposition of forest resources to the end
that public welfare is promoted. And it can hardly be
The reason for this is emphatically set forth in Nebia vs. New
York, 29 quoted in Philippine American Life Insurance Co. vs. Auditor
General,30 to wit:

Under our form of government the use of property and the


making of contracts are normally matters of private and
not of public concern. The general rule is that both shall be
free of governmental interference. But neither property
rights nor contract rights are absolute; for government
cannot exist if the citizen may at will use his property to
the detriment of his fellows, or exercise his freedom of
contract to work them harm. Equally fundamental with the
private right is that of the public to regulate it in the
common interest.

In short, the non-impairment clause must yield to the police power of the
state. 31

Finally, it is difficult to imagine, as the trial court did, how the non-
impairment clause could apply with respect to the prayer to enjoin the
respondent Secretary from receiving, accepting, processing, renewing or
approving new timber licenses for, save in cases of renewal, no contract
would have as of yet existed in the other instances. Moreover, with respect
to renewal, the holder is not entitled to it as a matter of right.

WHEREFORE, being impressed with merit, the instant Petition is hereby


GRANTED, and the challenged Order of respondent Judge of 18 July 1991
dismissing Civil Case No. 90-777 is hereby set aside. The petitioners may
therefore amend their complaint to implead as defendants the holders or
grantees of the questioned timber license agreements.

No pronouncement as to costs.

SO ORDERED.

Cruz, Padilla, Bidin, Griño-Aquino, Regalado, Romero, Nocon, Bellosillo,


Melo and Quiason, JJ., concur.

Narvasa, C.J., Puno and Vitug, JJ., took no part.


EN BANC I. EXECUTION OF THE NECESSARY CONTRACTS WITH GSIS/MHC -

1. The Highest Bidder must comply with the conditions set forth below by
October 23, 1995 (reset to November 3, 1995) or the Highest Bidder will
[G.R. No. 122156. February 3, 1997] lose the right to purchase the Block of Shares and GSIS will instead offer
the Block of Shares to the other Qualified Bidders:

a. The Highest Bidder must negotiate and execute with the GSIS/MHC the
MANILA PRINCE HOTEL, petitioner, vs. GOVERNMENT SERVICE Management Contract, International Marketing/Reservation System
INSURANCE SYSTEM, MANILA HOTEL CORPORATION, Contract or other type of contract specified by the Highest Bidder in its
COMMITTEE ON PRIVATIZATION and OFFICE OF THE strategic plan for the Manila Hotel x x x x
GOVERNMENT CORPORATE COUNSEL, respondents.
b. The Highest Bidder must execute the Stock Purchase and Sale
DECISION Agreement with GSIS x x x x

BELLOSILLO, J.:
K. DECLARATION OF THE WINNING BIDDER/STRATEGIC PARTNER
-
The Filipino First Policy enshrined in the 1987 Constitution, i.e., in the
grant of rights, privileges, and concessions covering the national economy
The Highest Bidder will be declared the Winning Bidder/Strategic Partner
and patrimony, the State shall give preference to qualified Filipinos,[1] is
after the following conditions are met:
invoked by petitioner in its bid to acquire 51% of the shares of the Manila
Hotel Corporation (MHC) which owns the historic Manila Hotel. Opposing,
respondents maintain that the provision is not self-executing but requires a. Execution of the necessary contracts with GSIS/MHC not later than
an implementing legislation for its enforcement. Corollarily, they ask October 23, 1995 (reset to November 3, 1995); and
whether the 51% shares form part of the national economy and patrimony
covered by the protective mantle of the Constitution. b. Requisite approvals from the GSIS/MHC and COP (Committee on
Privatization)/ OGCC (Office of the Government Corporate Counsel) are
The controversy arose when respondent Government Service
obtained.[3]
Insurance System (GSIS), pursuant to the privatization program of the
Philippine Government under Proclamation No. 50 dated 8 December
1986, decided to sell through public bidding 30% to 51% of the issued and Pending the declaration of Renong Berhard as the winning
outstanding shares of respondent MHC. The winning bidder, or the bidder/strategic partner and the execution of the necessary contracts,
eventual strategic partner, is to provide management expertise and/or an petitioner in a letter to respondent GSIS dated 28 September 1995
international marketing/reservation system, and financial support to matched the bid price of P44.00 per share tendered by Renong Berhad.
strengthen the profitability and performance of the Manila Hotel.[2] In a
[4]
In a subsequent letter dated 10 October 1995 petitioner sent a
close bidding held on 18 September 1995 only two (2) bidders managers check issued by Philtrust Bank for Thirty-three Million Pesos
participated: petitioner Manila Prince Hotel Corporation, a Filipino (P33,000,000.00) as Bid Security to match the bid of the Malaysian Group,
corporation, which offered to buy 51% of the MHC or 15,300,000 shares Messrs. Renong Berhad x x x x[5] which respondent GSIS refused to
at P41.58 per share, and Renong Berhad, a Malaysian firm, with ITT- accept.
Sheraton as its hotel operator, which bid for the same number of shares On 17 October 1995, perhaps apprehensive that respondent GSIS has
at P44.00 per share, or P2.42 more than the bid of petitioner. disregarded the tender of the matching bid and that the sale of 51% of the
Pertinent provisions of the bidding rules prepared by respondent GSIS MHC may be hastened by respondent GSIS and consummated with
state - Renong Berhad, petitioner came to this Court on prohibition and
mandamus. On 18 October 1995 the Court issued a temporary restraining the guests who have slept in the hotel and the events that have transpired
order enjoining respondents from perfecting and consummating the sale to therein which make the hotel historic, these alone do not make the hotel
the Malaysian firm. fall under the patrimony of the nation. What is more, the mandate of the
Constitution is addressed to the State, not to respondent GSIS which
On 10 September 1996 the instant case was accepted by the possesses a personality of its own separate and distinct from the
Court En Banc after it was referred to it by the First Division. The case was Philippines as a State.
then set for oral arguments with former Chief Justice Enrique M. Fernando
and Fr. Joaquin G. Bernas, S.J., as amici curiae. Third, granting that the Manila Hotel forms part of the national
patrimony, the constitutional provision invoked is still inapplicable since
In the main, petitioner invokes Sec. 10, second par., Art. XII, of the what is being sold is only 51% of the outstanding shares of the
1987 Constitution and submits that the Manila Hotel has been identified corporation, not the hotel building nor the land upon which the building
with the Filipino nation and has practically become a historical monument stands. Certainly, 51% of the equity of the MHC cannot be considered part
which reflects the vibrancy of Philippine heritage and culture. It is a proud of the national patrimony. Moreover, if the disposition of the shares of the
legacy of an earlier generation of Filipinos who believed in the nobility and MHC is really contrary to the Constitution, petitioner should have
sacredness of independence and its power and capacity to release the full questioned it right from the beginning and not after it had lost in the
potential of the Filipino people. To all intents and purposes, it has become bidding.
a part of the national patrimony.[6] Petitioner also argues that since 51% of
the shares of the MHC carries with it the ownership of the business of the Fourth, the reliance by petitioner on par. V., subpar. J. 1., of the
hotel which is owned by respondent GSIS, a government-owned and bidding rules which provides that if for any reason, the Highest Bidder
controlled corporation, the hotel business of respondent GSIS being a part cannot be awarded the Block of Shares, GSIS may offer this to the other
of the tourism industry is unquestionably a part of the national Qualified Bidders that have validly submitted bids provided that these
economy. Thus, any transaction involving 51% of the shares of stock of Qualified Bidders are willing to match the highest bid in terms of price per
the MHC is clearly covered by the term national economy, to which Sec. share, is misplaced. Respondents postulate that the privilege of submitting
10, second par., Art. XII, 1987 Constitution, applies. [7] a matching bid has not yet arisen since it only takes place if for any
reason, the Highest Bidder cannot be awarded the Block of Shares. Thus
It is also the thesis of petitioner that since Manila Hotel is part of the the submission by petitioner of a matching bid is premature since Renong
national patrimony and its business also unquestionably part of the Berhad could still very well be awarded the block of shares and the
national economy petitioner should be preferred after it has matched the condition giving rise to the exercise of the privilege to submit a matching
bid offer of the Malaysian firm. For the bidding rules mandate that if for bid had not yet taken place.
any reason, the Highest Bidder cannot be awarded the Block of Shares,
GSIS may offer this to the other Qualified Bidders that have validly Finally, the prayer for prohibition grounded on grave abuse of
submitted bids provided that these Qualified Bidders are willing to match discretion should fail since respondent GSIS did not exercise its discretion
the highest bid in terms of price per share. [8] in a capricious, whimsical manner, and if ever it did abuse its discretion it
was not so patent and gross as to amount to an evasion of a positive duty
Respondents except. They maintain that: First, Sec. 10, second par., or a virtual refusal to perform a duty enjoined by law. Similarly, the
Art. XII, of the 1987 Constitution is merely a statement of principle and petition for mandamus should fail as petitioner has no clear legal right to
policy since it is not a self-executing provision and requires implementing what it demands and respondents do not have an imperative duty to
legislation(s) x x x x Thus, for the said provision to operate, there must be perform the act required of them by petitioner.
existing laws to lay down conditions under which business may be done. [9]
We now resolve. A constitution is a system of fundamental laws for
Second, granting that this provision is self-executing, Manila Hotel the governance and administration of a nation. It is supreme, imperious,
does not fall under the term national patrimony which only refers to lands absolute and unalterable except by the authority from which it
of the public domain, waters, minerals, coal, petroleum and other mineral emanates. It has been defined as the fundamental and paramount law of
oils, all forces of potential energy, fisheries, forests or timber, wildlife, the nation.[10] It prescribes the permanent framework of a system of
flora and fauna and all marine wealth in its territorial sea, and exclusive government, assigns to the different departments their respective powers
marine zone as cited in the first and second paragraphs of Sec. 2, Art. XII, and duties, and establishes certain fixed principles on which government is
1987 Constitution. According to respondents, while petitioner speaks of founded. The fundamental conception in other words is that it is a
supreme law to which all other laws must conform and in accordance with would be subordinated to the will of the lawmaking body, which could
which all private rights must be determined and all public authority make them entirely meaningless by simply refusing to pass the needed
administered.[11] Under the doctrine of constitutional supremacy, if a law or implementing statute.[15]
contract violates any norm of the constitution that law or contract whether
promulgated by the legislative or by the executive branch or entered into Respondents argue that Sec. 10, second par., Art. XII, of the 1987
by private persons for private purposes is null and void and without any Constitution is clearly not self-executing, as they quote from discussions
force and effect. Thus, since the Constitution is the fundamental, on the floor of the 1986 Constitutional Commission -
paramount and supreme law of the nation, it is deemed written in every
statute and contract. MR. RODRIGO. Madam President, I am asking this question as
the Chairman of the Committee on Style. If the wording of
Admittedly, some constitutions are merely declarations of policies and PREFERENCE is given to QUALIFIED FILIPINOS, can it be
principles. Their provisions command the legislature to enact laws and understood as a preference to qualified Filipinos vis-a-
carry out the purposes of the framers who merely establish an outline of vis Filipinos who are not qualified. So, why do we not make
government providing for the different departments of the governmental it clear? To qualified Filipinos as against aliens?
machinery and securing certain fundamental and inalienable rights of
citizens.[12] A provision which lays down a general principle, such as those THE PRESIDENT. What is the question of Commissioner
found in Art. II of the 1987 Constitution, is usually not self-executing. But Rodrigo? Is it to remove the word QUALIFIED?
a provision which is complete in itself and becomes operative without the
aid of supplementary or enabling legislation, or that which supplies MR. RODRIGO. No, no, but say definitely TO QUALIFIED
FILIPINOS as against whom? As against aliens or over aliens
sufficient rule by means of which the right it grants may be enjoyed or
protected, is self-executing. Thus a constitutional provision is self- ?
executing if the nature and extent of the right conferred and the liability MR. NOLLEDO. Madam President, I think that is understood. We
imposed are fixed by the constitution itself, so that they can be use the word QUALIFIED because the existing laws or
determined by an examination and construction of its terms, and there is prospective laws will always lay down conditions under
no language indicating that the subject is referred to the legislature for which business may be done.For example, qualifications on
action.[13] capital, qualifications on the setting up of other financial
As against constitutions of the past, modern constitutions have been structures, et cetera (underscoring supplied by
respondents).
generally drafted upon a different principle and have often become in
effect extensive codes of laws intended to operate directly upon the people MR. RODRIGO. It is just a matter of style.
in a manner similar to that of statutory enactments, and the function of
constitutional conventions has evolved into one more like that of a MR. NOLLEDO. Yes.[16]
legislative body. Hence, unless it is expressly provided that a legislative
Quite apparently, Sec. 10, second par., of Art XII is couched in such a
act is necessary to enforce a constitutional mandate, the presumption now
way as not to make it appear that it is non-self-executing but simply for
is that all provisions of the constitution are self-executing. If the
purposes of style. But, certainly, the legislature is not precluded from
constitutional provisions are treated as requiring legislation instead of self-
enacting further laws to enforce the constitutional provision so long as the
executing, the legislature would have the power to ignore and practically
contemplated statute squares with the Constitution. Minor details may be
nullify the mandate of the fundamental law. [14] This can be
left to the legislature without impairing the self-executing nature of
cataclysmic. That is why the prevailing view is, as it has always been, that
constitutional provisions.
-
In self-executing constitutional provisions, the legislature may still
x x x x in case of doubt, the Constitution should be considered self- enact legislation to facilitate the exercise of powers directly granted by the
executing rather than non-self-executing x x x x Unless the contrary is constitution, further the operation of such a provision, prescribe a practice
clearly intended, the provisions of the Constitution should be considered to be used for its enforcement, provide a convenient remedy for the
self-executing, as a contrary rule would give the legislature discretion to protection of the rights secured or the determination thereof, or place
determine when, or whether, they shall be effective. These provisions reasonable safeguards around the exercise of the right. The mere fact that
legislation may supplement and add to or prescribe a penalty for the On the other hand, Sec. 10, second par., Art. XII of the 1987
violation of a self-executing constitutional provision does not render such a Constitution is a mandatory, positive command which is complete in itself
provision ineffective in the absence of such legislation. The omission from and which needs no further guidelines or implementing laws or rules for its
a constitution of any express provision for a remedy for enforcing a right enforcement. From its very words the provision does not require any
or liability is not necessarily an indication that it was not intended to be legislation to put it in operation. It is per se judicially enforceable. When
self-executing. The rule is that a self-executing provision of the our Constitution mandates that [i]n the grant of rights, privileges, and
constitution does not necessarily exhaust legislative power on the subject, concessions covering national economy and patrimony, the State shall
but any legislation must be in harmony with the constitution, further the give preference to qualified Filipinos, it means just that - qualified Filipinos
exercise of constitutional right and make it more available. [17] Subsequent shall be preferred. And when our Constitution declares that a right exists
legislation however does not necessarily mean that the subject in certain specified circumstances an action may be maintained to enforce
constitutional provision is not, by itself, fully enforceable. such right notwithstanding the absence of any legislation on the subject;
consequently, if there is no statute especially enacted to enforce such
Respondents also argue that the non-self-executing nature of Sec. 10, constitutional right, such right enforces itself by its own inherent potency
second par., of Art. XII is implied from the tenor of the first and third and puissance, and from which all legislations must take their
paragraphs of the same section which undoubtedly are not self-executing. bearings. Where there is a right there is a remedy. Ubi jus ibi remedium.
[18]
The argument is flawed. If the first and third paragraphs are not self-
executing because Congress is still to enact measures to encourage the As regards our national patrimony, a member of the 1986
formation and operation of enterprises fully owned by Filipinos, as in the Constitutional Commission[34] explains -
first paragraph, and the State still needs legislation to regulate and
exercise authority over foreign investments within its national jurisdiction, The patrimony of the Nation that should be conserved and
as in the third paragraph, then a fortiori, by the same logic, the second developed refers not only to our rich natural resources but also to
paragraph can only be self-executing as it does not by its language require the cultural heritage of our race. It also refers to our intelligence
any legislation in order to give preference to qualified Filipinos in the grant in arts, sciences and letters. Therefore, we should develop not
of rights, privileges and concessions covering the national economy and only our lands, forests, mines and other natural resources but
patrimony. A constitutional provision may be self-executing in one part also the mental ability or faculty of our people.
and non-self-executing in another.[19]

Even the cases cited by respondents holding that certain We agree. In its plain and ordinary meaning, the
constitutional provisions are merely statements of principles and policies, term patrimony pertains to heritage.[35] When the Constitution speaks
which are basically not self-executing and only placed in the Constitution of national patrimony, it refers not only to the natural resources of the
as moral incentives to legislation, not as judicially enforceable rights - are Philippines, as the Constitution could have very well used the term natural
simply not in point. Basco v. Philippine Amusements and Gaming resources, but also to the cultural heritage of the Filipinos.
Corporation[20] speaks of constitutional provisions on personal dignity,
Manila Hotel has become a landmark - a living testimonial of
[21]
the sanctity of family life, [22] the vital role of the youth in nation-
Philippine heritage. While it was restrictively an American hotel when it
building,[23] the promotion of social justice,[24] and the values of education.
first opened in 1912, it immediately evolved to be truly Filipino.Formerly a
[25]
Tolentino v. Secretary of Finance [26] refers to constitutional provisions
concourse for the elite, it has since then become the venue of various
on social justice and human rights [27] and on education.
significant events which have shaped Philippine history. It was called
[28]
Lastly, Kilosbayan, Inc. v. Morato[29] cites provisions on the promotion
the Cultural Center of the 1930s. It was the site of the festivities during
of general welfare,[30] the sanctity of family life,[31] the vital role of the
the inauguration of the Philippine Commonwealth. Dubbed as the Official
youth in nation-building[32] and the promotion of total human liberation and
Guest House of the Philippine Government it plays host to dignitaries and
development.[33] A reading of these provisions indeed clearly shows that
official visitors who are accorded the traditional Philippine hospitality. [36]
they are not judicially enforceable constitutional rights but merely
guidelines for legislation. The very terms of the provisions manifest that The history of the hotel has been chronicled in the book The Manila
they are only principles upon which legislations must be based. Res ipsa Hotel: The Heart and Memory of a City.[37] During World War II the hotel
loquitur. was converted by the Japanese Military Administration into a military
headquarters. When the American forces returned to recapture Manila the
hotel was selected by the Japanese together with Intramuros as the two MR. DAVIDE. The Nolledo amendment would refer to an
(2) places for their final stand.Thereafter, in the 1950s and 1960s, the individual Filipino. What about a corporation wholly owned
hotel became the center of political activities, playing host to almost every by Filipino citizens?
political convention. In 1970 the hotel reopened after a renovation and
reaped numerous international recognitions, an acknowledgment of the MR. MONSOD. At least 60 percent, Madam President.
Filipino talent and ingenuity. In 1986 the hotel was the site of a MR. DAVIDE. Is that the intention?
failed coup d etat where an aspirant for vice-president was proclaimed
President of the Philippine Republic. MR. MONSOD. Yes, because, in fact, we would be limiting it if we
say that the preference should only be 100-percent Filipino.
For more than eight (8) decades Manila Hotel has bore mute witness
to the triumphs and failures, loves and frustrations of the Filipinos; its MR. DAVIDE. I want to get that meaning clear because
existence is impressed with public interest; its own historicity associated QUALIFIED FILIPINOS may refer only to individuals and not
with our struggle for sovereignty, independence and nationhood. Verily, to juridical personalities or entities.
Manila Hotel has become part of our national economy and patrimony. For
sure, 51% of the equity of the MHC comes within the purview of the MR. MONSOD. We agree, Madam President.[39]
constitutional shelter for it comprises the majority and controlling stock, so xxxx
that anyone who acquires or owns the 51% will have actual control and
management of the hotel. In this instance, 51% of the MHC cannot be MR. RODRIGO. Before we vote, may I request that the
disassociated from the hotel and the land on which the hotel edifice amendment be read again.
stands. Consequently, we cannot sustain respondents claim that
the Filipino First Policy provision is not applicable since what is being sold MR. NOLLEDO. The amendment will read: IN THE GRANT OF
is only 51% of the outstanding shares of the corporation, not the Hotel RIGHTS, PRIVILEGES AND CONCESSIONS COVERING THE
building nor the land upon which the building stands. [38] NATIONAL ECONOMY AND PATRIMONY, THE STATE SHALL
GIVE PREFERENCE TO QUALIFIED FILIPINOS. And the word
The argument is pure sophistry. The term qualified Filipinos as used in Filipinos here, as intended by the proponents, will include
our Constitution also includes corporations at least 60% of which is owned not only individual Filipinos but also Filipino-controlled
by Filipinos. This is very clear from the proceedings of the 1986 entities or entities fully-controlled by Filipinos. [40]
Constitutional Commission -
The phrase preference to qualified Filipinos was explained thus -
THE PRESIDENT. Commissioner Davide is recognized.
MR. FOZ. Madam President, I would like to request Commissioner
MR. DAVIDE. I would like to introduce an amendment to the Nolledo to please restate his amendment so that I can ask a
Nolledo amendment. And the amendment would consist in question.
substituting the words QUALIFIED FILIPINOS with the
following: CITIZENS OF THE PHILIPPINES OR MR. NOLLEDO. IN THE GRANT OF RIGHTS, PRIVILEGES AND
CORPORATIONS OR ASSOCIATIONS WHOSE CAPITAL OR CONCESSIONS COVERING THE NATIONAL ECONOMY AND
CONTROLLING STOCK IS WHOLLY OWNED BY SUCH PATRIMONY, THE STATE SHALL GIVE PREFERENCE TO
CITIZENS. QUALIFIED FILIPINOS.

xxxx MR. FOZ. In connection with that amendment, if a foreign


enterprise is qualified and a Filipino enterprise is also
MR. MONSOD. Madam President, apparently the proponent is qualified, will the Filipino enterprise still be given a
agreeable, but we have to raise a question. Suppose it is a preference?
corporation that is 80-percent Filipino, do we not give it
preference? MR. NOLLEDO. Obviously.

MR. FOZ. If the foreigner is more qualified in some aspects than


the Filipino enterprise, will the Filipino still be preferred?
MR. NOLLEDO. The answer is yes. constitutional provision - by the government itself - is only too
distressing. To adopt such a line of reasoning is to renounce the duty to
MR. FOZ. Thank you. [41]
ensure faithfulness to the Constitution. For, even some of the provisions of
Expounding further on the Filipino First Policy provision Commissioner the Constitution which evidently need implementing legislation have
Nolledo continues juridical life of their own and can be the source of a judicial remedy. We
cannot simply afford the government a defense that arises out of the
MR. NOLLEDO. Yes, Madam President. Instead of MUST, it will be failure to enact further enabling, implementing or guiding legislation. In
SHALL - THE STATE SHALL GIVE PREFERENCE TO fine, the discourse of Fr. Joaquin G. Bernas, S.J., on constitutional
QUALIFIED FILIPINOS. This embodies the so-called Filipino government is apt -
First policy. That means that Filipinos should be given
preference in the grant of concessions, privileges and rights The executive department has a constitutional duty to implement laws,
covering the national patrimony.[42] including the Constitution, even before Congress acts - provided that there
The exchange of views in the sessions of the Constitutional are discoverable legal standards for executive action. When the executive
Commission regarding the subject provision was still further clarified by acts, it must be guided by its own understanding of the constitutional
Commissioner Nolledo[43] - command and of applicable laws. The responsibility for reading and
understanding the Constitution and the laws is not the sole prerogative of
Congress. If it were, the executive would have to ask Congress, or perhaps
Paragraph 2 of Section 10 explicitly mandates the Pro-Filipino bias in all the Court, for an interpretation every time the executive is confronted by a
economic concerns. It is better known as the FILIPINO FIRST Policy x x x constitutional command. That is not how constitutional government
x This provision was never found in previous Constitutions x x x x operates.[45]

The term qualified Filipinos simply means that preference shall be given to Respondents further argue that the constitutional provision is
those citizens who can make a viable contribution to the common good, addressed to the State, not to respondent GSIS which by itself possesses a
because of credible competence and efficiency. It certainly does NOT separate and distinct personality. This argument again is at best
mandate the pampering and preferential treatment to Filipino citizens or specious. It is undisputed that the sale of 51% of the MHC could only be
organizations that are incompetent or inefficient, since such an carried out with the prior approval of the State acting through respondent
indiscriminate preference would be counterproductive and inimical to the Committee on Privatization. As correctly pointed out by Fr. Joaquin G.
common good. Bernas, S.J., this fact alone makes the sale of the assets of respondents
GSIS and MHC a state action. In constitutional jurisprudence, the acts of
In the granting of economic rights, privileges, and concessions, when a persons distinct from the government are considered state action covered
choice has to be made between a qualified foreigner and a qualified by the Constitution (1) when the activity it engages in is a public
Filipino, the latter shall be chosen over the former. function; (2) when the government is so significantly involved with the
private actor as to make the government responsible for his action; and,
Lastly, the word qualified is also determinable. Petitioner was so (3) when the government has approved or authorized the action. It is
considered by respondent GSIS and selected as one of the qualified evident that the act of respondent GSIS in selling 51% of its share in
bidders. It was pre-qualified by respondent GSIS in accordance with its respondent MHC comes under the second and third categories of state
own guidelines so that the sole inference here is that petitioner has been action. Without doubt therefore the transaction, although entered into by
found to be possessed of proven management expertise in the hotel respondent GSIS, is in fact a transaction of the State and therefore subject
industry, or it has significant equity ownership in another hotel company, to the constitutional command.[46]
or it has an overall management and marketing proficiency to successfully
When the Constitution addresses the State it refers not only to the
operate the Manila Hotel.[44]
people but also to the government as elements of the State. After all,
The penchant to try to whittle away the mandate of the Constitution government is composed of three (3) divisions of power - legislative,
by arguing that the subject provision is not self-executory and requires executive and judicial. Accordingly, a constitutional mandate directed to
implementing legislation is quite disturbing. The attempt to violate a clear the State is correspondingly directed to the three (3) branches of
government. It is undeniable that in this case the subject constitutional This Court does not discount the apprehension that this policy may
injunction is addressed among others to the Executive Department and discourage foreign investors. But the Constitution and laws of the
respondent GSIS, a government instrumentality deriving its authority from Philippines are understood to be always open to public scrutiny. These are
the State. given factors which investors must consider when venturing into business
in a foreign jurisdiction. Any person therefore desiring to do business in
It should be stressed that while the Malaysian firm offered the higher the Philippines or with any of its agencies or instrumentalities is presumed
bid it is not yet the winning bidder. The bidding rules expressly provide to know his rights and obligations under the Constitution and the laws of
that the highest bidder shall only be declared the winning bidder after it the forum.
has negotiated and executed the necessary contracts, and secured the
requisite approvals. Since the Filipino First Policy provision of the The argument of respondents that petitioner is now estopped from
Constitution bestows preference on qualified Filipinos the mere tending of questioning the sale to Renong Berhad since petitioner was well aware
the highest bid is not an assurance that the highest bidder will be declared from the beginning that a foreigner could participate in the bidding is
the winning bidder. Resultantly, respondents are not bound to make the meritless. Undoubtedly, Filipinos and foreigners alike were invited to the
award yet, nor are they under obligation to enter into one with the highest bidding. But foreigners may be awarded the sale only if no Filipino
bidder. For in choosing the awardee respondents are mandated to abide by qualifies, or if the qualified Filipino fails to match the highest bid tendered
the dictates of the 1987 Constitution the provisions of which are presumed by the foreign entity. In the case before us, while petitioner was already
to be known to all the bidders and other interested parties. preferred at the inception of the bidding because of the constitutional
mandate, petitioner had not yet matched the bid offered by Renong
Adhering to the doctrine of constitutional supremacy, the subject Berhad. Thus it did not have the right or personality then to compel
constitutional provision is, as it should be, impliedly written in the bidding respondent GSIS to accept its earlier bid. Rightly, only after it had
rules issued by respondent GSIS, lest the bidding rules be nullified for matched the bid of the foreign firm and the apparent disregard by
being violative of the Constitution. It is a basic principle in constitutional respondent GSIS of petitioners matching bid did the latter have a cause of
law that all laws and contracts must conform with the fundamental law of action.
the land. Those which violate the Constitution lose their reason for being.
Besides, there is no time frame for invoking the constitutional
Paragraph V. J. 1 of the bidding rules provides that [i]f for any safeguard unless perhaps the award has been finally made. To insist on
reason the Highest Bidder cannot be awarded the Block of Shares, GSIS selling the Manila Hotel to foreigners when there is a Filipino group willing
may offer this to other Qualified Bidders that have validly submitted bids to match the bid of the foreign group is to insist that government be
provided that these Qualified Bidders are willing to match the highest bid treated as any other ordinary market player, and bound by its mistakes or
in terms of price per share. [47] Certainly, the constitutional mandate itself gross errors of judgment, regardless of the consequences to the Filipino
is reason enough not to award the block of shares immediately to the people. The miscomprehension of the Constitution is regrettable. Thus we
foreign bidder notwithstanding its submission of a higher, or even the would rather remedy the indiscretion while there is still an opportunity to
highest, bid. In fact, we cannot conceive of a stronger reason than the do so than let the government develop the habit of forgetting that the
constitutional injunction itself. Constitution lays down the basic conditions and parameters for its actions.
In the instant case, where a foreign firm submits the highest bid in a Since petitioner has already matched the bid price tendered by
public bidding concerning the grant of rights, privileges and concessions Renong Berhad pursuant to the bidding rules, respondent GSIS is left with
covering the national economy and patrimony, thereby exceeding the bid no alternative but to award to petitioner the block of shares of MHC and to
of a Filipino, there is no question that the Filipino will have to be allowed to execute the necessary agreements and documents to effect the sale in
match the bid of the foreign entity. And if the Filipino matches the bid of a accordance not only with the bidding guidelines and procedures but with
foreign firm the award should go to the Filipino. It must be so if we are to the Constitution as well. The refusal of respondent GSIS to execute the
give life and meaning to the Filipino First Policy provision of the 1987 corresponding documents with petitioner as provided in the bidding rules
Constitution. For, while this may neither be expressly stated nor after the latter has matched the bid of the Malaysian firm clearly
contemplated in the bidding rules, the constitutional fiat is omnipresent to constitutes grave abuse of discretion.
be simply disregarded. To ignore it would be to sanction a perilous skirting
of the basic law. The Filipino First Policy is a product of Philippine nationalism. It is
embodied in the 1987 Constitution not merely to be used as a guideline for
future legislation but primarily to be enforced; so must it be enforced. This state can have no higher purpose. Any interpretation of any constitutional
Court as the ultimate guardian of the Constitution will never shun, under provision must adhere to such basic concept. Protection of foreign
any reasonable circumstance, the duty of upholding the majesty of the investments, while laudible, is merely a policy. It cannot override the
Constitution which it is tasked to defend. It is worth emphasizing that it is demands of nationalism.[50]
not the intention of this Court to impede and diminish, much less
undermine, the influx of foreign investments. Far from it, the Court The Manila Hotel or, for that matter, 51% of the MHC, is not just any
encourages and welcomes more business opportunities but avowedly commodity to be sold to the highest bidder solely for the sake of
sanctions the preference for Filipinos whenever such preference is privatization. We are not talking about an ordinary piece of property in a
ordained by the Constitution. The position of the Court on this matter commercial district. We are talking about a historic relic that has hosted
could have not been more appropriately articulated by Chief Justice many of the most important events in the short history of the Philippines
Narvasa - as a nation. We are talking about a hotel where heads of states would
prefer to be housed as a strong manifestation of their desire to cloak the
dignity of the highest state function to their official visits to the
As scrupulously as it has tried to observe that it is not its function to Philippines. Thus the Manila Hotel has played and continues to play a
substitute its judgment for that of the legislature or the executive about significant role as an authentic repository of twentieth century Philippine
the wisdom and feasibility of legislation economic in nature, the Supreme history and culture. In this sense, it has become truly a reflection of the
Court has not been spared criticism for decisions perceived as obstacles to Filipino soul - a place with a history of grandeur; a most historical setting
economic progress and development x x x x in connection with a that has played a part in the shaping of a country. [51]
temporary injunction issued by the Courts First Division against the sale of
the Manila Hotel to a Malaysian Firm and its partner, certain statements This Court cannot extract rhyme nor reason from the determined
were published in a major daily to the effect that that injunction again efforts of respondents to sell the historical landmark - this Grand Old
demonstrates that the Philippine legal system can be a major obstacle to Dame of hotels in Asia - to a total stranger. For, indeed, the conveyance of
doing business here. this epic exponent of the Filipino psyche to alien hands cannot be less than
mephistophelian for it is, in whatever manner viewed, a veritable
Let it be stated for the record once again that while it is no business of the alienation of a nations soul for some pieces of foreign silver. And so we
Court to intervene in contracts of the kind referred to or set itself up as ask: What advantage, which cannot be equally drawn from a qualified
the judge of whether they are viable or attainable, it is its bounden duty to Filipino, can be gained by the Filipinos if Manila Hotel - and all that it
make sure that they do not violate the Constitution or the laws, or are not stands for - is sold to a non-Filipino? How much of national pride will
adopted or implemented with grave abuse of discretion amounting to lack vanish if the nations cultural heritage is entrusted to a foreign entity? On
or excess of jurisdiction. It will never shirk that duty, no matter how the other hand, how much dignity will be preserved and realized if the
buffeted by winds of unfair and ill-informed criticism. [48] national patrimony is safekept in the hands of a qualified, zealous and
well-meaning Filipino? This is the plain and simple meaning of the Filipino
First Policy provision of the Philippine Constitution. And this Court, heeding
Privatization of a business asset for purposes of enhancing its
the clarion call of the Constitution and accepting the duty of being the
business viability and preventing further losses, regardless of the
elderly watchman of the nation, will continue to respect and protect the
character of the asset, should not take precedence over non-material
sanctity of the Constitution.
values. A commercial, nay even a budgetary, objective should not be
pursued at the expense of national pride and dignity. For the Constitution WHEREFORE, respondents GOVERNMENT SERVICE INSURANCE
enshrines higher and nobler non-material values. Indeed, the Court will SYSTEM, MANILA HOTEL CORPORATION, COMMITTEE ON PRIVATIZATION
always defer to the Constitution in the proper governance of a free and OFFICE OF THE GOVERNMENT CORPORATE COUNSEL are directed to
society; after all, there is nothing so sacrosanct in any economic policy as CEASE and DESIST from selling 51% of the shares of the Manila Hotel
to draw itself beyond judicial review when the Constitution is involved. [49] Corporation to RENONG BERHAD, and to ACCEPT the matching bid of
petitioner MANILA PRINCE HOTEL CORPORATION to purchase the subject
Nationalism is inherent in the very concept of the Philippines being a
51% of the shares of the Manila Hotel Corporation at P44.00 per share and
democratic and republican state, with sovereignty residing in the Filipino
thereafter to execute the necessary agreements and documents to effect
people and from whom all government authority emanates. In nationalism,
the sale, to issue the necessary clearances and to do such other acts and
the happiness and welfare of the people must be the goal. The nation-
deeds as may be necessary for the purpose.
SO ORDERED.

Regalado, Davide, Jr., Romero, Kapunan,


Francisco, and Hermosisima, Jr., JJ, concur.
Narvasa, C.J., (Chairman), and Melo, J., joins J. Puno in his dissent.
Padilla, J., see concurring opinion.
Vitug, J., see separate concurring opinion
Mendoza, J., see concurring opinion
Torres, J., with separate opinion
Puno, J., see dissent.
Panganiban J., with separate dissenting opinion.
EN BANC The emergence on January 1, 1995 of the World Trade Organization,
abetted by the membership thereto of the vast majority of countries has
revolutionized international business and economic relations amongst
states. It has irreversibly propelled the world towards trade liberalization
[G.R. No. 118295. May 2, 1997] and economic globalization. Liberalization, globalization, deregulation and
privatization, the third-millennium buzz words, are ushering in a new
borderless world of business by sweeping away as mere historical relics
the heretofore traditional modes of promoting and protecting national
economies like tariffs, export subsidies, import quotas, quantitative
WIGBERTO E. TAADA and ANNA DOMINIQUE COSETENG, as
restrictions, tax exemptions and currency controls. Finding market niches
members of the Philippine Senate and as taxpayers;
and becoming the best in specific industries in a market-driven and
GREGORIO ANDOLANA and JOKER ARROYO as members of
export-oriented global scenario are replacing age-old beggar-thy-neighbor
the House of Representatives and as taxpayers; NICANOR
policies that unilaterally protect weak and inefficient domestic producers of
P. PERLAS and HORACIO R. MORALES, both as taxpayers;
goods and services. In the words of Peter Drucker, the well-known
CIVIL LIBERTIES UNION, NATIONAL ECONOMIC
management guru, Increased participation in the world economy has
PROTECTIONISM ASSOCIATION, CENTER FOR ALTERNATIVE
become the key to domestic economic growth and prosperity.
DEVELOPMENT INITIATIVES, LIKAS-KAYANG KAUNLARAN
FOUNDATION, INC., PHILIPPINE RURAL RECONSTRUCTION
MOVEMENT, DEMOKRATIKONG KILUSAN NG MAGBUBUKID
NG PILIPINAS, INC., and PHILIPPINE PEASANT INSTITUTE, Brief Historical Background
in representation of various taxpayers and as non-
governmental organizations, petitioners, vs. EDGARDO
ANGARA, ALBERTO ROMULO, LETICIA RAMOS-SHAHANI, To hasten worldwide recovery from the devastation wrought by the
HEHERSON ALVAREZ, AGAPITO AQUINO, RODOLFO BIAZON, Second World War, plans for the establishment of three multilateral
NEPTALI GONZALES, ERNESTO HERRERA, JOSE LINA, institutions -- inspired by that grand political body, the United Nations --
GLORIA MACAPAGAL-ARROYO, ORLANDO MERCADO, BLAS were discussed at Dumbarton Oaks and Bretton Woods. The first was the
OPLE, JOHN OSMEA, SANTANINA RASUL, RAMON REVILLA, World Bank (WB) which was to address the rehabilitation and
RAUL ROCO, FRANCISCO TATAD and FREDDIE WEBB, in reconstruction of war-ravaged and later developing countries; the second,
their respective capacities as members of the Philippine the International Monetary Fund (IMF) which was to deal with currency
Senate who concurred in the ratification by the President of problems; and the third, the International Trade Organization (ITO), which
the Philippines of the Agreement Establishing the World was to foster order and predictability in world trade and to minimize
Trade Organization; SALVADOR ENRIQUEZ, in his capacity unilateral protectionist policies that invite challenge, even retaliation, from
as Secretary of Budget and Management; CARIDAD other states. However, for a variety of reasons, including its non-
VALDEHUESA, in her capacity as National Treasurer; ratification by the United States, the ITO, unlike the IMF and WB, never
RIZALINO NAVARRO, in his capacity as Secretary of Trade took off. What remained was only GATT -- the General Agreement on
and Industry; ROBERTO SEBASTIAN, in his capacity as Tariffs and Trade. GATT was a collection of treaties governing access to
Secretary of Agriculture; ROBERTO DE OCAMPO, in his the economies of treaty adherents with no institutionalized body
capacity as Secretary of Finance; ROBERTO ROMULO, in his administering the agreements or dependable system of dispute
capacity as Secretary of Foreign Affairs; and TEOFISTO T. settlement.
GUINGONA, in his capacity as Executive
Secretary, respondents. After half a century and several dizzying rounds of negotiations,
principally the Kennedy Round, the Tokyo Round and the Uruguay Round,
the world finally gave birth to that administering body -- the World Trade
DECISION Organization -- with the signing of the Final Act in Marrakesh, Morocco and
PANGANIBAN, J.: the ratification of the WTO Agreement by its members. [1]
Like many other developing countries, the Philippines joined WTO as a executive offices concerned therewith. This concurrence is embodied in
founding member with the goal, as articulated by President Fidel V. Ramos Senate Resolution No. 97, dated December 14, 1994.
in two letters to the Senate (infra), of improving Philippine access to
foreign markets, especially its major trading partners, through the
reduction of tariffs on its exports, particularly agricultural and industrial
products. The President also saw in the WTO the opening of new The Facts
opportunities for the services sector x x x, (the reduction of) costs and
uncertainty associated with exporting x x x, and (the attraction of) more
investments into the country.Although the Chief Executive did not On April 15, 1994, Respondent Rizalino Navarro, then Secretary of
expressly mention it in his letter, the Philippines - - and this is of special the Department of Trade and Industry (Secretary Navarro, for brevity),
interest to the legal profession - - will benefit from the WTO system of representing the Government of the Republic of the Philippines, signed in
dispute settlement by judicial adjudication through the independent WTO Marrakesh, Morocco, the Final Act Embodying the Results of the Uruguay
settlement bodies called (1) Dispute Settlement Panels and (2) Appellate Round of Multilateral Negotiations (Final Act, for brevity).
Tribunal. Heretofore, trade disputes were settled mainly through By signing the Final Act,[2] Secretary Navarro on behalf of the Republic
negotiations where solutions were arrived at frequently on the basis of of the Philippines, agreed:
relative bargaining strengths, and where naturally, weak and
underdeveloped countries were at a disadvantage.
(a) to submit, as appropriate, the WTO Agreement for the consideration of
their respective competent authorities, with a view to seeking approval of
the Agreement in accordance with their procedures; and
The Petition in Brief
(b) to adopt the Ministerial Declarations and Decisions.

Arguing mainly (1) that the WTO requires the Philippines to place
On August 12, 1994, the members of the Philippine Senate received a
nationals and products of member-countries on the same footing as
letter dated August 11, 1994 from the President of the Philippines,
Filipinos and local products and (2) that the WTO intrudes, limits and/or [3]
stating among others that the Uruguay Round Final Act is hereby
impairs the constitutional powers of both Congress and the Supreme
submitted to the Senate for its concurrence pursuant to Section 21, Article
Court, the instant petition before this Court assails the WTO Agreement for
VII of the Constitution.
violating the mandate of the 1987 Constitution to develop a self-reliant
and independent national economy effectively controlled by Filipinos x x x On August 13, 1994, the members of the Philippine Senate received
(to) give preference to qualified Filipinos (and to) promote the preferential another letter from the President of the Philippines [4] likewise dated August
use of Filipino labor, domestic materials and locally produced goods. 11, 1994, which stated among others that the Uruguay Round Final Act,
the Agreement Establishing the World Trade Organization, the Ministerial
Simply stated, does the Philippine Constitution prohibit Philippine
Declarations and Decisions, and the Understanding on Commitments in
participation in worldwide trade liberalization and economic
Financial Services are hereby submitted to the Senate for its concurrence
globalization? Does it prescribe Philippine integration into a global
pursuant to Section 21, Article VII of the Constitution.
economy that is liberalized, deregulated and privatized? These are the
main questions raised in this petition for certiorari, prohibition On December 9, 1994, the President of the Philippines certified the
and mandamus under Rule 65 of the Rules of Court praying (1) for the necessity of the immediate adoption of P.S. 1083, a resolution entitled
nullification, on constitutional grounds, of the concurrence of the Philippine Concurring in the Ratification of the Agreement Establishing the World
Senate in the ratification by the President of the Philippines of the Trade Organization.[5]
Agreement Establishing the World Trade Organization (WTO Agreement,
for brevity) and (2) for the prohibition of its implementation and On December 14, 1994, the Philippine Senate adopted Resolution No.
enforcement through the release and utilization of public funds, the 97 which Resolved, as it is hereby resolved, that the Senate concur, as it
assignment of public officials and employees, as well as the use of hereby concurs, in the ratification by the President of the Philippines of the
government properties and resources by respondent-heads of various Agreement Establishing the World Trade Organization. [6] The text of the
WTO Agreement is written on pages 137 et seq. of Volume I of the 36- Annex 1B: General Agreement on Trade in Services and Annexes
volume Uruguay Round of Multilateral Trade Negotiations and includes
various agreements and associated legal instruments (identified in the said Annex 1C: Agreement on Trade-Related Aspects of Intellectual Property
Agreement as Annexes 1, 2 and 3 thereto and collectively referred to as Rights
Multilateral Trade Agreements, for brevity) as follows:
ANNEX 2
ANNEX 1
Understanding on Rules and Procedures Governing
Annex 1A: Multilateral Agreement on Trade in Goods the Settlement of Disputes

General Agreement on Tariffs and Trade 1994 ANNEX 3

Agreement on Agriculture Trade Policy Review Mechanism

Agreement on the Application of Sanitary and On December 16, 1994, the President of the Philippines signed [7] the
Instrument of Ratification, declaring:
Phytosanitary Measures
NOW THEREFORE, be it known that I, FIDEL V. RAMOS, President of the
Agreement on Textiles and Clothing Republic of the Philippines, after having seen and considered the
aforementioned Agreement Establishing the World Trade Organization and
Agreement on Technical Barriers to Trade the agreements and associated legal instruments included in Annexes one
(1), two (2) and three (3) of that Agreement which are integral parts
thereof, signed at Marrakesh, Morocco on 15 April 1994, do hereby ratify
Agreement on Trade-Related Investment Measures
and confirm the same and every Article and Clause thereof.

Agreement on Implementation of Article VI of


To emphasize, the WTO Agreement ratified by the President of the
the General Agreement on Tariffs and Trade
Philippines is composed of the Agreement Proper and the associated legal
1994
instruments included in Annexes one (1), two (2) and three (3) of that
Agreement which are integral parts thereof.
Agreement on Implementation of Article VII of the
General on Tariffs and Trade 1994 On the other hand, the Final Act signed by Secretary Navarro
embodies not only the WTO Agreement (and its integral annexes
aforementioned) but also (1) the Ministerial Declarations and Decisions
Agreement on Pre-Shipment Inspection
and (2) the Understanding on Commitments in Financial Services. In his
Memorandum dated May 13, 1996, [8] the Solicitor General describes these
Agreement on Rules of Origin two latter documents as follows:

Agreement on Imports Licensing Procedures The Ministerial Decisions and Declarations are twenty-five declarations and
decisions on a wide range of matters, such as measures in favor of least
Agreement on Subsidies and Coordinating Measures developed countries, notification procedures, relationship of WTO with the
International Monetary Fund (IMF), and agreements on technical barriers
Agreement on Safeguards to trade and on dispute settlement.
The Understanding on Commitments in Financial Services dwell on, among A. Whether the petition presents a political question or is otherwise not
other things, standstill or limitations and qualifications of commitments to justiciable.
existing non-conforming measures, market access, national treatment,
and definitions of non-resident supplier of financial services, commercial B. Whether the petitioner members of the Senate who participated in
presence and new financial service. the deliberations and voting leading to the concurrence are
estopped from impugning the validity of the Agreement
On December 29, 1994, the present petition was filed. After careful Establishing the World Trade Organization or of the validity of the
deliberation on respondents comment and petitioners reply thereto, the concurrence.
Court resolved on December 12, 1995, to give due course to the petition,
and the parties thereafter filed their respective memoranda. The Court C. Whether the provisions of the Agreement Establishing the World
also requested the Honorable Lilia R. Bautista, the Philippine Ambassador Trade Organization contravene the provisions of Sec. 19, Article
to the United Nations stationed in Geneva, Switzerland, to submit a paper, II, and Secs. 10 and 12, Article XII, all of the 1987 Philippine
hereafter referred to as Bautista Paper, [9] for brevity, (1) providing a Constitution.
historical background of and (2) summarizing the said agreements.

During the Oral Argument held on August 27, 1996, the Court D. Whether provisions of the Agreement Establishing the World Trade
directed: Organization unduly limit, restrict and impair Philippine
sovereignty specifically the legislative power which, under Sec. 2,
(a) the petitioners to submit the (1) Senate Committee Report on the Article VI, 1987 Philippine Constitution is vested in the Congress
matter in controversy and (2) the transcript of proceedings/hearings in the of the Philippines;
Senate; and
E. Whether provisions of the Agreement Establishing the World Trade
(b) the Solicitor General, as counsel for respondents, to file (1) a list of Organization interfere with the exercise of judicial power.
Philippine treaties signed prior to the Philippine adherence to the WTO
Agreement, which derogate from Philippine sovereignty and (2) copies of F. Whether the respondent members of the Senate acted in grave
the multi-volume WTO Agreement and other documents mentioned in the abuse of discretion amounting to lack or excess of jurisdiction
Final Act, as soon as possible. when they voted for concurrence in the ratification of the
constitutionally-infirm Agreement Establishing the World Trade
After receipt of the foregoing documents, the Court said it would Organization.
consider the case submitted for resolution. In a Compliance dated
September 16, 1996, the Solicitor General submitted a printed copy of the G. Whether the respondent members of the Senate acted in grave
36-volume Uruguay Round of Multilateral Trade Negotiations, and in abuse of discretion amounting to lack or excess of jurisdiction
another Compliance dated October 24, 1996, he listed the various bilateral when they concurred only in the ratification of the Agreement
or multilateral treaties or international instruments involving derogation of Establishing the World Trade Organization, and not with the
Philippine sovereignty. Petitioners, on the other hand, submitted their Presidential submission which included the Final Act, Ministerial
Compliance dated January 28, 1997, on January 30, 1997. Declaration and Decisions, and the Understanding on
Commitments in Financial Services.

On the other hand, the Solicitor General as counsel for respondents


The Issues
synthesized the several issues raised by petitioners into the following: [10]

In their Memorandum dated March 11, 1996, petitioners summarized 1. Whether or not the provisions of the Agreement Establishing the World
the issues as follows: Trade Organization and the Agreements and Associated Legal Instruments
included in Annexes one (1), two (2) and three (3) of that agreement cited
by petitioners directly contravene or undermine the letter, spirit and intent are also deemed to have waived the benefit of such issue.They probably
of Section 19, Article II and Sections 10 and 12, Article XII of the 1987 realized that grave constitutional issues, expenditures of public funds and
Constitution. serious international commitments of the nation are involved here, and
that transcendental public interest requires that the substantive issues be
2. Whether or not certain provisions of the Agreement unduly limit, restrict met head on and decided on the merits, rather than skirted or deflected by
or impair the exercise of legislative power by Congress. procedural matters.[11]

To recapitulate, the issues that will be ruled upon shortly are:


3. Whether or not certain provisions of the Agreement impair the exercise
of judicial power by this Honorable Court in promulgating the rules of (1) DOES THE PETITION PRESENT A JUSTICIABLE
evidence. CONTROVERSY? OTHERWISE STATED, DOES THE PETITION
INVOLVE A POLITICAL QUESTION OVER WHICH THIS
COURT HAS NO JURISDICTION?
4. Whether or not the concurrence of the Senate in the ratification by the
President of the Philippines of the Agreement establishing the World Trade (2) DO THE PROVISIONS OF THE WTO AGREEMENT AND ITS
Organization implied rejection of the treaty embodied in the Final Act. THREE ANNEXES CONTRAVENE SEC. 19, ARTICLE II, AND
SECS. 10 AND 12, ARTICLE XII, OF THE PHILIPPINE
By raising and arguing only four issues against the seven presented CONSTITUTION?
by petitioners, the Solicitor General has effectively ignored three, namely:
(3) DO THE PROVISIONS OF SAID AGREEMENT AND ITS
(1) whether the petition presents a political question or is otherwise not
ANNEXES LIMIT, RESTRICT, OR IMPAIR THE EXERCISE OF
justiciable; (2) whether petitioner-members of the Senate (Wigberto E.
LEGISLATIVE POWER BY CONGRESS?
Taada and Anna Dominique Coseteng) are estopped from joining this suit;
and (3) whether the respondent-members of the Senate acted in grave (4) DO SAID PROVISIONS UNDULY IMPAIR OR INTERFERE WITH
abuse of discretion when they voted for concurrence in the ratification of THE EXERCISE OF JUDICIAL POWER BY THIS COURT IN
the WTO Agreement. The foregoing notwithstanding, this Court resolved to PROMULGATING RULES ON EVIDENCE?
deal with these three issues thus:
(5) WAS THE CONCURRENCE OF THE SENATE IN THE WTO
AGREEMENT AND ITS ANNEXES SUFFICIENT AND/OR
(1) The political question issue -- being very fundamental and vital, and
VALID, CONSIDERING THAT IT DID NOT INCLUDE THE
being a matter that probes into the very jurisdiction of this Court to hear
FINAL ACT, MINISTERIAL DECLARATIONS AND DECISIONS,
and decide this case -- was deliberated upon by the Court and will thus be
AND THE UNDERSTANDING ON COMMITMENTS IN
ruled upon as the first issue;
FINANCIAL SERVICES?

(2) The matter of estoppel will not be taken up because this defense is
waivable and the respondents have effectively waived it by not pursuing it
in any of their pleadings; in any event, this issue, even if ruled in The First Issue: Does the Court Have Jurisdiction Over the
respondents favor, will not cause the petitions dismissal as there are Controversy?
petitioners other than the two senators, who are not vulnerable to the
defense of estoppel; and
In seeking to nullify an act of the Philippine Senate on the ground that
(3) The issue of alleged grave abuse of discretion on the part of the it contravenes the Constitution, the petition no doubt raises a justiciable
respondent senators will be taken up as an integral part of the disposition controversy. Where an action of the legislative branch is seriously alleged
of the four issues raised by the Solicitor General. to have infringed the Constitution, it becomes not only the right but in fact
the duty of the judiciary to settle the dispute. The question thus posed is
judicial rather than political. The duty (to adjudicate) remains to assure
During its deliberations on the case, the Court noted that the that the supremacy of the Constitution is upheld. [12] Once a controversy as
respondents did not question the locus standi of petitioners. Hence, they to the application or interpretation of a constitutional provision is raised
before this Court (as in the instant case), it becomes a legal issue which abuse of discretion amounting to lack or excess of jurisdiction on the part
the Court is bound by constitutional mandate to decide. [13] of the Senate in ratifying the WTO Agreement and its three annexes.

The jurisdiction of this Court to adjudicate the matters [14] raised in the
petition is clearly set out in the 1987 Constitution, [15] as follows:
Second Issue: The WTO Agreement and Economic Nationalism
Judicial power includes the duty of the courts of justice to settle actual
controversies involving rights which are legally demandable and
enforceable, and to determine whether or not there has been a grave This is the lis mota, the main issue, raised by the petition.
abuse of discretion amounting to lack or excess of jurisdiction on the part Petitioners vigorously argue that the letter, spirit and intent of the
of any branch or instrumentality of the government. Constitution mandating economic nationalism are violated by the so-called
parity provisions and national treatment clauses scattered in various parts
The foregoing text emphasizes the judicial departments duty and not only of the WTO Agreement and its annexes but also in the Ministerial
power to strike down grave abuse of discretion on the part of any branch Decisions and Declarations and in the Understanding on Commitments in
or instrumentality of government including Congress. It is an innovation in Financial Services.
our political law.[16] As explained by former Chief Justice Roberto
Concepcion,[17] the judiciary is the final arbiter on the question of whether Specifically, the flagship constitutional provisions referred to are Sec.
or not a branch of government or any of its officials has acted without 19, Article II, and Secs. 10 and 12, Article XII, of the Constitution, which
jurisdiction or in excess of jurisdiction or so capriciously as to constitute an are worded as follows:
abuse of discretion amounting to excess of jurisdiction. This is not only a
judicial power but a duty to pass judgment on matters of this nature. Article II

As this Court has repeatedly and firmly emphasized in many cases,


it will not shirk, digress from or abandon its sacred duty and authority
[18] DECLARATION OF PRINCIPLES AND STATE POLICIES
to uphold the Constitution in matters that involve grave abuse of discretion
brought before it in appropriate cases, committed by any officer, agency, xx xx xx xx
instrumentality or department of the government.
Sec. 19. The State shall develop a self-reliant and independent national
As the petition alleges grave abuse of discretion and as there is no
economy effectively controlled by Filipinos.
other plain, speedy or adequate remedy in the ordinary course of law, we
have no hesitation at all in holding that this petition should be given due
course and the vital questions raised therein ruled upon under Rule 65 of xx xx xx xx
the Rules of Court. Indeed, certiorari, prohibition and mandamus are
appropriate remedies to raise constitutional issues and to review and/or Article XII
prohibit/nullify, when proper, acts of legislative and executive officials. On
this, we have no equivocation. NATIONAL ECONOMY AND PATRIMONY
We should stress that, in deciding to take jurisdiction over this
petition, this Court will not review the wisdom of the decision of the xx xx xx xx
President and the Senate in enlisting the country into the WTO, or pass
upon the merits of trade liberalization as a policy espoused by said Sec. 10. x x x. The Congress shall enact measures that will encourage the
international body. Neither will it rule on the propriety of the governments formation and operation of enterprises whose capital is wholly owned by
economic policy of reducing/removing tariffs, taxes, subsidies, quantitative Filipinos.
restrictions, and other import/trade barriers. Rather, it will only exercise
its constitutional duty to determine whether or not there had been a grave
In the grant of rights, privileges, and concessions covering the national compliance with which is necessary to obtain an advantage,
economy and patrimony, the State shall give preference to qualified and which require:
Filipinos.
(a) the purchase or use by an enterprise of products of domestic
xx xx xx xx origin or from any domestic source, whether specified in
terms of particular products, in terms of volume or value of
Sec. 12. The State shall promote the preferential use of Filipino labor, products, or in terms of proportion of volume or value of its
domestic materials and locally produced goods, and adopt measures that local production; or
help make them competitive.
(b) that an enterprises purchases or use of imported products be
Petitioners aver that these sacred constitutional principles are limited to an amount related to the volume or value of local
desecrated by the following WTO provisions quoted in their memorandum: products that it exports.
[19]

2. TRIMS that are inconsistent with the obligations of general


a) In the area of investment measures related to trade in goods elimination of quantitative restrictions provided for in paragraph 1
(TRIMS, for brevity): of Article XI of GATT 1994 include those which are mandatory or
enforceable under domestic laws or under administrative rulings,
or compliance with which is necessary to obtain an advantage, and
Article 2
which restrict:

National Treatment and Quantitative Restrictions.


(a) the importation by an enterprise of products used in or
related to the local production that it exports;
1. Without prejudice to other rights and obligations under GATT
1994. no Member shall apply any TRIM that is inconsistent
(b) the importation by an enterprise of products used in or
with the provisions of Article III or Article XI of GATT 1994.
related to its local production by restricting its access to
foreign exchange inflows attributable to the enterprise; or
2. An Illustrative list of TRIMS that are inconsistent with the
obligations of general elimination of quantitative restrictions
(c) the exportation or sale for export specified in terms of
provided for in paragraph I of Article XI of GATT 1994 is
particular products, in terms of volume or value of products,
contained in the Annex to this Agreement. (Agreement on
or in terms of a preparation of volume or value of its local
Trade-Related Investment Measures, Vol. 27, Uruguay Round,
production. (Annex to the Agreement on Trade-Related
Legal Instruments, p.22121, emphasis supplied).
Investment Measures, Vol. 27, Uruguay Round Legal
Documents, p.22125, emphasis supplied).
The Annex referred to reads as follows:
The paragraph 4 of Article III of GATT 1994 referred to is quoted as
ANNEX follows:

Illustrative List The products of the territory of any contracting party imported into the
territory of any other contracting party shall be accorded treatment no
1. TRIMS that are inconsistent with the obligation of national less favorable than that accorded to like products of national
treatment provided for in paragraph 4 of Article III of GATT origin in respect of laws, regulations and requirements affecting their
1994 include those which are mandatory or enforceable internal sale, offering for sale, purchase, transportation, distribution or
under domestic law or under administrative rulings, or use. the provisions of this paragraph shall not prevent the application of
differential internal transportation charges which are based exclusively on It is petitioners position that the foregoing national treatment and
the economic operation of the means of transport and not on the parity provisions of the WTO Agreement place nationals and products of
nationality of the product. (Article III, GATT 1947, as amended by the member countries on the same footing as Filipinos and local products, in
Protocol Modifying Part II, and Article XXVI of GATT, 14 September 1948, contravention of the Filipino First policy of the Constitution. They allegedly
62 UMTS 82-84 in relation to paragraph 1(a) of the General Agreement on render meaningless the phrase effectively controlled by Filipinos. The
Tariffs and Trade 1994, Vol. 1, Uruguay Round, Legal Instruments p.177, constitutional conflict becomes more manifest when viewed in the context
emphasis supplied). of the clear duty imposed on the Philippines as a WTO member to ensure
the conformity of its laws, regulations and administrative procedures with
b) In the area of trade related aspects of intellectual property its obligations as provided in the annexed agreements. [20] Petitioners
rights (TRIPS, for brevity): further argue that these provisions contravene constitutional limitations on
the role exports play in national development and negate the preferential
treatment accorded to Filipino labor, domestic materials and locally
Each Member shall accord to the nationals of other Members
produced goods.
treatment no less favourable than that it accords to its own
nationals with regard to the protection of intellectual property... (par. 1, On the other hand, respondents through the Solicitor General counter
Article 3, Agreement on Trade-Related Aspect of Intellectual Property (1) that such Charter provisions are not self-executing and merely set out
rights, Vol. 31, Uruguay Round, Legal Instruments, p.25432 (emphasis general policies; (2) that these nationalistic portions of the Constitution
supplied) invoked by petitioners should not be read in isolation but should be related
to other relevant provisions of Art. XII, particularly Secs. 1 and 13 thereof;
(c) In the area of the General Agreement on Trade in Services: (3) that read properly, the cited WTO clauses do not conflict with the
Constitution; and (4) that the WTO Agreement contains sufficient
provisions to protect developing countries like the Philippines from the
National Treatment
harshness of sudden trade liberalization.

1. In the sectors inscribed in its schedule, and subject to any We shall now discuss and rule on these arguments.
conditions and qualifications set out therein, each Member
shall accord to services and service suppliers of any other
Member, in respect of all measures affecting the supply of
Declaration of Principles Not Self-Executing
services, treatment no less favourable than it accords to
its own like services and service suppliers.
By its very title, Article II of the Constitution is a declaration of
2. A Member may meet the requirement of paragraph I by principles and state policies. The counterpart of this article in the 1935
according to services and service suppliers of any other Constitution[21] is called the basic political creed of the nationby Dean
Member, either formally identical treatment or formally Vicente Sinco.[22] These principles in Article II are not intended to be self-
different treatment to that it accords to its own like services executing principles ready for enforcement through the courts. [23] They are
and service suppliers. used by the judiciary as aids or as guides in the exercise of its power of
judicial review, and by the legislature in its enactment of laws. As held in
3. Formally identical or formally different treatment shall be the leading case of Kilosbayan, Incorporated vs. Morato,[24] the principles
considered to be less favourable if it modifies the conditions of and state policies enumerated in Article II and some sections of Article XII
completion in favour of services or service suppliers of the are not self-executing provisions, the disregard of which can give rise to a
Member compared to like services or service suppliers of any cause of action in the courts. They do not embody judicially enforceable
other Member. (Article XVII, General Agreement on Trade in constitutional rights but guidelines for legislation.
Services, Vol. 28, Uruguay Round Legal Instruments, p.22610
emphasis supplied). In the same light, we held in Basco vs. Pagcor[25] that broad
constitutional principles need legislative enactments to implement them,
thus:
On petitioners allegation that P.D. 1869 violates Sections 11 (Personal The second is a broader-gauge consideration -- where a specific violation
Dignity) 12 (Family) and 13 (Role of Youth) of Article II; Section 13 (Social of law or applicable regulation is not alleged or proved, petitioners can be
Justice) of Article XIII and Section 2 (Educational Values) of Article XIV of expected to fall back on the expanded conception of judicial power in the
the 1987 Constitution, suffice it to state also that these are merely second paragraph of Section 1 of Article VIII of the Constitution which
statements of principles and policies. As such, they are basically not self- reads:
executing, meaning a law should be passed by Congress to clearly define
and effectuate such principles. Section 1. x x x

In general, therefore, the 1935 provisions were not intended to be self- Judicial power includes the duty of the courts of justice to settle actual
executing principles ready for enforcement through the courts. They were controversies involving rights which are legally demandable and
rather directives addressed to the executive and to the legislature. If the enforceable, and to determine whether or not there has been a grave
executive and the legislature failed to heed the directives of the article, the abuse of discretion amounting to lack or excess of jurisdiction on the part
available remedy was not judicial but political. The electorate could of any branch or instrumentality of the Government. (Emphases supplied)
express their displeasure with the failure of the executive and the
legislature through the language of the ballot. (Bernas, Vol. II, p. 2).
When substantive standards as general as the right to a balanced and
healthy ecology and the right to health are combined with remedial
The reasons for denying a cause of action to an alleged infringement standards as broad ranging as a grave abuse of discretion amounting to
of broad constitutional principles are sourced from basic considerations of lack or excess of jurisdiction, the result will be, it is respectfully submitted,
due process and the lack of judicial authority to wade into the uncharted to propel courts into the uncharted ocean of social and economic policy
ocean of social and economic policy making. Mr. Justice Florentino P. making. At least in respect of the vast area of environmental protection
Feliciano in his concurring opinion in Oposa vs. Factoran, Jr.,[26] explained and management, our courts have no claim to special technical
these reasons as follows: competence and experience and professional qualification. Where no
specific, operable norms and standards are shown to exist, then the policy
My suggestion is simply that petitioners must, before the trial court, show making departments -- the legislative and executive departments -- must
a more specific legal right -- a right cast in language of a significantly be given a real and effective opportunity to fashion and promulgate those
lower order of generality than Article II (15) of the Constitution -- that is norms and standards, and to implement them before the courts should
or may be violated by the actions, or failures to act, imputed to the public intervene.
respondent by petitioners so that the trial court can validly render
judgment granting all or part of the relief prayed for. To my mind, the
court should be understood as simply saying that such a more specific
legal right or rights may well exist in our corpus of law, considering the Economic Nationalism Should Be Read with Other Constitutional
general policy principles found in the Constitution and the existence of the Mandates to Attain Balanced Development of Economy
Philippine Environment Code, and that the trial court should have given
petitioners an effective opportunity so to demonstrate, instead of aborting
the proceedings on a motion to dismiss. On the other hand, Secs. 10 and 12 of Article XII, apart from merely
laying down general principles relating to the national economy and
patrimony, should be read and understood in relation to the other sections
It seems to me important that the legal right which is an essential in said article, especially Secs. 1 and 13 thereof which read:
component of a cause of action be a specific, operable legal right, rather
than a constitutional or statutory policy, for at least two (2) reasons. One
is that unless the legal right claimed to have been violated or disregarded Section 1. The goals of the national economy are a more equitable
is given specification in operational terms, defendants may well be unable distribution of opportunities, income, and wealth; a sustained increase in
to defend themselves intelligently and effectively; in other words, there the amount of goods and services produced by the nation for the benefit of
are due process dimensions to this matter. the people; and an expanding productivity as the key to raising the quality
of life for all, especially the underprivileged.
The State shall promote industrialization and full employment based on or implementing laws or rules for its enforcement. From its very words the
sound agricultural development and agrarian reform, through industries provision does not require any legislation to put it in operation. It is per
that make full and efficient use of human and natural resources, and which se judicially enforceable. However, as the constitutional provision itself
are competitive in both domestic and foreign markets. However, the State states, it is enforceable only in regard to the grants of rights, privileges
shall protect Filipino enterprises against unfair foreign competition and and concessions covering national economy and patrimony and not to
trade practices. every aspect of trade and commerce. It refers to exceptions rather than
the rule. The issue here is not whether this paragraph of Sec. 10 of Art.
In the pursuit of these goals, all sectors of the economy and all regions of XII is self-executing or not. Rather, the issue is whether, as a rule, there
the country shall be given optimum opportunity to develop. x x x are enough balancing provisions in the Constitution to allow the Senate to
ratify the Philippine concurrence in the WTO Agreement. And we hold that
there are.
xxxxxxxxx
All told, while the Constitution indeed mandates a bias in favor of
Sec. 13. The State shall pursue a trade policy that serves the general Filipino goods, services, labor and enterprises, at the same time, it
welfare and utilizes all forms and arrangements of exchange on the basis recognizes the need for business exchange with the rest of the world on
of equality and reciprocity. the bases of equality and reciprocity and limits protection of Filipino
enterprises only against foreign competition and trade practices that are
unfair.[32] In other words, the Constitution did not intend to pursue an
As pointed out by the Solicitor General, Sec. 1 lays down the basic
isolationist policy. It did not shut out foreign investments, goods and
goals of national economic development, as follows:
services in the development of the Philippine economy. While the
1. A more equitable distribution of opportunities, income and wealth; Constitution does not encourage the unlimited entry of foreign goods,
services and investments into the country, it does not prohibit them
2. A sustained increase in the amount of goods and services provided either. In fact, it allows an exchange on the basis of equality and
by the nation for the benefit of the people; and reciprocity, frowning only on foreign competition that is unfair.
3. An expanding productivity as the key to raising the quality of life
for all especially the underprivileged.
WTO Recognizes Need to Protect Weak Economies
With these goals in context, the Constitution then ordains the ideals
of economic nationalism (1) by expressing preference in favor of qualified
Filipinos in the grant of rights, privileges and concessions covering the
Upon the other hand, respondents maintain that the WTO itself has
national economy and patrimony[27] and in the use of Filipino labor,
some built-in advantages to protect weak and developing economies,
domestic materials and locally-produced goods; (2) by mandating the
which comprise the vast majority of its members. Unlike in the UN where
State to adopt measures that help make them competitive; [28] and (3) by
major states have permanent seats and veto powers in the Security
requiring the State to develop a self-reliant and independent national
Council, in the WTO, decisions are made on the basis of sovereign
economy effectively controlled by Filipinos. [29] In similar language, the
equality, with each members vote equal in weight to that of any
Constitution takes into account the realities of the outside world as it
other. There is no WTO equivalent of the UN Security Council.
requires the pursuit of a trade policy that serves the general welfare and
utilizes all forms and arrangements of exchange on the basis of equality
and reciprocity;[30] and speaks of industries which are competitive in both WTO decides by consensus whenever possible, otherwise, decisions of the
domestic and foreign markets as well as of the protection of Filipino Ministerial Conference and the General Council shall be taken by the
enterprises against unfair foreign competition and trade practices. majority of the votes cast, except in cases of interpretation of the
Agreement or waiver of the obligation of a member which would require
It is true that in the recent case of Manila Prince Hotel vs. three fourths vote. Amendments would require two thirds vote in
Government Service Insurance System, et al.,[31] this Court held that Sec. general. Amendments to MFN provisions and the Amendments provision
10, second par., Art. XII of the 1987 Constitution is a mandatory, positive will require assent of all members. Any member may withdraw from the
command which is complete in itself and which needs no further guidelines
Agreement upon the expiration of six months from the date of notice of Specific WTO Provisos Protect Developing Countries
withdrawals.[33]

Hence, poor countries can protect their common interests more So too, the Solicitor General points out that pursuant to and
effectively through the WTO than through one-on-one negotiations with consistent with the foregoing basic principles, the WTO Agreement grants
developed countries. Within the WTO, developing countries can form developing countries a more lenient treatment, giving their domestic
powerful blocs to push their economic agenda more decisively than outside industries some protection from the rush of foreign competition. Thus, with
the Organization. This is not merely a matter of practical alliances but a respect to tariffs in general, preferential treatment is given to developing
negotiating strategy rooted in law. Thus, the basic principles underlying countries in terms of the amount of tariff reduction and the period within
the WTO Agreement recognize the need of developing countries like the which the reduction is to be spread out. Specifically, GATT requires an
Philippines to share in the growth in international trade commensurate average tariff reduction rate of 36% for developed countries to be effected
with the needs of their economic development. These basic principles are within a period of six (6) years while developing countries -- including the
found in the preamble[34] of the WTO Agreement as follows: Philippines -- are required to effect an average tariff reduction of only 24%
within ten (10) years.
The Parties to this Agreement, In respect to domestic subsidy, GATT requires developed countries to
reduce domestic support to agricultural products by 20% over six (6)
Recognizing that their relations in the field of trade and economic years, as compared to only 13% for developing countries to be effected
endeavour should be conducted with a view to raising standards of living, within ten (10) years.
ensuring full employment and a large and steadily growing volume of real
In regard to export subsidy for agricultural products, GATT requires
income and effective demand, and expanding the production of and trade
developed countries to reduce their budgetary outlays for export
in goods and services, while allowing for the optimal use of the worlds
subsidy by 36% and export volumes receiving export subsidy by 21%
resources in accordance with the objective of sustainable development,
within a period of six (6) years. For developing countries, however, the
seeking both to protect and preserve the environment and to enhance the
reduction rate is only two-thirds of that prescribed for developed countries
means for doing so in a manner consistent with their respective needs and
and a longer period of ten (10) years within which to effect such reduction.
concerns at different levels of economic development,
Moreover, GATT itself has provided built-in protection from unfair
Recognizing further that there is need for positive efforts designed to foreign competition and trade practices including anti-dumping measures,
ensure that developing countries, and especially the least developed countervailing measures and safeguards against import surges. Where
among them, secure a share in the growth in international trade local businesses are jeopardized by unfair foreign competition, the
commensurate with the needs of their economic development, Philippines can avail of these measures. There is hardly therefore any
basis for the statement that under the WTO, local industries and
enterprises will all be wiped out and that Filipinos will be deprived of
Being desirous of contributing to these objectives by entering into
control of the economy. Quite the contrary, the weaker situations of
reciprocal and mutually advantageous arrangements directed to the
developing nations like the Philippines have been taken into account; thus,
substantial reduction of tariffs and other barriers to trade and to
there would be no basis to say that in joining the WTO, the respondents
the elimination of discriminatory treatment in international trade relations,
have gravely abused their discretion. True, they have made a bold decision
to steer the ship of state into the yet uncharted sea of economic
Resolved, therefore, to develop an integrated, more viable and durable liberalization. But such decision cannot be set aside on the ground of grave
multilateral trading system encompassing the General Agreement on abuse of discretion, simply because we disagree with it or simply because
Tariffs and Trade, the results of past trade liberalization efforts, and all of we believe only in other economic policies. As earlier stated, the Court in
the results of the Uruguay Round of Multilateral Trade Negotiations, taking jurisdiction of this case will not pass upon the advantages and
disadvantages of trade liberalization as an economic policy. It will only
Determined to preserve the basic principles and to further the objectives perform its constitutional duty of determining whether the Senate
underlying this multilateral trading system, x x x. (underscoring supplied.) committed grave abuse of discretion.
Constitution Does Not Rule Out Foreign Competition Will adherence to the WTO treaty bring this ideal (of favoring the
general welfare) to reality?

Furthermore, the constitutional policy of a self-reliant and Will WTO/GATT succeed in promoting the Filipinos general welfare
independent national economy[35] does not necessarily rule out the entry of because it will -- as promised by its promoters -- expand the countrys
foreign investments, goods and services. It contemplates neither economic exports and generate more employment?
seclusion nor mendicancy in the international community. As explained by Will it bring more prosperity, employment, purchasing power and
Constitutional Commissioner Bernardo Villegas, sponsor of this quality products at the most reasonable rates to the Filipino public?
constitutional policy:
The responses to these questions involve judgment calls by our policy
Economic self-reliance is a primary objective of a developing country that makers, for which they are answerable to our people during appropriate
is keenly aware of overdependence on external assistance for even its electoral exercises. Such questions and the answers thereto are not
most basic needs. It does not mean autarky or economic seclusion; rather, subject to judicial pronouncements based on grave abuse of discretion.
it means avoiding mendicancy in the international
community. Independence refers to the freedom from undue foreign
control of the national economy, especially in such strategic industries as Constitution Designed to Meet Future Events and Contingencies
in the development of natural resources and public utilities. [36]

The WTO reliance on most favored nation, national treatment, and No doubt, the WTO Agreement was not yet in existence when the
trade without discrimination cannot be struck down as unconstitutional as Constitution was drafted and ratified in 1987. That does not mean however
in fact they are rules of equality and reciprocity that apply to all WTO that the Charter is necessarily flawed in the sense that its framers might
members. Aside from envisioning a trade policy based on equality and not have anticipated the advent of a borderless world of business. By the
reciprocity,[37] the fundamental law encourages industries that are same token, the United Nations was not yet in existence when the 1935
competitive in both domestic and foreign markets, thereby demonstrating Constitution became effective.Did that necessarily mean that the then
a clear policy against a sheltered domestic trade environment, but one in Constitution might not have contemplated a diminution of the
favor of the gradual development of robust industries that can compete absoluteness of sovereignty when the Philippines signed the UN Charter,
with the best in the foreign markets. Indeed, Filipino managers and thereby effectively surrendering part of its control over its foreign relations
Filipino enterprises have shown capability and tenacity to compete to the decisions of various UN organs like the Security Council?
internationally. And given a free trade environment, Filipino entrepreneurs
and managers in Hongkong have demonstrated the Filipino capacity to It is not difficult to answer this question. Constitutions are designed to
grow and to prosper against the best offered under a policy of laissez faire. meet not only the vagaries of contemporary events. They should be
interpreted to cover even future and unknown circumstances. It is to the
credit of its drafters that a Constitution can withstand the assaults of
bigots and infidels but at the same time bend with the refreshing winds of
Constitution Favors Consumers, Not Industries or Enterprises change necessitated by unfolding events. As one eminent political law
writer and respected jurist[38] explains:

The Constitution has not really shown any unbalanced bias in favor of The Constitution must be quintessential rather than superficial, the root
any business or enterprise, nor does it contain any specific pronouncement and not the blossom, the base and framework only of the edifice that is
that Filipino companies should be pampered with a total yet to rise. It is but the core of the dream that must take shape, not in a
proscription of foreign competition. On the other hand, respondents claim twinkling by mandate of our delegates, but slowly in the crucible of Filipino
that WTO/GATT aims to make available to the Filipino consumer the best minds and hearts, where it will in time develop its sinews and gradually
goods and services obtainable anywhere in the world at the most gather its strength and finally achieve its substance. In fine, the
reasonable prices. Consequently, the question boils down to whether Constitution cannot, like the goddess Athena, rise full-grown from the
WTO/GATT will favor the general welfare of the public at large. brow of the Constitutional Convention, nor can it conjure by mere fiat an
instant Utopia. It must grow with the society it seeks to re-structure and incorporation, the country is bound by generally accepted principles of
march apace with the progress of the race, drawing from the vicissitudes international law, which are considered to be automatically part of our own
of history the dynamism and vitality that will keep it, far from becoming a laws.[44] One of the oldest and most fundamental rules in international law
petrified rule, a pulsing, living law attuned to the heartbeat of the nation. is pacta sunt servanda -- international agreements must be performed in
good faith. A treaty engagement is not a mere moral obligation but creates
a legally binding obligation on the parties x x x. A state which has
contracted valid international obligations is bound to make in its
Third Issue: The WTO Agreement and Legislative Power legislations such modifications as may be necessary to ensure the
fulfillment of the obligations undertaken.[45]
The WTO Agreement provides that (e)ach Member shall ensure the By their inherent nature, treaties really limit or restrict the
conformity of its laws, regulations and administrative procedures with its absoluteness of sovereignty. By their voluntary act, nations may surrender
obligations as provided in the annexed Agreements. [39] Petitioners maintain some aspects of their state power in exchange for greater benefits granted
that this undertaking unduly limits, restricts and impairs Philippine by or derived from a convention or pact. After all, states, like individuals,
sovereignty, specifically the legislative power which under Sec. 2, Article live with coequals, and in pursuit of mutually covenanted objectives and
VI of the 1987 Philippine Constitution is vested in the Congress of the benefits, they also commonly agree to limit the exercise of their otherwise
Philippines. It is an assault on the sovereign powers of the Philippines absolute rights. Thus, treaties have been used to record agreements
because this means that Congress could not pass legislation that will be between States concerning such widely diverse matters as, for example,
good for our national interest and general welfare if such legislation will the lease of naval bases, the sale or cession of territory, the termination of
not conform with the WTO Agreement, which not only relates to the trade war, the regulation of conduct of hostilities, the formation of alliances, the
in goods x x x but also to the flow of investments and money x x x as well regulation of commercial relations, the settling of claims, the laying down
as to a whole slew of agreements on socio-cultural matters x x x. [40] of rules governing conduct in peace and the establishment of international
organizations.[46] The sovereignty of a state therefore cannot in fact and in
More specifically, petitioners claim that said WTO proviso derogates reality be considered absolute. Certain restrictions enter into the picture:
from the power to tax, which is lodged in the Congress. [41] And while the (1) limitations imposed by the very nature of membership in the family of
Constitution allows Congress to authorize the President to fix tariff rates, nations and (2) limitations imposed by treaty stipulations. As aptly put by
import and export quotas, tonnage and wharfage dues, and other duties or John F. Kennedy, Today, no nation can build its destiny alone. The age of
imposts, such authority is subject to specified limits and x x x such self-sufficient nationalism is over. The age of interdependence is here.[47]
limitations and restrictions as Congress may provide, [42] as in fact it did
under Sec. 401 of the Tariff and Customs Code.

UN Charter and Other Treaties Limit Sovereignty


Sovereignty Limited by International Law and Treaties
Thus, when the Philippines joined the United Nations as one of its 51
charter members, it consented to restrict its sovereign rights under the
This Court notes and appreciates the ferocity and passion by which concept of sovereignty as auto-limitation. 47-A Under Article 2 of the UN
petitioners stressed their arguments on this issue. However, while Charter, (a)ll members shall give the United Nations every assistance in
sovereignty has traditionally been deemed absolute and all-encompassing any action it takes in accordance with the present Charter, and shall
on the domestic level, it is however subject to restrictions and limitations refrain from giving assistance to any state against which the United
voluntarily agreed to by the Philippines, expressly or impliedly, as a Nations is taking preventive or enforcement action. Such assistance
member of the family of nations.Unquestionably, the Constitution did not includes payment of its corresponding share not merely in administrative
envision a hermit-type isolation of the country from the rest of the expenses but also in expenditures for the peace-keeping operations of the
world. In its Declaration of Principles and State Policies, the Constitution organization. In its advisory opinion of July 20, 1961, the International
adopts the generally accepted principles of international law as part of the Court of Justice held that money used by the United Nations Emergency
law of the land, and adheres to the policy of peace, equality, justice, Force in the Middle East and in the Congo were expenses of the United
freedom, cooperation and amity, with all nations." [43] By the doctrine of
Nations under Article 17, paragraph 2, of the UN Charter. Hence, all its (e) Bilateral air transport agreement with Korea where the Philippines
members must bear their corresponding share in such expenses. In this agreed to exempt from all customs duties, inspection fees and
sense, the Philippine Congress is restricted in its power to appropriate. It other duties or taxes aircrafts of South Korea and the regular
is compelled to appropriate funds whether it agrees with such peace- equipment, spare parts and supplies arriving with said aircrafts.
keeping expenses or not. So too, under Article 105 of the said Charter, the
UN and its representatives enjoy diplomatic privileges and immunities, (f) Bilateral air service agreement with Japan, where the Philippines
thereby limiting again the exercise of sovereignty of members within their agreed to exempt from customs duties, excise taxes, inspection
own territory.Another example: although sovereign equality and domestic fees and other similar duties, taxes or charges fuel, lubricating
jurisdiction of all members are set forth as underlying principles in the UN oils, spare parts, regular equipment, stores on board Japanese
Charter, such provisos are however subject to enforcement measures aircrafts while on Philippine soil.
decided by the Security Council for the maintenance of international peace
and security under Chapter VII of the Charter. A final example: under
(g) Bilateral air service agreement with Belgium where the Philippines
Article 103, (i)n the event of a conflict between the obligations of the
granted Belgian air carriers the same privileges as those granted
Members of the United Nations under the present Charter and their
to Japanese and Korean air carriers under separate air service
obligations under any other international agreement, their obligation under
agreements.
the present charter shall prevail, thus unquestionably denying the
Philippines -- as a member -- the sovereign power to make a choice as to
which of conflicting obligations, if any, to honor. (h) Bilateral notes with Israel for the abolition of transit and visitor
visas where the Philippines exempted Israeli nationals from the
Apart from the UN Treaty, the Philippines has entered into many other requirement of obtaining transit or visitor visas for a sojourn in
international pacts -- both bilateral and multilateral -- that involve the Philippines not exceeding 59 days.
limitations on Philippine sovereignty. These are enumerated by the
Solicitor General in his Compliance dated October 24, 1996, as follows:
(I) Bilateral agreement with France exempting French nationals from
the requirement of obtaining transit and visitor visa for a sojourn
(a) Bilateral convention with the United States regarding taxes on not exceeding 59 days.
income, where the Philippines agreed, among others, to exempt
from tax, income received in the Philippines by, among others,
(j) Multilateral Convention on Special Missions, where the Philippines
the Federal Reserve Bank of the United States, the
agreed that premises of Special Missions in the Philippines are
Export/Import Bank of the United States, the Overseas Private
inviolable and its agents can not enter said premises without
Investment Corporation of the United States. Likewise, in said
consent of the Head of Mission concerned. Special Missions are
convention, wages, salaries and similar remunerations paid by
also exempted from customs duties, taxes and related charges.
the United States to its citizens for labor and personal services
performed by them as employees or officials of the United States
are exempt from income tax by the Philippines. (k) Multilateral Convention on the Law of Treaties. In this convention,
the Philippines agreed to be governed by the Vienna Convention
on the Law of Treaties.
(b) Bilateral agreement with Belgium, providing, among others, for
the avoidance of double taxation with respect to taxes on
income. (l) Declaration of the President of the Philippines accepting
compulsory jurisdiction of the International Court of Justice. The
International Court of Justice has jurisdiction in all legal disputes
(c) Bilateral convention with the Kingdom of Sweden for the
concerning the interpretation of a treaty, any question of
avoidance of double taxation.
international law, the existence of any fact which, if established,
would constitute a breach of international obligation.
(d) Bilateral convention with the French Republic for the avoidance of
double taxation.
In the foregoing treaties, the Philippines has effectively agreed to limit Process Patents: Burden of Proof
the exercise of its sovereign powers of taxation, eminent domain and
police power. The underlying consideration in this partial surrender of 1. For the purposes of civil proceedings in respect of the infringement
sovereignty is the reciprocal commitment of the other contracting states in of the rights of the owner referred to in paragraph 1(b) of Article 28,
granting the same privilege and immunities to the Philippines, its officials if the subject matter of a patent is a process for obtaining a product,
and its citizens. The same reciprocity characterizes the Philippine the judicial authorities shall have the authority to order the defendant
commitments under WTO-GATT. to prove that the process to obtain an identical product is different
from the patented process. Therefore, Members shall provide, in at
International treaties, whether relating to nuclear disarmament, human least one of the following circumstances, that any identical product
rights, the environment, the law of the sea, or trade, constrain domestic when produced without the consent of the patent owner shall, in the
political sovereignty through the assumption of external obligations. But absence of proof to the contrary, be deemed to have been obtained
unless anarchy in international relations is preferred as an alternative, in by the patented process:
most cases we accept that the benefits of the reciprocal obligations
involved outweigh the costs associated with any loss of political (a) if the product obtained by the patented process is new;
sovereignty.(T)rade treaties that structure relations by reference to
durable, well-defined substantive norms and objective dispute resolution
(b) if there is a substantial likelihood that the identical product
procedures reduce the risks of larger countries exploiting raw economic
was made by the process and the owner of the patent has
power to bully smaller countries, by subjecting power relations to some
been unable through reasonable efforts to determine the
form of legal ordering. In addition, smaller countries typically stand to gain
process actually used.
disproportionately from trade liberalization. This is due to the simple fact
that liberalization will provide access to a larger set of potential new
trading relationship than in case of the larger country gaining enhanced 2. Any Member shall be free to provide that the burden of proof
success to the smaller countrys market.[48] indicated in paragraph 1 shall be on the alleged infringer only if the
condition referred to in subparagraph (a) is fulfilled or only if the
condition referred to in subparagraph (b) is fulfilled.
The point is that, as shown by the foregoing treaties, a portion of
sovereignty may be waived without violating the Constitution, based on
the rationale that the Philippines adopts the generally accepted principles 3. In the adduction of proof to the contrary, the legitimate interests of
of international law as part of the law of the land and adheres to the policy defendants in protecting their manufacturing and business secrets
of x x x cooperation and amity with all nations. shall be taken into account.

From the above, a WTO Member is required to provide a rule of


disputable (note the words in the absence of proof to the contrary)
Fourth Issue: The WTO Agreement and Judicial Power
presumption that a product shown to be identical to one produced with the
use of a patented process shall be deemed to have been obtained by the
(illegal) use of the said patented process, (1) where such product obtained
Petitioners aver that paragraph 1, Article 34 of the General Provisions
by the patented product is new, or (2) where there is substantial likelihood
and Basic Principles of the Agreement on Trade-Related Aspects of
that the identical product was made with the use of the said patented
Intellectual Property Rights (TRIPS) [49] intrudes on the power of the
process but the owner of the patent could not determine the exact process
Supreme Court to promulgate rules concerning pleading, practice and
used in obtaining such identical product. Hence, the burden of proof
procedures.[50]
contemplated by Article 34 should actually be understood as the duty of
To understand the scope and meaning of Article 34, TRIPS, [51] it will the alleged patent infringer to overthrow such presumption. Such burden,
be fruitful to restate its full text as follows: properly understood, actually refers to the burden of evidence (burden of
going forward) placed on the producer of the identical (or fake) product to
show that his product was produced without the use of the patented
Article 34
process.
The foregoing notwithstanding, the patent owner still has the burden Fifth Issue: Concurrence Only in the WTO Agreement and Not in
of proof since, regardless of the presumption provided under paragraph 1 Other Documents Contained in the Final Act
of Article 34, such owner still has to introduce evidence of the existence of
the alleged identical product, the fact that it is identical to the genuine one
produced by the patented process and the fact of newness of the genuine Petitioners allege that the Senate concurrence in the WTO Agreement
product or the fact of substantial likelihood that the identical product was and its annexes -- but not in the other documents referred to in the Final
made by the patented process. Act, namely the Ministerial Declaration and Decisions and the
Understanding on Commitments in Financial Services -- is defective and
The foregoing should really present no problem in changing the rules insufficient and thus constitutes abuse of discretion. They submit that such
of evidence as the present law on the subject, Republic Act No. 165, as concurrence in the WTO Agreement alone is flawed because it is in effect a
amended, otherwise known as the Patent Law, provides a similar rejection of the Final Act, which in turn was the document signed by
presumption in cases of infringement of patented design or utility model, Secretary Navarro, in representation of the Republic upon authority of the
thus: President. They contend that the second letter of the President to the
Senate[53] which enumerated what constitutes the Final Act should have
SEC. 60. Infringement. - Infringement of a design patent or of a patent been the subject of concurrence of the Senate.
for utility model shall consist in unauthorized copying of the patented
design or utility model for the purpose of trade or industry in the article or A final act, sometimes called protocol de clture, is an instrument
product and in the making, using or selling of the article or product which records the winding up of the proceedings of a diplomatic
copying the patented design or utility model. Identity or substantial conference and usually includes a reproduction of the texts of treaties,
identity with the patented design or utility model shall constitute evidence conventions, recommendations and other acts agreed upon and signed by
of copying. (underscoring supplied) the plenipotentiaries attending the conference. [54] It is not the treaty
itself. It is rather a summary of the proceedings of a protracted conference
which may have taken place over several years. The text of the Final Act
Moreover, it should be noted that the requirement of Article 34 to
Embodying the Results of the Uruguay Round of Multilateral Trade
provide a disputable presumption applies only if (1) the product obtained Negotiations is contained in just one page [55] in Vol. I of the 36-
by the patented process is NEW or (2) there is a substantial likelihood that
volume Uruguay Round of Multilateral Trade Negotiations. By signing said
the identical product was made by the process and the process owner has Final Act, Secretary Navarro as representative of the Republic of the
not been able through reasonable effort to determine the process
Philippines undertook:
used. Where either of these two provisos does not obtain, members shall
be free to determine the appropriate method of implementing the
provisions of TRIPS within their own internal systems and processes. "(a) to submit, as appropriate, the WTO Agreement for the
consideration of their respective competent authorities with a
By and large, the arguments adduced in connection with our view to seeking approval of the Agreement in accordance with
disposition of the third issue -- derogation of legislative power - will apply their procedures; and
to this fourth issue also. Suffice it to say that the reciprocity clause more
than justifies such intrusion, if any actually exists. Besides, Article 34 does (b) to adopt the Ministerial Declarations and Decisions."
not contain an unreasonable burden, consistent as it is with due process
and the concept of adversarial dispute settlement inherent in our judicial
system. The assailed Senate Resolution No. 97 expressed concurrence in
exactly what the Final Act required from its signatories, namely,
So too, since the Philippine is a signatory to most international concurrence of the Senate in the WTO Agreement.
conventions on patents, trademarks and copyrights, the adjustment in
The Ministerial Declarations and Decisions were deemed adopted
legislation and rules of procedure will not be substantial. [52]
without need for ratification. They were approved by the ministers by
virtue of Article XXV: 1 of GATT which provides that representatives of the
members can meet to give effect to those provisions of this Agreement
which invoke joint action, and generally with a view to facilitating the It should be added that the Senate was well-aware of what it was
operation and furthering the objectives of this Agreement. [56] concurring in as shown by the members deliberation on August 25,
1994. After reading the letter of President Ramos dated August 11, 1994,
The Understanding on Commitments in Financial Services also [59]
the senators of the Republic minutely dissected what the Senate was
approved in Marrakesh does not apply to the Philippines. It applies only to concurring in, as follows: [60]
those 27 Members which have indicated in their respective schedules of
commitments on standstill, elimination of monopoly, expansion of
operation of existing financial service suppliers, temporary entry of THE CHAIRMAN: Yes. Now, the question of the validity of the submission
personnel, free transfer and processing of information, and national came up in the first day hearing of this Committee yesterday. Was the
treatment with respect to access to payment, clearing systems and observation made by Senator Taada that what was submitted to the
refinancing available in the normal course of business. [57] Senate was not the agreement on establishing the World Trade
Organization by the final act of the Uruguay Round which is not the same
On the other hand, the WTO Agreement itself expresses what as the agreement establishing the World Trade Organization? And on that
multilateral agreements are deemed included as its integral parts, [58] as basis, Senator Tolentino raised a point of order which, however, he agreed
follows: to withdraw upon understanding that his suggestion for an alternative
solution at that time was acceptable. That suggestion was to treat the
Article II proceedings of the Committee as being in the nature of briefings for
Senators until the question of the submission could be clarified.
Scope of the WTO
And so, Secretary Romulo, in effect, is the President submitting a new... is
he making a new submission which improves on the clarity of the first
1. The WTO shall provide the common institutional framework for the submission?
conduct of trade relations among its Members in matters to the
agreements and associated legal instruments included in the Annexes
to this Agreement. MR. ROMULO: Mr. Chairman, to make sure that it is clear cut and there
should be no misunderstanding, it was his intention to clarify all matters
by giving this letter.
2. The Agreements and associated legal instruments included in
Annexes 1, 2, and 3 (hereinafter referred to as Multilateral
Agreements) are integral parts of this Agreement, binding on all THE CHAIRMAN: Thank you.
Members.
Can this Committee hear from Senator Taada and later on Senator
3. The Agreements and associated legal instruments included in Tolentino since they were the ones that raised this question yesterday?
Annex 4 (hereinafter referred to as Plurilateral Trade Agreements) are
also part of this Agreement for those Members that have accepted Senator Taada, please.
them, and are binding on those Members. The Plurilateral Trade
Agreements do not create either obligation or rights for Members that SEN. TAADA: Thank you, Mr. Chairman.
have not accepted them.
Based on what Secretary Romulo has read, it would now clearly appear
4. The General Agreement on Tariffs and Trade 1994 as specified in that what is being submitted to the Senate for ratification is not the Final
annex 1A (hereinafter referred to as GATT 1994) is legally distinct Act of the Uruguay Round, but rather the Agreement on the World Trade
from the General Agreement on Tariffs and Trade, dated 30 October Organization as well as the Ministerial Declarations and Decisions, and the
1947, annexed to the Final Act adopted at the conclusion of the Understanding and Commitments in Financial Services.
Second Session of the Preparatory Committee of the United Nations
Conference on Trade and Employment, as subsequently rectified,
I am now satisfied with the wording of the new submission of President
amended or modified (hereinafter referred to as GATT 1947).
Ramos.
SEN. TAADA. . . . of President Ramos, Mr. Chairman. submission is, I believe, stating the obvious and therefore I have no
further comment to make.
THE CHAIRMAN. Thank you, Senator Taada. Can we hear from Senator
Tolentino? And after him Senator Neptali Gonzales and Senator Lina.
Epilogue
SEN TOLENTINO, Mr. Chairman, I have not seen the new submission
actually transmitted to us but I saw the draft of his earlier, and I think it
now complies with the provisions of the Constitution, and with the Final In praying for the nullification of the Philippine ratification of the WTO
Act itself. The Constitution does not require us to ratify the Final Act. It Agreement, petitioners are invoking this Courts constitutionally imposed
requires us to ratify the Agreement which is now being submitted. The duty to determine whether or not there has been grave abuse of discretion
Final Act itself specifies what is going to be submitted to with the amounting to lack or excess of jurisdiction on the part of the Senate in
governments of the participants. giving its concurrence therein via Senate Resolution No. 97. Procedurally,
a writ of certiorari grounded on grave abuse of discretion may be issued
In paragraph 2 of the Final Act, we read and I quote: by the Court under Rule 65 of the Rules of Court when it is amply shown
that petitioners have no other plain, speedy and adequate remedy in the
ordinary course of law.
By signing the present Final Act, the representatives agree: (a) to submit
as appropriate the WTO Agreement for the consideration of the respective By grave abuse of discretion is meant such capricious and whimsical
competent authorities with a view to seeking approval of the Agreement in exercise of judgment as is equivalent to lack of jurisdiction. [61] Mere abuse
accordance with their procedures. of discretion is not enough. It must be grave abuse of discretion as when
the power is exercised in an arbitrary or despotic manner by reason of
In other words, it is not the Final Act that was agreed to be submitted to passion or personal hostility, and must be so patent and so gross as to
the governments for ratification or acceptance as whatever their amount to an evasion of a positive duty or to a virtual refusal to perform
constitutional procedures may provide but it is the World Trade the duty enjoined or to act at all in contemplation of law. [62] Failure on the
Organization Agreement. And if that is the one that is being submitted part of the petitioner to show grave abuse of discretion will result in the
now, I think it satisfies both the Constitution and the Final Act itself. dismissal of the petition.[63]

In rendering this Decision, this Court never forgets that the Senate,
Thank you, Mr. Chairman. whose act is under review, is one of two sovereign houses of Congress and
is thus entitled to great respect in its actions. It is itself a constitutional
THE CHAIRMAN. Thank you, Senator Tolentino, May I call on Senator body independent and coordinate, and thus its actions are presumed
Gonzales. regular and done in good faith. Unless convincing proof and persuasive
arguments are presented to overthrow such presumptions, this Court will
SEN. GONZALES. Mr. Chairman, my views on this matter are already a resolve every doubt in its favor. Using the foregoing well-accepted
matter of record. And they had been adequately reflected in the journal of definition of grave abuse of discretion and the presumption of regularity in
yesterdays session and I dont see any need for repeating the same. the Senates processes, this Court cannot find any cogent reason to impute
grave abuse of discretion to the Senates exercise of its power of
concurrence in the WTO Agreement granted it by Sec. 21 of Article VII of
Now, I would consider the new submission as an act ex abudante cautela. the Constitution.[64]

THE CHAIRMAN. Thank you, Senator Gonzales. Senator Lina, do you want It is true, as alleged by petitioners, that broad constitutional principles
to make any comment on this? require the State to develop an independent national economy effectively
controlled by Filipinos; and to protect and/or prefer Filipino labor,
products, domestic materials and locally produced goods. But it is equally
SEN. LINA. Mr. President, I agree with the observation just made by
true that such principles -- while serving as judicial and legislative guides
Senator Gonzales out of the abundance of question. Then the new
-- are not in themselves sources of causes of action. Moreover, there are
other equally fundamental constitutional principles relied upon by the the people, through their duly authorized elected officers, make their free
Senate which mandate the pursuit of a trade policy that serves the general choice.
welfare and utilizes all forms and arrangements of exchange on the basis
of equality and reciprocity and the promotion of industries which are WHEREFORE, the petition is DISMISSED for lack of merit.
competitive in both domestic and foreign markets, thereby justifying its SO ORDERED.
acceptance of said treaty. So too, the alleged impairment of sovereignty in
the exercise of legislative and judicial powers is balanced by the adoption Narvasa, C.J., Regalado, Davide, Jr., Romero, Bellosillo, Melo, Puno,
of the generally accepted principles of international law as part of the law Kapunan, Mendoza, Francisco, Hermosisima, Jr., and Torres, Jr.,
of the land and the adherence of the Constitution to the policy of JJ., concur.
cooperation and amity with all nations. Padilla, and Vitug, JJ., in the result.

That the Senate, after deliberation and voting, voluntarily and


overwhelmingly gave its consent to the WTO Agreement thereby making it
a part of the law of the land is a legitimate exercise of its sovereign duty
and power. We find no patent and gross arbitrariness or despotism by
reason of passion or personal hostility in such exercise. It is not impossible
to surmise that this Court, or at least some of its members, may even
agree with petitioners that it is more advantageous to the national interest
to strike down Senate Resolution No. 97. But that is not a legal
reason to attribute grave abuse of discretion to the Senate and to nullify
its decision. To do so would constitute grave abuse in the exercise of our
own judicial power and duty. Ineludably, what the Senate did was a valid
exercise of its authority. As to whether such exercise was wise, beneficial
or viable is outside the realm of judicial inquiry and review. That is a
matter between the elected policy makers and the people. As to whether
the nation should join the worldwide march toward trade liberalization and
economic globalization is a matter that our people should determine in
electing their policy makers.After all, the WTO Agreement allows
withdrawal of membership, should this be the political desire of a member.

The eminent futurist John Naisbitt, author of the best


seller Megatrends, predicts an Asian Renaissance[65] where the East will
become the dominant region of the world economically, politically and
culturally in the next century. He refers to the free market espoused by
WTO as the catalyst in this coming Asian ascendancy. There are at present
about 31 countries including China, Russia and Saudi Arabia negotiating
for membership in the WTO. Notwithstanding objections against possible
limitations on national sovereignty, the WTO remains as the only viable
structure for multilateral trading and the veritable forum for the
development of international trade law. The alternative to WTO is isolation,
stagnation, if not economic self-destruction. Duly enriched with original
membership, keenly aware of the advantages and disadvantages of
globalization with its on-line experience, and endowed with a vision of the
future, the Philippines now straddles the crossroads of an international
strategy for economic prosperity and stability in the new millennium. Let
the power of the sovereign people to choose their leaders. The COMELEC
EN BANC supposedly erred in disqualifying him since he is the most qualified among
all the presidential candidates, i.e., he possesses all the constitutional and
G.R. No. 161872 April 13, 2004 legal qualifications for the office of the president, he is capable of waging a
national campaign since he has numerous national organizations under his
leadership, he also has the capacity to wage an international campaign
REV. ELLY CHAVEZ PAMATONG, ESQUIRE, petitioner,
since he has practiced law in other countries, and he has a platform of
vs.
government. Petitioner likewise attacks the validity of the form for
COMMISSION ON ELECTIONS, respondent.
the Certificate of Candidacy prepared by the COMELEC. Petitioner claims
that the form does not provide clear and reasonable guidelines for
RESOLUTION determining the qualifications of candidates since it does not ask for the
candidate’s bio-data and his program of government.
TINGA, J.:
First, the constitutional and legal dimensions involved.
Petitioner Rev. Elly Velez Pamatong filed his Certificate of Candidacy for
President on December 17, 2003. Respondent Commission on Elections Implicit in the petitioner’s invocation of the constitutional provision
(COMELEC) refused to give due course to petitioner’s Certificate of ensuring "equal access to opportunities for public office" is the claim that
Candidacy in its Resolution No. 6558 dated January 17, 2004. The there is a constitutional right to run for or hold public office and,
decision, however, was not unanimous since Commissioners Luzviminda G. particularly in his case, to seek the presidency. There is none. What is
Tancangco and Mehol K. Sadain voted to include petitioner as they recognized is merely a privilege subject to limitations imposed by law.
believed he had parties or movements to back up his candidacy. Section 26, Article II of the Constitution neither bestows such a right nor
elevates the privilege to the level of an enforceable right. There is nothing
On January 15, 2004, petitioner moved for reconsideration of Resolution in the plain language of the provision which suggests such a thrust or
No. 6558. Petitioner’s Motion for Reconsideration was docketed as SPP justifies an interpretation of the sort.
(MP) No. 04-001. The COMELEC, acting on petitioner’s Motion for
Reconsideration and on similar motions filed by other aspirants for national The "equal access" provision is a subsumed part of Article II of the
elective positions, denied the same under the aegis of Omnibus Resolution Constitution, entitled "Declaration of Principles and State Policies." The
No. 6604 dated February 11, 2004. The COMELEC declared petitioner and provisions under the Article are generally considered not self-
thirty-five (35) others nuisance candidates who could not wage a executing,2 and there is no plausible reason for according a different
nationwide campaign and/or are not nominated by a political party or are treatment to the "equal access" provision. Like the rest of the policies
not supported by a registered political party with a national constituency. enumerated in Article II, the provision does not contain any judicially
Commissioner Sadain maintained his vote for petitioner. By then, enforceable constitutional right but merely specifies a guideline for
Commissioner Tancangco had retired. legislative or executive action.3 The disregard of the provision does not
give rise to any cause of action before the courts. 4
In this Petition For Writ of Certiorari, petitioner seeks to reverse the
resolutions which were allegedly rendered in violation of his right to "equal An inquiry into the intent of the framers5 produces the same determination
access to opportunities for public service" under Section 26, Article II of that the provision is not self-executory. The original wording of the present
the 1987 Section 26, Article II had read, "The State shall broaden opportunities to
public office and prohibit public dynasties."6 Commissioner (now Chief
Constitution,1 by limiting the number of qualified candidates only to those Justice) Hilario Davide, Jr. successfully brought forth an amendment that
who can afford to wage a nationwide campaign and/or are nominated by changed the word "broaden" to the phrase "ensure equal access," and the
political parties. In so doing, petitioner argues that the COMELEC indirectly substitution of the word "office" to "service." He explained his proposal in
amended the constitutional provisions on the electoral process and limited this wise:
I changed the word "broaden" to "ENSURE EQUAL ACCESS TO" Significantly, petitioner does not challenge the constitutionality or validity
because what is important would be equal access to the of Section 69 of the Omnibus Election Code and COMELEC Resolution No.
opportunity. If you broaden, it would necessarily mean that 6452 dated 10 December 2003. Thus, their presumed validity stands and
the government would be mandated to create as many has to be accorded due weight.
offices as are possible to accommodate as many people as
are also possible. That is the meaning of broadening Clearly, therefore, petitioner’s reliance on the equal access clause in
opportunities to public service. So, in order that we should not Section 26, Article II of the Constitution is misplaced.
mandate the State to make the government the number one
employer and to limit offices only to what may be necessary
The rationale behind the prohibition against nuisance candidates and the
and expedient yet offering equal opportunities to access to
disqualification of candidates who have not evinced a bona fide intention to
it, I change the word "broaden."7 (emphasis supplied)
run for office is easy to divine. The State has a compelling interest to
ensure that its electoral exercises are rational, objective, and orderly.
Obviously, the provision is not intended to compel the State to enact Towards this end, the State takes into account the practical considerations
positive measures that would accommodate as many people as possible in conducting elections. Inevitably, the greater the number of candidates,
into public office. The approval of the "Davide amendment" indicates the the greater the opportunities for logistical confusion, not to mention the
design of the framers to cast the provision as simply enunciatory of a increased allocation of time and resources in preparation for the election.
desired policy objective and not reflective of the imposition of a clear State These practical difficulties should, of course, never exempt the State from
burden. the conduct of a mandated electoral exercise. At the same time, remedial
actions should be available to alleviate these logistical hardships,
Moreover, the provision as written leaves much to be desired if it is to be whenever necessary and proper. Ultimately, a disorderly election is not
regarded as the source of positive rights. It is difficult to interpret the merely a textbook example of inefficiency, but a rot that erodes faith in
clause as operative in the absence of legislation since its effective means our democratic institutions. As the United States Supreme Court held:
and reach are not properly defined. Broadly written, the myriad of claims
that can be subsumed under this rubric appear to be entirely open- [T]here is surely an important state interest in requiring some
ended.8 Words and phrases such as "equal access," "opportunities," and preliminary showing of a significant modicum of support before
"public service" are susceptible to countless interpretations owing to their printing the name of a political organization and its candidates on
inherent impreciseness. Certainly, it was not the intention of the framers the ballot – the interest, if no other, in avoiding confusion,
to inflict on the people an operative but amorphous foundation from which deception and even frustration of the democratic [process]. 11
innately unenforceable rights may be sourced.
The COMELEC itself recognized these practical considerations when it
As earlier noted, the privilege of equal access to opportunities to public promulgated Resolution No. 6558 on 17 January 2004, adopting the study
office may be subjected to limitations. Some valid limitations specifically Memorandum of its Law Department dated 11 January 2004. As observed
on the privilege to seek elective office are found in the provisions 9 of the in the COMELEC’s Comment:
Omnibus Election Code on "Nuisance Candidates" and COMELEC Resolution
No. 645210 dated December 10, 2002 outlining the instances wherein the
There is a need to limit the number of candidates especially in the
COMELEC may motu proprio refuse to give due course to or cancel
case of candidates for national positions because the election
a Certificate of Candidacy.
process becomes a mockery even if those who cannot clearly wage
a national campaign are allowed to run. Their names would have
As long as the limitations apply to everybody equally without to be printed in the Certified List of Candidates, Voters Information
discrimination, however, the equal access clause is not violated. Equality is Sheet and the Official Ballots. These would entail additional costs
not sacrificed as long as the burdens engendered by the limitations are to the government. For the official ballots in automated counting
meant to be borne by any one who is minded to file a certificate of and canvassing of votes, an additional page would amount to more
candidacy. In the case at bar, there is no showing that any person is or less FOUR HUNDRED FIFTY MILLION PESOS (₱450,000,000.00).
exempt from the limitations or the burdens which they create.
xxx[I]t serves no practical purpose to allow those candidates to reviewed by this Court on the basis of what is now before it. The assailed
continue if they cannot wage a decent campaign enough to project resolutions of the COMELEC do not direct the Court to the evidence which
the prospect of winning, no matter how slim. 12 it considered in determining that petitioner was a nuisance candidate. This
precludes the Court from reviewing at this instance whether the COMELEC
The preparation of ballots is but one aspect that would be affected by committed grave abuse of discretion in disqualifying petitioner, since such
allowance of "nuisance candidates" to run in the elections. Our election a review would necessarily take into account the matters which the
laws provide various entitlements for candidates for public office, such as COMELEC considered in arriving at its decisions.
watchers in every polling place,13 watchers in the board of canvassers,14 or
even the receipt of electoral contributions. 15Moreover, there are election Petitioner has submitted to this Court mere photocopies of various
rules and regulations the formulations of which are dependent on the documents purportedly evincing his credentials as an eligible candidate for
number of candidates in a given election. the presidency. Yet this Court, not being a trier of facts, can not properly
pass upon the reproductions as evidence at this level. Neither the
Given these considerations, the ignominious nature of a nuisance COMELEC nor the Solicitor General appended any document to their
candidacy becomes even more galling. The organization of an election respective Comments.
with bona fide candidates standing is onerous enough. To add into the mix
candidates with no serious intentions or capabilities to run a viable The question of whether a candidate is a nuisance candidate or not is both
campaign would actually impair the electoral process. This is not to legal and factual. The basis of the factual determination is not before this
mention the candidacies which are palpably ridiculous so as to constitute a Court. Thus, the remand of this case for the reception of further evidence
one-note joke. The poll body would be bogged by irrelevant minutiae is in order.
covering every step of the electoral process, most probably posed at the
instance of these nuisance candidates. It would be a senseless sacrifice on A word of caution is in order. What is at stake is petitioner’s aspiration and
the part of the State. offer to serve in the government. It deserves not a cursory treatment but
a hearing which conforms to the requirements of due process.
Owing to the superior interest in ensuring a credible and orderly election,
the State could exclude nuisance candidates and need not indulge in, as As to petitioner’s attacks on the validity of the form for the certificate of
the song goes, "their trips to the moon on gossamer wings." candidacy, suffice it to say that the form strictly complies with Section 74
of the Omnibus Election Code. This provision specifically enumerates what
The Omnibus Election Code and COMELEC Resolution No. 6452 are a certificate of candidacy should contain, with the required information
cognizant of the compelling State interest to ensure orderly and credible tending to show that the candidate possesses the minimum qualifications
elections by excising impediments thereto, such as nuisance candidacies for the position aspired for as established by the Constitution and other
that distract and detract from the larger purpose. The COMELEC is election laws.
mandated by the Constitution with the administration of elections 16 and
endowed with considerable latitude in adopting means and methods that IN VIEW OF THE FOREGOING, COMELEC Case No. SPP (MP) No. 04-001 is
will ensure the promotion of free, orderly and honest elections. 17 Moreover, hereby remanded to the COMELEC for the reception of further evidence, to
the Constitution guarantees that only bona fide candidates for public office determine the question on whether petitioner Elly Velez Lao Pamatong is a
shall be free from any form of harassment and discrimination. 18 The nuisance candidate as contemplated in Section 69 of the Omnibus Election
determination of bona fidecandidates is governed by the statutes, and the Code.
concept, to our mind is, satisfactorily defined in the Omnibus Election
Code.
The COMELEC is directed to hold and complete the reception of evidence
and report its findings to this Court with deliberate dispatch.
Now, the needed factual premises.
SO ORDERED.
However valid the law and the COMELEC issuance involved are, their
proper application in the case of the petitioner cannot be tested and
Davide, Jr., Puno, Vitug*, Panganiban, Quisumbing, Ynares-Santiago,
Sandoval-Gutierrez, Carpio, Austria-Martinez, Corona, Carpio-Morales,
Callejo, Sr., and Azcuna, JJ., concur.
Republic of the Philippines Regional Office No. VII, Cebu City of the P10,000.00 Food Basket
Supreme Court Allowance granted by BFAR to each of its employees in 1999, and COA
Manila Resolution[2] dated August 5, 2005, denying petitioners motion for
reconsideration of said Decision.

EN BANC First, the facts:


On October 18, 1999, petitioner Bureau of Fisheries and Aquatic
Resources (BFAR) Employees Union, Regional Office No.
BUREAU OF FISHERIES AND G.R. No. 169815 VII, Cebu City issued Resolution No. 01, series of 1999 requesting the
AQUATIC RESOURCES (BFAR) BFAR Central Office for a Food Basket Allowance. It justified its request on
EMPLOYEES UNION, REGIONAL Present: the high cost of living, i.e., the increase in prices of petroleum products
OFFICE NO. VII, CEBU CITY, which catapulted the cost of food commodities, has greatly affected the
Petitioner, Puno, C.J.,
economic conditions and living standard of the government employees of
Quisumbing,
BFAR Region VII and could hardly sustain its need to cope up with the four
Ynares-Santiago,
(4) basic needs, i.e., food, shelter, clothing and education. [3] It also relied
Carpio,
on the Employees Suggestions and Incentive Awards System (ESIAS),
Austria-Martinez,
pursuant to Book V of Executive Order No. 292, or the Administrative Code
Corona,
of 1987, and approved by the Civil Service Commission on December 3,
Carpio Morales,
1996. The ESIAS includes the granting of incentives that will help
Azcuna,
employees overcome present economic difficulties, boost their morale, and
- versus - Tinga,
further commitment and dedication to public service. [4] Regional Director
Chico-Nazario,
Corazon M. Corrales of BFAR Region VII indorsed the Resolution, and
Velasco, Jr.,
Malcolm I. Sarmiento, Jr., Director of BFAR recommended its
Nachura,*
approval. Honorable Cesar M. Drilon, Jr., Undersecretary for Fisheries and
Reyes,
Livestock of the Department of Agriculture, approved the request for
Leonardo-De Castro,
Authority to Grant a Gift Check or the Food Basket Allowance at the rate
Brion, JJ.
of P10,000.00 each to the 130 employees of BFAR Region VII, or in the
total amount of P1,322,682.00.[5] On the strength of the approval,
COMMISSION ON AUDIT, Promulgated:
Regional Director Corrales released the allowance to the BFAR employees.
Respondent.
August 13, 2008
On post audit, the Commission on Audit Legal and Adjudication
x- - - - - - - - - - - - - - - - - - - - - - - - - - - - - - - - - - - - - - - - - - - - - - Office (COA-LAO) Regional Office No. VII, Cebu City disallowed the grant
--------------x of Food Basket Allowance under Notice of Disallowance No. 2003-022-101
(1999) dated September 19, 2003. It ruled that the allowance had no legal
basis and that it violated: a) Sec. 15(d) of the General Appropriations Act
DECISION of 1999, prohibiting the payment of honoraria, allowances, or other forms
of compensation to any government official or employee, except those
specifically authorized by law; b) par. 4.5 of Budget Circular No. 16 dated
PUNO, C.J.: November 28, 1998, prohibiting the grant of food, rice, gift checks, or any
other form of incentives/allowances, except those authorized via
Administrative Order by the Office of the President; and c) Sec. 12 of
Republic Act (R.A.) No. 6758, or the Salary Standardization Law of 1989,
On appeal are the Decision[1] dated April 8, 2005 of respondent
which includes all allowances in the standardized salary rates, subject to
Commission on Audit (COA) in LAO-N-2005-119 upholding the
certain exceptions.
disallowance by the COA Legal and Adjudication Office (COA-LAO),
On February 26, 2004, BFAR Regional Office No. VII, through We deny the petition.
Regional Director Corrales, moved for reconsideration and prayed for the
lifting of the disallowance. It argued that the grant of Food Basket First, we rule on the issue of constitutionality. Petitioner invokes
Allowance would enhance the welfare and productivity of the the provisions of the 1987 Constitution on social justice to warrant the
employees. Further, it contended that the approval by the Honorable grant of the Food Basket Allowance. Time and again, we have ruled that
Drilon, Undersecretary for Fisheries and Livestock, of the said benefit was the social justice provisions of the Constitution are not self-
the law itself which vested the specific authority for its executing principles ready for enforcement through the courts. They are
release. The Commission on Audit Legal and Adjudication Office (COA- merely statements of principles and policies. To give them effect,
LAO) Regional Office No. VII, Cebu City denied the motion. legislative enactment is required. As we held in Kilosbayan,
Incorporated v. Morato,[9] the principles and state policies enumerated
Petitioner appealed to the Commission on Audit Legal and in Article II and some sections of Article XII are "not self-executing
Adjudication Office (COA-LAO) National, Quezon City. The appeal was provisions, the disregard of which can give rise to a cause of action in the
denied in a Decision dated April 8, 2005.Petitioners motion for courts. They do not embody judicially enforceable constitutional rights but
reconsideration was likewise denied in a Resolution dated August 5, 2005. guidelines for legislation."[10]

Hence, this appeal. Second, petitioner contends that the approval of the Department
of Agriculture (DA) Undersecretary for Fisheries and Livestock of the Food
Petitioner cites the following grounds for its appeal: Basket Allowance is the law which authorizes its release. It is crystal clear
1. The disallowance in question is unconstitutional as it that the DA Undersecretary has no authority to grant any allowance to the
contravenes the fundamental principle of the State employees of BFAR. Section 4.5 of Budget Circular No. 16 dated November
enshrined under Sections 9 and 10, Article II of the 28, 1998 states:
1987 Constitution, which provide as follows:
All agencies are hereby prohibited from granting
SEC. 9. The State shall promote a just and any food, rice, gift checks, or any other form of
dynamic social order that will ensure the prosperity incentives/allowances except those authorized via
and independence of the nation and free the Administrative Order by the Office of the President.
people from poverty through policies that provide
adequate social services, promote full
employment, a rising standard of living, and an In the instant case, no Administrative Order has been issued by
improved quality of life for all. the Office of the President to exempt BFAR from the express prohibition
against the grant of any food, rice, gift checks, or any other form of
SEC. 10. The State shall promote social justice in incentive/allowance to its employees.
all phases of national development. [6]
Petitioner argues that the grant of the Food Basket Allowance does
2. The Undersecretary for Fisheries and Livestock is an not violate Sec. 12 of R.A. No. 6758 or the Salary Standardization
extension of the Secretary of Agriculture who is an Law. This law was passed to standardize salary rates among government
alter-ego of the President. His approval was personnel and do away with multiple allowances and other incentive
tantamount to the authority from the Office of the packages and the resulting differences in compensation among them.
President, as contemplated in DBM Budget Circular No. [11]
Sec. 12 of the law provides:
16, dated November 28, 1998.[7]
Consolidation of Allowances and Compensation. All
3. The grant of the Food Basket Allowance is in allowances, except for representation and transportation
conformity with Sec. 12 of the Salary Standardization allowances; clothing and laundry allowances; subsistence
Law.[8] allowance of marine officers and crew on board
government vessels and hospital personnel; hazard pay;
allowances of foreign service personnel stationed abroad; which are usually granted to officials and employees of the government to
and such other additional compensation not otherwise defray or reimburse the expenses incurred in the performance of their
specified herein as may be determined by the DBM official functions. These are the RATA, clothing and laundry allowance,
[Department of Budget and Management], shall be subsistence allowance of marine officers and crew on board government
deemed included in the standardized salary rates herein vessels and hospital personnel, hazard pay, and others, as enumerated
prescribed. Such other additional compensation, whether in the first sentence of Section 12. We further ruled that the phrase "and
in cash or in kind, being received by incumbents only as of such other additional compensation not otherwise specified herein as may
July 1, 1989 not integrated into the standardized salary be determined by the DBM" is a catch-all proviso for benefits in the nature
rates shall continue to be authorized. of allowances similar to those enumerated. In Philippine Ports Authority
v. Commission on Audit,[13] we explained that if these allowances were
Existing additional compensation of any national consolidated with the standardized salary rates, then government officials
government official or employee paid from local funds of a or employees would be compelled to spend their personal funds in
local government unit shall be absorbed into the basic attending to their duties.
salary of said official or employee and shall be paid by the
National Government. In the instant case, the Food Basket Allowance is definitely not in
the nature of an allowance to reimburse expenses incurred by officials and
employees of the government in the performance of their official
Under Sec. 12, as quoted, all kinds of allowances are integrated in functions. It is not payment in consideration of the fulfillment of official
the standardized salary rates. The exceptions are: duty. It is a form of financial assistance to all officials and employees of
BFAR.Petitioner itself stated that the Food Basket Allowance has the
1. representation and transportation allowance purpose of alleviating the economic condition of BFAR employees.
(RATA);
Next, petitioner relies on National Compensation Circular No. 59
2. clothing and laundry allowance; dated September 30, 1989, issued by the DBM, which is the List of
3. subsistence allowance of marine officers and Allowances/Additional Compensation of Government Officials and
crew on board government vessels; Employees which shall be Deemed Integrated into the Basic Salary. The
list enumerates the following allowances/additional compensation which
4. subsistence allowance of hospital personnel; shall be incorporated in the basic salary, hence, may no longer be granted
5. hazard pay; to government employees:

6. allowances of foreign service personnel stationed 1. Cost of Living Allowance (COLA);


abroad; and 2. Inflation connected allowance;
7. such other additional compensation not 3. Living Allowance;
otherwise specified herein as may be determined by 4. Emergency Allowance;
the DBM. 5. Additional Compensation of Public Health Nurses
assigned to public health nursing;
6. Additional Compensation of Rural Health
Physicians;
Petitioner contends that the Food Basket Allowance falls under the
7. Additional Compensation of Nurses in Malacaang
7th category above, that of other additional compensation not otherwise
Clinic;
specified herein as may be determined by the DBM.
8. Nurses Allowance in the Air Transportation
Office;
The Court has had the occasion to interpret Sec. 12 of R.A. No. 9. Assignment Allowance of School
6758. In National Tobacco Administration v. Commission on Audit, Superintendents;
[12]
we held that under the first sentence of Section 12, the benefits 10. Post allowance of Postal Service Office employees;
excluded from the standardized salary rates are the "allowances" or those
11. Honoraria/allowances which are regularly given Decree (P.D.) No. 807. Sec. 15(d) of the GAA for Fiscal Year 1999 or R.A.
except the following: No. 8745 clearly prohibits the payment of honoraria, allowances or other
a. those for teaching overload; forms of compensation to any government official or employee, except
b. in lieu of overtime pay; those specifically authorized by law. There is no law authorizing the grant
c. for employees on detail with task of the subject Food Basket Allowance. Further, Sec. 33 of P.D. No. 807 or
forces/special projects; the Civil Service Decree of the Philippines does not exempt the Food
d. researchers, experts and specialists who are Basket Allowance from the general rule. Sec. 33 states:
acknowledged authorities in their field of
specialization; Section 33. Employee Suggestions and Incentive
e. lecturers and resource persons; Award System. There shall be established a government-
wide employee suggestions and incentive awards system
f. Municipal Treasurers deputized by the Bureau
which shall be administered under such rules, regulations,
of Internal Revenue to collect and remit internal
and standards as may be promulgated by the Commission.
revenue collections; and
In accordance with rules, regulations, and
g. Executive positions in State Universities and
standards promulgated by the Commission, the President
Colleges filled by designation from among their
or the head of each department or agency is authorized to
faculty members.
incur whatever necessary expenses involved in the
12. Subsistence Allowance of employees except those honorary recognition of subordinate officers and employees
authorized under EO [Executive Order] No. 346 and of the government who by their suggestions, inventions,
uniformed personnel of the Armed Forces of superior accomplishment, and other personal efforts
the Philippines and Integrated National Police; contribute to the efficiency, economy, or other
improvement of government operations, or who perform
13. Laundry Allowance of employees except those
such other extraordinary acts or services in the public
hospital/sanitaria personnel who attend directly to
interest in connection with, or in relation to, their official
patients and who by the nature of their duties are
employment.
required to wear uniforms, prison guards and
uniformed personnel of the Armed Forces of the
Philippines and Integrated National Police; and
We are not convinced that the Food Basket Allowance falls under
14. Incentive allowance/fee/pay except those the incentive award system contemplated above. The decree speaks
authorized under the General Appropriations Act and of suggestions, inventions, superior accomplishments, and other personal
Section 33 of P.D. No. 807. efforts contributed by an employee to the efficiency, economy, or other
improvement of government operations, or other extraordinary acts or
services performed by an employee in the public interest in connection
Petitioner invokes the rule of statutory construction that what is
with, or in relation to, his official employment. In the instant case, the
not included is excluded. Inclusio unius est exclusio alterius. Petitioner
Food Basket Allowance was granted to all BFAR employees, without
claims that the Food Basket Allowance is distinct and separate from the
distinction. It was not granted due to any extraordinary contribution or
specific allowances/additional compensation listed in the circular.
exceptional accomplishment by an employee. The Food Basket Allowance
was primarily an economic monetary assistance to the employees.
Again, we reject petitioners contention. The Food Basket Allowance
falls under the 14th category, that of incentive
Lastly, we note, as the Office of the Solicitor General, on behalf of
allowance/fee/pay. Petitioner itself justified the Food Basket Allowance as
respondent did, that petitioner failed to exhaust its administrative
an incentive to the employees to encourage them to be more productive
remedies. It stopped seeking remedies at the level of respondents Legal
and efficient.[14] Under National Compensation Circular No. 59, exceptions
and Adjudication Office. It failed to appeal the latters adverse decision to
to the incentive allowance/fee/pay category are those authorized under
the Commission on Audit proper. The consequence for failure to exhaust
the General Appropriations Act (GAA) and Section 33 of Presidential
administrative remedies is clear: the disallowance, as ruled
by the Commission on Audit Legal and Adjudication Office Regional Office RAFAEL MUSNGI, MICHAEL OCAMPO, JAKLYN HANNA PINEDA,
No. VII, Cebu City and upheld by the Commission on Audit Legal and WILLIAM RAGAMAT, MARICAR RAMOS, ENRIK FORT REVILLAS,
Adjudication Office National, Quezon City, became final and JAMES MARK TERRY RIDON, JOHANN FRANTZ RIVERA IV,
executory. Sections 48 and 51 of Presidential Decree No. 1445, or the CHRISTIAN RIVERO, DIANNE MARIE ROA, NICHOLAS SANTIZO,
Government Auditing Code of the Philippines provide: MELISSA CHRISTINA SANTOS, CRISTINE MAE TABING, VANESSA
ANNE TORNO, MARIA ESTER VANGUARDIA, and MARCELINO
Section 48. Appeal from decision of auditors. Any VELOSO III, Petitioners,
person aggrieved by the decision of an auditor of any vs.
government agency in the settlement of an account or HON. EDUARDO ERMITA, IN HIS CAPACITY AS EXECUTIVE
claim may, within six months from receipt of a copy of the SECRETARY, HON. ALBERTO ROMULO, IN HIS CAPACITY AS
decision, appeal in writing to the Commission. SECRETARY OF THE DEPARTMENT OF FOREIGN AFFAIRS, HON.
ROLANDO ANDAYA, IN HIS CAPACITY AS SECRETARY OF THE
Section 51. Finality of decisions of the Commission DEPARTMENT OF BUDGET AND MANAGEMENT, HON. DIONY
or any auditor. A decision of the Commission or of any VENTURA, IN HIS CAPACITY AS ADMINISTRATOR OF THE
auditor upon any matter within its or his jurisdiction, if not NATIONAL MAPPING & RESOURCE INFORMATION AUTHORITY, and
appealed as herein provided, shall be final and executory. HON. HILARIO DAVIDE, JR., IN HIS CAPACITY AS
REPRESENTATIVE OF THE PERMANENT MISSION OF THE REPUBLIC
OF THE PHILIPPINES TO THE UNITED NATIONS,Respondents.
IN VIEW WHEREOF, the petition is DENIED. The Decision and
Resolution of the Commission on Audit Legal and Adjudication Office dated
DECISION
April 8, 2005 and August 5, 2005, respectively, in LAO-N-2005-119, are
AFFIRMED.
CARPIO, J.:
SO ORDERED.
The Case

Republic of the Philippines


This original action for the writs of certiorari and prohibition assails the
SUPREME COURT
constitutionality of Republic Act No. 9522 1(RA 9522) adjusting the
Manila
country’s archipelagic baselines and classifying the baseline regime of
nearby territories.
EN BANC
The Antecedents
G.R No. 187167 August 16, 2011
In 1961, Congress passed Republic Act No. 3046 (RA 3046) 2 demarcating
PROF. MERLIN M. MAGALLONA, AKBAYAN PARTY-LIST REP. RISA the maritime baselines of the Philippines as an archipelagic State. 3 This
HONTIVEROS, PROF. HARRY C. ROQUE, JR., AND UNIVERSITY OF law followed the framing of the Convention on the Territorial Sea and the
THE PHILIPPINES COLLEGE OF LAW STUDENTS, ALITHEA BARBARA Contiguous Zone in 1958 (UNCLOS I),4 codifying, among others, the
ACAS, VOLTAIRE ALFERES, CZARINA MAY ALTEZ, FRANCIS ALVIN sovereign right of States parties over their "territorial sea," the breadth of
ASILO, SHERYL BALOT, RUBY AMOR BARRACA, JOSE JAVIER which, however, was left undetermined. Attempts to fill this void during
BAUTISTA, ROMINA BERNARDO, VALERIE PAGASA the second round of negotiations in Geneva in 1960 (UNCLOS II) proved
BUENAVENTURA, EDAN MARRI CAÑETE, VANN ALLEN DELA CRUZ, futile. Thus, domestically, RA 3046 remained unchanged for nearly five
RENE DELORINO, PAULYN MAY DUMAN, SHARON ESCOTO, decades, save for legislation passed in 1968 (Republic Act No. 5446 [RA
RODRIGO FAJARDO III, GIRLIE FERRER, RAOULLE OSEN FERRER, 5446]) correcting typographical errors and reserving the drawing of
CARLA REGINA GREPO, ANNA MARIE CECILIA GO, IRISH KAY baselines around Sabah in North Borneo.
KALAW, MARY ANN JOY LEE, MARIA LUISA MANALAYSAY, MIGUEL
In March 2009, Congress amended RA 3046 by enacting RA 9522, the Respondents also question the normative force, under international law, of
statute now under scrutiny. The change was prompted by the need to petitioners’ assertion that what Spain ceded to the United States under the
make RA 3046 compliant with the terms of the United Nations Convention Treaty of Paris were the islands and all the waters found within the
on the Law of the Sea (UNCLOS III),5 which the Philippines ratified on 27 boundaries of the rectangular area drawn under the Treaty of Paris.
February 1984.6 Among others, UNCLOS III prescribes the water-land
ratio, length, and contour of baselines of archipelagic States like the We left unacted petitioners’ prayer for an injunctive writ.
Philippines7 and sets the deadline for the filing of application for the
extended continental shelf.8 Complying with these requirements, RA 9522
The Issues
shortened one baseline, optimized the location of some basepoints around
the Philippine archipelago and classified adjacent territories, namely, the
Kalayaan Island Group (KIG) and the Scarborough Shoal, as "regimes of The petition raises the following issues:
islands" whose islands generate their own applicable maritime zones.
1. Preliminarily –
Petitioners, professors of law, law students and a legislator, in their
respective capacities as "citizens, taxpayers or x x x legislators," 9 as the 1. Whether petitioners possess locus standi to bring this
case may be, assail the constitutionality of RA 9522 on two principal suit; and
grounds, namely: (1) RA 9522 reduces Philippine maritime territory, and
logically, the reach of the Philippine state’s sovereign power, in violation of 2. Whether the writs of certiorari and prohibition are the
Article 1 of the 1987 Constitution,10 embodying the terms of the Treaty of proper remedies to assail the constitutionality of RA 9522.
Paris11 and ancillary treaties,12 and (2) RA 9522 opens the country’s waters
landward of the baselines to maritime passage by all vessels and aircrafts,
undermining Philippine sovereignty and national security, contravening the 2. On the merits, whether RA 9522 is unconstitutional.
country’s nuclear-free policy, and damaging marine resources, in violation
of relevant constitutional provisions.13 The Ruling of the Court

In addition, petitioners contend that RA 9522’s treatment of the KIG as On the threshold issues, we hold that (1) petitioners possess locus
"regime of islands" not only results in the loss of a large maritime area but standi to bring this suit as citizens and (2) the writs of certiorari and
also prejudices the livelihood of subsistence fishermen. 14 To buttress their prohibition are proper remedies to test the constitutionality of RA 9522. On
argument of territorial diminution, petitioners facially attack RA 9522 for the merits, we find no basis to declare RA 9522 unconstitutional.
what it excluded and included – its failure to reference either the Treaty of
Paris or Sabah and its use of UNCLOS III’s framework of regime of islands On the Threshold Issues
to determine the maritime zones of the KIG and the Scarborough Shoal. Petitioners Possess Locus
Standi as Citizens
Commenting on the petition, respondent officials raised threshold issues
questioning (1) the petition’s compliance with the case or controversy Petitioners themselves undermine their assertion of locus standi as
requirement for judicial review grounded on petitioners’ alleged lack legislators and taxpayers because the petition alleges neither infringement
of locus standi and (2) the propriety of the writs of certiorari and of legislative prerogative15 nor misuse of public funds,16 occasioned by the
prohibition to assail the constitutionality of RA 9522. On the merits, passage and implementation of RA 9522. Nonetheless, we recognize
respondents defended RA 9522 as the country’s compliance with the terms petitioners’ locus standi as citizens with constitutionally sufficient interest
of UNCLOS III, preserving Philippine territory over the KIG or Scarborough in the resolution of the merits of the case which undoubtedly raises issues
Shoal. Respondents add that RA 9522 does not undermine the country’s of national significance necessitating urgent resolution. Indeed, owing to
security, environment and economic interests or relinquish the Philippines’ the peculiar nature of RA 9522, it is understandably difficult to find other
claim over Sabah. litigants possessing "a more direct and specific interest" to bring the suit,
thus satisfying one of the requirements for granting citizenship standing. 17
The Writs of Certiorari and Prohibition UNCLOS III has nothing to do with the acquisition (or loss) of territory. It
Are Proper Remedies to Test is a multilateral treaty regulating, among others, sea-use rights over
the Constitutionality of Statutes maritime zones (i.e., the territorial waters [12 nautical miles from the
baselines], contiguous zone [24 nautical miles from the baselines],
In praying for the dismissal of the petition on preliminary grounds, exclusive economic zone [200 nautical miles from the baselines]), and
respondents seek a strict observance of the offices of the writs of certiorari continental shelves that UNCLOS III delimits. 23 UNCLOS III was the
and prohibition, noting that the writs cannot issue absent any showing of culmination of decades-long negotiations among United Nations members
grave abuse of discretion in the exercise of judicial, quasi-judicial or to codify norms regulating the conduct of States in the world’s oceans and
ministerial powers on the part of respondents and resulting prejudice on submarine areas, recognizing coastal and archipelagic States’ graduated
the part of petitioners.18 authority over a limited span of waters and submarine lands along their
coasts.
Respondents’ submission holds true in ordinary civil proceedings. When
this Court exercises its constitutional power of judicial review, however, we On the other hand, baselines laws such as RA 9522 are enacted by
have, by tradition, viewed the writs of certiorari and prohibition as proper UNCLOS III States parties to mark-out specific basepoints along their
remedial vehicles to test the constitutionality of statutes, 19 and indeed, of coasts from which baselines are drawn, either straight or contoured, to
acts of other branches of government.20 Issues of constitutional import are serve as geographic starting points to measure the breadth of the
sometimes crafted out of statutes which, while having no bearing on the maritime zones and continental shelf. Article 48 of UNCLOS III on
personal interests of the petitioners, carry such relevance in the life of this archipelagic States like ours could not be any clearer:
nation that the Court inevitably finds itself constrained to take cognizance
of the case and pass upon the issues raised, non-compliance with the Article 48. Measurement of the breadth of the territorial sea, the
letter of procedural rules notwithstanding. The statute sought to be contiguous zone, the exclusive economic zone and the continental shelf. –
reviewed here is one such law. The breadth of the territorial sea, the contiguous zone, the exclusive
economic zone and the continental shelf shall be measured from
RA 9522 is Not Unconstitutional archipelagic baselines drawn in accordance with article 47. (Emphasis
RA 9522 is a Statutory Tool supplied)
to Demarcate the Country’s
Maritime Zones and Continental Thus, baselines laws are nothing but statutory mechanisms for UNCLOS III
Shelf Under UNCLOS III, not to States parties to delimit with precision the extent of their maritime zones
Delineate Philippine Territory and continental shelves. In turn, this gives notice to the rest of the
international community of the scope of the maritime space and submarine
Petitioners submit that RA 9522 "dismembers a large portion of the areas within which States parties exercise treaty-based rights, namely, the
national territory"21 because it discards the pre-UNCLOS III demarcation of exercise of sovereignty over territorial waters (Article 2), the jurisdiction to
Philippine territory under the Treaty of Paris and related treaties, enforce customs, fiscal, immigration, and sanitation laws in the contiguous
successively encoded in the definition of national territory under the 1935, zone (Article 33), and the right to exploit the living and non-living
1973 and 1987 Constitutions. Petitioners theorize that this constitutional resources in the exclusive economic zone (Article 56) and continental shelf
definition trumps any treaty or statutory provision denying the Philippines (Article 77).
sovereign control over waters, beyond the territorial sea recognized at the
time of the Treaty of Paris, that Spain supposedly ceded to the United Even under petitioners’ theory that the Philippine territory embraces the
States. Petitioners argue that from the Treaty of Paris’ technical islands and all the waters within the rectangular area delimited in the
description, Philippine sovereignty over territorial waters extends hundreds Treaty of Paris, the baselines of the Philippines would still have to be
of nautical miles around the Philippine archipelago, embracing the drawn in accordance with RA 9522 because this is the only way to draw
rectangular area delineated in the Treaty of Paris.22 the baselines in conformity with UNCLOS III. The baselines cannot be
drawn from the boundaries or other portions of the rectangular area
Petitioners’ theory fails to persuade us.
delineated in the Treaty of Paris, but from the "outermost islands and Petitioners’ assertion of loss of "about 15,000 square nautical miles of
drying reefs of the archipelago."24 territorial waters" under RA 9522 is similarly unfounded both in fact and
law. On the contrary, RA 9522, by optimizing the location of
UNCLOS III and its ancillary baselines laws play no role in the acquisition, basepoints, increased the Philippines’ total maritime space (covering its
enlargement or, as petitioners claim, diminution of territory. Under internal waters, territorial sea and exclusive economic zone) by 145,216
traditional international law typology, States acquire (or conversely, lose) square nautical miles, as shown in the table below: 29
territory through occupation, accretion, cession and prescription, 25 not by
executing multilateral treaties on the regulations of sea-use rights or
enacting statutes to comply with the treaty’s terms to delimit maritime
zones and continental shelves. Territorial claims to land features are Extent of maritime
outside UNCLOS III, and are instead governed by the rules on general area using RA 3046, Extent of maritime
international law.26 as amended, taking area using RA 9522,
into account the taking into account
Treaty of Paris’ UNCLOS III (in
RA 9522’s Use of the Framework delimitation (in square nautical
of Regime of Islands to Determine the square nautical miles)
Maritime Zones of the KIG and the miles)
Scarborough Shoal, not Inconsistent
with the Philippines’ Claim of Sovereignty
Over these Areas Internal or
archipelagic
Petitioners next submit that RA 9522’s use of UNCLOS III’s regime of waters 166,858 171,435
islands framework to draw the baselines, and to measure the breadth of
the applicable maritime zones of the KIG, "weakens our territorial claim" Territorial Sea 274,136 32,106
over that area.27Petitioners add that the KIG’s (and Scarborough Shoal’s) Exclusive
exclusion from the Philippine archipelagic baselines results in the loss of Economic Zone 382,669
"about 15,000 square nautical miles of territorial waters," prejudicing the
livelihood of subsistence fishermen.28 A comparison of the configuration of TOTAL 440,994 586,210
the baselines drawn under RA 3046 and RA 9522 and the extent of
maritime space encompassed by each law, coupled with a reading of the
text of RA 9522 and its congressional deliberations, vis-à-vis the Thus, as the map below shows, the reach of the exclusive economic zone
Philippines’ obligations under UNCLOS III, belie this view.1avvphi1 drawn under RA 9522 even extends way beyond the waters covered by the
rectangular demarcation under the Treaty of Paris. Of course, where there
are overlapping exclusive economic zones of opposite or adjacent States,
The configuration of the baselines drawn under RA 3046 and RA 9522
there will have to be a delineation of maritime boundaries in accordance
shows that RA 9522 merely followed the basepoints mapped by RA 3046,
with UNCLOS III.30
save for at least nine basepoints that RA 9522 skipped to optimize the
location of basepoints and adjust the length of one baseline (and thus
comply with UNCLOS III’s limitation on the maximum length of baselines).
Under RA 3046, as under RA 9522, the KIG and the Scarborough Shoal lie
outside of the baselines drawn around the Philippine archipelago. This
undeniable cartographic fact takes the wind out of petitioners’ argument
branding RA 9522 as a statutory renunciation of the Philippines’ claim over
the KIG, assuming that baselines are relevant for this purpose.
Philippines’ continued claim of sovereignty and jurisdiction over the KIG
and the Scarborough Shoal:

SEC. 2. The baselines in the following areas over which the Philippines
likewise exercises sovereignty and jurisdiction shall be determined
as "Regime of Islands" under the Republic of the Philippines consistent
with Article 121 of the United Nations Convention on the Law of the Sea
(UNCLOS):

a) The Kalayaan Island Group as constituted under Presidential


Decree No. 1596 and

b) Bajo de Masinloc, also known as Scarborough Shoal. (Emphasis


supplied)

Had Congress in RA 9522 enclosed the KIG and the Scarborough Shoal as
part of the Philippine archipelago, adverse legal effects would have
ensued. The Philippines would have committed a breach of two provisions
of UNCLOS III. First, Article 47 (3) of UNCLOS III requires that "[t]he
drawing of such baselines shall not depart to any appreciable extent from
the general configuration of the archipelago." Second, Article 47 (2) of
UNCLOS III requires that "the length of the baselines shall not exceed 100
nautical miles," save for three per cent (3%) of the total number of
baselines which can reach up to 125 nautical miles. 31

Although the Philippines has consistently claimed sovereignty over the


KIG32 and the Scarborough Shoal for several decades, these outlying areas
are located at an appreciable distance from the nearest shoreline of the
Philippine archipelago,33 such that any straight baseline loped around them
from the nearest basepoint will inevitably "depart to an appreciable extent
from the general configuration of the archipelago."

The principal sponsor of RA 9522 in the Senate, Senator Miriam Defensor-


Santiago, took pains to emphasize the foregoing during the Senate
deliberations:

What we call the Kalayaan Island Group or what the rest of the world call[]
the Spratlys and the Scarborough Shoal are outside our archipelagic
baseline because if we put them inside our baselines we might be accused
of violating the provision of international law which states: "The drawing of
Further, petitioners’ argument that the KIG now lies outside Philippine such baseline shall not depart to any appreciable extent from the general
territory because the baselines that RA 9522 draws do not enclose the KIG configuration of the archipelago." So sa loob ng ating baseline, dapat
is negated by RA 9522 itself. Section 2 of the law commits to text the magkalapit ang mga islands. Dahil malayo ang Scarborough Shoal, hindi
natin masasabing malapit sila sa atin although we are still allowed by Hence, far from surrendering the Philippines’ claim over the KIG and the
international law to claim them as our own. Scarborough Shoal, Congress’ decision to classify the KIG and the
Scarborough Shoal as "‘Regime[s] of Islands’ under the Republic of the
This is called contested islands outside our configuration. We see that our Philippines consistent with Article 121" 36 of UNCLOS III manifests the
archipelago is defined by the orange line which [we] call[] archipelagic Philippine State’s responsible observance of its pacta sunt
baseline. Ngayon, tingnan ninyo ang maliit na circle doon sa itaas, that is servanda obligation under UNCLOS III. Under Article 121 of UNCLOS III,
Scarborough Shoal, itong malaking circle sa ibaba, that is Kalayaan Group any "naturally formed area of land, surrounded by water, which is above
or the Spratlys. Malayo na sila sa ating archipelago kaya kung ilihis pa water at high tide," such as portions of the KIG, qualifies under the
natin ang dating archipelagic baselines para lamang masama itong category of "regime of islands," whose islands generate their own
dalawang circles, hindi na sila magkalapit at baka hindi na tatanggapin ng applicable maritime zones.37
United Nations because of the rule that it should follow the natural
configuration of the archipelago.34 (Emphasis supplied) Statutory Claim Over Sabah under
RA 5446 Retained
Similarly, the length of one baseline that RA 3046 drew exceeded UNCLOS
III’s limits.1avvphi1 The need to shorten this baseline, and in addition, to Petitioners’ argument for the invalidity of RA 9522 for its failure to
optimize the location of basepoints using current maps, became imperative textualize the Philippines’ claim over Sabah in North Borneo is also
as discussed by respondents: untenable. Section 2 of RA 5446, which RA 9522 did not repeal, keeps
open the door for drawing the baselines of Sabah:
[T]he amendment of the baselines law was necessary to enable the
Philippines to draw the outer limits of its maritime zones including the Section 2. The definition of the baselines of the territorial sea of the
extended continental shelf in the manner provided by Article 47 of Philippine Archipelago as provided in this Act is without prejudice to the
[UNCLOS III]. As defined by R.A. 3046, as amended by R.A. 5446, the delineation of the baselines of the territorial sea around the
baselines suffer from some technical deficiencies, to wit: territory of Sabah, situated in North Borneo, over which the
Republic of the Philippines has acquired dominion and sovereignty.
1. The length of the baseline across Moro Gulf (from Middle of 3 (Emphasis supplied)
Rock Awash to Tongquil Point) is 140.06 nautical miles x x x. This
exceeds the maximum length allowed under Article 47(2) of the UNCLOS III and RA 9522 not
[UNCLOS III], which states that "The length of such baselines shall Incompatible with the Constitution’s
not exceed 100 nautical miles, except that up to 3 per cent of the Delineation of Internal Waters
total number of baselines enclosing any archipelago may exceed
that length, up to a maximum length of 125 nautical miles." As their final argument against the validity of RA 9522, petitioners contend
that the law unconstitutionally "converts" internal waters into archipelagic
2. The selection of basepoints is not optimal. At least 9 basepoints waters, hence subjecting these waters to the right of innocent and sea
can be skipped or deleted from the baselines system. This will lanes passage under UNCLOS III, including overflight. Petitioners
enclose an additional 2,195 nautical miles of water. extrapolate that these passage rights indubitably expose Philippine internal
waters to nuclear and maritime pollution hazards, in violation of the
3. Finally, the basepoints were drawn from maps existing in 1968, Constitution.38
and not established by geodetic survey methods. Accordingly,
some of the points, particularly along the west coasts of Luzon Whether referred to as Philippine "internal waters" under Article I of the
down to Palawan were later found to be located either inland or on Constitution39 or as "archipelagic waters" under UNCLOS III (Article 49
water, not on low-water line and drying reefs as prescribed by [1]), the Philippines exercises sovereignty over the body of water lying
Article 47.35 landward of the baselines, including the air space over it and the
submarine areas underneath. UNCLOS III affirms this:
Article 49. Legal status of archipelagic waters, of the air space over place them in lesser footing vis-à-vis continental coastal States which are
archipelagic waters and of their bed and subsoil. – subject, in their territorial sea, to the right of innocent passage and the
right of transit passage through international straits. The imposition of
1. The sovereignty of an archipelagic State extends to the these passage rights through archipelagic waters under UNCLOS III was a
waters enclosed by the archipelagic baselines drawn in concession by archipelagic States, in exchange for their right to claim all
accordance with article 47, described as archipelagic waters, the waters landward of their baselines, regardless of their depth or
regardless of their depth or distance from the coast. distance from the coast, as archipelagic waters subject to their territorial
sovereignty. More importantly, the recognition of archipelagic States’
archipelago and the waters enclosed by their baselines as one cohesive
2. This sovereignty extends to the air space over the
entity prevents the treatment of their islands as separate islands under
archipelagic waters, as well as to their bed and subsoil, and
UNCLOS III.46 Separate islands generate their own maritime zones, placing
the resources contained therein.
the waters between islands separated by more than 24 nautical miles
beyond the States’ territorial sovereignty, subjecting these waters to the
xxxx rights of other States under UNCLOS III. 47

4. The regime of archipelagic sea lanes passage established in this Petitioners’ invocation of non-executory constitutional provisions in Article
Part shall not in other respects affect the status of the II (Declaration of Principles and State Policies)48 must also fail. Our present
archipelagic waters, including the sea lanes, or the exercise by state of jurisprudence considers the provisions in Article II as mere
the archipelagic State of its sovereignty over such waters legislative guides, which, absent enabling legislation, "do not embody
and their air space, bed and subsoil, and the resources judicially enforceable constitutional rights x x x."49 Article II provisions
contained therein. (Emphasis supplied) serve as guides in formulating and interpreting implementing legislation,
as well as in interpreting executory provisions of the Constitution.
The fact of sovereignty, however, does not preclude the operation of Although Oposa v. Factoran50 treated the right to a healthful and balanced
municipal and international law norms subjecting the territorial sea or ecology under Section 16 of Article II as an exception, the present petition
archipelagic waters to necessary, if not marginal, burdens in the interest of lacks factual basis to substantiate the claimed constitutional violation. The
maintaining unimpeded, expeditious international navigation, consistent other provisions petitioners cite, relating to the protection of marine
with the international law principle of freedom of navigation. Thus, wealth (Article XII, Section 2, paragraph 251 ) and subsistence fishermen
domestically, the political branches of the Philippine government, in the (Article XIII, Section 752 ), are not violated by RA 9522.
competent discharge of their constitutional powers, may pass legislation
designating routes within the archipelagic waters to regulate innocent and In fact, the demarcation of the baselines enables the Philippines to delimit
sea lanes passage.40 Indeed, bills drawing nautical highways for sea lanes its exclusive economic zone, reserving solely to the Philippines the
passage are now pending in Congress.41 exploitation of all living and non-living resources within such zone. Such a
maritime delineation binds the international community since the
In the absence of municipal legislation, international law norms, now delineation is in strict observance of UNCLOS III. If the maritime
codified in UNCLOS III, operate to grant innocent passage rights over the delineation is contrary to UNCLOS III, the international community will of
territorial sea or archipelagic waters, subject to the treaty’s limitations and course reject it and will refuse to be bound by it.
conditions for their exercise.42 Significantly, the right of innocent passage
is a customary international law,43 thus automatically incorporated in the UNCLOS III favors States with a long coastline like the Philippines.
corpus of Philippine law.44 No modern State can validly invoke its UNCLOS III creates a sui generis maritime space – the exclusive economic
sovereignty to absolutely forbid innocent passage that is exercised in zone – in waters previously part of the high seas. UNCLOS III grants new
accordance with customary international law without risking retaliatory rights to coastal States to exclusively exploit the resources found within
measures from the international community. this zone up to 200 nautical miles.53 UNCLOS III, however, preserves the
traditional freedom of navigation of other States that attached to this zone
The fact that for archipelagic States, their archipelagic waters are subject beyond the territorial sea before UNCLOS III.
to both the right of innocent passage and sea lanes passage 45 does not
RA 9522 and the Philippines’ Maritime Zones

Petitioners hold the view that, based on the permissive text of UNCLOS III,
Congress was not bound to pass RA 9522.54 We have looked at the
relevant provision of UNCLOS III55 and we find petitioners’ reading
plausible. Nevertheless, the prerogative of choosing this option belongs to
Congress, not to this Court. Moreover, the luxury of choosing this option
comes at a very steep price. Absent an UNCLOS III compliant baselines
law, an archipelagic State like the Philippines will find itself devoid of
internationally acceptable baselines from where the breadth of its maritime
zones and continental shelf is measured. This is recipe for a two-fronted
disaster: first, it sends an open invitation to the seafaring powers to freely
enter and exploit the resources in the waters and submarine areas around
our archipelago; and second, it weakens the country’s case in any
international dispute over Philippine maritime space. These are
consequences Congress wisely avoided.

The enactment of UNCLOS III compliant baselines law for the Philippine
archipelago and adjacent areas, as embodied in RA 9522, allows an
internationally-recognized delimitation of the breadth of the Philippines’
maritime zones and continental shelf. RA 9522 is therefore a most vital
step on the part of the Philippines in safeguarding its maritime zones,
consistent with the Constitution and our national interest.

WHEREFORE, we DISMISS the petition.

SO ORDERED.

Pursuant to Section 13, Article VIII of the Constitution, I certify that the
conclusions in the above Decision had been reached in consultation before
the case was assigned to the writer of the opinion of the Court.

RENATO C. CORONA
Chief Justice
Republic of the Philippines Manolo Racho, Spouses Alfred R. Racho & Francine V. Racho for
SUPREME COURT themselves and on behalf of their minor children Michael Racho,
Baguio City Mariana Racho, Rafael Racho, Maxi Racho, Chessie Racho & Laura
Racho, Spouses David R. Racho & Armilyn A. Racho for themselves
EN BANC and on behalf of their minor child Gabriel Racho, Mindy M. Juatas
and on behalf of her minor children Elijah Gerald Juatas and Elian
Gabriel Juatas, Salvacion M. Monteiro, Emily R. Laws, Joseph R .
G.R. No. 204819 April 8, 2014
Laws & Katrina R. Laws, Petitioners,
vs.
JAMES M. IMBONG and LOVELY-ANN C. IMBONG, for themselves HON. PAQUITO N. OCHOA, JR., Executive Secretary, HON. ENRIQUE
and in behalf of their minor children, LUCIA CARLOS IMBONG and T. ONA, Secretary, Department of Health, HON. ARMIN A. LUISTRO,
BERNADETTE CARLOS IMBONG and MAGNIFICAT CHILD Secretary, Department of Education, Culture and Sports, HON.
DEVELOPMENT CENTER, INC., Petitioners, CORAZON SOLIMAN, Secretary, Department of Social Welfare and
vs. Development, HON. MANUELA. ROXAS II, Secretary, Department of
HON. PAQUITO N. OCHOA, JR., Executive Secretary, HON. Interior and Local Government, HON. FLORENCIO B. ABAD,
FLORENCIO B. ABAD, Secretary, Department of Budget and Secretary, Department of Budget and Management, HON. ARSENIO
Management, HON. ENRIQUE T. ONA, Secretary, Department of M. BALISACAN, Socio-Economic Planning Secretary and NEDA
Health, HON. ARMIN A. LUISTRO, Secretary, Department of Director-General, THE PHILIPPINE COMMISSION ON WOMEN,
Education, Culture and Sports and HON. MANUELA. ROXAS II, represented by its Chairperson, Remedios lgnacio-Rikken, THE
Secretary, Department of Interior and Local PHILIPPINE HEALTH INSURANCE CORPORATION, represented by
Government, Respondents. its President Eduardo Banzon, THE LEAGUE OF PROVINCES OF THE
PHILIPPINES, represented by its President Alfonso Umali, THE
x---------------------------------x LEAGUE OF CITIES OF THE PHILIPPINES, represented by its
President Oscar Rodriguez, and THE LEAGUE OF MUNICIPALITIES
G.R. No. 204934 OF THE PHILIPPINES, represented by its President Donato
Marcos,Respondents.

ALLIANCE FOR THE FAMILY FOUNDATION PHILIPPINES, INC.


[ALFI], represented by its President, Maria Concepcion S. Noche, x---------------------------------x
Spouses Reynaldo S. Luistro & Rosie B . Luistro, Jose S. Sandejas &
Elenita S.A. Sandejas, Arturo M. Gorrez & Marietta C. Gorrez, G.R. No. 204957
Salvador S. Mante, Jr. & Hazeleen L. Mante, Rolando M. Bautista &
Maria Felisa S. Bautista, Desiderio Racho & Traquilina Racho, F TASK FORCE FOR FAMILY AND LIFE VISAYAS, INC. and VALERIANO
emand Antonio A. Tansingco & Carol Anne C. Tansingco for S. AVILA, Petitioners,
themselves and on behalf of their minor children, Therese vs.
Antonette C. Tansingco, Lorenzo Jose C. Tansingco, Miguel F HON. PAQUITO N. OCHOA, JR., Executive Secretary; HON.
emando C. Tangsingco, Carlo Josemaria C. Tansingco & Juan Paolo FLORENCIO B. ABAD, Secretary, Department of Budget and
C. Tansingco, Spouses Mariano V. Araneta & Eileen Z. Araneta for Management; HON. ENRIQUE T. ONA, Secretary, Department of
themselves and on behalf of their minor children, Ramon Carlos Z. Education; and HON. MANUELA. ROXAS II, Secretary, Department
Araneta & Maya Angelica Z. Araneta, Spouses Renato C. Castor & of Interior and Local Government, Respondents.
Mildred C. Castor for themselves and on behalf of their minor
children, Renz Jeffrey C. Castor, Joseph Ramil C. Castor, John Paul
x---------------------------------x
C. Castor & Raphael C. Castor, Spouses Alexander R. Racho & Zara
Z. Racho for themselves and on behalf of their minor children
Margarita Racho, Mikaela Racho, Martin Racho, Mari Racho & G.R. No. 204988
SERVE LIFE CAGAYAN DE ORO CITY, INC., represented by Dr. PHILIPPINE ALLIANCE OF XSEMINARIANS, INC. (PAX), herein
Nestor B. Lumicao, M.D., as President and in his personal capacity, represented by its National President, Atty. Ricardo M . Ribo, and
ROSEVALE FOUNDATION INC., represented by Dr. Rodrigo M. in his own behalf, Atty. Lino E.A. Dumas, Romeo B. Almonte,
Alenton, M.D., as member of the school board and in his personal Osmundo C. Orlanes, Arsenio Z. Menor, Samuel J. Yap, Jaime F.
capacity, ROSEMARIE R. ALENTON, IMELDA G. IBARRA, CPA, Mateo, Rolly Siguan, Dante E. Magdangal, Michael Eugenio O.
LOVENIAP. NACES, Phd., ANTHONY G. NAGAC, EARL ANTHONY C. Plana, Bienvenido C. Miguel, Jr., Landrito M. Diokno and Baldomero
GAMBE and MARLON I. YAP,Petitioners, Falcone, Petitioners,
vs. vs.
OFFICE OF THE PRESIDENT, SENATE OF THE PHILIPPINES, HOUSE HON. PAQUITO N. OCHOA, JR., Executive Secretary, HON.
OF REPRESENTATIVES, HON. PAQUITO N. OCHOA, JR., Executive FLORENCIO B. ABAD, Secretary, Department of Budget and
Secretary, HON. FLORENCIO B. ABAD, Secretary, Department of Management, HON. ENRIQUE T. ONA, Secretary, Department of
Budget and Management; HON. ENRIQUE T. ONA, Secretary, Health, HON. ARMIN A. LUISTRO, Secretary, Department of
Department of Health; HON. ARMIN A. LUISTRO, Secretary, Education, HON. MANUELA. ROXAS II, Secretary, Department of
Department of Education and HON. MANUELA. ROXAS II, Secretary, Interior and Local Government, HON. CORAZON J. SOLIMAN,
Department of Interior and Local Government, Respondents. Secretary, Department of Social Welfare and Development, HON.
ARSENIO BALISACAN, Director-General, National Economic and
x---------------------------------x Development Authority, HON. SUZETTE H. LAZO, Director-General,
Food and Drugs Administration, THE BOARD OF DIRECTORS,
Philippine Health Insurance Corporation, and THE BOARD OF
G.R. No. 205003
COMMISSIONERS, Philippine Commission on Women, Respondents.

EXPEDITO A. BUGARIN, JR., Petitioner,


x---------------------------------x
vs.
OFFICE OF THE PRESIDENT OF THE REPUBLIC OF THE
PHILIPPINES, HON. SENATE PRESIDENT, HON. SPEAKER OF THE G.R. No. 205478
HOUSE OF REPRESENTATIVES and HON. SOLICITOR
GENERAL, Respondents. REYNALDO J. ECHAVEZ, M.D., JACQUELINE H. KING, M.D., CYNTHIA
T. DOMINGO, M.D., AND JOSEPHINE MILLADO-LUMITAO, M.D.,
x---------------------------------x collectively known as Doctors For Life, and ANTHONY PEREZ,
MICHAEL ANTHONY G. MAPA, CARLOS ANTONIO PALAD, WILFREDO
JOSE, CLAIRE NAVARRO, ANNA COSIO, and GABRIEL DY LIACCO
G.R. No. 205043
collectively known as Filipinos For Life, Petitioners,
vs.
EDUARDO B. OLAGUER and THE CATHOLIC XYBRSPACE HON. PAQUITO N. OCHOA, JR., Executive Secretary; HON.
APOSTOLATE OF THE PHILIPPINES, Petitioners, FLORENCIO B. ABAD, Secretary of the Department of Budget and
vs. Management; HON. ENRIQUE T. ONA, Secretary of the Department
DOH SECRETARY ENRIQUE T. ONA, FDA DIRECTOR SUZETTE H. of Health; HON. ARMIN A. LUISTRO, Secretary of the Department
LAZO, DBM SECRETARY FLORENCIO B. ABAD, DILG SECRETARY of Education; and HON. MANUELA. ROXAS II, Secretary of the
MANUELA. ROXAS II, DECS SECRETARY ARMIN A. Department of Interior and Local Government, Respondents.
LUISTRO, Respondents.
x---------------------------------x
x---------------------------------x
G.R. No. 205491
G.R. No. 205138
SPOUSES FRANCISCO S. TATAD AND MARIA FENNY C. TATAD & ALA JOHN WALTER B. JUAT, MARY M. IMBONG, ANTHONY VICTORIO B.
F. PAGUIA, for themselves, their Posterity, and the rest of Filipino LUMICAO, JOSEPH MARTIN Q. VERDEJO, ANTONIA EMMA R. ROXAS
posterity, Petitioners, and LOTA LAT-GUERRERO, Petitioners,
vs. vs.
OFFICE OF THE PRESIDENT of the Republic of the HON. PAQUITO N. OCHOA, JR., Executive Secretary, HON.
Philippines, Respondent. FLORENCIO ABAD, Secretary, Department of Budget and
Management, HON. ENRIQUE T. ONA, Secretary, Department of
x---------------------------------x Health, HON. ARMIN A. LUISTRO, Secretary, Department of
Education, Culture and Sports and HON. MANUEL A. ROXAS II,
Secretary, Department of Interior and Local
G.R. No. 205720
Government, Respondents.

PRO-LIFE PHILIPPINES FOUNDATION, Inc., represented by Loma


x---------------------------------x
Melegrito, as Executive Director, and in her personal capacity,
JOSELYN B. BASILIO, ROBERT Z. CORTES, ARIEL A. CRISOSTOMO,
JEREMY I. GATDULA, CRISTINA A. MONTES, RAUL ANTONIO A. G.R. No. 207172
NIDOY, WINSTON CONRAD B. PADOJINOG, RUFINO L. POLICARPIO
III, Petitioners, COUPLES FOR CHRIST FOUNDATION, INC., SPOUSES JUAN CARLOS
vs. ARTADI SARMIENTO AND FRANCESCA ISABELLE BESINGA-
OFFICE OF THE PRESIDENT, SENATE OF THE PHILIPPINES, HOUSE SARMIENTO, AND SPOUSES LUIS FRANCIS A. RODRIGO, JR. and
OF REPRESENTATIVES, HON. PAQUITO N. OCHOA, JR., Executive DEBORAH MARIE VERONICA N. RODRIGO, Petitioners,
Secretary, HON. FLORENCIO B. ABAD, Secretary, Department of vs.
Budget and Management, HON. ENRIQUE T. ONA, Secretary, HON. PAQUITO N. OCHOA, JR., Executive Secretary, HON.
Department of Health, HON. ARMIN A. LUISTRO, Secretary, FLORENCIO B. ABAD, Secretary, Department of Budget and
Department of Education and HON. MANUEL A. ROXAS II, Management, HON. ENRIQUE T. ONA, Secretary, Department of
Secretary, Department of Interior and Local Health, HON. ARMIN A. LUISTRO, Secretary, Department of
Government, Respondents. Education, Culture and Sports and HON. MANUELA. ROXAS II,
Secretary, Department of Interior and Local
x---------------------------------x Government, Respondents.

G.R. No. 206355 x---------------------------------x

MILLENNIUM SAINT FOUNDATION, INC., ATTY. RAMON PEDROSA, G.R. No. 207563
ATTY. CITA BORROMEO-GARCIA, STELLAACEDERA, ATTY. BERTENI
CATALUNA CAUSING, Petitioners, ALMARIM CENTI TILLAH and ABDULHUSSEIN M.
vs. KASHIM, Petitioners,
OFFICE OF THE PRESIDENT, OFFICE OF THE EXECUTIVE vs.
SECRETARY, DEPARTMENT OF HEALTH, DEPARTMENT OF HON. PAQUITO N. OCHOA, JR., Executive Secretary, HON. ENRIQUE
EDUCATION, Respondents. T. ONA, Secretary of the Department of Health, and HON. ARMIN A.
LUISTRO,Secretary of the Department of Budget and
x---------------------------------x Management,Respondents.

G.R. No. 207111 DECISION


MENDOZA, J.: the Court, beckoning it to wield the sword that strikes down constitutional
disobedience. Aware of the profound and lasting impact that its decision
Freedom of religion was accorded preferred status by the framers of our may produce, the Court now faces the iuris controversy, as presented in
fundamental law. And this Court has consistently affirmed this preferred fourteen (14) petitions and two (2) petitions- in-intervention, to wit:
status, well aware that it is "designed to protect the broadest possible
liberty of conscience, to allow each man to believe as his conscience (1) Petition for Certiorari and Prohibition,5 filed by spouses Attys.
directs, to profess his beliefs , and to live as he believes he ought to live, James M. Imbong and Lovely Ann C. Imbong, in their personal
consistent with the liberty of others and with the common good." 1 capacities as citizens, lawyers and taxpayers and on behalf of their
minor children; and the Magnificat Child Leaming Center, Inc., a
To this day, poverty is still a major stumbling block to the nation's domestic, privately-owned educational institution (Jmbong);
emergence as a developed country, leaving our people beleaguered in a
state of hunger, illiteracy and unemployment. While governmental policies (2) Petition for Prohibition,6 filed by the Alliance for the Family
have been geared towards the revitalization of the economy, the Foundation Philippines, Inc., through its president, Atty. Maria
bludgeoning dearth in social services remains to be a problem that Concepcion S. Noche7 and several others8 in their personal
concerns not only the poor, but every member of society. The government capacities as citizens and on behalf of the generations unborn
continues to tread on a trying path to the realization of its very purpose, (ALFI);
that is, the general welfare of the Filipino people and the development of
the country as a whole. The legislative branch, as the main facet of a (3) Petition for Certiorari,9 filed by the Task Force for Family and
representative government, endeavors to enact laws and policies that aim Life Visayas, Inc., and Valeriano S. Avila, in their capacities as
to remedy looming societal woes, while the executive is closed set to fully citizens and taxpayers (Task Force Family);
implement these measures and bring concrete and substantial solutions
within the reach of Juan dela Cruz. Seemingly distant is the judicial
(4) Petition for Certiorari and Prohibition,10 filed by Serve Life
branch, oftentimes regarded as an inert governmental body that merely
Cagayan De Oro City, Inc.,11 Rosevale Foundation, Inc.,12 a
casts its watchful eyes on clashing stakeholders until it is called upon to
domestic, privately-owned educational institution, and several
adjudicate. Passive, yet reflexive when called into action, the Judiciary
others,13 in their capacities as citizens (Serve Life);
then willingly embarks on its solemn duty to interpret legislation vis-a-vis
the most vital and enduring principle that holds Philippine society together
- the supremacy of the Philippine Constitution. (5) Petition,14 filed by Expedito A. Bugarin, Jr. in his capacity as a
citizen (Bugarin);
Nothing has polarized the nation more in recent years than the issues of
population growth control, abortion and contraception. As in every (6) Petition for Certiorari and Prohibition,15 filed by Eduardo
democratic society, diametrically opposed views on the subjects and their Olaguer and the Catholic Xybrspace Apostolate of the
perceived consequences freely circulate in various media. From television Philippines,16 in their capacities as a citizens and taxpayers
debates2 to sticker campaigns,3 from rallies by socio-political activists to (Olaguer);
mass gatherings organized by members of the clergy4 - the clash between
the seemingly antithetical ideologies of the religious conservatives and (7) Petition for Certiorari and Prohibition,17 filed by the Philippine
progressive liberals has caused a deep division in every level of the Alliance of Xseminarians Inc.,18 and several others19 in their
society. Despite calls to withhold support thereto, however, Republic Act capacities as citizens and taxpayers (PAX);
(R.A.) No. 10354, otherwise known as the Responsible Parenthood and
Reproductive Health Act of 2012 (RH Law), was enacted by Congress on (8) Petition,20 filed by Reynaldo J. Echavez, M.D. and several
December 21, 2012. others,21 in their capacities as citizens and taxpayers (Echavez);

Shortly after the President placed his imprimatur on the said law, (9) Petition for Certiorari and Prohibition,22 filed by spouses
challengers from various sectors of society came knocking on the doors of Francisco and Maria Fenny C. Tatad and Atty. Alan F. Paguia, in
their capacities as citizens, taxpayers and on behalf of those yet • The RH Law violates the right to health and the right to
unborn. Atty. Alan F. Paguia is also proceeding in his capacity as a protection against hazardous products. The petitioners posit that
member of the Bar (Tatad); the RH Law provides universal access to contraceptives which are
hazardous to one's health, as it causes cancer and other health
(10) Petition for Certiorari and Prohibition,23 filed by Pro-Life problems.36
Philippines Foundation Inc.24 and several others,25 in their
capacities as citizens and taxpayers and on behalf of its associates • The RH Law violates the right to religious freedom. The
who are members of the Bar (Pro-Life); petitioners contend that the RH Law violates the constitutional
guarantee respecting religion as it authorizes the use of public
(11) Petition for Prohibition,26 filed by Millennium Saint Foundation, funds for the procurement of contraceptives. For the petitioners,
Inc.,27 Attys. Ramon Pedrosa, Cita Borromeo-Garcia, Stella the use of public funds for purposes that are believed to be
Acedera, and Berteni Catalufia Causing, in their capacities as contrary to their beliefs is included in the constitutional mandate
citizens, taxpayers and members of the Bar (MSF); ensuring religious freedom.37

(12) Petition for Certiorari and Prohibition,28 filed by John Walter B. It is also contended that the RH Law threatens conscientious objectors of
Juat and several others,29 in their capacities as citizens (Juat) ; criminal prosecution, imprisonment and other forms of punishment, as it
compels medical practitioners 1] to refer patients who seek advice on
reproductive health programs to other doctors; and 2] to provide full and
(13) Petition for Certiorari and Prohibition,30 filed by Couples for
correct information on reproductive health programs and service, although
Christ Foundation, Inc. and several others, 31in their capacities as
it is against their religious beliefs and convictions. 38
citizens (CFC);

In this connection, Section 5 .23 of the Implementing Rules and


(14) Petition for Prohibition32 filed by Almarim Centi Tillah and
Regulations of the RH Law (RH-IRR),39 provides that skilled health
Abdulhussein M. Kashim in their capacities as citizens and
professionals who are public officers such as, but not limited to, Provincial,
taxpayers (Tillah); and
City, or Municipal Health Officers, medical officers, medical specialists,
rural health physicians, hospital staff nurses, public health nurses, or rural
(15) Petition-In-Intervention,33 filed by Atty. Samson S. Alcantara health midwives, who are specifically charged with the duty to implement
in his capacity as a citizen and a taxpayer (Alcantara); and these Rules, cannot be considered as conscientious objectors. 40

(16) Petition-In-Intervention,34 filed by Buhay Hayaang Yumabong It is also argued that the RH Law providing for the formulation of
(B UHAY) , an accredited political party. mandatory sex education in schools should not be allowed as it is an
affront to their religious beliefs.41
A perusal of the foregoing petitions shows that the petitioners are assailing
the constitutionality of RH Law on the following GROUNDS: While the petit10ners recognize that the guarantee of religious freedom is
not absolute, they argue that the RH Law fails to satisfy the "clear and
• The RH Law violates the right to life of the unborn. According to present danger test" and the "compelling state interest test" to justify the
the petitioners, notwithstanding its declared policy against regulation of the right to free exercise of religion and the right to free
abortion, the implementation of the RH Law would authorize the speech.42
purchase of hormonal contraceptives, intra-uterine devices and
injectables which are abortives, in violation of Section 12, Article II • The RH Law violates the constitutional provision on involuntary
of the Constitution which guarantees protection of both the life of servitude. According to the petitioners, the RH Law subjects
the mother and the life of the unborn from conception. 35 medical practitioners to involuntary servitude because, to be
accredited under the PhilHealth program, they are compelled to
provide forty-eight (48) hours of pro bona services for indigent
women, under threat of criminal prosecution, imprisonment and It is claimed that, by giving absolute authority to the person who will
other forms of punishment.43 undergo reproductive health procedure, the RH Law forsakes any real
dialogue between the spouses and impedes the right of spouses to
The petitioners explain that since a majority of patients are covered by mutually decide on matters pertaining to the overall well-being of their
PhilHealth, a medical practitioner would effectively be forced to render family. In the same breath, it is also claimed that the parents of a child
reproductive health services since the lack of PhilHealth accreditation who has suffered a miscarriage are deprived of parental authority to
would mean that the majority of the public would no longer be able to determine whether their child should use contraceptives. 50
avail of the practitioners services.44
• The RH Law violates the constitutional principle of non-delegation
• The RH Law violates the right to equal protection of the law. It is of legislative authority. The petitioners question the delegation by
claimed that the RH Law discriminates against the poor as it Congress to the FDA of the power to determine whether a product
makes them the primary target of the government program that is non-abortifacient and to be included in the Emergency Drugs
promotes contraceptive use. The petitioners argue that, rather List (EDL).51
than promoting reproductive health among the poor, the RH Law
seeks to introduce contraceptives that would effectively reduce the • The RH Law violates the one subject/one bill rule provision under
number of the poor.45 Section 26( 1 ), Article VI of the Constitution. 52

• The RH Law is "void-for-vagueness" in violation of the due • The RH Law violates Natural Law.53
process clause of the Constitution. In imposing the penalty of
imprisonment and/or fine for "any violation," it is vague because it • The RH Law violates the principle of Autonomy of Local
does not define the type of conduct to be treated as "violation" of Government Units (LGUs) and the Autonomous Region of Muslim
the RH Law.46 Mindanao {ARMM). It is contended that the RH Law, providing for
reproductive health measures at the local government level and
In this connection, it is claimed that "Section 7 of the RH Law violates the the ARMM, infringes upon the powers devolved to LGUs and the
right to due process by removing from them (the people) the right to ARMM under the Local Government Code and R.A . No. 9054. 54
manage their own affairs and to decide what kind of health facility they
shall be and what kind of services they shall offer."47 It ignores the Various parties also sought and were granted leave to file their respective
management prerogative inherent in corporations for employers to comments-in-intervention in defense of the constitutionality of the RH
conduct their affairs in accordance with their own discretion and judgment. Law. Aside from the Office of the Solicitor General (OSG) which
commented on the petitions in behalf of the respondents, 55 Congressman
• The RH Law violates the right to free speech. To compel a person Edcel C. Lagman,56 former officials of the Department of Health Dr.
to explain a full range of family planning methods is plainly to Esperanza I. Cabral, Jamie Galvez-Tan, and Dr. Alberto G.
curtail his right to expound only his own preferred way of family Romualdez,57 the Filipino Catholic Voices for Reproductive Health
planning. The petitioners note that although exemption is granted (C4RH),58 Ana Theresa "Risa" Hontiveros,59 and Atty. Joan De
to institutions owned and operated by religious groups, they are Venecia60 also filed their respective Comments-in-Intervention in
still forced to refer their patients to another healthcare facility conjunction with several others. On June 4, 2013, Senator Pia Juliana S.
willing to perform the service or procedure. 48 Cayetano was also granted leave to intervene.61

• The RH Law intrudes into the zone of privacy of one's family The respondents, aside from traversing the substantive arguments of the
protected by the Constitution. It is contended that the RH Law petitioners, pray for the dismissal of the petitions for the principal reasons
providing for mandatory reproductive health education intrudes that 1] there is no actual case or controversy and, therefore, the issues
upon their constitutional right to raise their children in accordance are not yet ripe for judicial determination.; 2] some petitioners lack
with their beliefs.49 standing to question the RH Law; and 3] the petitions are essentially
petitions for declaratory relief over which the Court has no original shall be delivered or sold to any person without a proper prescription by a
jurisdiction. duly licensed physician."

Meanwhile, on March 15, 2013, the RH-IRR for the enforcement of the On December 11, 1967, the Philippines, adhering to the UN Declaration on
assailed legislation took effect. Population, which recognized that the population problem should be
considered as the principal element for long-term economic development,
On March 19, 2013, after considering the issues and arguments raised, the enacted measures that promoted male vasectomy and tubal ligation to
Court issued the Status Quo Ante Order (SQAO), enjoining the effects and mitigate population growth.67 Among these measures included R.A. No.
implementation of the assailed legislation for a period of one hundred and 6365, approved on August 16, 1971, entitled "An Act Establishing a
twenty (120) days, or until July 17, 2013.62 National Policy on Population, Creating the Commission on Population and
for Other Purposes. " The law envisioned that "family planning will be
made part of a broad educational program; safe and effective means will
On May 30, 2013, the Court held a preliminary conference with the
be provided to couples desiring to space or limit family size; mortality and
counsels of the parties to determine and/or identify the pertinent issues
morbidity rates will be further reduced."
raised by the parties and the sequence by which these issues were to be
discussed in the oral arguments. On July 9 and 23, 2013, and on August 6,
13, and 27, 2013, the cases were heard on oral argument. On July 16, To further strengthen R.A. No. 6365, then President Ferdinand E . Marcos
2013, the SQAO was ordered extended until further orders of the Court. 63 issued Presidential Decree. (P.D.) No. 79,68 dated December 8, 1972,
which, among others, made "family planning a part of a broad educational
program," provided "family planning services as a part of over-all health
Thereafter, the Court directed the parties to submit their respective
care," and made "available all acceptable methods of contraception, except
memoranda within sixty (60) days and, at the same time posed several
abortion, to all Filipino citizens desirous of spacing, limiting or preventing
questions for their clarification on some contentions of the parties. 64
pregnancies."

The Status Quo Ante


Through the years, however, the use of contraceptives and family planning
methods evolved from being a component of demographic management,
(Population, Contraceptive and Reproductive Health Laws to one centered on the promotion of public health, particularly,
reproductive health.69 Under that policy, the country gave priority to one's
Prior to the RH Law right to freely choose the method of family planning to be adopted, in
conformity with its adherence to the commitments made in the
Long before the incipience of the RH Law, the country has allowed the International Conference on Population and Development. 70 Thus, on
sale, dispensation and distribution of contraceptive drugs and devices. As August 14, 2009, the country enacted R.A. No. 9710 or "The Magna Carta
far back as June 18, 1966, the country enacted R.A. No. 4729 entitled "An for Women, " which, among others, mandated the State to provide for
Act to Regu,late the Sale, Dispensation, and/or Distribution of comprehensive health services and programs for women, including family
Contraceptive Drugs and Devices." Although contraceptive drugs and planning and sex education.71
devices were allowed, they could not be sold, dispensed or distributed
"unless such sale, dispensation and distribution is by a duly licensed drug The RH Law
store or pharmaceutical company and with the prescription of a qualified
medical practitioner."65 Despite the foregoing legislative measures, the population of the country
kept on galloping at an uncontrollable pace. From a paltry number of just
In addition, R.A. No. 5921,66 approved on June 21, 1969, contained over 27 million Filipinos in 1960, the population of the country reached
provisions relative to "dispensing of abortifacients or anti-conceptional over 76 million in the year 2000 and over 92 million in 2010. 72 The
substances and devices." Under Section 37 thereof, it was provided that executive and the legislative, thus, felt that the measures were still not
"no drug or chemical product or device capable of provoking abortion or adequate. To rein in the problem, the RH Law was enacted to provide
preventing conception as classified by the Food and Drug Administration Filipinos, especially the poor and the marginalized, access and information
to the full range of modem family planning methods, and to ensure that its 1] Power of Judicial Review
objective to provide for the peoples' right to reproductive health be
achieved. To make it more effective, the RH Law made it mandatory for 2] Actual Case or Controversy
health providers to provide information on the full range of modem family
planning methods, supplies and services, and for schools to provide
3] Facial Challenge
reproductive health education. To put teeth to it, the RH Law criminalizes
certain acts of refusals to carry out its mandates.
4] Locus Standi
Stated differently, the RH Law is an enhancement measure to fortify and
make effective the current laws on contraception, women's health and 5] Declaratory Relief
population control.
6] One Subject/One Title Rule
Prayer of the Petitioners - Maintain the Status Quo
II. SUBSTANTIVE: Whether the RH law is unconstitutional:
The petitioners are one in praying that the entire RH Law be declared
unconstitutional. Petitioner ALFI, in particular, argues that the government 1] Right to Life
sponsored contraception program, the very essence of the RH Law,
violates the right to health of women and the sanctity of life, which the 2] Right to Health
State is mandated to protect and promote. Thus, ALFI prays that "the
status quo ante - the situation prior to the passage of the RH Law - must
be maintained."73 It explains: 3] Freedom of Religion and the Right to Free Speech

x x x. The instant Petition does not question contraception and 4] The Family
contraceptives per se. As provided under Republic Act No. 5921 and
Republic Act No. 4729, the sale and distribution of contraceptives are 5] Freedom of Expression and Academic Freedom
prohibited unless dispensed by a prescription duly licensed by a physician.
What the Petitioners find deplorable and repugnant under the RH Law is 6] Due Process
the role that the State and its agencies - the entire bureaucracy, from the
cabinet secretaries down to the barangay officials in the remotest areas of
7] Equal Protection
the country - is made to play in the implementation of the contraception
program to the fullest extent possible using taxpayers' money. The State
then will be the funder and provider of all forms of family planning 8] Involuntary Servitude
methods and the implementer of the program by ensuring the widespread
dissemination of, and universal access to, a full range of family planning 9] Delegation of Authority to the FDA
methods, devices and supplies.74
10] Autonomy of Local Govemments/ARMM
ISSUES
DISCUSSION
After a scrutiny of the various arguments and contentions of the parties,
the Court has synthesized and refined them to the following principal Before delving into the constitutionality of the RH Law and its
issues: implementing rules, it behooves the Court to resolve some procedural
impediments.
I. PROCEDURAL: Whether the Court may exercise its power of judicial
review over the controversy.
I. PROCEDURAL ISSUE: Whether the Court can exercise its power of In its relationship with its co-equals, the Judiciary recognizes the doctrine
judicial review over the controversy. of separation of powers which imposes upon the courts proper restraint,
born of the nature of their functions and of their respect for the other
The Power of Judicial Review branches of government, in striking down the acts of the Executive or the
Legislature as unconstitutional. Verily, the policy is a harmonious blend of
courtesy and caution.86
In its attempt to persuade the Court to stay its judicial hand, the OSG
asserts that it should submit to the legislative and political wisdom of
Congress and respect the compromises made in the crafting of the RH It has also long been observed, however, that in times of social
Law, it being "a product of a majoritarian democratic process" 75 and disquietude or political instability, the great landmarks of the Constitution
"characterized by an inordinate amount of transparency." 76 The OSG posits are apt to be forgotten or marred, if not entirely obliterated. 87 In order to
that the authority of the Court to review social legislation like the RH Law address this, the Constitution impresses upon the Court to respect the acts
by certiorari is "weak," since the Constitution vests the discretion to performed by a co-equal branch done within its sphere of competence and
implement the constitutional policies and positive norms with the political authority, but at the same time, allows it to cross the line of separation -
departments, in particular, with Congress.77 It further asserts that in view but only at a very limited and specific point - to determine whether the
of the Court's ruling in Southern Hemisphere v. Anti-Terrorism acts of the executive and the legislative branches are null because they
Council,78 the remedies of certiorari and prohibition utilized by the were undertaken with grave abuse of discretion. 88 Thus, while the Court
petitioners are improper to assail the validity of the acts of the may not pass upon questions of wisdom, justice or expediency of the RH
legislature.79 Law, it may do so where an attendant unconstitutionality or grave abuse
of discretion results.89 The Court must demonstrate its unflinching
commitment to protect those cherished rights and principles embodied in
Moreover, the OSG submits that as an "as applied challenge," it cannot
the Constitution.
prosper considering that the assailed law has yet to be enforced and
applied to the petitioners, and that the government has yet to distribute
reproductive health devices that are abortive. It claims that the RH Law In this connection, it bears adding that while the scope of judicial power of
cannot be challenged "on its face" as it is not a speech-regulating review may be limited, the Constitution makes no distinction as to the kind
measure.80 of legislation that may be subject to judicial scrutiny, be it in the form of
social legislation or otherwise. The reason is simple and goes back to the
earlier point. The Court may pass upon the constitutionality of acts of the
In many cases involving the determination of the constitutionality of the
legislative and the executive branches, since its duty is not to review their
actions of the Executive and the Legislature, it is often sought that the
collective wisdom but, rather, to make sure that they have acted in
Court temper its exercise of judicial power and accord due respect to the
consonance with their respective authorities and rights as mandated of
wisdom of its co-equal branch on the basis of the principle of separation of
them by the Constitution. If after said review, the Court finds no
powers. To be clear, the separation of powers is a fundamental principle in
constitutional violations of any sort, then, it has no more authority of
our system of government, which obtains not through express provision
proscribing the actions under review.90 This is in line with Article VIII,
but by actual division in our Constitution. Each department of the
Section 1 of the Constitution which expressly provides:
government has exclusive cognizance of matters within its jurisdiction and
is supreme within its own sphere.81
Section 1. The judicial power shall be vested in one Supreme Court and in
such lower courts as may be established by law.
Thus, the 1987 Constitution provides that: (a) the legislative power shall
be vested in the Congress of the Philippines; 82 (b) the executive power
shall be vested in the President of the Philippines; 83 and (c) the judicial Judicial power includes the duty of the courts of justice to settle actual
power shall be vested in one Supreme Court and in such lower courts as controversies involving rights which are legally demandable and
may be established by law.84 The Constitution has truly blocked out with enforceable, and to determine whether or not there has been a grave
deft strokes and in bold lines, the allotment of powers among the three abuse of discretion amounting to lack or excess of jurisdiction on the part
branches of government.85 of any branch or instrumentality of the Government. [Emphases supplied]
As far back as Tanada v. Angara,91 the Court has unequivocally declared not yet concrete and ripe for adjudication since no one has been charged
that certiorari, prohibition and mandamus are appropriate remedies to with violating any of its provisions and that there is no showing that any of
raise constitutional issues and to review and/or prohibit/nullify, when the petitioners' rights has been adversely affected by its operation. 98 In
proper, acts of legislative and executive officials, as there is no other plain, short, it is contended that judicial review of the RH Law is premature.
speedy or adequate remedy in the ordinary course of law. This ruling was
later on applied in Macalintal v. COMELEC,92 Aldaba v. An actual case or controversy means an existing case or controversy that
COMELEC,93 Magallona v. Ermita,94 and countless others. In Tanada, the is appropriate or ripe for determination, not conjectural or anticipatory,
Court wrote: lest the decision of the court would amount to an advisory opinion. 99 The
rule is that courts do not sit to adjudicate mere academic questions to
In seeking to nullify an act of the Philippine Senate on the ground that it satisfy scholarly interest, however intellectually challenging. The
contravenes the Constitution, the petition no doubt raises a justiciable controversy must be justiciable-definite and concrete, touching on the
controversy. Where an action of the legislative branch is seriously alleged legal relations of parties having adverse legal interests. In other words,
to have infringed the Constitution, it becomes not only the right but in fact the pleadings must show an active antagonistic assertion of a legal right,
the duty of the judiciary to settle the dispute. "The question thus posed is on the one hand, and a denial thereof, on the other; that is, it must
judicial rather than political. The duty (to adjudicate) remains to assure concern a real, tangible and not merely a theoretical question or issue.
that the supremacy of the Constitution is upheld. " Once a "controversy as There ought to be an actual and substantial controversy admitting of
to the application or interpretation of constitutional provision is raised specific relief through a decree conclusive in nature, as distinguished from
before this Court (as in the instant case), it becomes a legal issue which an opinion advising what the law would be upon a hypothetical state of
the Court is bound by constitutional mandate to decide. [Emphasis facts.100
supplied]
Corollary to the requirement of an actual case or controversy is the
In the scholarly estimation of former Supreme Court Justice Florentino requirement of ripeness.101 A question is ripe for adjudication when the act
Feliciano, "judicial review is essential for the maintenance and being challenged has had a direct adverse effect on the individual
enforcement of the separation of powers and the balancing of powers challenging it. For a case to be considered ripe for adjudication, it is a
among the three great departments of government through the definition prerequisite that something has then been accomplished or performed by
and maintenance of the boundaries of authority and control between them. either branch before a court may come into the picture, and the petitioner
To him, judicial review is the chief, indeed the only, medium of must allege the existence of an immediate or threatened injury to himself
participation - or instrument of intervention - of the judiciary in that as a result of the challenged action. He must show that he has sustained
balancing operation.95 or is immediately in danger of sustaining some direct injury as a result of
the act complained of102
Lest it be misunderstood, it bears emphasizing that the Court does not
have the unbridled authority to rule on just any and every claim of In The Province of North Cotabato v. The Government of the Republic of
constitutional violation. Jurisprudence is replete with the rule that the the Philippines,103 where the constitutionality of an unimplemented
power of judicial review is limited by four exacting requisites, viz : (a) Memorandum of Agreement on the Ancestral Domain (MOA-AD) was put in
there must be an actual case or controversy; (b) the petitioners must question, it was argued that the Court has no authority to pass upon the
possess locus standi; (c) the question of constitutionality must be raised at issues raised as there was yet no concrete act performed that could
the earliest opportunity; and (d) the issue of constitutionality must be the possibly violate the petitioners' and the intervenors' rights. Citing
lis mota of the case.96 precedents, the Court ruled that the fact of the law or act in question
being not yet effective does not negate ripeness. Concrete acts under a
Actual Case or Controversy law are not necessary to render the controversy ripe. Even a singular
violation of the Constitution and/or the law is enough to awaken judicial
duty.
Proponents of the RH Law submit that the subj ect petitions do not present
any actual case or controversy because the RH Law has yet to be
implemented.97 They claim that the questions raised by the petitions are
In this case, the Court is of the view that an actual case or controversy grave abuse of discretion amounting to lack or excess of jurisdiction on the
exists and that the same is ripe for judicial determination. Considering that part of any branch or instrumentality of the Government. 110 Verily, the
the RH Law and its implementing rules have already taken effect and that framers of Our Constitution envisioned a proactive Judiciary, ever vigilant
budgetary measures to carry out the law have already been passed, it is with its duty to maintain the supremacy of the Constitution.
evident that the subject petitions present a justiciable controversy. As
stated earlier, when an action of the legislative branch is seriously alleged Consequently, considering that the foregoing petitions have seriously
to have infringed the Constitution, it not only becomes a right, but also a alleged that the constitutional human rights to life, speech and religion and
duty of the Judiciary to settle the dispute.104 other fundamental rights mentioned above have been violated by the
assailed legislation, the Court has authority to take cognizance of these
Moreover, the petitioners have shown that the case is so because medical kindred petitions and to determine if the RH Law can indeed pass
practitioners or medical providers are in danger of being criminally constitutional scrutiny. To dismiss these petitions on the simple expedient
prosecuted under the RH Law for vague violations thereof, particularly that there exist no actual case or controversy, would diminish this Court as
public health officers who are threatened to be dismissed from the service a reactive branch of government, acting only when the Fundamental Law
with forfeiture of retirement and other benefits. They must, at least, be has been transgressed, to the detriment of the Filipino people.
heard on the matter NOW.
Locus Standi
Facial Challenge
The OSG also attacks the legal personality of the petitioners to file their
The OSG also assails the propriety of the facial challenge lodged by the respective petitions. It contends that the "as applied challenge" lodged by
subject petitions, contending that the RH Law cannot be challenged "on its the petitioners cannot prosper as the assailed law has yet to be enforced
face" as it is not a speech regulating measure. 105 and applied against them,111 and the government has yet to distribute
reproductive health devices that are abortive.112
The Court is not persuaded.
The petitioners, for their part, invariably invoke the "transcendental
In United States (US) constitutional law, a facial challenge, also known as importance" doctrine and their status as citizens and taxpayers in
a First Amendment Challenge, is one that is launched to assail the validity establishing the requisite locus standi.
of statutes concerning not only protected speech, but also all other rights
in the First Amendment.106 These include religious freedom, freedom of the Locus standi or legal standing is defined as a personal and substantial
press, and the right of the people to peaceably assemble, and to petition interest in a case such that the party has sustained or will sustain direct
the Government for a redress of grievances.107 After all, the fundamental injury as a result of the challenged governmental act. 113 It requires a
right to religious freedom, freedom of the press and peaceful assembly are personal stake in the outcome of the controversy as to assure the concrete
but component rights of the right to one's freedom of expression, as they adverseness which sharpens the presentation of issues upon which the
are modes which one's thoughts are externalized. court so largely depends for illumination of difficult constitutional
questions.114
In this jurisdiction, the application of doctrines originating from the U.S.
has been generally maintained, albeit with some modifications. While this In relation to locus standi, the "as applied challenge" embodies the rule
Court has withheld the application of facial challenges to strictly penal that one can challenge the constitutionality of a statute only if he asserts a
statues,108 it has expanded its scope to cover statutes not only regulating violation of his own rights. The rule prohibits one from challenging the
free speech, but also those involving religious freedom, and other constitutionality of the statute grounded on a violation of the rights of third
fundamental rights.109 The underlying reason for this modification is persons not before the court. This rule is also known as the prohibition
simple. For unlike its counterpart in the U.S., this Court, under its against third-party standing.115
expanded jurisdiction, is mandated by the Fundamental Law not only to
settle actual controversies involving rights which are legally demandable Transcendental Importance
and enforceable, but also to determine whether or not there has been a
Notwithstanding, the Court leans on the doctrine that "the rule on standing constitutional provisions on the right to life and health, the freedom of
is a matter of procedure, hence, can be relaxed for non-traditional religion and expression and other constitutional rights. Mindful of all these
plaintiffs like ordinary citizens, taxpayers, and legislators when the public and the fact that the issues of contraception and reproductive health have
interest so requires, such as when the matter is of transcendental already caused deep division among a broad spectrum of society, the
importance, of overreaching significance to society, or of paramount public Court entertains no doubt that the petitions raise issues of transcendental
interest."116 importance warranting immediate court adjudication. More importantly,
considering that it is the right to life of the mother and the unborn which is
In Coconut Oil Refiners Association, Inc. v. Torres, 117 the Court held that in primarily at issue, the Court need not wait for a life to be taken away
cases of paramount importance where serious constitutional questions are before taking action.
involved, the standing requirement may be relaxed and a suit may be
allowed to prosper even where there is no direct injury to the party The Court cannot, and should not, exercise judicial restraint at this time
claiming the right of judicial review. In the first Emergency Powers when rights enshrined in the Constitution are being imperilled to be
Cases,118 ordinary citizens and taxpayers were allowed to question the violated. To do so, when the life of either the mother or her child is at
constitutionality of several executive orders although they had only an stake, would lead to irreparable consequences.
indirect and general interest shared in common with the public.
Declaratory Relief
With these said, even if the constitutionality of the RH Law may not be
assailed through an "as-applied challenge, still, the Court has time and The respondents also assail the petitions because they are essentially
again acted liberally on the locus s tandi requirement. It has accorded petitions for declaratory relief over which the Court has no original
certain individuals standing to sue, not otherwise directly injured or with jurisdiction.120 Suffice it to state that most of the petitions are praying for
material interest affected by a Government act, provided a constitutional injunctive reliefs and so the Court would just consider them as petitions for
issue of transcendental importance is invoked. The rule on locus standi is, prohibition under Rule 65, over which it has original jurisdiction. Where the
after all, a procedural technicality which the Court has, on more than one case has far-reaching implications and prays for injunctive reliefs, the
occasion, waived or relaxed, thus allowing non-traditional plaintiffs, such Court may consider them as petitions for prohibition under Rule 65. 121
as concerned citizens, taxpayers, voters or legislators, to sue in the public
interest, albeit they may not have been directly injured by the operation of
One Subject-One Title
a law or any other government act. As held in Jaworski v. PAGCOR: 119

The petitioners also question the constitutionality of the RH Law, claiming


Granting arguendo that the present action cannot be properly treated as a
that it violates Section 26(1 ), Article VI of the Constitution, 122 prescribing
petition for prohibition, the transcendental importance of the issues
the one subject-one title rule. According to them, being one for
involved in this case warrants that we set aside the technical defects and
reproductive health with responsible parenthood, the assailed legislation
take primary jurisdiction over the petition at bar. One cannot deny that the
violates the constitutional standards of due process by concealing its true
issues raised herein have potentially pervasive influence on the social and
intent - to act as a population control measure. 123
moral well being of this nation, specially the youth; hence, their proper
and just determination is an imperative need. This is in accordance with
the well-entrenched principle that rules of procedure are not inflexible To belittle the challenge, the respondents insist that the RH Law is not a
tools designed to hinder or delay, but to facilitate and promote the birth or population control measure,124 and that the concepts of
administration of justice. Their strict and rigid application, which would "responsible parenthood" and "reproductive health" are both interrelated
result in technicalities that tend to frustrate, rather than promote as they are inseparable.125
substantial justice, must always be eschewed. (Emphasis supplied)
Despite efforts to push the RH Law as a reproductive health law, the Court
In view of the seriousness, novelty and weight as precedents, not only to sees it as principally a population control measure. The corpus of the RH
the public, but also to the bench and bar, the issues raised must be Law is geared towards the reduction of the country's population. While it
resolved for the guidance of all. After all, the RH Law drastically affects the claims to save lives and keep our women and children healthy, it also
promotes pregnancy-preventing products. As stated earlier, the RH Law
emphasizes the need to provide Filipinos, especially the poor and the population growth. As expressed in the first paragraph of Section 2 of the
marginalized, with access to information on the full range of modem family RH Law:
planning products and methods. These family planning methods, natural or
modem, however, are clearly geared towards the prevention of pregnancy. SEC. 2. Declaration of Policy. - The State recognizes and guarantees the
human rights of all persons including their right to equality and
For said reason, the manifest underlying objective of the RH Law is to nondiscrimination of these rights, the right to sustainable human
reduce the number of births in the country. development, the right to health which includes reproductive health, the
right to education and information, and the right to choose and make
It cannot be denied that the measure also seeks to provide pre-natal and decisions for themselves in accordance with their religious convictions,
post-natal care as well. A large portion of the law, however, covers the ethics, cultural beliefs, and the demands of responsible parenthood.
dissemination of information and provisions on access to medically-safe,
non-abortifacient, effective, legal, affordable, and quality reproductive The one subject/one title rule expresses the principle that the title of a law
health care services, methods, devices, and supplies, which are all must not be "so uncertain that the average person reading it would not be
intended to prevent pregnancy. informed of the purpose of the enactment or put on inquiry as to its
contents, or which is misleading, either in referring to or indicating one
The Court, thus, agrees with the petitioners' contention that the whole subject where another or different one is really embraced in the act, or in
idea of contraception pervades the entire RH Law. It is, in fact, the central omitting any expression or indication of the real subject or scope of the
idea of the RH Law.126 Indeed, remove the provisions that refer to act."129
contraception or are related to it and the RH Law loses its very
foundation.127 As earlier explained, "the other positive provisions such as Considering the close intimacy between "reproductive health" and
skilled birth attendance, maternal care including pre-and post-natal "responsible parenthood" which bears to the attainment of the goal of
services, prevention and management of reproductive tract infections achieving "sustainable human development" as stated under its terms, the
including HIV/AIDS are already provided for in the Magna Carta for Court finds no reason to believe that Congress intentionally sought to
Women."128 deceive the public as to the contents of the assailed legislation.

Be that as it may, the RH Law does not violate the one subject/one bill II - SUBSTANTIVE ISSUES:
rule. In Benjamin E. Cawaling, Jr. v. The Commission on Elections and
Rep. Francis Joseph G Escudero, it was written: 1-The Right to Life
Position of the Petitioners
It is well-settled that the "one title-one subject" rule does not require the
Congress to employ in the title of the enactment language of such The petitioners assail the RH Law because it violates the right to life and
precision as to mirror, fully index or catalogue all the contents and the health of the unborn child under Section 12, Article II of the Constitution.
minute details therein. The rule is sufficiently complied with if the title is The assailed legislation allowing access to abortifacients/abortives
comprehensive enough as to include the general object which the statute effectively sanctions abortion.130
seeks to effect, and where, as here, the persons interested are informed of
the nature, scope and consequences of the proposed law and its operation.
According to the petitioners, despite its express terms prohibiting abortion,
Moreover, this Court has invariably adopted a liberal rather than technical
Section 4(a) of the RH Law considers contraceptives that prevent the
construction of the rule "so as not to cripple or impede legislation."
fertilized ovum to reach and be implanted in the mother's womb as an
[Emphases supplied]
abortifacient; thus, sanctioning contraceptives that take effect after
fertilization and prior to implantation, contrary to the intent of the Framers
In this case, a textual analysis of the various provisions of the law shows of the Constitution to afford protection to the fertilized ovum which already
that both "reproductive health" and "responsible parenthood" are has life.
interrelated and germane to the overriding objective to control the
They argue that even if Section 9 of the RH Law allows only "non- The Court's Position
abortifacient" hormonal contraceptives, intrauterine devices, injectables
and other safe, legal, non-abortifacient and effective family planning It is a universally accepted principle that every human being enjoys the
products and supplies, medical research shows that contraceptives use right to life.137
results in abortion as they operate to kill the fertilized ovum which already
has life.131
Even if not formally established, the right to life, being grounded on
natural law, is inherent and, therefore, not a creation of, or dependent
As it opposes the initiation of life, which is a fundamental human good, the upon a particular law, custom, or belief. It precedes and transcends any
petitioners assert that the State sanction of contraceptive use contravenes authority or the laws of men.
natural law and is an affront to the dignity of man. 132
In this jurisdiction, the right to life is given more than ample protection.
Finally, it is contended that since Section 9 of the RH Law requires the Section 1, Article III of the Constitution provides:
Food and Drug Administration (FDA) to certify that the product or supply is
not to be used as an abortifacient, the assailed legislation effectively
Section 1. No person shall be deprived of life, liberty, or property without
confirms that abortifacients are not prohibited. Also considering that the
due process of law, nor shall any person be denied the equal protection of
FDA is not the agency that will actually supervise or administer the use of
the laws.
these products and supplies to prospective patients, there is no way it can
truthfully make a certification that it shall not be used for abortifacient
purposes.133 As expounded earlier, the use of contraceptives and family planning
methods in the Philippines is not of recent vintage. From the enactment of
R.A. No. 4729, entitled "An Act To Regulate The Sale, Dispensation, and/or
Position of the Respondents
Distribution of Contraceptive Drugs and Devices "on June 18, 1966,
prescribing rules on contraceptive drugs and devices which prevent
For their part, the defenders of the RH Law point out that the intent of the fertilization,138 to the promotion of male vasectomy and tubal
Framers of the Constitution was simply the prohibition of abortion. They ligation,139 and the ratification of numerous international agreements, the
contend that the RH Law does not violate the Constitution since the said country has long recognized the need to promote population control
law emphasizes that only "non-abortifacient" reproductive health care through the use of contraceptives in order to achieve long-term economic
services, methods, devices products and supplies shall be made accessible development. Through the years, however, the use of contraceptives and
to the public.134 other family planning methods evolved from being a component of
demographic management, to one centered on the promotion of public
According to the OSG, Congress has made a legislative determination that health, particularly, reproductive health. 140
contraceptives are not abortifacients by enacting the RH Law. As the RH
Law was enacted with due consideration to various studies and This has resulted in the enactment of various measures promoting
consultations with the World Health Organization (WHO) and other experts women's rights and health and the overall promotion of the family's well-
in the medical field, it is asserted that the Court afford deference and being. Thus, aside from R.A. No. 4729, R.A. No. 6365 or "The Population
respect to such a determination and pass judgment only when a particular Act of the Philippines" and R.A. No. 9710, otherwise known as the "The
drug or device is later on determined as an abortive. 135 Magna Carta of Women" were legislated. Notwithstanding this paradigm
shift, the Philippine national population program has always been
For his part, respondent Lagman argues that the constitutional protection grounded two cornerstone principles: "principle of no-abortion" and the
of one's right to life is not violated considering that various studies of the "principle of non-coercion."141 As will be discussed later, these principles
WHO show that life begins from the implantation of the fertilized ovum. are not merely grounded on administrative policy, but rather, originates
Consequently, he argues that the RH Law is constitutional since the law from the constitutional protection expressly provided to afford protection
specifically provides that only contraceptives that do not prevent the to life and guarantee religious freedom.
implantation of the fertilized ovum are allowed.136
When Life Begins*
Majority of the Members of the Court are of the position that the question a well-settled principle of constitutional construction that the language
of when life begins is a scientific and medical issue that should not be employed in the Constitution must be given their ordinary meaning except
decided, at this stage, without proper hearing and evidence. During the where technical terms are employed. As much as possible, the words of
deliberation, however, it was agreed upon that the individual members of the Constitution should be understood in the sense they have in common
the Court could express their own views on this matter. use. What it says according to the text of the provision to be construed
compels acceptance and negates the power of the courts to alter it, based
In this regard, the ponente, is of the strong view that life begins at on the postulate that the framers and the people mean what they say.
fertilization. Verba legis non est recedendum - from the words of a statute there should
be no departure.
In answering the question of when life begins, focus should be made on
the particular phrase of Section 12 which reads: The raison d' etre for the rule is essentially two-fold: First, because it is
assumed that the words in which constitutional provisions are couched
express the objective sought to be attained; and second, because the
Section 12. The State recognizes the sanctity of family life and shall
Constitution is not primarily a lawyer's document but essentially that of
protect and strengthen the family as a basic autonomous social institution.
the people, in whose consciousness it should ever be present as an
It shall equally protect the life of the mother and the life of the unborn
important condition for the rule of law to prevail.
from conception. The natural and primary right and duty of parents in the
rearing of the youth for civic efficiency and the development of moral
character shall receive the support of the Government. In conformity with the above principle, the traditional meaning of the word
"conception" which, as described and defined by all reliable and reputable
sources, means that life begins at fertilization.
Textually, the Constitution affords protection to the unborn from
conception. This is undisputable because before conception, there is no
unborn to speak of. For said reason, it is no surprise that the Constitution Webster's Third New International Dictionary describes it as the act of
is mute as to any proscription prior to conception or when life begins. The becoming pregnant, formation of a viable zygote; the fertilization that
problem has arisen because, amazingly, there are quarters who have results in a new entity capable of developing into a being like its
conveniently disregarded the scientific fact that conception is reckoned parents.145
from fertilization. They are waving the view that life begins at
implantation. Hence, the issue of when life begins. Black's Law Dictionary gives legal meaning to the term "conception" as the
fecundation of the female ovum by the male spermatozoon resulting in
In a nutshell, those opposing the RH Law contend that conception is human life capable of survival and maturation under normal conditions. 146
synonymous with "fertilization" of the female ovum by the male
sperm.142 On the other side of the spectrum are those who assert that Even in jurisprudence, an unborn child has already a legal personality. In
conception refers to the "implantation" of the fertilized ovum in the Continental Steel Manufacturing Corporation v. Hon. Accredited Voluntary
uterus.143 Arbitrator Allan S. Montano,147 it was written:

Plain and Legal Meaning Life is not synonymous with civil personality. One need not acquire civil
personality first before he/she could die. Even a child inside the womb
It is a canon in statutory construction that the words of the Constitution already has life. No less than the Constitution recognizes the life of the
should be interpreted in their plain and ordinary meaning. As held in the unborn from conception, that the State must protect equally with the life
recent case of Chavez v. Judicial Bar Council:144 of the mother. If the unborn already has life, then the cessation thereof
even prior to the child being delivered, qualifies as death. [Emphases in
the original]
One of the primary and basic rules in statutory construction is that where
the words of a statute are clear, plain, and free from ambiguity, it must be
given its literal meaning and applied without attempted interpretation. It is In Gonzales v. Carhart,148 Justice Anthony Kennedy, writing for the US
Supreme Court, said that the State "has respect for human life at all
stages in the pregnancy" and "a legitimate and substantial interest in chromosome count of 46 is found only - and I repeat, only in human cells.
preserving and promoting fetal life." Invariably, in the decision, the fetus Therefore, the fertilized ovum is human.
was referred to, or cited, as a baby or a child.149
Since these questions have been answered affirmatively, we must
Intent of the Framers conclude that if the fertilized ovum is both alive and human, then, as night
follows day, it must be human life. Its nature is human. 151
Records of the Constitutional Convention also shed light on the intention of
the Framers regarding the term "conception" used in Section 12, Article II Why the Constitution used the phrase "from the moment of conception"
of the Constitution. From their deliberations, it clearly refers to the and not "from the moment of fertilization" was not because of doubt when
moment of "fertilization." The records reflect the following: human life begins, but rather, because:

Rev. Rigos: In Section 9, page 3, there is a sentence which reads: Mr. Tingson: x x x x the phrase from the moment of conception" was
described by us here before with the scientific phrase "fertilized ovum"
"The State shall equally protect the life of the mother and the life of the may be beyond the comprehension of some people; we want to use the
unborn from the moment of conception." simpler phrase "from the moment of conception."152

When is the moment of conception? Thus, in order to ensure that the fertilized ovum is given ample protection
under the Constitution, it was discussed:
xxx
Rev. Rigos: Yes, we think that the word "unborn" is sufficient for the
purpose of writing a Constitution, without specifying "from the moment of
Mr. Villegas: As I explained in the sponsorship speech, it is when the ovum
conception."
is fertilized by the sperm that there is human life. x x x. 150

Mr. Davide: I would not subscribe to that particular view because


xxx
according to the Commissioner's own admission, he would leave it to
Congress to define when life begins. So, Congress can define life to begin
As to why conception is reckoned from fertilization and, as such, the from six months after fertilization; and that would really be very, very,
beginning of human life, it was explained: dangerous. It is now determined by science that life begins from the
moment of conception. There can be no doubt about it. So we should not
Mr. Villegas: I propose to review this issue in a biological manner. The first give any doubt to Congress, too.153
question that needs to be answered is: Is the fertilized ovum alive?
Biologically categorically says yes, the fertilized ovum is alive. First of all, Upon further inquiry, it was asked:
like all living organisms, it takes in nutrients which it processes by itself. It
begins doing this upon fertilization. Secondly, as it takes in these
Mr. Gascon: Mr. Presiding Officer, I would like to ask a question on that
nutrients, it grows from within. Thirdly, it multiplies itself at a geometric
point. Actually, that is one of the questions I was going to raise during the
rate in the continuous process of cell division. All these processes are vital
period of interpellations but it has been expressed already. The provision,
signs of life. Therefore, there is no question that biologically the fertilized
as proposed right now states:
ovum has life.

The State shall equally protect the life of the mother and the life of the
The second question: Is it human? Genetics gives an equally categorical
unborn from the moment of conception.
"yes." At the moment of conception, the nuclei of the ovum and the sperm
rupture. As this happens 23 chromosomes from the ovum combine with 23
chromosomes of the sperm to form a total of 46 chromosomes. A When it speaks of "from the moment of conception," does this mean when
the egg meets the sperm?
Mr. Villegas: Yes, the ovum is fertilized by the sperm. As emphasized by the Framers of the Constitution:

Mr. Gascon: Therefore that does not leave to Congress the right to xxx xxx xxx
determine whether certain contraceptives that we know today are
abortifacient or not because it is a fact that some of the so-called Mr. Gascon: xx xx. As I mentioned in my speech on the US bases, I am
contraceptives deter the rooting of the ovum in the uterus. If fertilization pro-life, to the point that I would like not only to protect the life of the
has already occurred, the next process is for the fertilized ovum to travel unborn, but also the lives of the millions of people in the world by fighting
towards the uterus and to take root. What happens with some for a nuclear-free world. I would just like to be assured of the legal and
contraceptives is that they stop the opportunity for the fertilized ovum to pragmatic implications of the term "protection of the life of the unborn
reach the uterus. Therefore, if we take the provision as it is proposed, from the moment of conception." I raised some of these implications this
these so called contraceptives should be banned. afternoon when I interjected in the interpellation of Commissioner
Regalado. I would like to ask that question again for a categorical answer.
Mr. Villegas: Yes, if that physical fact is established, then that is what is
called abortifacient and, therefore, would be unconstitutional and should I mentioned that if we institutionalize the term "the life of the unborn from
be banned under this provision. the moment of conception" we are also actually saying "no," not "maybe,"
to certain contraceptives which are already being encouraged at this point
Mr. Gascon: Yes. So my point is that I do not think it is up to Congress to in time. Is that the sense of the committee or does it disagree with me?
state whether or not these certain contraceptives are abortifacient.
Scientifically and based on the provision as it is now proposed, they are Mr. Azcuna: No, Mr. Presiding Officer, because contraceptives would be
already considered abortifacient.154 preventive. There is no unborn yet. That is yet unshaped.

From the deliberations above-quoted, it is apparent that the Framers of Mr. Gascon: Yes, Mr. Presiding Officer, but I was speaking more about
the Constitution emphasized that the State shall provide equal protection some contraceptives, such as the intra-uterine device which actually stops
to both the mother and the unborn child from the earliest opportunity of the egg which has already been fertilized from taking route to the uterus.
life, that is, upon fertilization or upon the union of the male sperm and the So if we say "from the moment of conception," what really occurs is that
female ovum. It is also apparent is that the Framers of the Constitution some of these contraceptives will have to be unconstitutionalized.
intended that to prohibit Congress from enacting measures that would
allow it determine when life begins.
Mr. Azcuna: Yes, to the extent that it is after the fertilization.

Equally apparent, however, is that the Framers of the Constitution did not
Mr. Gascon: Thank you, Mr. Presiding Officer. 156
intend to ban all contraceptives for being unconstitutional. In fact,
Commissioner Bernardo Villegas, spearheading the need to have a
constitutional provision on the right to life, recognized that the The fact that not all contraceptives are prohibited by the 1987 Constitution
determination of whether a contraceptive device is an abortifacient is a is even admitted by petitioners during the oral arguments. There it was
question of fact which should be left to the courts to decide on based on conceded that tubal ligation, vasectomy, even condoms are not classified
established evidence.155 as abortifacients.157

From the discussions above, contraceptives that kill or destroy the Atty. Noche:
fertilized ovum should be deemed an abortive and thus prohibited.
Conversely, contraceptives that actually prevent the union of the male Before the union of the eggs, egg and the sperm, there is no life yet.
sperm and the female ovum, and those that similarly take action prior to
fertilization should be deemed non-abortive, and thus, constitutionally Justice Bersamin:
permissible.
There is no life. Medical Meaning

Atty. Noche: That conception begins at fertilization is not bereft of medical foundation.
Mosby s Medical, Nursing, and Allied Health Dictionary defines conception
So, there is no life to be protected. as "the beginning of pregnancy usually taken to be the instant a
spermatozoon enters an ovum and forms a viable zygote." 159
Justice Bersamin:
It describes fertilization as "the union of male and female gametes to form
a zygote from which the embryo develops."160
To be protected.

The Textbook of Obstetrics (Physiological & Pathological


Atty. Noche:
Obstetrics),161 used by medical schools in the Philippines, also concludes
that human life (human person) begins at the moment of fertilization with
Under Section 12, yes. the union of the egg and the sperm resulting in the formation of a new
individual, with a unique genetic composition that dictates all
Justice Bersamin: developmental stages that ensue.

So you have no objection to condoms? Similarly, recent medical research on the matter also reveals that: "Human
development begins after the union of male and female gametes or germ
Atty. Noche: cells during a process known as fertilization (conception). Fertilization is a
sequence of events that begins with the contact of a sperm
(spermatozoon) with a secondary oocyte (ovum) and ends with the fusion
Not under Section 12, Article II. of their pronuclei (the haploid nuclei of the sperm and ovum) and the
mingling of their chromosomes to form a new cell. This fertilized ovum,
Justice Bersamin: known as a zygote, is a large diploid cell that is the beginning, or
primordium, of a human being."162
Even if there is already information that condoms sometimes have
porosity? The authors of Human Embryology & Teratology163 mirror the same
position. They wrote: "Although life is a continuous process, fertilization is
Atty. Noche: a critical landmark because, under ordinary circumstances, a new,
genetically distinct human organism is thereby formed.... The combination
of 23 chromosomes present in each pronucleus results in 46 chromosomes
Well, yes, Your Honor, there are scientific findings to that effect, Your
in the zygote. Thus the diploid number is restored and the embryonic
Honor, but I am discussing here Section 12, Article II, Your Honor, yes.
genome is formed. The embryo now exists as a genetic unity."

Justice Bersamin:
In support of the RH Bill, The Philippine Medical Association came out with
a "Paper on the Reproductive Health Bill (Responsible Parenthood Bill)"
Alright. and therein concluded that:

Atty. Noche: CONCLUSION

And it's not, I have to admit it's not an abortifacient, Your Honor. 158 The PMA throws its full weight in supporting the RH Bill at the same time
that PMA maintains its strong position that fertilization is sacred because it
is at this stage that conception, and thus human life, begins. Human lives If such theory would be accepted, it would unnervingly legitimize the
are sacred from the moment of conception, and that destroying those new utilization of any drug or device that would prevent the implantation of the
lives is never licit, no matter what the purported good outcome would be. fetus at the uterine wall. It would be provocative and further aggravate
In terms of biology and human embryology, a human being begins religious-based divisiveness.
immediately at fertilization and after that, there is no point along the
continuous line of human embryogenesis where only a "potential" human It would legally permit what the Constitution proscribes - abortion and
being can be posited. Any philosophical, legal, or political conclusion abortifacients.
cannot escape this objective scientific fact.
The RH Law and Abortion
The scientific evidence supports the conclusion that a zygote is a human
organism and that the life of a new human being commences at a
The clear and unequivocal intent of the Framers of the 1987 Constitution
scientifically well defined "moment of conception." This conclusion is
in protecting the life of the unborn from conception was to prevent the
objective, consistent with the factual evidence, and independent of any
Legislature from enacting a measure legalizing abortion. It was so clear
specific ethical, moral, political, or religious view of human life or of human
that even the Court cannot interpret it otherwise. This intent of the
embryos.164
Framers was captured in the record of the proceedings of the 1986
Constitutional Commission. Commissioner Bernardo Villegas, the principal
Conclusion: The Moment of Conception is Reckoned from proponent of the protection of the unborn from conception, explained:
Fertilization
The intention .. .is to make sure that there would be no pro-abortion laws
In all, whether it be taken from a plain meaning, or understood under ever passed by Congress or any pro-abortion decision passed by the
medical parlance, and more importantly, following the intention of the Supreme Court.169
Framers of the Constitution, the undeniable conclusion is that a zygote is a
human organism and that the life of a new human being commences at a
A reading of the RH Law would show that it is in line with this intent and
scientifically well-defined moment of conception, that is, upon fertilization.
actually proscribes abortion. While the Court has opted not to make any
determination, at this stage, when life begins, it finds that the RH Law
For the above reasons, the Court cannot subscribe to the theory advocated itself clearly mandates that protection be afforded from the moment of
by Hon. Lagman that life begins at implantation.165 According to him, fertilization. As pointed out by Justice Carpio, the RH Law is replete with
"fertilization and conception are two distinct and successive stages in the provisions that embody the policy of the law to protect to the fertilized
reproductive process. They are not identical and synonymous." 166 Citing a ovum and that it should be afforded safe travel to the uterus for
letter of the WHO, he wrote that "medical authorities confirm that the implantation.170
implantation of the fertilized ovum is the commencement of conception
and it is only after implantation that pregnancy can be medically
Moreover, the RH Law recognizes that abortion is a crime under Article
detected."167
256 of the Revised Penal Code, which penalizes the destruction or
expulsion of the fertilized ovum. Thus:
This theory of implantation as the beginning of life is devoid of any legal or
scientific mooring. It does not pertain to the beginning of life but to the
1] xx x.
viability of the fetus. The fertilized ovum/zygote is not an inanimate object
- it is a living human being complete with DNA and 46
chromosomes.168 Implantation has been conceptualized only for Section 4. Definition of Terms. - For the purpose of this Act, the following
convenience by those who had population control in mind. To adopt it terms shall be defined as follows:
would constitute textual infidelity not only to the RH Law but also to the
Constitution. xxx.

Not surprisingly, even the OSG does not support this position.
(q) Reproductive health care refers to the access to a full range of (a) Abortifacient refers to any drug or device that induces abortion or the
methods, facilities, services and supplies that contribute to reproductive destruction of a fetus inside the mother's womb or the prevention of the
health and well-being by addressing reproductive health-related problems. fertilized ovum to reach and be implanted in the mother's womb upon
It also includes sexual health, the purpose of which is the enhancement of determination of the FDA.
life and personal relations. The elements of reproductive health care
include the following: As stated above, the RH Law mandates that protection must be afforded
from the moment of fertilization. By using the word " or," the RH Law
xxx. prohibits not only drugs or devices that prevent implantation, but also
those that induce abortion and those that induce the destruction of a fetus
(3) Proscription of abortion and management of abortion complications; inside the mother's womb. Thus, an abortifacient is any drug or device
that either:
xxx.
(a) Induces abortion; or
2] xx x.
(b) Induces the destruction of a fetus inside the mother's womb;
or
Section 4. x x x.

(c) Prevents the fertilized ovum to reach and be implanted in the


(s) Reproductive health rights refers to the rights of individuals and
mother's womb, upon determination of the FDA.
couples, to decide freely and responsibly whether or not to have children;
the number, spacing and timing of their children; to make other decisions
concerning reproduction, free of discrimination, coercion and violence; to Contrary to the assertions made by the petitioners, the Court finds that
have the information and means to do so; and to attain the highest the RH Law, consistent with the Constitution, recognizes that the fertilized
standard of sexual health and reproductive health: Provided, however, ovum already has life and that the State has a bounden duty to protect it.
That reproductive health rights do not include abortion, and access to The conclusion becomes clear because the RH Law, first, prohibits any
abortifacients. drug or device that induces abortion (first kind), which, as discussed
exhaustively above, refers to that which induces the killing or the
destruction of the fertilized ovum, and, second, prohibits any drug or
3] xx x.
device the fertilized ovum to reach and be implanted in the mother's womb
(third kind).
SEC. 29. Repealing Clause. - Except for prevailing laws against abortion,
any law, presidential decree or issuance, executive order, letter of
By expressly declaring that any drug or device that prevents the fertilized
instruction, administrative order, rule or regulation contrary to or is
ovum to reach and be implanted in the mother's womb is an abortifacient
inconsistent with the provisions of this Act including Republic Act No.
(third kind), the RH Law does not intend to mean at all that life only
7392, otherwise known as the Midwifery Act, is hereby repealed, modified
begins only at implantation, as Hon. Lagman suggests. It also does not
or amended accordingly.
declare either that protection will only be given upon implantation, as the
petitioners likewise suggest. Rather, it recognizes that: one, there is a
The RH Law and Abortifacients need to protect the fertilized ovum which already has life, and two, the
fertilized ovum must be protected the moment it becomes existent - all
In carrying out its declared policy, the RH Law is consistent in prohibiting the way until it reaches and implants in the mother's womb. After all, if life
abortifacients. To be clear, Section 4(a) of the RH Law defines an is only recognized and afforded protection from the moment the fertilized
abortifacient as: ovum implants - there is nothing to prevent any drug or device from killing
or destroying the fertilized ovum prior to implantation.
Section 4. Definition of Terms - x x x x
From the foregoing, the Court finds that inasmuch as it affords protection (a) Abortifacient refers to any drug or device that induces abortion or the
to the fertilized ovum, the RH Law does not sanction abortion. To repeat, it destruction of a fetus inside the mother's womb or the prevention of the
is the Court's position that life begins at fertilization, not at implantation. fertilized ovum to reach and be implanted in the mother's womb upon
When a fertilized ovum is implanted in the uterine wall , its viability is determination of the FDA.
sustained but that instance of implantation is not the point of beginning of
life. It started earlier. And as defined by the RH Law, any drug or device Section 3.0l (a) of the IRR, however, redefines "abortifacient" as:
that induces abortion, that is, which kills or destroys the fertilized ovum or
prevents the fertilized ovum to reach and be implanted in the mother's
Section 3.01 For purposes of these Rules, the terms shall be defined as
womb, is an abortifacient.
follows:

Proviso Under Section 9 of the RH Law


a) Abortifacient refers to any drug or device that primarily induces
abortion or the destruction of a fetus inside the mother's womb or the
This notwithstanding, the Court finds that the proviso under Section 9 of prevention of the fertilized ovum to reach and be implanted in the
the law that "any product or supply included or to be included in the EDL mother's womb upon determination of the Food and Drug Administration
must have a certification from the FDA that said product and supply is (FDA). [Emphasis supplied]
made available on the condition that it is not to be used as an
abortifacient" as empty as it is absurd. The FDA, with all its expertise,
Again in Section 3.0lG) of the RH-IRR, "contraceptive," is redefined, viz:
cannot fully attest that a drug or device will not all be used as an
abortifacient, since the agency cannot be present in every instance when
the contraceptive product or supply will be used. 171 j) Contraceptive refers to any safe, legal, effective and scientifically proven
modern family planning method, device, or health product, whether
natural or artificial, that prevents pregnancy but does not primarily destroy
Pursuant to its declared policy of providing access only to safe, legal and
a fertilized ovum or prevent a fertilized ovum from being implanted in the
non-abortifacient contraceptives, however, the Court finds that the proviso
mother's womb in doses of its approved indication as determined by the
of Section 9, as worded, should bend to the legislative intent and mean
Food and Drug Administration (FDA).
that "any product or supply included or to be included in the EDL must
have a certification from the FDA that said product and supply is made
available on the condition that it cannot be used as abortifacient." Such a The above-mentioned section of the RH-IRR allows "contraceptives" and
construction is consistent with the proviso under the second paragraph of recognizes as "abortifacient" only those that primarily induce abortion or
the same section that provides: the destruction of a fetus inside the mother's womb or the prevention of
the fertilized ovum to reach and be implanted in the mother's womb. 172
Provided, further, That the foregoing offices shall not purchase or acquire
by any means emergency contraceptive pills, postcoital pills, abortifacients This cannot be done.
that will be used for such purpose and their other forms or equivalent.
In this regard, the observations of Justice Brion and Justice Del Castillo are
Abortifacients under the RH-IRR well taken. As they pointed out, with the insertion of the word "primarily,"
Section 3.0l(a) and G) of the RH-IRR173 must be struck down for being
ultra vires.
At this juncture, the Court agrees with ALFI that the authors of the RH-IRR
gravely abused their office when they redefined the meaning of
abortifacient. The RH Law defines "abortifacient" as follows: Evidently, with the addition of the word "primarily," in Section 3.0l(a) and
G) of the RH-IRR is indeed ultra vires. It contravenes Section 4(a) of the
RH Law and should, therefore, be declared invalid. There is danger that
SEC. 4. Definition of Terms. - For the purpose of this Act, the following
the insertion of the qualifier "primarily" will pave the way for the approval
terms shall be defined as follows:
of contraceptives which may harm or destroy the life of the unborn from
conception/fertilization in violation of Article II, Section 12 of the
Constitution. With such qualification in the RH-IRR, it appears to insinuate contraceptives as compared to women who never use them. They point
that a contraceptive will only be considered as an "abortifacient" if its sole out that the risk is decreased when the use of contraceptives is
known effect is abortion or, as pertinent here, the prevention of the discontinued. Further, it is contended that the use of combined oral
implantation of the fertilized ovum. contraceptive pills is associated with a threefold increased risk of venous
thromboembolism, a twofold increased risk of ischematic stroke, and an
For the same reason, this definition of "contraceptive" would permit the indeterminate effect on risk of myocardial infarction. 177 Given the definition
approval of contraceptives which are actually abortifacients because of of "reproductive health" and "sexual health" under Sections 4(p) 178 and
their fail-safe mechanism.174 (w)179 of the RH Law, the petitioners assert that the assailed legislation
only seeks to ensure that women have pleasurable and satisfying sex
lives.180
Also, as discussed earlier, Section 9 calls for the certification by the FDA
that these contraceptives cannot act as abortive. With this, together with
the definition of an abortifacient under Section 4 (a) of the RH Law and its The OSG, however, points out that Section 15, Article II of the
declared policy against abortion, the undeniable conclusion is that Constitution is not self-executory, it being a mere statement of the
contraceptives to be included in the PNDFS and the EDL will not only be administration's principle and policy. Even if it were self-executory, the
those contraceptives that do not have the primary action of causing OSG posits that medical authorities refute the claim that contraceptive
abortion or the destruction of a fetus inside the mother's womb or the pose a danger to the health of women.181
prevention of the fertilized ovum to reach and be implanted in the
mother's womb, but also those that do not have the secondary action of The Court's Position
acting the same way.
A component to the right to life is the constitutional right to health. In this
Indeed, consistent with the constitutional policy prohibiting abortion, and regard, the Constitution is replete with provisions protecting and
in line with the principle that laws should be construed in a manner that its promoting the right to health. Section 15, Article II of the Constitution
constitutionality is sustained, the RH Law and its implementing rules must provides:
be consistent with each other in prohibiting abortion. Thus, the word "
primarily" in Section 3.0l(a) and G) of the RH-IRR should be declared void. Section 15. The State shall protect and promote the right to health of the
To uphold the validity of Section 3.0l(a) and G) of the RH-IRR and prohibit people and instill health consciousness among them.
only those contraceptives that have the primary effect of being an abortive
would effectively "open the floodgates to the approval of contraceptives
A portion of Article XIII also specifically provides for the States' duty to
which may harm or destroy the life of the unborn from
provide for the health of the people, viz:
conception/fertilization in violation of Article II, Section 12 of the
Constitution."175
HEALTH
To repeat and emphasize, in all cases, the "principle of no abortion"
embodied in the constitutional protection of life must be upheld. Section 11. The State shall adopt an integrated and comprehensive
approach to health development which shall endeavor to make essential
goods, health and other social services available to all the people at
2-The Right to Health
affordable cost. There shall be priority for the needs of the
underprivileged, sick, elderly, disabled, women, and children. The State
The petitioners claim that the RH Law violates the right to health because shall endeavor to provide free medical care to paupers.
it requires the inclusion of hormonal contraceptives, intrauterine devices,
injectables and family products and supplies in the National Drug
Section 12. The State shall establish and maintain an effective food and
Formulary and the inclusion of the same in the regular purchase of
drug regulatory system and undertake appropriate health, manpower
essential medicines and supplies of all national hospitals. 176Citing various
development, and research, responsive to the country's health needs and
studies on the matter, the petitioners posit that the risk of developing
problems.
breast and cervical cancer is greatly increased in women who use oral
Section 13. The State shall establish a special agency for disabled person proliferation of contraceptives since the sale, distribution and dispensation
for their rehabilitation, self-development, and self-reliance, and their of contraceptive drugs and devices will still require the prescription of a
integration into the mainstream of society. licensed physician. With R.A. No. 4729 in place, there exists adequate
safeguards to ensure the public that only contraceptives that are safe are
Finally, Section 9, Article XVI provides: made available to the public. As aptly explained by respondent Lagman:

Section 9. The State shall protect consumers from trade malpractices and D. Contraceptives cannot be
from substandard or hazardous products. dispensed and used without
prescription
Contrary to the respondent's notion, however, these provisions are self-
executing. Unless the provisions clearly express the contrary, the 108. As an added protection to voluntary users of contraceptives, the
provisions of the Constitution should be considered self-executory. There is same cannot be dispensed and used without prescription.
no need for legislation to implement these self-executing provisions. 182 In
Manila Prince Hotel v. GSIS,183 it was stated: 109. Republic Act No. 4729 or "An Act to Regulate the Sale, Dispensation,
and/ or Distribution of Contraceptive Drugs and Devices" and Republic Act
x x x Hence, unless it is expressly provided that a legislative act is No. 5921 or "An Act Regulating the Practice of Pharmacy and Setting
necessary to enforce a constitutional mandate, the presumption now is Standards of Pharmaceutical Education in the Philippines and for Other
that all provisions of the constitution are self-executing. If the Purposes" are not repealed by the RH Law and the provisions of said Acts
constitutional provisions are treated as requiring legislation instead of self- are not inconsistent with the RH Law.
executing, the legislature would have the power to ignore and practically
nullify the mandate of the fundamental law. This can be cataclysmic. That 110. Consequently, the sale, distribution and dispensation of contraceptive
is why the prevailing view is, as it has always been, that – drugs and devices are particularly governed by RA No. 4729 which
provides in full:
... in case of doubt, the Constitution should be considered self-executing
rather than non-self-executing. . . . Unless the contrary is clearly intended, "Section 1. It shall be unlawful for any person, partnership, or corporation,
the provisions of the Constitution should be considered self-executing, as a to sell, dispense or otherwise distribute whether for or without
contrary rule would give the legislature discretion to determine when, or consideration, any contraceptive drug or device, unless such sale,
whether, they shall be effective. These provisions would be subordinated dispensation or distribution is by a duly licensed drug store or
to the will of the lawmaking body, which could make them entirely pharmaceutical company and with the prescription of a qualified medical
meaningless by simply refusing to pass the needed implementing statute. practitioner.
(Emphases supplied)
"Sec. 2 . For the purpose of this Act:
This notwithstanding, it bears mentioning that the petitioners, particularly
ALFI, do not question contraception and contraceptives per se. 184 In fact, "(a) "Contraceptive drug" is any medicine, drug, chemical, or
ALFI prays that the status quo - under R.A. No. 5921 and R.A. No. 4729, portion which is used exclusively for the purpose of preventing
the sale and distribution of contraceptives are not prohibited when they fertilization of the female ovum: and
are dispensed by a prescription of a duly licensed by a physician - be
maintained.185
"(b) "Contraceptive device" is any instrument, device, material, or
agent introduced into the female reproductive system for the
The legislative intent in the enactment of the RH Law in this regard is to primary purpose of preventing conception.
leave intact the provisions of R.A. No. 4729. There is no intention at all to
do away with it. It is still a good law and its requirements are still in to be
"Sec. 3 Any person, partnership, or corporation, violating the provisions of
complied with. Thus, the Court agrees with the observation of respondent
this Act shall be punished with a fine of not more than five hundred pesos
Lagman that the effectivity of the RH Law will not lead to the unmitigated
or an imprisonment of not less than six months or more than one year or Provided, That LGUs may implement its own procurement, distribution and
both in the discretion of the Court. monitoring program consistent with the overall provisions of this Act and
the guidelines of the DOH.
"This Act shall take effect upon its approval.
Thus, in the distribution by the DOH of contraceptive drugs and devices, it
"Approved: June 18, 1966" must consider the provisions of R.A. No. 4729, which is still in effect, and
ensure that the contraceptives that it will procure shall be from a duly
licensed drug store or pharmaceutical company and that the actual
111. Of the same import, but in a general manner, Section 25 of RA No.
dispensation of these contraceptive drugs and devices will done following a
5921 provides:
prescription of a qualified medical practitioner. The distribution of
contraceptive drugs and devices must not be indiscriminately done. The
"Section 25. Sale of medicine, pharmaceuticals, drugs and devices. No public health must be protected by all possible means. As pointed out by
medicine, pharmaceutical, or drug of whatever nature and kind or device Justice De Castro, a heavy responsibility and burden are assumed by the
shall be compounded, dispensed, sold or resold, or otherwise be made government in supplying contraceptive drugs and devices, for it may be
available to the consuming public except through a prescription drugstore held accountable for any injury, illness or loss of life resulting from or
or hospital pharmacy, duly established in accordance with the provisions of incidental to their use.187
this Act.
At any rate, it bears pointing out that not a single contraceptive has yet
112. With all of the foregoing safeguards, as provided for in the RH Law been submitted to the FDA pursuant to the RH Law. It behooves the Court
and other relevant statutes, the pretension of the petitioners that the RH to await its determination which drugs or devices are declared by the FDA
Law will lead to the unmitigated proliferation of contraceptives, whether as safe, it being the agency tasked to ensure that food and medicines
harmful or not, is completely unwarranted and baseless. 186 [Emphases in available to the public are safe for public consumption. Consequently, the
the Original. Underlining supplied.] Court finds that, at this point, the attack on the RH Law on this ground is
premature. Indeed, the various kinds of contraceptives must first be
In Re: Section 10 of the RH Law: measured up to the constitutional yardstick as expounded herein, to be
determined as the case presents itself.
The foregoing safeguards should be read in connection with Section 10 of
the RH Law which provides: At this point, the Court is of the strong view that Congress cannot legislate
that hormonal contraceptives and intra-uterine devices are safe and non-
SEC. 10. Procurement and Distribution of Family Planning Supplies. - The abortifacient. The first sentence of Section 9 that ordains their inclusion by
DOH shall procure, distribute to LGUs and monitor the usage of family the National Drug Formulary in the EDL by using the mandatory "shall" is
planning supplies for the whole country. The DOH shall coordinate with all to be construed as operative only after they have been tested, evaluated,
appropriate local government bodies to plan and implement this and approved by the FDA. The FDA, not Congress, has the expertise to
procurement and distribution program. The supply and budget allotments determine whether a particular hormonal contraceptive or intrauterine
shall be based on, among others, the current levels and projections of the device is safe and non-abortifacient. The provision of the third sentence
following: concerning the requirements for the inclusion or removal of a particular
family planning supply from the EDL supports this construction.

(a) Number of women of reproductive age and couples who want


to space or limit their children; Stated differently, the provision in Section 9 covering the inclusion of
hormonal contraceptives, intra-uterine devices, injectables, and other safe,
legal, non-abortifacient and effective family planning products and supplies
(b) Contraceptive prevalence rate, by type of method used; and by the National Drug Formulary in the EDL is not mandatory. There must
first be a determination by the FDA that they are in fact safe, legal, non-
(c) Cost of family planning supplies. abortifacient and effective family planning products and supplies. There
can be no predetermination by Congress that the gamut of contraceptives conscientious objector in Section 23 (a)(3) the option to refer a patient
are "safe, legal, non-abortifacient and effective" without the proper seeking reproductive health services and information - no escape is
scientific examination. afforded the conscientious objector in Section 23 (a)(l) and (2), i.e.
against a patient seeking reproductive health procedures. They claim that
3 -Freedom of Religion the right of other individuals to conscientiously object, such as: a) those
and the Right to Free Speech working in public health facilities referred to in Section 7; b) public officers
involved in the implementation of the law referred to in Section 23(b );
and c) teachers in public schools referred to in Section 14 of the RH Law,
Position of the Petitioners:
are also not recognize.191

1. On Contraception
Petitioner Echavez and the other medical practitioners meanwhile, contend
that the requirement to refer the matter to another health care service
While contraceptives and procedures like vasectomy and tubal ligation are provider is still considered a compulsion on those objecting healthcare
not covered by the constitutional proscription, there are those who, service providers. They add that compelling them to do the act against
because of their religious education and background, sincerely believe that their will violates the Doctrine of Benevolent Neutrality. Sections 9, 14 and
contraceptives, whether abortifacient or not, are evil. Some of these are 1 7 of the law are too secular that they tend to disregard the religion of
medical practitioners who essentially claim that their beliefs prohibit not Filipinos. Authorizing the use of contraceptives with abortive effects,
only the use of contraceptives but also the willing participation and mandatory sex education, mandatory pro-bono reproductive health
cooperation in all things dealing with contraceptive use. Petitioner PAX services to indigents encroach upon the religious freedom of those upon
explained that "contraception is gravely opposed to marital chastity, it is whom they are required.192
contrary to the good of the transmission of life, and to the reciprocal self-
giving of the spouses; it harms true love and denies the sovereign rule of
Petitioner CFC also argues that the requirement for a conscientious
God in the transmission of Human life."188
objector to refer the person seeking reproductive health care services to
another provider infringes on one's freedom of religion as it forces the
The petitioners question the State-sponsored procurement of objector to become an unwilling participant in the commission of a serious
contraceptives, arguing that the expenditure of their taxes on sin under Catholic teachings. While the right to act on one's belief may be
contraceptives violates the guarantee of religious freedom since regulated by the State, the acts prohibited by the RH Law are passive acts
contraceptives contravene their religious beliefs. 189 which produce neither harm nor injury to the public. 193

2. On Religious Accommodation and Petitioner CFC adds that the RH Law does not show compelling state
The Duty to Refer interest to justify regulation of religious freedom because it mentions no
emergency, risk or threat that endangers state interests. It does not
Petitioners Imbong and Luat note that while the RH Law attempts to explain how the rights of the people (to equality, non-discrimination of
address religious sentiments by making provisions for a conscientious rights, sustainable human development, health, education, information,
objector, the constitutional guarantee is nonetheless violated because the choice and to make decisions according to religious convictions, ethics,
law also imposes upon the conscientious objector the duty to refer the cultural beliefs and the demands of responsible parenthood) are being
patient seeking reproductive health services to another medical threatened or are not being met as to justify the impairment of religious
practitioner who would be able to provide for the patient's needs. For the freedom.194
petitioners, this amounts to requiring the conscientious objector to
cooperate with the very thing he refuses to do without violating his/her Finally, the petitioners also question Section 15 of the RH Law requiring
religious beliefs.190 would-be couples to attend family planning and responsible parenthood
seminars and to obtain a certificate of compliance. They claim that the
They further argue that even if the conscientious objector's duty to refer is provision forces individuals to participate in the implementation of the RH
recognized, the recognition is unduly limited, because although it allows a Law even if it contravenes their religious beliefs. 195 As the assailed law
dangles the threat of penalty of fine and/or imprisonment in case of non- Regarding mandatory family planning seminars under Section 15 , the
compliance with its provisions, the petitioners claim that the RH Law respondents claim that it is a reasonable regulation providing an
forcing them to provide, support and facilitate access and information to opportunity for would-be couples to have access to information regarding
contraception against their beliefs must be struck down as it runs afoul to parenthood, family planning, breastfeeding and infant nutrition. It is
the constitutional guarantee of religious freedom. argued that those who object to any information received on account of
their attendance in the required seminars are not compelled to accept
The Respondents' Positions information given to them. They are completely free to reject any
information they do not agree with and retain the freedom to decide on
matters of family life without intervention of the State. 204
The respondents, on the other hand, contend that the RH Law does not
provide that a specific mode or type of contraceptives be used, be it
natural or artificial. It neither imposes nor sanctions any religion or For their part, respondents De Venecia et al., dispute the notion that
belief.196 They point out that the RH Law only seeks to serve the public natural family planning is the only method acceptable to Catholics and the
interest by providing accessible, effective and quality reproductive health Catholic hierarchy. Citing various studies and surveys on the matter, they
services to ensure maternal and child health, in line with the State's duty highlight the changing stand of the Catholic Church on contraception
to bring to reality the social justice health guarantees of the throughout the years and note the general acceptance of the benefits of
Constitution,197 and that what the law only prohibits are those acts or contraceptives by its followers in planning their families.
practices, which deprive others of their right to reproductive
health.198 They assert that the assailed law only seeks to guarantee The Church and The State
informed choice, which is an assurance that no one will be compelled to
violate his religion against his free will.199 At the outset, it cannot be denied that we all live in a heterogeneous
society. It is made up of people of diverse ethnic, cultural and religious
The respondents add that by asserting that only natural family planning beliefs and backgrounds. History has shown us that our government, in
should be allowed, the petitioners are effectively going against the law and in practice, has allowed these various religious, cultural, social and
constitutional right to religious freedom, the same right they invoked to racial groups to thrive in a single society together. It has embraced
assail the constitutionality of the RH Law.200 In other words, by seeking the minority groups and is tolerant towards all - the religious people of
declaration that the RH Law is unconstitutional, the petitioners are asking different sects and the non-believers. The undisputed fact is that our
that the Court recognize only the Catholic Church's sanctioned natural people generally believe in a deity, whatever they conceived Him to be,
family planning methods and impose this on the entire citizenry. 201 and to whom they call for guidance and enlightenment in crafting our
fundamental law. Thus, the preamble of the present Constitution reads:
With respect to the duty to refer, the respondents insist that the same
does not violate the constitutional guarantee of religious freedom, it being We, the sovereign Filipino people, imploring the aid of Almighty God, in
a carefully balanced compromise between the interests of the religious order to build a just and humane society, and establish a Government that
objector, on one hand, who is allowed to keep silent but is required to shall embody our ideals and aspirations, promote the common good,
refer -and that of the citizen who needs access to information and who has conserve and develop our patrimony, and secure to ourselves and our
the right to expect that the health care professional in front of her will act posterity, the blessings of independence and democracy under the rule of
professionally. For the respondents, the concession given by the State law and a regime of truth, justice, freedom, love, equality, and peace, do
under Section 7 and 23(a)(3) is sufficient accommodation to the right to ordain and promulgate this Constitution.
freely exercise one's religion without unnecessarily infringing on the rights
of others.202 The Filipino people in "imploring the aid of Almighty God " manifested their
spirituality innate in our nature and consciousness as a people, shaped by
Whatever burden is placed on the petitioner's religious freedom is minimal tradition and historical experience. As this is embodied in the preamble, it
as the duty to refer is limited in duration, location and impact. 203 means that the State recognizes with respect the influence of religion in so
far as it instills into the mind the purest principles of morality. 205 Moreover,
in recognition of the contributions of religion to society, the 1935, 1973
and 1987 constitutions contain benevolent and accommodating provisions No public money or property shall be appropriated, applied, paid, or
towards religions such as tax exemption of church property, salary of employed, directly or indirectly, for the use, benefit, or support of any
religious officers in government institutions, and optional religious sect, church, denomination, sectarian institution, or system of religion, or
instructions in public schools. of any priest, preacher, minister, other religious teacher, or dignitary as
such, except when such priest, preacher, minister, or dignitary is assigned
The Framers, however, felt the need to put up a strong barrier so that the to the armed forces, or to any penal institution, or government orphanage
State would not encroach into the affairs of the church, and vice-versa. or leprosarium.
The principle of separation of Church and State was, thus, enshrined in
Article II, Section 6 of the 1987 Constitution, viz: In short, the constitutional assurance of religious freedom provides two
guarantees: the Establishment Clause and the Free Exercise Clause.
Section 6. The separation of Church and State shall be inviolable.
The establishment clause "principally prohibits the State from sponsoring
Verily, the principle of separation of Church and State is based on mutual any religion or favoring any religion as against other religions. It mandates
respect.1âwphi1 Generally, the State cannot meddle in the internal affairs a strict neutrality in affairs among religious groups."206 Essentially, it
of the church, much less question its faith and dogmas or dictate upon it. prohibits the establishment of a state religion and the use of public
It cannot favor one religion and discriminate against another. On the other resources for the support or prohibition of a religion.
hand, the church cannot impose its beliefs and convictions on the State
and the rest of the citizenry. It cannot demand that the nation follow its On the other hand, the basis of the free exercise clause is the respect for
beliefs, even if it sincerely believes that they are good for the country. the inviolability of the human conscience. 207 Under this part of religious
freedom guarantee, the State is prohibited from unduly interfering with
Consistent with the principle that not any one religion should ever be the outside manifestations of one's belief and faith. 208 Explaining the
preferred over another, the Constitution in the above-cited provision concept of religious freedom, the Court, in Victoriano v. Elizalde Rope
utilizes the term "church" in its generic sense, which refers to a temple, a Workers Union209 wrote:
mosque, an iglesia, or any other house of God which metaphorically
symbolizes a religious organization. Thus, the "Church" means the The constitutional provisions not only prohibits legislation for the support
religious congregations collectively. of any religious tenets or the modes of worship of any sect, thus
forestalling compulsion by law of the acceptance of any creed or the
Balancing the benefits that religion affords and the need to provide an practice of any form of worship (U.S. Ballard, 322 U.S. 78, 88 L. ed. 1148,
ample barrier to protect the State from the pursuit of its secular 1153), but also assures the free exercise of one's chosen form of religion
objectives, the Constitution lays down the following mandate in Article III, within limits of utmost amplitude. It has been said that the religion clauses
Section 5 and Article VI, Section 29 (2), of the 1987 Constitution: of the Constitution are all designed to protect the broadest possible liberty
of conscience, to allow each man to believe as his conscience directs, to
profess his beliefs, and to live as he believes he ought to live, consistent
Section. 5. No law shall be made respecting an establishment of religion,
with the liberty of others and with the common good. Any legislation
or prohibiting the free exercise thereof. The free exercise and enjoyment
whose effect or purpose is to impede the observance of one or all
of religious profession and worship, without discrimination or preference,
religions, or to discriminate invidiously between the religions, is invalid,
shall forever be allowed. No religious test shall be required for the exercise
even though the burden may be characterized as being only indirect.
of civil or political rights.
(Sherbert v. Verner, 374 U.S. 398, 10 L.ed.2d 965, 83 S. Ct. 1970) But if
the state regulates conduct by enacting, within its power, a general law
Section 29. which has for its purpose and effect to advance the state's secular goals,
the statute is valid despite its indirect burden on religious observance,
xxx. unless the state can accomplish its purpose without imposing such burden.
(Braunfeld v. Brown, 366 U.S. 599, 6 Led. 2d. 563, 81 S. Ct. 144;
McGowan v. Maryland, 366 U.S. 420, 444-5 and 449).
As expounded in Escritor, The benevolent neutrality theory believes that with respect to these
governmental actions, accommodation of religion may be allowed, not to
The establishment and free exercise clauses were not designed to serve promote the government's favored form of religion, but to allow individuals
contradictory purposes. They have a single goal-to promote freedom of and groups to exercise their religion without hindrance. "The purpose of
individual religious beliefs and practices. In simplest terms, the free accommodation is to remove a burden on, or facilitate the exercise of, a
exercise clause prohibits government from inhibiting religious beliefs with person's or institution's religion."216 "What is sought under the theory of
penalties for religious beliefs and practice, while the establishment clause accommodation is not a declaration of unconstitutionality of a facially
prohibits government from inhibiting religious belief with rewards for neutral law, but an exemption from its application or its 'burdensome
religious beliefs and practices. In other words, the two religion clauses effect,' whether by the legislature or the courts."217
were intended to deny government the power to use either the carrot or
the stick to influence individual religious beliefs and practices. 210 In ascertaining the limits of the exercise of religious freedom, the
compelling state interest test is proper.218Underlying the compelling state
Corollary to the guarantee of free exercise of one's religion is the principle interest test is the notion that free exercise is a fundamental right and that
that the guarantee of religious freedom is comprised of two parts: the laws burdening it should be subject to strict scrutiny. 219 In Escritor, it was
freedom to believe, and the freedom to act on one's belief. The first part is written:
absolute. As explained in Gerona v. Secretary of Education: 211
Philippine jurisprudence articulates several tests to determine these limits.
The realm of belief and creed is infinite and limitless bounded only by Beginning with the first case on the Free Exercise Clause, American Bible
one's imagination and thought. So is the freedom of belief, including Society, the Court mentioned the "clear and present danger" test but did
religious belief, limitless and without bounds. One may believe in most not employ it. Nevertheless, this test continued to be cited in subsequent
anything, however strange, bizarre and unreasonable the same may cases on religious liberty. The Gerona case then pronounced that the test
appear to others, even heretical when weighed in the scales of orthodoxy of permissibility of religious freedom is whether it violates the established
or doctrinal standards. But between the freedom of belief and the exercise institutions of society and law. The Victoriano case mentioned the
of said belief, there is quite a stretch of road to travel. 212 "immediate and grave danger" test as well as the doctrine that a law of
general applicability may burden religious exercise provided the law is the
least restrictive means to accomplish the goal of the law. The case also
The second part however, is limited and subject to the awesome power of
used, albeit inappropriately, the "compelling state interest" test. After
the State and can be enjoyed only with proper regard to the rights of
Victoriano , German went back to the Gerona rule. Ebralinag then
others. It is "subject to regulation where the belief is translated into
employed the "grave and immediate danger" test and overruled the
external acts that affect the public welfare."213
Gerona test. The fairly recent case of Iglesia ni Cristo went back to the "
clear and present danger" test in the maiden case of A merican Bible
Legislative Acts and the Society. Not surprisingly, all the cases which employed the "clear and
present danger" or "grave and immediate danger" test involved, in one
Free Exercise Clause form or another, religious speech as this test is often used in cases on
freedom of expression. On the other hand, the Gerona and German cases
Thus, in case of conflict between the free exercise clause and the State, set the rule that religious freedom will not prevail over established
the Court adheres to the doctrine of benevolent neutrality. This has been institutions of society and law. Gerona, however, which was the authority
clearly decided by the Court in Estrada v. Escritor, (Escritor) 214 where it cited by German has been overruled by Ebralinag which employed the
was stated "that benevolent neutrality-accommodation, whether "grave and immediate danger" test . Victoriano was the only case that
mandatory or permissive, is the spirit, intent and framework underlying employed the "compelling state interest" test, but as explained previously,
the Philippine Constitution."215 In the same case, it was further explained the use of the test was inappropriate to the facts of the case.
that"
The case at bar does not involve speech as in A merican Bible Society,
Ebralinag and Iglesia ni Cristo where the "clear and present danger" and
"grave and immediate danger" tests were appropriate as speech has easily
discernible or immediate effects. The Gerona and German doctrine, aside extends only to public and secular morality. Whatever pronouncement the
from having been overruled, is not congruent with the benevolent Court makes in the case at bench should be understood only in this realm
neutrality approach, thus not appropriate in this jurisdiction. Similar to where it has authority. Stated otherwise, while the Court stands without
Victoriano, the present case involves purely conduct arising from religious authority to rule on ecclesiastical matters, as vanguard of the Constitution,
belief. The "compelling state interest" test is proper where conduct is it does have authority to determine whether the RH Law contravenes the
involved for the whole gamut of human conduct has different effects on guarantee of religious freedom.
the state's interests: some effects may be immediate and short-term while
others delayed and far-reaching. A test that would protect the interests of At first blush, it appears that the RH Law recognizes and respects religion
the state in preventing a substantive evil, whether immediate or delayed, and religious beliefs and convictions. It is replete with assurances the no
is therefore necessary. However, not any interest of the state would suffice one can be compelled to violate the tenets of his religion or defy his
to prevail over the right to religious freedom as this is a fundamental right religious convictions against his free will. Provisions in the RH Law
that enjoys a preferred position in the hierarchy of rights - "the most respecting religious freedom are the following:
inalienable and sacred of all human rights", in the words of Jefferson. This
right is sacred for an invocation of the Free Exercise Clause is an appeal to
1. The State recognizes and guarantees the human rights of all persons
a higher sovereignty. The entire constitutional order of limited government
including their right to equality and nondiscrimination of these rights, the
is premised upon an acknowledgment of such higher sovereignty, thus the
right to sustainable human development, the right to health which includes
Filipinos implore the "aid of Almighty God in order to build a just and
reproductive health, the right to education and information, and the right
humane society and establish a government." As held in Sherbert, only the
to choose and make decisions for themselves in accordance with their
gravest abuses, endangering paramount interests can limit this
religious convictions, ethics, cultural beliefs, and the demands of
fundamental right. A mere balancing of interests which balances a right
responsible parenthood. [Section 2, Declaration of Policy]
with just a colorable state interest is therefore not appropriate. Instead,
only a compelling interest of the state can prevail over the fundamental
right to religious liberty. The test requires the state to carry a heavy 2 . The State recognizes marriage as an inviolable social institution and the
burden, a compelling one, for to do otherwise would allow the state to foundation of the family which in turn is the foundation of the nation.
batter religion, especially the less powerful ones until they are destroyed. Pursuant thereto, the State shall defend:
In determining which shall prevail between the state's interest and
religious liberty, reasonableness shall be the guide. The "compelling state (a) The right of spouses to found a family in accordance with their religious
interest" serves the purpose of revering religious liberty while at the same convictions and the demands of responsible parenthood." [Section 2,
time affording protection to the paramount interests of the state. This was Declaration of Policy]
the test used in Sherbert which involved conduct, i.e. refusal to work on
Saturdays. In the end, the "compelling state interest" test, by upholding 3. The State shall promote and provide information and access, without
the paramount interests of the state, seeks to protect the very state, bias, to all methods of family planning, including effective natural and
without which, religious liberty will not be preserved. [Emphases in the modern methods which have been proven medically safe, legal, non-
original. Underlining supplied.] abortifacient, and effective in accordance with scientific and evidence-
based medical research standards such as those registered and approved
The Court's Position by the FDA for the poor and marginalized as identified through the NHTS-
PR and other government measures of identifying marginalization:
In the case at bench, it is not within the province of the Court to Provided, That the State shall also provide funding support to promote
determine whether the use of contraceptives or one's participation in the modern natural methods of family planning, especially the Billings
support of modem reproductive health measures is moral from a religious Ovulation Method, consistent with the needs of acceptors and their
standpoint or whether the same is right or wrong according to one's religious convictions. [Section 3(e), Declaration of Policy]
dogma or belief. For the Court has declared that matters dealing with
"faith, practice, doctrine, form of worship, ecclesiastical law, custom and 4. The State shall promote programs that: (1) enable individuals and
rule of a church ... are unquestionably ecclesiastical matters which are couples to have the number of children they desire with due consideration
outside the province of the civil courts." 220 The jurisdiction of the Court to the health, particularly of women, and the resources available and
affordable to them and in accordance with existing laws, public morals and simply because the promotion of contraceptive use is contrary to their
their religious convictions. [Section 3CDJ religious beliefs. Indeed, the State is not precluded to pursue its legitimate
secular objectives without being dictated upon by the policies of any one
5. The State shall respect individuals' preferences and choice of family religion. One cannot refuse to pay his taxes simply because it will cloud his
planning methods that are in accordance with their religious convictions conscience. The demarcation line between Church and State demands that
and cultural beliefs, taking into consideration the State's obligations under one render unto Caesar the things that are Caesar's and unto God the
various human rights instruments. [Section 3(h)] things that are God's.221

6. Active participation by nongovernment organizations (NGOs) , women's The Free Exercise Clause and the Duty to Refer
and people's organizations, civil society, faith-based organizations, the
religious sector and communities is crucial to ensure that reproductive While the RH Law, in espousing state policy to promote reproductive
health and population and development policies, plans, and programs will health manifestly respects diverse religious beliefs in line with the Non-
address the priority needs of women, the poor, and the marginalized. Establishment Clause, the same conclusion cannot be reached with respect
[Section 3(i)] to Sections 7, 23 and 24 thereof. The said provisions commonly mandate
that a hospital or a medical practitioner to immediately refer a person
7. Responsible parenthood refers to the will and ability of a parent to seeking health care and services under the law to another accessible
respond to the needs and aspirations of the family and children. It is healthcare provider despite their conscientious objections based on
likewise a shared responsibility between parents to determine and achieve religious or ethical beliefs.
the desired number of children, spacing and timing of their children
according to their own family life aspirations, taking into account In a situation where the free exercise of religion is allegedly burdened by
psychological preparedness, health status, sociocultural and economic government legislation or practice, the compelling state interest test in line
concerns consistent with their religious convictions. [Section 4(v)] with the Court's espousal of the Doctrine of Benevolent Neutrality in
(Emphases supplied) Escritor, finds application. In this case, the conscientious objector's claim
to religious freedom would warrant an exemption from obligations under
While the Constitution prohibits abortion, laws were enacted allowing the the RH Law, unless the government succeeds in demonstrating a more
use of contraceptives. To some medical practitioners, however, the whole compelling state interest in the accomplishment of an important secular
idea of using contraceptives is an anathema. Consistent with the principle objective. Necessarily so, the plea of conscientious objectors for exemption
of benevolent neutrality, their beliefs should be respected. from the RH Law deserves no less than strict scrutiny.

The Establishment Clause In applying the test, the first inquiry is whether a conscientious objector's
right to religious freedom has been burdened. As in Escritor, there is no
doubt that an intense tug-of-war plagues a conscientious objector. One
and Contraceptives
side coaxes him into obedience to the law and the abandonment of his
religious beliefs, while the other entices him to a clean conscience yet
In the same breath that the establishment clause restricts what the under the pain of penalty. The scenario is an illustration of the
government can do with religion, it also limits what religious sects can or predicament of medical practitioners whose religious beliefs are
cannot do with the government. They can neither cause the government to incongruent with what the RH Law promotes.
adopt their particular doctrines as policy for everyone, nor can they not
cause the government to restrict other groups. To do so, in simple terms,
The Court is of the view that the obligation to refer imposed by the RH Law
would cause the State to adhere to a particular religion and, thus,
violates the religious belief and conviction of a conscientious objector.
establishing a state religion.
Once the medical practitioner, against his will, refers a patient seeking
information on modem reproductive health products, services, procedures
Consequently, the petitioners are misguided in their supposition that the and methods, his conscience is immediately burdened as he has been
State cannot enhance its population control program through the RH Law compelled to perform an act against his beliefs. As Commissioner Joaquin
A. Bernas (Commissioner Bernas) has written, "at the basis of the free part 'directly' or ' indirectly' this would actually mean more complexity and
exercise clause is the respect for the inviolability of the human uncertainty."227
conscience.222
While the said case did not cover the act of referral, the applicable
Though it has been said that the act of referral is an opt-out clause, it is, principle was the same - they could not be forced to assist abortions if it
however, a false compromise because it makes pro-life health providers would be against their conscience or will.
complicit in the performance of an act that they find morally repugnant or
offensive. They cannot, in conscience, do indirectly what they cannot do Institutional Health Providers
directly. One may not be the principal, but he is equally guilty if he abets
the offensive act by indirect participation.
The same holds true with respect to non-maternity specialty hospitals and
hospitals owned and operated by a religious group and health care service
Moreover, the guarantee of religious freedom is necessarily intertwined providers. Considering that Section 24 of the RH Law penalizes such
with the right to free speech, it being an externalization of one's thought institutions should they fail or refuse to comply with their duty to refer
and conscience. This in turn includes the right to be silent. With the under Section 7 and Section 23(a)(3), the Court deems that it must be
constitutional guarantee of religious freedom follows the protection that struck down for being violative of the freedom of religion. The same
should be afforded to individuals in communicating their beliefs to others applies to Section 23(a)(l) and (a)(2) in relation to Section 24, considering
as well as the protection for simply being silent. The Bill of Rights that in the dissemination of information regarding programs and services
guarantees the liberty of the individual to utter what is in his mind and the and in the performance of reproductive health procedures, the religious
liberty not to utter what is not in his mind. 223 While the RH Law seeks to freedom of health care service providers should be respected.
provide freedom of choice through informed consent, freedom of choice
guarantees the liberty of the religious conscience and prohibits any degree
In the case of Islamic Da'wah Council of the Philippines, Inc. v. Office of
of compulsion or burden, whether direct or indirect, in the practice of one's
the Executive Secretary228 it was stressed:
religion.224

Freedom of religion was accorded preferred status by the framers of our


In case of conflict between the religious beliefs and moral convictions of
fundamental law. And this Court has consistently affirmed this preferred
individuals, on one hand, and the interest of the State, on the other, to
status, well aware that it is "designed to protect the broadest possible
provide access and information on reproductive health products, services,
liberty of conscience, to allow each man to believe as his conscience
procedures and methods to enable the people to determine the timing,
directs, to profess his beliefs, and to live as he believes he ought to live,
number and spacing of the birth of their children, the Court is of the strong
consistent with the liberty of others and with the common good." 10
view that the religious freedom of health providers, whether public or
private, should be accorded primacy. Accordingly, a conscientious objector
should be exempt from compliance with the mandates of the RH Law. If he The Court is not oblivious to the view that penalties provided by law
would be compelled to act contrary to his religious belief and conviction, it endeavour to ensure compliance. Without set consequences for either an
would be violative of "the principle of non-coercion" enshrined in the active violation or mere inaction, a law tends to be toothless and
constitutional right to free exercise of religion. ineffectual. Nonetheless, when what is bartered for an effective
implementation of a law is a constitutionally-protected right the Court
firmly chooses to stamp its disapproval. The punishment of a healthcare
Interestingly, on April 24, 2013, Scotland's Inner House of the Court of
service provider, who fails and/or refuses to refer a patient to another, or
Session, found in the case of Doogan and Wood v. NHS Greater Glasgow
who declines to perform reproductive health procedure on a patient
and Clyde Health Board,225 that the midwives claiming to be conscientious
because incompatible religious beliefs, is a clear inhibition of a
objectors under the provisions of Scotland's Abortion Act of 1967, could
constitutional guarantee which the Court cannot allow.
not be required to delegate, supervise or support staff on their labor ward
who were involved in abortions.226 The Inner House stated "that if
'participation' were defined according to whether the person was taking The Implementing Rules and Regulation (RH-IRR)
The last paragraph of Section 5.24 of the RH-IRR reads: Yes, Your Honor, I have read but I have to admit, it's a long IRR and I
have not thoroughly dissected the nuances of the provisions.
Provided, That skilled health professional such as provincial, city or
municipal health officers, chiefs of hospital, head nurses, supervising Justice Mendoza:
midwives, among others, who by virtue of their office are specifically
charged with the duty to implement the provisions of the RPRH Act and I will read to you one provision. It's Section 5.24. This I cannot find in the
these Rules, cannot be considered as conscientious objectors. RH Law. But in the IRR it says: " .... skilled health professionals such as
provincial, city or municipal health officers, chief of hospitals, head nurses,
This is discriminatory and violative of the equal protection clause. The supervising midwives, among others, who by virtue of their office are
conscientious objection clause should be equally protective of the religious specifically charged with the duty to implement the provisions of the RPRH
belief of public health officers. There is no perceptible distinction why they Act and these Rules, cannot be considered as conscientious objectors." Do
should not be considered exempt from the mandates of the law. The you agree with this?
protection accorded to other conscientious objectors should equally apply
to all medical practitioners without distinction whether they belong to the Congressman Lagman:
public or private sector. After all, the freedom to believe is intrinsic in
every individual and the protective robe that guarantees its free exercise is
I will have to go over again the provisions, Your Honor.
not taken off even if one acquires employment in the government.

Justice Mendoza:
It should be stressed that intellectual liberty occupies a place inferior to
none in the hierarchy of human values. The mind must be free to think
what it wills, whether in the secular or religious sphere, to give expression In other words, public health officers in contrast to the private
to its beliefs by oral discourse or through the media and, thus, seek other practitioners who can be conscientious objectors, skilled health
candid views in occasions or gatherings or in more permanent professionals cannot be considered conscientious objectors. Do you agree
aggrupation. Embraced in such concept then are freedom of religion, with this? Is this not against the constitutional right to the religious belief?
freedom of speech, of the press, assembly and petition, and freedom of
association.229 Congressman Lagman:

The discriminatory provision is void not only because no such exception is Your Honor, if there is any conflict between the IRR and the law, the law
stated in the RH Law itself but also because it is violative of the equal must prevail.230
protection clause in the Constitution. Quoting respondent Lagman, if there
is any conflict between the RH-IRR and the RH Law, the law must prevail. Compelling State Interest

Justice Mendoza: The foregoing discussion then begets the question on whether the
respondents, in defense of the subject provisions, were able to: 1]
I'll go to another point. The RH law .. .in your Comment- in-Intervention demonstrate a more compelling state interest to restrain conscientious
on page 52, you mentioned RH Law is replete with provisions in upholding objectors in their choice of services to render; and 2] discharge the burden
the freedom of religion and respecting religious convictions. Earlier, you of proof that the obligatory character of the law is the least intrusive
affirmed this with qualifications. Now, you have read, I presumed you means to achieve the objectives of the law.
have read the IRR-Implementing Rules and Regulations of the RH Bill?
Unfortunately, a deep scrutiny of the respondents' submissions proved to
Congressman Lagman: be in vain. The OSG was curiously silent in the establishment of a more
compelling state interest that would rationalize the curbing of a
conscientious objector's right not to adhere to an action contrary to his
religious convictions. During the oral arguments, the OSG maintained the Freedom of religion means more than just the freedom to believe. It also
same silence and evasion. The Transcripts of the Stenographic Notes means the freedom to act or not to act according to what one believes.
disclose the following: And this freedom is violated when one is compelled to act against one's
belief or is prevented from acting according to one's belief. 233
Justice De Castro:
Apparently, in these cases, there is no immediate danger to the life or
Let's go back to the duty of the conscientious objector to refer. .. health of an individual in the perceived scenario of the subject provisions.
After all, a couple who plans the timing, number and spacing of the birth
of their children refers to a future event that is contingent on whether or
Senior State Solicitor Hilbay:
not the mother decides to adopt or use the information, product, method
or supply given to her or whether she even decides to become pregnant at
Yes, Justice. all. On the other hand, the burden placed upon those who object to
contraceptive use is immediate and occurs the moment a patient seeks
Justice De Castro: consultation on reproductive health matters.

... which you are discussing awhile ago with Justice Abad. What is the Moreover, granting that a compelling interest exists to justify the
compelling State interest in imposing this duty to refer to a conscientious infringement of the conscientious objector's religious freedom, the
objector which refuses to do so because of his religious belief? respondents have failed to demonstrate "the gravest abuses, endangering
paramount interests" which could limit or override a person's fundamental
Senior State Solicitor Hilbay: right to religious freedom. Also, the respondents have not presented any
government effort exerted to show that the means it takes to achieve its
legitimate state objective is the least intrusive means. 234 Other than the
Ahh, Your Honor, .. assertion that the act of referring would only be momentary, considering
that the act of referral by a conscientious objector is the very action being
Justice De Castro: contested as violative of religious freedom, it behooves the respondents to
demonstrate that no other means can be undertaken by the State to
What is the compelling State interest to impose this burden? achieve its objective without violating the rights of the conscientious
objector. The health concerns of women may still be addressed by other
practitioners who may perform reproductive health-related procedures
Senior State Solicitor Hilbay:
with open willingness and motivation. Suffice it to say, a person who is
forced to perform an act in utter reluctance deserves the protection of the
In the first place, Your Honor, I don't believe that the standard is a Court as the last vanguard of constitutional freedoms.
compelling State interest, this is an ordinary health legislation involving
professionals. This is not a free speech matter or a pure free exercise
At any rate, there are other secular steps already taken by the Legislature
matter. This is a regulation by the State of the relationship between
to ensure that the right to health is protected. Considering other
medical doctors and their patients. 231
legislations as they stand now, R.A . No. 4 729 or the Contraceptive Act,
R.A. No. 6365 or "The Population Act of the Philippines" and R.A. No.
Resultantly, the Court finds no compelling state interest which would limit 9710, otherwise known as "The Magna Carta of Women," amply cater to
the free exercise clause of the conscientious objectors, however few in the needs of women in relation to health services and programs. The
number. Only the prevention of an immediate and grave danger to the pertinent provision of Magna Carta on comprehensive health services and
security and welfare of the community can justify the infringement of programs for women, in fact, reads:
religious freedom. If the government fails to show the seriousness and
immediacy of the threat, State intrusion is constitutionally unacceptable. 232
Section 17. Women's Right to Health. - (a) Comprehensive Health
Services. - The State shall, at all times, provide for a comprehensive,
culture-sensitive, and gender-responsive health services and programs (9) Prevention and management of infertility and sexual
covering all stages of a woman's life cycle and which addresses the major dysfunction pursuant to ethical norms and medical
causes of women's mortality and morbidity: Provided, That in the provision standards;
for comprehensive health services, due respect shall be accorded to
women's religious convictions, the rights of the spouses to found a family (10) Care of the elderly women beyond their child-bearing
in accordance with their religious convictions, and the demands of years; and
responsible parenthood, and the right of women to protection from
hazardous drugs, devices, interventions, and substances.
(11) Management, treatment, and intervention of mental
health problems of women and girls. In addition, healthy
Access to the following services shall be ensured: lifestyle activities are encouraged and promoted through
programs and projects as strategies in the prevention of
(1) Maternal care to include pre- and post-natal services to diseases.
address pregnancy and infant health and nutrition;
(b) Comprehensive Health Information and Education. - The State shall
(2) Promotion of breastfeeding; provide women in all sectors with appropriate, timely, complete, and
accurate information and education on all the above-stated aspects of
(3) Responsible, ethical, legal, safe, and effective methods women's health in government education and training programs, with due
of family planning; regard to the following:

(4) Family and State collaboration in youth sexuality (1) The natural and primary right and duty of parents in
education and health services without prejudice to the the rearing of the youth and the development of moral
primary right and duty of parents to educate their character and the right of children to be brought up in an
children; atmosphere of morality and rectitude for the enrichment
and strengthening of character;
(5) Prevention and management of reproductive tract
infections, including sexually transmitted diseases, HIV, (2) The formation of a person's sexuality that affirms
and AIDS; human dignity; and

(6) Prevention and management of reproductive tract (3) Ethical, legal, safe, and effective family planning
cancers like breast and cervical cancers, and other methods including fertility awareness.
gynecological conditions and disorders;
As an afterthought, Asst. Solicitor General Hilbay eventually replied that
(7) Prevention of abortion and management of pregnancy- the compelling state interest was "Fifteen maternal deaths per day,
related complications; hundreds of thousands of unintended pregnancies, lives changed, x x
x."235 He, however, failed to substantiate this point by concrete facts and
figures from reputable sources.
(8) In cases of violence against women and children,
women and children victims and survivors shall be
provided with comprehensive health services that include The undisputed fact, however, is that the World Health Organization
psychosocial, therapeutic, medical, and legal interventions reported that the Filipino maternal mortality rate dropped to 48 percent
and assistance towards healing, recovery, and from 1990 to 2008, 236 although there was still no RH Law at that time.
empowerment; Despite such revelation, the proponents still insist that such number of
maternal deaths constitute a compelling state interest.
Granting that there are still deficiencies and flaws in the delivery of social Family Planning Seminars
healthcare programs for Filipino women, they could not be solved by a
measure that puts an unwarrantable stranglehold on religious beliefs in Anent the requirement imposed under Section 15239 as a condition for the
exchange for blind conformity. issuance of a marriage license, the Court finds the same to be a
reasonable exercise of police power by the government. A cursory reading
Exception: Life Threatening Cases of the assailed provision bares that the religious freedom of the petitioners
is not at all violated. All the law requires is for would-be spouses to attend
All this notwithstanding, the Court properly recognizes a valid exception a seminar on parenthood, family planning breastfeeding and infant
set forth in the law. While generally healthcare service providers cannot be nutrition. It does not even mandate the type of family planning methods to
forced to render reproductive health care procedures if doing it would be included in the seminar, whether they be natural or artificial. As
contravene their religious beliefs, an exception must be made in life- correctly noted by the OSG, those who receive any information during
threatening cases that require the performance of emergency procedures. their attendance in the required seminars are not compelled to accept the
In these situations, the right to life of the mother should be given information given to them, are completely free to reject the information
preference, considering that a referral by a medical practitioner would they find unacceptable, and retain the freedom to decide on matters of
amount to a denial of service, resulting to unnecessarily placing the life of family life without the intervention of the State.
a mother in grave danger. Thus, during the oral arguments, Atty. Liban,
representing CFC, manifested: "the forced referral clause that we are 4-The Family and the Right to Privacy
objecting on grounds of violation of freedom of religion does not
contemplate an emergency."237 Petitioner CFC assails the RH Law because Section 23(a) (2) (i) thereof
violates the provisions of the Constitution by intruding into marital privacy
In a conflict situation between the life of the mother and the life of a child, and autonomy. It argues that it cultivates disunity and fosters animosity in
the doctor is morally obliged always to try to save both lives. If, however, the family rather than promote its solidarity and total development. 240
it is impossible, the resulting death to one should not be deliberate. Atty.
Noche explained: The Court cannot but agree.

Principle of Double-Effect. - May we please remind the principal author of The 1987 Constitution is replete with provisions strengthening the family
the RH Bill in the House of Representatives of the principle of double-effect as it is the basic social institution. In fact, one article, Article XV, is
wherein intentional harm on the life of either the mother of the child is devoted entirely to the family.
never justified to bring about a "good" effect. In a conflict situation
between the life of the child and the life of the mother, the doctor is
ARTICLE XV
morally obliged always to try to save both lives. However, he can act in
THE FAMILY
favor of one (not necessarily the mother) when it is medically impossible
to save both, provided that no direct harm is intended to the other. If the
above principles are observed, the loss of the child's life or the mother's Section 1. The State recognizes the Filipino family as the foundation of the
life is not intentional and, therefore, unavoidable. Hence, the doctor would nation. Accordingly, it shall strengthen its solidarity and actively promote
not be guilty of abortion or murder. The mother is never pitted against the its total development.
child because both their lives are equally valuable. 238
Section 2. Marriage, as an inviolable social institution, is the foundation of
Accordingly, if it is necessary to save the life of a mother, procedures the family and shall be protected by the State.
endangering the life of the child may be resorted to even if is against the
religious sentiments of the medical practitioner. As quoted above, Section 3. The State shall defend:
whatever burden imposed upon a medical practitioner in this case would
have been more than justified considering the life he would be able to The right of spouses to found a family in accordance with their religious
save. convictions and the demands of responsible parenthood;
The right of children to assistance, including proper care and nutrition, and The RH Law cannot be allowed to infringe upon this mutual decision-
special protection from all forms of neglect, abuse, cruelty, exploitation making. By giving absolute authority to the spouse who would undergo a
and other conditions prejudicial to their development; procedure, and barring the other spouse from participating in the decision
would drive a wedge between the husband and wife, possibly result in
The right of the family to a family living wage and income; and bitter animosity, and endanger the marriage and the family, all for the
sake of reducing the population. This would be a marked departure from
the policy of the State to protect marriage as an inviolable social
The right of families or family assoc1at1ons to participate in the planning
institution.241
and implementation of policies and programs that affect them.

Decision-making involving a reproductive health procedure is a private


In this case, the RH Law, in its not-so-hidden desire to control population
matter which belongs to the couple, not just one of them. Any decision
growth, contains provisions which tend to wreck the family as a solid social
they would reach would affect their future as a family because the size of
institution. It bars the husband and/or the father from participating in the
the family or the number of their children significantly matters. The
decision making process regarding their common future progeny. It
decision whether or not to undergo the procedure belongs exclusively to,
likewise deprives the parents of their authority over their minor daughter
and shared by, both spouses as one cohesive unit as they chart their own
simply because she is already a parent or had suffered a miscarriage.
destiny. It is a constitutionally guaranteed private right. Unless it
prejudices the State, which has not shown any compelling interest, the
The Family and Spousal Consent State should see to it that they chart their destiny together as one family.

Section 23(a) (2) (i) of the RH Law states: As highlighted by Justice Leonardo-De Castro, Section 19( c) of R.A. No.
9710, otherwise known as the "Magna Carta for Women," provides that
The following acts are prohibited: women shall have equal rights in all matters relating to marriage and
family relations, including the joint decision on the number and spacing of
(a) Any health care service provider, whether public or private, who their children. Indeed, responsible parenthood, as Section 3(v) of the RH
shall: ... Law states, is a shared responsibility between parents. Section 23(a)(2)(i)
of the RH Law should not be allowed to betray the constitutional mandate
to protect and strengthen the family by giving to only one spouse the
(2) refuse to perform legal and medically-safe reproductive health absolute authority to decide whether to undergo reproductive health
procedures on any person of legal age on the ground of lack of consent or procedure.242
authorization of the following persons in the following instances:
The right to chart their own destiny together falls within the protected
(i) Spousal consent in case of married persons: provided, That in case of zone of marital privacy and such state intervention would encroach into
disagreement, the decision of the one undergoing the procedures shall the zones of spousal privacy guaranteed by the Constitution. In our
prevail. [Emphasis supplied] jurisdiction, the right to privacy was first recognized in Marje v.
Mutuc,243 where the Court, speaking through Chief Justice Fernando, held
The above provision refers to reproductive health procedures like tubal that "the right to privacy as such is accorded recognition independently of
litigation and vasectomy which, by their very nature, should require its identification with liberty; in itself, it is fully deserving of constitutional
mutual consent and decision between the husband and the wife as they protection."244 Marje adopted the ruling of the US Supreme Court in
affect issues intimately related to the founding of a family. Section 3, Art. Griswold v. Connecticut,245 where Justice William O. Douglas wrote:
XV of the Constitution espouses that the State shall defend the "right of
the spouses to found a family." One person cannot found a family. The We deal with a right of privacy older than the Bill of Rights -older than our
right, therefore, is shared by both spouses. In the same Section 3, their political parties, older than our school system. Marriage is a coming
right "to participate in the planning and implementation of policies and together for better or for worse, hopefully enduring, and intimate to the
programs that affect them " is equally recognized. degree of being sacred. It is an association that promotes a way of life, not
causes; a harmony in living, not political faiths; a bilateral loyalty, not More alarmingly, it disregards and disobeys the constitutional mandate
commercial or social projects. Yet it is an association for as noble a that "the natural and primary right and duty of parents in the rearing of
purpose as any involved in our prior decisions. the youth for civic efficiency and the development of moral character shall
receive the support of the Government."247 In this regard, Commissioner
Ironically, Griswold invalidated a Connecticut statute which made the use Bernas wrote:
of contraceptives a criminal offense on the ground of its amounting to an
unconstitutional invasion of the right to privacy of married persons. The 1987 provision has added the adjective "primary" to modify the right
Nevertheless, it recognized the zone of privacy rightfully enjoyed by of parents. It imports the assertion that the right of parents is superior to
couples. Justice Douglas in Grisworld wrote that "specific guarantees in the that of the State.248 [Emphases supplied]
Bill of Rights have penumbras, formed by emanations from those
guarantees that help give them life and substance. Various guarantees To insist on a rule that interferes with the right of parents to exercise
create zones of privacy."246 parental control over their minor-child or the right of the spouses to
mutually decide on matters which very well affect the very purpose of
At any rate, in case of conflict between the couple, the courts will decide. marriage, that is, the establishment of conjugal and family life, would
result in the violation of one's privacy with respect to his family. It would
The Family and Parental Consent be dismissive of the unique and strongly-held Filipino tradition of
maintaining close family ties and violative of the recognition that the State
affords couples entering into the special contract of marriage to as one
Equally deplorable is the debarment of parental consent in cases where the
unit in forming the foundation of the family and society.
minor, who will be undergoing a procedure, is already a parent or has had
a miscarriage. Section 7 of the RH law provides:
The State cannot, without a compelling state interest, take over the role of
parents in the care and custody of a minor child, whether or not the latter
SEC. 7. Access to Family Planning. – x x x.
is already a parent or has had a miscarriage. Only a compelling state
interest can justify a state substitution of their parental authority.
No person shall be denied information and access to family planning
services, whether natural or artificial: Provided, That minors will not be
First Exception: Access to Information
allowed access to modern methods of family planning without written
consent from their parents or guardian/s except when the minor is already
a parent or has had a miscarriage. Whether with respect to the minor referred to under the exception
provided in the second paragraph of Section 7 or with respect to the
consenting spouse under Section 23(a)(2)(i), a distinction must be made.
There can be no other interpretation of this provision except that when a
There must be a differentiation between access to information about family
minor is already a parent or has had a miscarriage, the parents are
planning services, on one hand, and access to the reproductive health
excluded from the decision making process of the minor with regard to
procedures and modern family planning methods themselves, on the
family planning. Even if she is not yet emancipated, the parental authority
other. Insofar as access to information is concerned, the Court finds no
is already cut off just because there is a need to tame population growth.
constitutional objection to the acquisition of information by the minor
referred to under the exception in the second paragraph of Section 7 that
It is precisely in such situations when a minor parent needs the comfort, would enable her to take proper care of her own body and that of her
care, advice, and guidance of her own parents. The State cannot replace unborn child. After all, Section 12, Article II of the Constitution mandates
her natural mother and father when it comes to providing her needs and the State to protect both the life of the mother as that of the unborn child.
comfort. To say that their consent is no longer relevant is clearly anti- Considering that information to enable a person to make informed
family. It does not promote unity in the family. It is an affront to the decisions is essential in the protection and maintenance of ones' health,
constitutional mandate to protect and strengthen the family as an access to such information with respect to reproductive health must be
inviolable social institution. allowed. In this situation, the fear that parents might be deprived of their
parental control is unfounded because they are not prohibited to exercise
parental guidance and control over their minor child and assist her in petitioners and validate their apprehensions. Thus, considering the
deciding whether to accept or reject the information received. premature nature of this particular issue, the Court declines to rule on its
constitutionality or validity.
Second Exception: Life Threatening Cases
At any rate, Section 12, Article II of the 1987 Constitution provides that
As in the case of the conscientious objector, an exception must be made in the natural and primary right and duty of parents in the rearing of the
life-threatening cases that require the performance of emergency youth for civic efficiency and development of moral character shall receive
procedures. In such cases, the life of the minor who has already suffered a the support of the Government. Like the 1973 Constitution and the 1935
miscarriage and that of the spouse should not be put at grave risk simply Constitution, the 1987 Constitution affirms the State recognition of the
for lack of consent. It should be emphasized that no person should be invaluable role of parents in preparing the youth to become productive
denied the appropriate medical care urgently needed to preserve the members of society. Notably, it places more importance on the role of
primordial right, that is, the right to life. parents in the development of their children by recognizing that said role
shall be "primary," that is, that the right of parents in upbringing the youth
is superior to that of the State.252
In this connection, the second sentence of Section 23(a)(2)(ii) 249 should be
struck down. By effectively limiting the requirement of parental consent to
"only in elective surgical procedures," it denies the parents their right of It is also the inherent right of the State to act as parens patriae to aid
parental authority in cases where what is involved are "non-surgical parents in the moral development of the youth. Indeed, the Constitution
procedures." Save for the two exceptions discussed above, and in the case makes mention of the importance of developing the youth and their
of an abused child as provided in the first sentence of Section 23(a)(2)(ii), important role in nation building.253 Considering that Section 14 provides
the parents should not be deprived of their constitutional right of parental not only for the age-appropriate-reproductive health education, but also
authority. To deny them of this right would be an affront to the for values formation; the development of knowledge and skills in self-
constitutional mandate to protect and strengthen the family. protection against discrimination; sexual abuse and violence against
women and children and other forms of gender based violence and teen
pregnancy; physical, social and emotional changes in adolescents;
5 - Academic Freedom
women's rights and children's rights; responsible teenage behavior;
gender and development; and responsible parenthood, and that Rule 10,
It is asserted that Section 14 of the RH Law, in relation to Section 24 Section 11.01 of the RH-IRR and Section 4(t) of the RH Law itself provides
thereof, mandating the teaching of Age-and Development-Appropriate for the teaching of responsible teenage behavior, gender sensitivity and
Reproductive Health Education under threat of fine and/or imprisonment physical and emotional changes among adolescents - the Court finds that
violates the principle of academic freedom . According to the petitioners, the legal mandate provided under the assailed provision supplements,
these provisions effectively force educational institutions to teach rather than supplants, the rights and duties of the parents in the moral
reproductive health education even if they believe that the same is not development of their children.
suitable to be taught to their students.250 Citing various studies conducted
in the United States and statistical data gathered in the country, the
Furthermore, as Section 14 also mandates that the mandatory
petitioners aver that the prevalence of contraceptives has led to an
reproductive health education program shall be developed in conjunction
increase of out-of-wedlock births; divorce and breakdown of families; the
with parent-teacher-community associations, school officials and other
acceptance of abortion and euthanasia; the "feminization of poverty"; the
interest groups, it could very well be said that it will be in line with the
aging of society; and promotion of promiscuity among the youth. 251
religious beliefs of the petitioners. By imposing such a condition, it
becomes apparent that the petitioners' contention that Section 14 violates
At this point, suffice it to state that any attack on the validity of Section 14 Article XV, Section 3(1) of the Constitution is without merit. 254
of the RH Law is premature because the Department of Education, Culture
and Sports has yet to formulate a curriculum on age-appropriate
While the Court notes the possibility that educators might raise their
reproductive health education. One can only speculate on the content,
objection to their participation in the reproductive health education
manner and medium of instruction that will be used to educate the
program provided under Section 14 of the RH Law on the ground that the
adolescents and whether they will contradict the religious beliefs of the
same violates their religious beliefs, the Court reserves its judgment (n) Public health care service provider refers to: (1) public health care
should an actual case be filed before it. institution, which is duly licensed and accredited and devoted primarily to
the maintenance and operation of facilities for health promotion, disease
6 - Due Process prevention, diagnosis, treatment and care of individuals suffering from
illness, disease, injury, disability or deformity, or in need of obstetrical or
other medical and nursing care; (2) public health care professional, who is
The petitioners contend that the RH Law suffers from vagueness and, thus
a doctor of medicine, a nurse or a midvvife; (3) public health worker
violates the due process clause of the Constitution. According to them,
engaged in the delivery of health care services; or (4) barangay health
Section 23 (a)(l) mentions a "private health service provider" among those
worker who has undergone training programs under any accredited
who may be held punishable but does not define who is a "private health
government and NGO and who voluntarily renders primarily health care
care service provider." They argue that confusion further results since
services in the community after having been accredited to function as such
Section 7 only makes reference to a "private health care institution."
by the local health board in accordance with the guidelines promulgated by
the Department of Health (DOH) .
The petitioners also point out that Section 7 of the assailed legislation
exempts hospitals operated by religious groups from rendering
Further, the use of the term "private health care institution" in Section 7 of
reproductive health service and modern family planning methods. It is
the law, instead of "private health care service provider," should not be a
unclear, however, if these institutions are also exempt from giving
cause of confusion for the obvious reason that they are used
reproductive health information under Section 23(a)(l), or from rendering
synonymously.
reproductive health procedures under Section 23(a)(2).

The Court need not belabor the issue of whether the right to be exempt
Finally, it is averred that the RH Law punishes the withholding, restricting
from being obligated to render reproductive health service and modem
and providing of incorrect information, but at the same time fails to define
family planning methods, includes exemption from being obligated to give
"incorrect information."
reproductive health information and to render reproductive health
procedures. Clearly, subject to the qualifications and exemptions earlier
The arguments fail to persuade. discussed, the right to be exempt from being obligated to render
reproductive health service and modem family planning methods,
A statute or act suffers from the defect of vagueness when it lacks necessarily includes exemption from being obligated to give reproductive
comprehensible standards that men of common intelligence must health information and to render reproductive health procedures. The
necessarily guess its meaning and differ as to its application. It is terms "service" and "methods" are broad enough to include the providing
repugnant to the Constitution in two respects: (1) it violates due process of information and the rendering of medical procedures.
for failure to accord persons, especially the parties targeted by it, fair
notice of the conduct to avoid; and (2) it leaves law enforcers unbridled The same can be said with respect to the contention that the RH Law
discretion in carrying out its provisions and becomes an arbitrary flexing of punishes health care service providers who intentionally withhold, restrict
the Government muscle.255 Moreover, in determining whether the words and provide incorrect information regarding reproductive health programs
used in a statute are vague, words must not only be taken in accordance and services. For ready reference, the assailed provision is hereby quoted
with their plain meaning alone, but also in relation to other parts of the as follows:
statute. It is a rule that every part of the statute must be interpreted with
reference to the context, that is, every part of it must be construed
SEC. 23. Prohibited Acts. - The following acts are prohibited:
together with the other parts and kept subservient to the general intent of
the whole enactment.256
(a) Any health care service provider, whether public or private, who shall:
As correctly noted by the OSG, in determining the definition of "private
health care service provider," reference must be made to Section 4(n) of (1) Knowingly withhold information or restrict the dissemination thereof,
the RH Law which defines a "public health service provider," viz: and/ or intentionally provide incorrect information regarding programs and
services on reproductive health including the right to informed choice and
access to a full range of legal, medically-safe, non-abortifacient and the particular act assailed partakes of an unwarranted partiality or
effective family planning methods; prejudice, the sharper weapon to cut it down is the equal protection
clause.
From its plain meaning, the word "incorrect" here denotes failing to agree
with a copy or model or with established rules; inaccurate, faulty; failing to "According to a long line of decisions, equal protection simply requires that
agree with the requirements of duty, morality or propriety; and failing to all persons or things similarly situated should be treated alike, both as to
coincide with the truth. 257 On the other hand, the word "knowingly" means rights conferred and responsibilities imposed." It "requires public bodies
with awareness or deliberateness that is intentional. 258 Used together in and inst itutions to treat similarly situated individuals in a similar manner."
relation to Section 23(a)(l), they connote a sense of malice and ill motive "The purpose of the equal protection clause is to secure every person
to mislead or misrepresent the public as to the nature and effect of within a state's jurisdiction against intentional and arbitrary discrimination,
programs and services on reproductive health. Public health and safety whether occasioned by the express terms of a statue or by its improper
demand that health care service providers give their honest and correct execution through the state's duly constituted authorities." "In other
medical information in accordance with what is acceptable in medical words, the concept of equal justice under the law requires the state to
practice. While health care service providers are not barred from govern impartially, and it may not draw distinctions between individuals
expressing their own personal opinions regarding the programs and solely on differences that are irrelevant to a legitimate governmental
services on reproductive health, their right must be tempered with the objective."
need to provide public health and safety. The public deserves no less.
The equal protection clause is aimed at all official state actions, not just
7-Egual Protection those of the legislature. Its inhibitions cover all the departments of the
government including the political and executive departments, and extend
The petitioners also claim that the RH Law violates the equal protection to all actions of a state denying equal protection of the laws, through
clause under the Constitution as it discriminates against the poor because whatever agency or whatever guise is taken.
it makes them the primary target of the government program that
promotes contraceptive use . They argue that, rather than promoting It, however, does not require the universal application of the laws to all
reproductive health among the poor, the RH Law introduces contraceptives persons or things without distinction. What it simply requires is equality
that would effectively reduce the number of the poor. Their bases are the among equals as determined according to a valid classification. Indeed,
various provisions in the RH Law dealing with the poor, especially those the equal protection clause permits classification. Such classification,
mentioned in the guiding principles259 and definition of terms260 of the law. however, to be valid must pass the test of reasonableness. The test has
four requisites: (1) The classification rests on substantial distinctions; (2)
They add that the exclusion of private educational institutions from the It is germane to the purpose of the law; (3) It is not limited to existing
mandatory reproductive health education program imposed by the RH Law conditions only; and (4) It applies equally to all members of the same
renders it unconstitutional. class. "Superficial differences do not make for a valid classification."

In Biraogo v. Philippine Truth Commission,261 the Court had the occasion to For a classification to meet the requirements of constitutionality, it must
expound on the concept of equal protection. Thus: include or embrace all persons who naturally belong to the class. "The
classification will be regarded as invalid if all the members of the class are
not similarly treated, both as to rights conferred and obligations imposed.
One of the basic principles on which this government was founded is that
It is not necessary that the classification be made with absolute symmetry,
of the equality of right which is embodied in Section 1, Article III of the
in the sense that the members of the class should possess the same
1987 Constitution. The equal protection of the laws is embraced in the
characteristics in equal degree. Substantial similarity will suffice; and as
concept of due process, as every unfair discrimination offends the
long as this is achieved, all those covered by the classification are to be
requirements of justice and fair play. It has been embodied in a separate
treated equally. The mere fact that an individual belonging to a class
clause, however, to provide for a more specific guaranty against any form
differs from the other members, as long as that class is substantially
of undue favoritism or hostility from the government. Arbitrariness in
general may be challenged on the basis of the due process clause. But if
distinguishable from all others, does not justify the non-application of the suffice it to state that the mere fact that the children of those who are less
law to him." fortunate attend public educational institutions does not amount to
substantial distinction sufficient to annul the assailed provision. On the
The classification must not be based on existing circumstances only, or so other hand, substantial distinction rests between public educational
constituted as to preclude addition to the number included in the class. It institutions and private educational institutions, particularly because there
must be of such a nature as to embrace all those who may thereafter be in is a need to recognize the academic freedom of private educational
similar circumstances and conditions. It must not leave out or institutions especially with respect to religious instruction and to consider
"underinclude" those that should otherwise fall into a certain classification. their sensitivity towards the teaching of reproductive health education.
[Emphases supplied; citations excluded]
8-Involuntary Servitude
To provide that the poor are to be given priority in the government's
reproductive health care program is not a violation of the equal protection The petitioners also aver that the RH Law is constitutionally infirm as it
clause. In fact, it is pursuant to Section 11, Article XIII of the Constitution violates the constitutional prohibition against involuntary servitude. They
which recognizes the distinct necessity to address the needs of the posit that Section 17 of the assailed legislation requiring private and non-
underprivileged by providing that they be given priority in addressing the government health care service providers to render forty-eight (48) hours
health development of the people. Thus: of pro bono reproductive health services, actually amounts to involuntary
servitude because it requires medical practitioners to perform acts against
Section 11. The State shall adopt an integrated and comprehensive their will.262
approach to health development which shall endeavor to make essential
goods, health and other social services available to all the people at The OSG counters that the rendition of pro bono services envisioned in
affordable cost. There shall be priority for the needs of the Section 17 can hardly be considered as forced labor analogous to slavery,
underprivileged, sick, elderly, disabled, women, and children. The State as reproductive health care service providers have the discretion as to the
shall endeavor to provide free medical care to paupers. manner and time of giving pro bono services. Moreover, the OSG points
out that the imposition is within the powers of the government, the
It should be noted that Section 7 of the RH Law prioritizes poor and accreditation of medical practitioners with PhilHealth being a privilege and
marginalized couples who are suffering from fertility issues and desire to not a right.
have children. There is, therefore, no merit to the contention that the RH
Law only seeks to target the poor to reduce their number. While the RH The point of the OSG is well-taken.
Law admits the use of contraceptives, it does not, as elucidated above,
sanction abortion. As Section 3(1) explains, the "promotion and/or It should first be mentioned that the practice of medicine is undeniably
stabilization of the population growth rate is incidental to the advancement imbued with public interest that it is both a power and a duty of the State
of reproductive health." to control and regulate it in order to protect and promote the public
welfare. Like the legal profession, the practice of medicine is not a right
Moreover, the RH Law does not prescribe the number of children a couple but a privileged burdened with conditions as it directly involves the very
may have and does not impose conditions upon couples who intend to lives of the people. A fortiori, this power includes the power of
have children. While the petitioners surmise that the assailed law seeks to Congress263 to prescribe the qualifications for the practice of professions or
charge couples with the duty to have children only if they would raise trades which affect the public welfare, the public health, the public morals,
them in a truly humane way, a deeper look into its provisions shows that and the public safety; and to regulate or control such professions or
what the law seeks to do is to simply provide priority to the poor in the trades, even to the point of revoking such right altogether. 264
implementation of government programs to promote basic reproductive
health care. Moreover, as some petitioners put it, the notion of involuntary servitude
connotes the presence of force, threats, intimidation or other similar
With respect to the exclusion of private educational institutions from the means of coercion and compulsion.265 A reading of the assailed provision,
mandatory reproductive health education program under Section 14, however, reveals that it only encourages private and non- government
reproductive healthcare service providers to render pro bono service. "(b) To assume primary jurisdiction in the collection of samples of
Other than non-accreditation with PhilHealth, no penalty is imposed should health products;
they choose to do otherwise. Private and non-government reproductive
healthcare service providers also enjoy the liberty to choose which kind of "(c) To analyze and inspect health products in connection with the
health service they wish to provide, when, where and how to provide it or implementation of this Act;
whether to provide it all. Clearly, therefore, no compulsion, force or threat
is made upon them to render pro bono service against their will. While the
"(d) To establish analytical data to serve as basis for the
rendering of such service was made a prerequisite to accreditation with
preparation of health products standards, and to recommend
PhilHealth, the Court does not consider the same to be an unreasonable
standards of identity, purity, safety, efficacy, quality and fill of
burden, but rather, a necessary incentive imposed by Congress in the
container;
furtherance of a perceived legitimate state interest.

"(e) To issue certificates of compliance with technical requirements


Consistent with what the Court had earlier discussed, however, it should
to serve as basis for the issuance of appropriate authorization and
be emphasized that conscientious objectors are exempt from this provision
spot-check for compliance with regulations regarding operation of
as long as their religious beliefs and convictions do not allow them to
manufacturers, importers, exporters, distributors, wholesalers,
render reproductive health service, pro bona or otherwise.
drug outlets, and other establishments and facilities of health
products, as determined by the FDA;
9-Delegation of Authority to the FDA
"x x x
The petitioners likewise question the delegation by Congress to the FDA of
the power to determine whether or not a supply or product is to be
"(h) To conduct appropriate tests on all applicable health products
included in the Essential Drugs List (EDL).266
prior to the issuance of appropriate authorizations to ensure
safety, efficacy, purity, and quality;
The Court finds nothing wrong with the delegation. The FDA does not only
have the power but also the competency to evaluate, register and cover
"(i) To require all manufacturers, traders, distributors, importers,
health services and methods. It is the only government entity empowered
exporters, wholesalers, retailers, consumers, and non-consumer
to render such services and highly proficient to do so. It should be
users of health products to report to the FDA any incident that
understood that health services and methods fall under the gamut of
reasonably indicates that said product has caused or contributed to
terms that are associated with what is ordinarily understood as "health
the death, serious illness or serious injury to a consumer, a
products."
patient, or any person;

In this connection, Section 4 of R.A. No. 3 720, as amended by R.A. No.


"(j) To issue cease and desist orders motu propio or upon verified
9711 reads:
complaint for health products, whether or not registered with the
FDA Provided, That for registered health products, the cease and
SEC. 4. To carry out the provisions of this Act, there is hereby created an desist order is valid for thirty (30) days and may be extended for
office to be called the Food and Drug Administration (FDA) in the sixty ( 60) days only after due process has been observed;
Department of Health (DOH). Said Administration shall be under the Office
of the Secretary and shall have the following functions, powers and duties:
"(k) After due process, to order the ban, recall, and/or withdrawal
of any health product found to have caused death, serious illness
"(a) To administer the effective implementation of this Act and of or serious injury to a consumer or patient, or is found to be
the rules and regulations issued pursuant to the same; imminently injurious, unsafe, dangerous, or grossly deceptive, and
to require all concerned to implement the risk management plan
which is a requirement for the issuance of the appropriate and functions currently vested upon them. They shall also
authorization; discharge the functions and responsibilities of national agencies
and offices devolved to them pursuant to this Code. Local
x x x. government units shall likewise exercise such other powers and
discharge such other functions and responsibilities as are
necessary, appropriate, or incidental to efficient and effective
As can be gleaned from the above, the functions, powers and duties of the
provision of the basic services and facilities enumerated herein.
FDA are specific to enable the agency to carry out the mandates of the
law. Being the country's premiere and sole agency that ensures the safety
of food and medicines available to the public, the FDA was equipped with (b) Such basic services and facilities include, but are not limited to,
the necessary powers and functions to make it effective. Pursuant to the x x x.
principle of necessary implication, the mandate by Congress to the FDA to
ensure public health and safety by permitting only food and medicines that While the aforementioned provision charges the LGUs to take on
are safe includes "service" and "methods." From the declared policy of the the functions and responsibilities that have already been devolved
RH Law, it is clear that Congress intended that the public be given only upon them from the national agencies on the aspect of providing
those medicines that are proven medically safe, legal, non-abortifacient, for basic services and facilities in their respective jurisdictions,
and effective in accordance with scientific and evidence-based medical paragraph (c) of the same provision provides a categorical
research standards. The philosophy behind the permitted delegation was exception of cases involving nationally-funded projects, facilities,
explained in Echagaray v. Secretary of Justice,267 as follows: programs and services.268 Thus:

The reason is the increasing complexity of the task of the government and (c) Notwithstanding the provisions of subsection (b) hereof, public
the growing inability of the legislature to cope directly with the many works and infrastructure projects and other facilities, programs
problems demanding its attention. The growth of society has ramified its and services funded by the National Government under the annual
activities and created peculiar and sophisticated problems that the General Appropriations Act, other special laws, pertinent executive
legislature cannot be expected reasonably to comprehend. Specialization orders, and those wholly or partially funded from foreign sources,
even in legislation has become necessary. To many of the problems are not covered under this Section, except in those cases where
attendant upon present day undertakings, the legislature may not have the local government unit concerned is duly designated as the
the competence, let alone the interest and the time, to provide the implementing agency for such projects, facilities, programs and
required direct and efficacious, not to say specific solutions. services. [Emphases supplied]

10- Autonomy of Local Governments and the Autonomous Region The essence of this express reservation of power by the national
government is that, unless an LGU is particularly designated as the
of Muslim Mindanao (ARMM) implementing agency, it has no power over a program for which funding
has been provided by the national government under the annual general
appropriations act, even if the program involves the delivery of basic
As for the autonomy of local governments, the petitioners claim that the
services within the jurisdiction of the LGU. 269 A complete relinquishment of
RH Law infringes upon the powers devolved to local government units
central government powers on the matter of providing basic facilities and
(LGUs) under Section 17 of the Local Government Code. Said Section 17
services cannot be implied as the Local Government Code itself weighs
vested upon the LGUs the duties and functions pertaining to the delivery of
against it.270
basic services and facilities, as follows:

In this case, a reading of the RH Law clearly shows that whether it


SECTION 17. Basic Services and Facilities. –
pertains to the establishment of health care facilities, 271 the hiring of skilled
health professionals,272 or the training of barangay health workers,273 it will
(a) Local government units shall endeavor to be self-reliant and be the national government that will provide for the funding of its
shall continue exercising the powers and discharging the duties implementation. Local autonomy is not absolute. The national government
still has the say when it comes to national priority programs which the written law, it cannot serve as a basis to strike down a law. In Republic v.
local government is called upon to implement like the RH Law. Sandiganbayan,278 the very case cited by the petitioners, it was explained
that the Court is not duty-bound to examine every law or action and
Moreover, from the use of the word "endeavor," the LG Us are merely whether it conforms with both the Constitution and natural law. Rather,
encouraged to provide these services. There is nothing in the wording of natural law is to be used sparingly only in the most peculiar of
the law which can be construed as making the availability of these services circumstances involving rights inherent to man where no law is
mandatory for the LGUs. For said reason, it cannot be said that the RH applicable.279
Law amounts to an undue encroachment by the national government upon
the autonomy enjoyed by the local governments. At any rate, as earlier expounded, the RH Law does not sanction the
taking away of life. It does not allow abortion in any shape or form. It only
The ARMM seeks to enhance the population control program of the government by
providing information and making non-abortifacient contraceptives more
readily available to the public, especially to the poor.
The fact that the RH Law does not intrude in the autonomy of local
governments can be equally applied to the ARMM. The RH Law does not
infringe upon its autonomy. Moreover, Article III, Sections 6, 10 and 11 of Facts and Fallacies
R.A. No. 9054, or the organic act of the ARMM, alluded to by petitioner
Tillah to justify the exemption of the operation of the RH Law in the and the Wisdom of the Law
autonomous region, refer to the policy statements for the guidance of the
regional government. These provisions relied upon by the petitioners In general, the Court does not find the RH Law as unconstitutional insofar
simply delineate the powers that may be exercised by the regional as it seeks to provide access to medically-safe, non-abortifacient,
government, which can, in no manner, be characterized as an abdication effective, legal, affordable, and quality reproductive healthcare services,
by the State of its power to enact legislation that would benefit the general methods, devices, and supplies. As earlier pointed out, however, the
welfare. After all, despite the veritable autonomy granted the ARMM, the religious freedom of some sectors of society cannot be trampled upon in
Constitution and the supporting jurisprudence, as they now stand, reject pursuit of what the law hopes to achieve. After all, the Constitutional
the notion of imperium et imperio in the relationship between the national safeguard to religious freedom is a recognition that man stands
and the regional governments.274 Except for the express and implied accountable to an authority higher than the State.
limitations imposed on it by the Constitution, Congress cannot be
restricted to exercise its inherent and plenary power to legislate on all
In conformity with the principle of separation of Church and State, one
subjects which extends to all matters of general concern or common
religious group cannot be allowed to impose its beliefs on the rest of the
interest.275
society. Philippine modem society leaves enough room for diversity and
pluralism. As such, everyone should be tolerant and open-minded so that
11 - Natural Law peace and harmony may continue to reign as we exist alongside each
other.
With respect to the argument that the RH Law violates natural
law,276 suffice it to say that the Court does not duly recognize it as a legal As healthful as the intention of the RH Law may be, the idea does not
basis for upholding or invalidating a law. Our only guidepost is the escape the Court that what it seeks to address is the problem of rising
Constitution. While every law enacted by man emanated from what is poverty and unemployment in the country. Let it be said that the cause of
perceived as natural law, the Court is not obliged to see if a statute, these perennial issues is not the large population but the unequal
executive issuance or ordinance is in conformity to it. To begin with, it is distribution of wealth. Even if population growth is controlled, poverty will
not enacted by an acceptable legitimate body. Moreover, natural laws are remain as long as the country's wealth remains in the hands of the very
mere thoughts and notions on inherent rights espoused by theorists, few.
philosophers and theologists. The jurists of the philosophical school are
interested in the law as an abstraction, rather than in the actual law of the
At any rate, population control may not be beneficial for the country in the
past or present.277 Unless, a natural right has been transformed into a
long run. The European and Asian countries, which embarked on such a
program generations ago , are now burdened with ageing populations. The same, the principle of "no-abortion" and "non-coercion" in the adoption of
number of their young workers is dwindling with adverse effects on their any family planning method should be maintained.
economy. These young workers represent a significant human capital
which could have helped them invigorate, innovate and fuel their WHEREFORE, the petitions are PARTIALLY GRANTED. Accordingly, the
economy. These countries are now trying to reverse their programs, but Court declares R.A. No. 10354 as NOT UNCONSTITUTIONAL except with
they are still struggling. For one, Singapore, even with incentives, is respect to the following provisions which are declared
failing. UNCONSTITUTIONAL:

And in this country, the economy is being propped up by remittances from 1) Section 7 and the corresponding provision in the RH-IRR insofar
our Overseas Filipino Workers. This is because we have an ample supply of as they: a) require private health facilities and non-maternity
young able-bodied workers. What would happen if the country would be specialty hospitals and hospitals owned and operated by a religious
weighed down by an ageing population and the fewer younger generation group to refer patients, not in an emergency or life-threatening
would not be able to support them? This would be the situation when our case, as defined under Republic Act No. 8344, to another health
total fertility rate would go down below the replacement level of two (2) facility which is conveniently accessible; and b) allow minor-
children per woman.280 parents or minors who have suffered a miscarriage access to
modem methods of family planning without written consent from
Indeed, at the present, the country has a population problem, but the their parents or guardian/s;
State should not use coercive measures (like the penal provisions of the
RH Law against conscientious objectors) to solve it. Nonetheless, the 2) Section 23(a)(l) and the corresponding provision in the RH-IRR,
policy of the Court is non-interference in the wisdom of a law. particularly Section 5 .24 thereof, insofar as they punish any
healthcare service provider who fails and or refuses to disseminate
x x x. But this Court cannot go beyond what the legislature has laid down. information regarding programs and services on reproductive
Its duty is to say what the law is as enacted by the lawmaking body. That health regardless of his or her religious beliefs.
is not the same as saying what the law should be or what is the correct
rule in a given set of circumstances. It is not the province of the judiciary 3) Section 23(a)(2)(i) and the corresponding provision in the RH-
to look into the wisdom of the law nor to question the policies adopted by IRR insofar as they allow a married individual, not in an
the legislative branch. Nor is it the business of this Tribunal to remedy emergency or life-threatening case, as defined under Republic Act
every unjust situation that may arise from the application of a particular No. 8344, to undergo reproductive health procedures without the
law. It is for the legislature to enact remedial legislation if that would be consent of the spouse;
necessary in the premises. But as always, with apt judicial caution and
cold neutrality, the Court must carry out the delicate function of
4) Section 23(a)(2)(ii) and the corresponding provision in the RH-
interpreting the law, guided by the Constitution and existing legislation
IRR insofar as they limit the requirement of parental consent only
and mindful of settled jurisprudence. The Court's function is therefore
to elective surgical procedures.
limited, and accordingly, must confine itself to the judicial task of saying
what the law is, as enacted by the lawmaking body.281
5) Section 23(a)(3) and the corresponding provision in the RH-
IRR, particularly Section 5.24 thereof, insofar as they punish any
Be that as it may, it bears reiterating that the RH Law is a mere
healthcare service provider who fails and/or refuses to refer a
compilation and enhancement of the prior existing contraceptive and
patient not in an emergency or life-threatening case, as defined
reproductive health laws, but with coercive measures. Even if the Court
under Republic Act No. 8344, to another health care service
decrees the RH Law as entirely unconstitutional, there will still be the
provider within the same facility or one which is conveniently
Population Act (R.A. No. 6365), the Contraceptive Act (R.A. No. 4729) and
accessible regardless of his or her religious beliefs;
the reproductive health for women or The Magna Carta of Women (R.A.
No. 9710), sans the coercive provisions of the assailed legislation. All the
6) Section 23(b) and the corresponding provision in the RH-IRR,
particularly Section 5 .24 thereof, insofar as they punish any public
officer who refuses to support reproductive health programs or
shall do any act that hinders the full implementation of a
reproductive health program, regardless of his or her religious
beliefs;

7) Section 17 and the corresponding prov1s10n in the RH-IRR


regarding the rendering of pro bona reproductive health service in
so far as they affect the conscientious objector in securing
PhilHealth accreditation; and

8) Section 3.0l(a) and Section 3.01 G) of the RH-IRR, which added


the qualifier "primarily" in defining abortifacients and
contraceptives, as they are ultra vires and, therefore, null and void
for contravening Section 4(a) of the RH Law and violating Section
12, Article II of the Constitution.

The Status Quo Ante Order issued by the Court on March 19, 2013 as
extended by its Order, dated July 16, 2013 , is hereby LIFTED, insofar as
the provisions of R.A. No. 10354 which have been herein declared as
constitutional.

SO ORDERED.
December 3, 2009

EN BANC

G.R. No. 182498


GEN. AVELINO I. RAZON, JR.,
Chief, Philippine National Police Present:
(PNP); Police Chief Superintendent
RAUL CASTAEDA, Chief, Criminal
Investigation and Detection Group PUNO, C.J.,
(CIDG); Police Senior
Superintendent LEONARDO A. CARPIO,
ESPINA, Chief, Police Anti-Crime
and Emergency Response CORONA,
(PACER); and GEN. JOEL R.
GOLTIAO, Regional Director of CARPIO MORALES,
ARMM, PNP,
Petitioners, CHICO-NAZARIO,
- versus -
VELASCO, JR.,

NACHURA,

LEONARDO-DE CASTRO,
MARY JEAN B. TAGITIS, herein
represented by ATTY. FELIPE P. BRION,
ARCILLA, JR., Attorney-in-Fact,
PERALTA,
Respondent.
BERSAMIN,

DEL CASTILLO,

ABAD, and x----------------------------------------------------------------------------------


-------x
VILLARAMA, JR., JJ.
DECISION

BRION, J.:

We review in this petition for review on certiorari[1] the decision


Promulgated: dated March 7, 2008 of the Court of Appeals (CA) in C.A-G.R. AMPARO No.
00009.[2] This CA decision confirmed the enforced disappearance of
Engineer Morced N. Tagitis (Tagitis) and granted the Writ of Amparo at the
petition of his wife, Mary Jean B. Tagitis (respondent). The dispositive This Decision reflects the nature of the Writ of Amparo a protective
portion of the CA decision reads: remedy against violations or threats of violation against the rights to
life, liberty and security.[3] It embodies, as a remedy, the courts
directive to police agencies to undertake specified courses of action to
address the disappearance of an individual, in this case, Engr. Morced
WHEREFORE, premises considered, petition is N. Tagitis. It does not determine guilt nor pinpoint criminal culpability
hereby GRANTED. The Court hereby FINDS that this is for the disappearance; rather, it determines responsibility, or at
an enforced disappearance within the meaning of the least accountability, for the enforced disappearance for purposes of
United Nations instruments, as used in the Amparo imposing the appropriate remedies to address the
Rules. The privileges of the writ of amparo are hereby disappearance. Responsibility refers to the extent the actors have
extended to Engr. Morced Tagitis. been established by substantial evidence to have participated in
whatever way, by action or omission, in an enforced disappearance, as
a measure of the remedies this Court shall craft, among them, the
directive to file the appropriate criminal and civil cases against the
Consequently: (1) respondent GEN. EDGARDO M. responsible parties in the proper courts. Accountability, on the other
DOROMAL, Chief, Criminal Investigation and Detention hand, refers to the measure of remedies that should be addressed to
Group (CIDG) who should order COL. JOSE VOLPANE those who exhibited involvement in the enforced disappearance
PANTE, CIDG-9 Chief, Zamboanga City, to aid him; (2) without bringing the level of their complicity to the level of
respondent GEN. AVELINO I. RAZON, Chief, PNP, who responsibility defined above; or who are imputed with knowledge
should order his men, namely: (a) respondent GEN. JOEL relating to the enforced disappearance and who carry the burden of
GOLTIAO, Regional Director of ARMM PNP, (b) COL. disclosure; or those who carry, but have failed to discharge, the
AHIRON AJIRIM, both head of TASK FORCE TAGITIS, burden of extraordinary diligence in the investigation of the enforced
and (c) respondent SR. SUPERINTENDENT LEONARDO disappearance. In all these cases, the issuance of the Writ
A. ESPINA, Chief, Police Anti-Crime and Emergency of Amparo is justified by our primary goal of addressing the
Response, to aid him as their superior- are disappearance, so that the life of the victim is preserved and his liberty
hereby DIRECTED to exert extraordinary diligence and and security are restored.
efforts, not only to protect the life, liberty and security of
Engr. Morced Tagitis, but also to extend the privilegesof We highlight this nature of a Writ of Amparo case at the outset to
the writ of amparo to Engr. Morced Tagitis and his family, stress that the unique situations that call for the issuance of the writ,
and to submit a monthly report of their actions to this as well as the considerations and measures necessary to address these
Court, as a way of PERIODIC REVIEW to enable this situations, may not at all be the same as the standard measures and
Court to monitor the action of respondents. procedures in ordinary court actions and proceedings. In this sense,
the Rule on the Writ of Amparo[4](Amparo Rule) issued by this Court is
unique. The Amparo Rule should be read, too, as a work in progress,
as its directions and finer points remain to evolve through time and
This amparo case is hereby DISMISSED as to
jurisprudence and through the substantive laws that Congress may
respondent LT. GEN. ALEXANDER YANO, Commanding
promulgate.
General, Philippine Army, and as to respondent GEN.
RUBEN RAFAEL, Chief Anti-Terror Task Force Comet,
THE FACTUAL ANTECEDENTS
Zamboanga City, both being with the military, which is a
separate and distinct organization from the police and the
The background facts, based on the petition and the records of the
CIDG, in terms of operations, chain of command and
case, are summarized below.
budget.
The established facts show that Tagitis, a consultant for the World
Bank and the Senior Honorary Counselor for the Islamic Development
Bank (IDB) Scholarship Programme, was last seen in Jolo,
Sulu. Together with Arsimin Kunnong (Kunnong), an IDB scholar,
Tagitis arrived in Jolo by boat in the early morning of October 31, 2007 9. Kunnong requested for the key from the desk of the pension
from a seminar in Zamboanga City.They immediately checked-in at house who [sic] assisted him to open the room of Engr.
ASY Pension House. Tagitis asked Kunnong to buy him a boat ticket for Tagitis, where they discovered that the personal
his return trip the following day to Zamboanga. When Kunnong belongings of Engr. Tagitis, including cell phones,
returned from this errand, Tagitis was no longer around. [5] The documents and other personal belongings were all intact
receptionist related that Tagitis went out to buy food at around 12:30 inside the room;
in the afternoon and even left his room key with the desk. [6]Kunnong
looked for Tagitis and even sent a text message to the latters Manila- 10. When Kunnong could not locate Engr. Tagitis, the former
based secretary who did not know of Tagitis whereabouts and sought the help of another IDB scholar and reported the
activities either; she advised Kunnong to simply wait. [7] matter to the local police agency;

On November 4, 2007, Kunnong and Muhammad Abdulnazeir N. Matli, 11. Arsimin Kunnong including his friends and companions in Jolo,
a UP professor of Muslim studies and Tagitis fellow student counselor exerted efforts in trying to locate the whereabouts of Engr.
at the IDB, reported Tagitis disappearance to the Jolo Police Station. Tagitis and when he reported the matter to the police
[8]
On November 7, 2007, Kunnong executed a sworn affidavit attesting authorities in Jolo, he was immediately given a ready
to what he knew of the circumstances surrounding Tagitis answer that Engr. Tagitis could have been abducted by the
disappearance.[9] Abu Sayyaf group and other groups known to be fighting
against the government;
More than a month later (on December 28, 2007), the respondent filed
a Petition for the Writ of Amparo (petition) with the CA through her 12. Being scared with [sic] these suggestions and insinuations of
Attorney-in-Fact, Atty. Felipe P. Arcilla.[10]The petition was directed the police officers, Kunnong reported the matter to the
against Lt. Gen. Alexander Yano, Commanding General, Philippine [respondent, wife of Engr. Tagitis] by phone and other
Army; Gen. Avelino I. Razon, Chief, Philippine National Police (PNP); responsible officers and coordinators of the IDB
Gen. Edgardo M. Doromal, Chief, Criminal Investigation and Detention Scholarship Programme in the Philippines, who alerted the
Group (CIDG); Sr. Supt. Leonardo A. Espina, Chief, Police Anti-Crime office of the Governor of ARMM who was then preparing to
and Emergency Response; Gen. Joel Goltiao, Regional Director, ARMM- attend the OIC meeting in Jeddah, Saudi Arabia;
PNP; and Gen. Ruben Rafael, Chief, Anti-Terror Task Force Comet
[collectively referred to as petitioners]. After reciting Tagitis personal 13. [Respondent], on the other hand, approached some of her co-
circumstances and the facts outlined above, the petition went on to employees with the Land Bank in Digos branch, Digos City,
state: Davao del Sur who likewise sought help from some of their
xxxx friends in the military who could help them find/locate the
7. Soon after the student left the room, Engr. Tagitis went out of whereabouts of her husband;
the pension house to take his early lunch but while out on
the street, a couple of burly men believed to be police 14. All of these efforts of the [respondent] did not produce any
intelligence operatives, forcibly took him and positive results except the information from persons in the
boarded the latter on a motor vehicle then sped military who do not want to be identified that Engr. Tagitis
away without the knowledge of his student, Arsimin is in the hands of the uniformed men;
Kunnong;
15. According to reliable information received by the
8. As instructed, in the late afternoon of the same day, Kunnong [respondent], subject Engr. Tagitis is in the custody of
returned to the pension house, and was surprised to find police intelligence operatives, specifically with the
out that subject Engr. Tagitis cannot [sic] be contacted by CIDG, PNP Zamboanga City, being held against his
phone and was not also around and his room was closed will in an earnest attempt of the police to involve
and locked; and connect Engr. Tagitis with the different terrorist
groups;
xxxx almost two (2) months now, clearly indicates that the
17. [Respondent] filed her complaint with the PNP Police Station in [petitioners] are actually in physical possession and
the ARMM in Cotobato and in Jolo, as suggested by her custody of [respondents] husband, Engr. Tagitis;
friends, seeking their help to find her husband, but
[respondents] request and pleadings failed to produce any xxxx
positive results;
25. [The respondent] has exhausted all administrative avenues
18. Instead of helping the [respondent], she [sic] was told of an and remedies but to no avail, and under the
intriguing tale by the police that her husband, subject of circumstances, [the respondent] has no other plain,
the petition, was not missing but was with another woman speedy and adequate remedy to protect and get the
having good time somewhere, which is a clear indication of release of subject Engr. Morced Tagitis from the illegal
the [petitioners] refusal to help and provide police clutches of the [petitioners], their intelligence operatives
assistance in locating her missing husband; and the like which are in total violation of the subjects
human and constitutional rights, except the issuance of
19. The continued failure and refusal of the [petitioners] to release a WRIT OF AMPARO. [Emphasis supplied]
and/or turn-over subject Engr. Tagitis to his family or even
to provide truthful information to [the respondent] of the On the same day the petition was filed, the CA immediately issued the
subjects whereabouts, and/or allow [the respondent] to Writ of Amparo, set the case for hearing on January 7, 2008, and
visit her husband Engr. Morced Tagitis, caused so much directed the petitioners to file their verified return within seventy-two
sleepless nights and serious anxieties; (72) hours from service of the writ.[11]

20. Lately, [the respondent] was again advised by one of the In their verified Return filed during the hearing of January 27,
[petitioners] to go to the ARMM Police Headquarters again 2008, the petitioners denied any involvement in or knowledge of
in Cotobato City and also to the different Police Tagitis alleged abduction. They argued that the allegations of the
Headquarters including [those] in Davao City, in petition were incomplete and did not constitute a cause of action
Zamboanga City, in Jolo, and in Camp Crame, Quezon against them; were baseless, or at best speculative; and were merely
City, and all these places have been visited by the based on hearsay evidence. [12]
[respondent] in search for her husband, which entailed
expenses for her trips to these places thereby resorting The affidavit of PNP Chief Gen. Avelino I. Razon, attached to
her to borrowings and beggings [sic] for financial help the Return, stated that: he did not have any personal knowledge of, or
from friends and relatives only to try complying [sic] to the any participation in, the alleged disappearance; that he had been
different suggestions of these police officers, despite of designated by President Gloria Macapagal Arroyo as the head of a
which, her efforts produced no positive results up to the special body called TASK FORCE USIG, to address concerns about
present time; extralegal killings and enforced disappearances; the Task Force, inter
alia, coordinated with the investigators and local police, held case
21. In fact at times, some police officers, who [sympathized conferences, rendered legal advice in connection to these cases; and
with] the sufferings undergone by the gave the following summary:[13]
[respondent], informed her that they are not the proper
persons that she should approach, but assured her not to xxxx
worry because her husband is [sic] in good hands; 4.
a) On November 5, 2007, the Regional
22. The unexplained uncooperative behavior of the [petitioners] to Director, Police Regional Office ARMM submitted a report
the [respondents] request for help and failure and refusal on the alleged disappearance of one Engr. Morced
of the [petitioners] to extend the needed help, support and Tagitis. According to the said report, the victim checked-in
assistance in locating the whereabouts of Engr. Tagitis who at ASY Pension House on October 30, 2007 at about 6:00
had been declared missing since October 30, 2007 which is in the morning and then roamed around Jolo, Sulu with an
unidentified companion. It was only after a few days when perpetrators to bring them before the bar of justice and
the said victim did not return that the matter was reported secure their conviction in court.
to Jolo MPS. Afterwards, elements of Sulu PPO conducted a
thorough investigation to trace and locate the whereabouts
of the said missing person, but to no avail. The said PPO is The petitioner PNP-CIDG Chief, Gen. Edgardo M. Doromal, submitted
still conducting investigation that will lead to the as well his affidavit, also attached to the Return of the Writ, attesting
immediate findings of the whereabouts of the person. that upon receipt of the Writ of Amparo, he caused the following:[14]
xxxx
b) Likewise, the Regional Chief, 9RCIDU That immediately upon receipt on December 29, 2007 of the
submitted a Progress Report to the Director, CIDG. The Resolution of the Honorable Special Fourth Division of the
said report stated among others that: subject person Court of Appeals, I immediately directed the Investigation
attended an Education Development Seminar set on Division of this Group [CIDG] to conduct urgent
October 28, 2007 conducted at Ateneo de Zamboanga, investigation on the alleged enforced disappearance of
Zamboanga City together with a Prof. Matli. On October Engineer Morced Tagitis.
30, 2007, at around 5:00 oclock in the morning, Engr.
Tagitis reportedly arrived at Jolo Sulu wharf aboard That based on record, Engr. Morced N. Tagitis attended an
M/V Bounty Cruise, he was then billeted at ASY Pension Education Development Seminar on October 28, 2007 at
House. At about 6:15 oclock in the morning of the same Ateneo de Zamboanga at Zamboanga City together with
date, he instructed his student to purchase a fast craft Prof. Abdulnasser Matli. On October 30, 2007, at around
ticket bound for Zamboanga City and will depart from Jolo, six oclock in the morning he arrived at Jolo, Sulu. He was
Sulu on October 31, 2007. That on or about 10:00 oclock assisted by his student identified as Arsimin Kunnong of
in the morning, Engr. Tagitis left the premises of ASY the Islamic Development Bank who was also one of the
Pension House as stated by the cashier of the said pension participants of the said seminar. He checked in at ASY
house. Later in the afternoon, the student instructed to pension house located [sic] Kakuyagan, Patikul, Sulu on
purchase the ticket arrived at the pension house and October 30, 2007 with [sic] unidentified companion. At
waited for Engr. Tagitis, but the latter did not return. On around six oclock in the morning of even date, Engr.
its part, the elements of 9RCIDU is now conducting a Tagitis instructed his student to purchase a fast craft ticket
continuous case build up and information gathering to for Zamboanga City. In the afternoon of the same date,
locate the whereabouts of Engr. Tagitis. Kunnong arrived at the pension house carrying the ticket
he purchased for Engr. Tagitis, but the latter was nowhere
c) That the Director, CIDG directed the to be found anymore. Kunnong immediately informed Prof.
conduct of the search in all divisions of the CIDG to find Abdulnasser Matli who reported the incident to the
Engr. Tagitis who was allegedly abducted or illegally police. The CIDG is not involved in the disappearance of
detained by covert CIDG-PNP Intelligence Operatives since Engr. Morced Tagitis to make out a case of an enforced
October 30, 2007, but after diligent and thorough search, disappearance which presupposes a direct or indirect
records show that no such person is being detained in involvement of the government.
CIDG or any of its department or divisions.
That herein [petitioner] searched all divisions and
5. On this particular case, the Philippine National Police departments for a person named Engr. Morced N. Tagitis,
exhausted all possible efforts, steps and actions available who was allegedly abducted or illegally detained by covert
under the circumstances and continuously search and CIDG-PNP Intelligence Operatives since October 30, 2007
investigate [sic] the instant case.This immense mandate, and after a diligent and thorough research records show
however, necessitates the indispensable role of the that no such person is being detained in CIDG or any of its
citizenry, as the PNP cannot stand alone without the department or divisions.
cooperation of the victims and witnesses to identify the
That nevertheless, in order to determine the circumstances or practice that may have brought about the
surrounding Engr. Morced Tagitis [sic] alleged enforced disappearance.
disappearance, the undersigned had undertaken immediate
investigation and will pursue investigations up to its full That I further directed the chief of PACER-MOR, Police
completion in order to aid in the prosecution of the person Superintendent JOSE ARNALDO BRIONES JR., to submit a
or persons responsible therefore. written report regarding the disappearance of ENGR.
MORCED.
Likewise attached to the Return of the Writ was PNP-PACER [15] Chief PS
Supt. Leonardo A. Espinas affidavit which alleged that: [16] That in compliance with my directive, the chief of PACER-
MOR sent through fax his written report.
xxxx
That the investigation and measures being undertaken to
That, I and our men and women in PACER vehemently locate/search the subject in coordination with Police
deny any participation in the alleged abduction or illegally Regional Office, Autonomous Region of Muslim Mindanao
[sic] detention of ENGR. MORCED N. TAGITS on October (PRO-ARMM) and Jolo Police Provincial Office (PPO) and
30, 2007. As a matter of fact, nowhere in the writ was other AFP and PNP units/agencies in the area are ongoing
mentioned that the alleged abduction was perpetrated by with the instruction not to leave any stone unturned so to
elements of PACER nor was there any indication that the speak in the investigation until the perpetrators in the
alleged abduction or illegal detention of ENGR. TAGITIS instant case are brought to the bar of justice.
was undertaken jointly by our men and by
the alleged covert CIDG-PNP intelligence That I have exercised EXTRAORDINARY DILIGENCE in
operatives alleged to have abducted or illegally detained dealing with the WRIT OF AMPARO just issued.
ENGR. TAGITIS.
That I was shocked when I learned that I was implicated in
the alleged disappearance of ENGR. MORCED in my Finally, the PNP PRO ARMM Regional Director PC Supt. Joel R. Goltiao
capacity as the chief PACER [sic] considering that our (Gen. Goltiao), also submitted his affidavit detailing the actions that he
office, the Police Anti-Crime and Emergency Response had taken upon receipt of the report on Tagitis disappearance, viz:[17]
(PACER), a special task force created for the purpose of xxxx
neutralizing or eradicating kidnap-for-ransom groups which
until now continue to be one of the menace of our society 3) For the record:
is a respondent in kidnapping or illegal detention
case. Simply put, our task is to go after kidnappers and 1. I am the Regional Director of Police
charge them in court and to abduct or illegally detain or Regional Office ARMM now and during the time of the
kidnap anyone is anathema to our mission. incident;

That right after I learned of the receipt of the WRIT OF xxxx


AMPARO, I directed the Chief of PACER Mindanao Oriental
(PACER-MOR) to conduct pro-active measures to 4. It is my duty to look into and take appropriate
investigate, locate/search the subject, identify and measures on any cases of reported enforced
apprehend the persons responsible, to recover and disappearances and when they are being alluded to my
preserve evidence related to the disappearance of ENGR. office;
MORCED TAGITIS, which may aid in the prosecution of the
person or persons responsible, to identify witnesses and 5. On November 5, 2007, the Provincial Director of Sulu
obtain statements from them concerning the Police Provincial Office reported to me through Radio
disappearance and to determine the cause, manner, Message Cite No. SPNP3-1105-07-2007 that on November
location and time of disappearance as well as any pattern 4, 2007 at around 3:30 p.m., a certain Abdulnasser Matli,
an employee of Islamic Development Bank, appeared
before the Office of the Chief of Police, Jolo Police Station, b) Radio Message Cite No. RIDMD-1128-07-361
and reported the disappearance of Engr. Morced Tagitis, dated November 28, 2007 directing PD Sulu PPO to
scholarship coordinator of Islamic Development Bank, expedite compliance to my previous directive;
Manila;
c) Memorandum dated December 14, 2007
6. There was no report that Engr. Tagibis was last seen in addressed to PD Sulu PPO reiterating our series of
the company of or taken by any member of the Philippine directives for investigation and directing him to
National Police but rather he just disappeared from ASY undertake exhaustive coordination efforts with the
Pension House situated at Kakuyagan Village, Village, owner of ASY Pension House and student scholars
Patikul, Sulu, on October 30, 2007, without any trace of of IDB in order to secure corroborative statements
forcible abduction or arrest; regarding the disappearance and whereabouts of
said personality;
7. The last known instance of communication with him was
when Arsimin Kunnong, a student scholar, was requested d) Memorandum dated December 24, 2007
by him to purchase a vessel ticket at the Office of Weezam addressed to PD Sulu PPO directing him to
Express, however, when the student returned back to ASY maximize efforts to establish clues on the
Pension House, he no longer found Engr. Tagitis there and whereabouts of Engr. Tagitis by seeking the
when he immediately inquired at the information counter cooperation of Prof. Abdulnasser Matli and Arsimin
regarding his whereabouts [sic], the person in charge in Kunnong and/or whenever necessary, for them to
the counter informed him that Engr. Tagitis had left the voluntarily submit for polygraph examination with
premises on October 30, 2007 around 1 oclock p.m. and the NBI so as to expunge all clouds of doubt that
never returned back to his room; they may somehow have knowledge or idea to his
disappearance;
8. Immediately after learning the incident, I called and
directed the Provincial Director of Sulu Police Provincial e) Memorandum dated December 27, 2007
Office and other units through phone call and text addressed to the Regional Chief, Criminal
messages to conduct investigation [sic] to determine the Investigation and Detection Group, Police Regional
whereabouts of the aggrieved party and the person or Office 9, Zamboanga City, requesting assistance to
persons responsible for the threat, act or omission, to investigate the cause and unknown disappearance
recover and preserve evidence related to the of Engr. Tagitis considering that it is within their
disappearance of Engr. Tagitis, to identify witnesses and area of operational jurisdiction;
obtain statements from them concerning his
disappearance, to determine the cause and manner of his f) Memorandum from Chief, Intelligence Division,
disappearance, to identify and apprehend the person or PRO ARMM dated December 30, 2007 addressed to
persons involved in the disappearance so that they shall be PD Sulu PPO requiring them to submit complete
brought before a competent court; investigation report regarding the case of Engr.
Tagitis;
9. Thereafter, through my Chief of the Regional
Investigation and Detection Management Division, I have 10. In compliance to our directives, PD Sulu PPO has
caused the following directives: exerted his [sic] efforts to conduct investigation [sic] on
the matter to determine the whereabouts of Engr. Tagitis
a) Radio Message Cite No. RIDMD-1122-07-358 and the circumstances related to his disappearance and
dated November 22, 2007 directing PD Sulu PPO to submitted the following:
conduct joint investigation with CIDG and CIDU
ARMM on the matter;
a) Progress Report dated November 6, 2007 through third hearing would be to mobilize the Chief of Police of Jolo, Sulu and
Radio Message Cite No. SPNP3-1106-10-2007; the Chief of Police of Zamboanga City and other police operatives. [21]
In the hearing on January 17, 2008, TASK FORCE TAGITIS submitted
b) Radio Message Cite No. SPIDMS-1205-47-07 to the CA an intelligence report from PSL Usman S. Pingay, the Chief
informing this office that they are still monitoring the of Police of the Jolo Police Station, stating a possible motive for Tagitis
whereabouts of Engr. Tagitis; disappearance.[22] The intelligence report was apparently based on the
sworn affidavit dated January 4, 2008 of Muhammad Abdulnazeir N.
c) Investigation Report dated December 31, 2007 Matli (Prof. Matli), Professor of Islamic Studies at the University of the
from the Chief of Police, Jolo Police Station, Sulu PPO; Philippines and an Honorary Student Counselor of the IDB Scholarship
Program in the Philippines, who told the Provincial Governor of Sulu
11. This incident was properly reported to the PNP Higher that:[23]
Headquarters as shown in the following:
[Based] on reliable information from the Office of Muslim
a) Memorandum dated November 6, 2007 addressed to Affairs in Manila, Tagitis has reportedly taken and carried
the Chief, PNP informing him of the facts of the away more or less Five Million Pesos (P5,000,000.00)
disappearance and the action being taken by our deposited and entrusted to his [personal] bank accounts by
office; the Central Office of IDB, Jeddah, Kingdom of Saudi
Arabia, which [was] intended for the IDB Scholarship Fund.
b) Memorandum dated November 6, 2007 addressed to
the Director, Directorate for Investigation and In the same hearing, PS Supt. Ajirim testified that since the CIDG was
Detection Management, NHQ PNP; alleged to be responsible, he personally went to the CIDG office in
Zamboanga City to conduct an ocular inspection/investigation,
c) Memorandum dated December 30, 2007 addressed particularly of their detention cells. [24] PS Supt. Ajirim stated that the
to the Director, DIDM; CIDG, while helping TASK FORCE TAGITIS investigate the
disappearance of Tagitis, persistently denied any knowledge or
4) In spite of our exhaustive efforts, the whereabouts of complicity in any abduction. [25] He further testified that prior to the
Engr. Tagitis cannot be determined but our office is hearing, he had already mobilized and given specific instructions to
continuously intensifying the conduct of information their supporting units to perform their respective tasks; that they even
gathering, monitoring and coordination for the immediate talked to, but failed to get any lead from the respondent in Jolo. [26] In
solution of the case. his submitted investigation report dated January 16, 2008, PS Supt.
Ajirim concluded:[27]
Since the disappearance of Tagistis was practically admitted and
taking note of favorable actions so far taken on the disappearance, the 9. Gleaned from the undersigned inspection and observation at the
CA directed Gen. Goltiao as the officer in command of the area of Headquarters 9 RCIDU and the documents at hand, it is
disappearance to form TASK FORCE TAGITIS.[18] my own initial conclusion that the 9RCIDU and other PNP
units in the area had no participation neither [sic]
Task Force Tagitis something to do with [sic] mysterious disappearance of
Engr. Morced Tagitis last October 30, 2007. Since doubt
On January 11, 2008, Gen. Goltiao designated PS Supt. Ahiron Ajirim has been raised regarding the emolument on the Islamic
(PS Supt. Ajirim) to head TASK FORCE TAGITIS. [19] The CA Development Bank Scholar program of IDB that was
subsequently set three hearings to monitor whether TASK FORCE reportedly deposited in the personal account of Engr.
TAGITIS was exerting extraordinary efforts in handling the Tagitis by the IDB central office in Jeddah, Kingdom of
disappearance of Tagitis.[20] As planned, (1) the first hearing would be Saudi Arabia. Secondly, it could might [sic] be done by
to mobilize the CIDG, Zamboanga City; (2) the second hearing would resentment or sour grape among students who are
be to mobilize intelligence with Abu Sayyaf and ARMM; and (3) the applying for the scholar [sic] and were denied which was
allegedly conducted/screened by the subject being the despite the Task Force Tagitis claim that they already had
coordinator of said program. an all points bulletin, since November 5, 2007, on the
missing person, Engr. Morced Tagitis. How could the police
20. It is also premature to conclude but it does or it may and [sic] look for someone who disappeared if no clear photograph
presumed that the motive behind the disappearance of the had been disseminated?
subject might be due to the funds he maliciously spent for
his personal interest and wanted to elude responsibilities (2) Furthermore, Task Force Tagitis COL.
from the institution where he belong as well as to the AHIROM AJIRIM informed this Court that P/Supt KASIM
Islamic student scholars should the statement of Prof. Matli was designated as Col. Ahirom Ajirims replacement in the
be true or there might be a professional jealousy among latters official designated post.Yet, P/Supt KASIMs
them. subpoena was returned to this Court unserved. Since this
Court was made to understand that it was P/Supt KASIM
xxxx who was the petitioners unofficial source of the military
It is recommended that the Writ of Amparo filed against intelligence information that Engr. Morced Tagitis was
the respondents be dropped and dismissed considering on abducted by bad elements of the CIDG (par. 15 of the
[sic] the police and military actions in the area particularly Petition), the close contact between P/Supt KASIM and Col.
the CIDG are exerting their efforts and religiously doing Ahirom Ajirim of TASK FORCE TAGITIS should have
their tasked [sic] in the conduct of its intelligence ensured the appearance of Col. KASIM in response to this
monitoring and investigation for the early resolution of this courts subpoena and COL. KASIM could have confirmed
instant case. But rest assured, our office, in coordination the military intelligence information that bad elements of
with other law-enforcement agencies in the area, are the CIDG had abducted Engr. Morced Tagitis.
continuously and religiously conducting our investigation
for the resolution of this case.

On February 4, 2008, the CA issued an ALARM WARNING that TASK Testimonies for the Respondent
FORCE TAGITIS did not appear to be exerting extraordinary efforts in On January 7, 2008, the respondent, Mary Jean B. Tagitis, testified on
resolving Tagitis disappearance on the following grounds: [28] direct examination that she went to Jolo and Zamboanga in her efforts
to locate her husband. She said that a friend from Zamboanga holding
(1) This Court FOUND that it was only as a high position in the military (whom she did not then identify) gave
late as January 28, 2008, after the hearing, that GEN. her information that allowed her to specify her allegations, particularly
JOEL GOLTIAO and COL. AHIRON AJIRIM had requested for paragraph 15 of the petition. [29] This friend also told her that her
clear photographs when it should have been standard husband [was] in good hands. [30] The respondent also testified that she
operating procedure in kidnappings or disappearances that sought the assistance of her former boss in Davao City, Land Bank
the first agenda was for the police to secure clear Bajada Branch Manager Rudy Salvador, who told her that PNP CIDG is
pictures of the missing person, Engr. Morced Tagitis, for holding [her husband], Engineer Morced Tagitis. [31] The respondent
dissemination to all parts of the country and to neighboring recounted that she went to Camp Katitipan in Davao City where she
countries. It had been three (3) months since GEN. JOEL met Col. Julasirim Ahadin Kasim (Col. Kasim/Sr. Supt Kasim) who read
GOLTIAO admitted having been informed on November 5, to her and her friends (who were then with her) a highly confidential
2007 of the alleged abduction of Engr. Morced Tagitis by report that contained the alleged activities of Engineer Tagitis and
alleged bad elements of the CIDG. It had been more than informed her that her husband was abducted because he is under
one (1) month since the Writ of Amparo had been issued custodial investigation for being a liaison for J.I. or Jemaah Islamiah.
on December 28, 2007. It had been three (3) weeks when [32]

battle formation was ordered through Task Force Tagitis,


on January 17, 2008. It was only on January 28, 2008 On January 17, 2008, the respondent on cross-examination testified
when the Task Force Tagitis requested for clear and recent that she is Tagitis second wife, and they have been married for
photographs of the missing person, Engr. Morced Tagitis, thirteen years; Tagitis was divorced from his first wife. [33] She last
communicated with her husband on October 29, 2007 at around 7:31 subject Engr. Tagitis. One of the very important text
p.m. through text messaging; Tagitis was then on his way to Jolo, messages of Engr. Tagitis sent to his daughter Zaynah
Sulu, from Zamboanga City.[34] Tagitis was that she was not allowed to answer any
telephone calls in his condominium unit.
The respondent narrated that she learned of her husbands
disappearance on October 30, 2007 when her stepdaughter, Zaynah While we were there he did not tell us any information of
Tagitis (Zaynah), informed her that she had not heard from her father the whereabouts of Engr. Tagitis. After the said meeting
since the time they arranged to meet in Manila on October 31, 2007. with Col. Ancanan, he treated us as guests to the city. His
[35]
The respondent explained that it took her a few days (or on two staffs accompanied us to the mall to purchase our
November 5, 2007) to personally ask Kunnong to report her husbands plane ticket going back to Davao City on November 12,
disappearance to the Jolo Police Station, since she had the impression 2007.
that her husband could not communicate with her because his cellular When we arrived in Davao City on November 12, 2007 at
phones battery did not have enough power, and that he would call her 9:00 in the morning, Col. Ancanan and I were discussing
when he had fully-charged his cellular phones battery. [36] some points through phone calls. He assured me that my
husband is alive and hes last looked [sic] in Talipapao,
The respondent also identified the high-ranking military friend, who Jolo, Sulu. Yet I did not believe his given statements of the
gave her the information found in paragraph 15 of her petition, as Lt. whereabouts of my husband, because I contacted some of
Col. Pedro L. Ancanan, Jr (Col. Ancanan). She met him in Camp my friends who have access to the groups of MILF, MNLF
Karingal, Zamboanga through her boss. [37] She also testified that she and ASG. I called up Col. Ancanan several times begging to
was with three other people, namely, Mrs. Marydel Martin Talbin and tell me the exact location of my husband and who held him
her two friends from Mati City, Davao Oriental, when Col. Kasim read but he refused.
to them the contents of the highly confidential report at Camp
Katitipan, Davao City. The respondent further narrated that the report While I was in Jolo, Sulu on November 30, 2007, I called
indicated that her husband met with people belonging to a terrorist him up again because the PNP, Jolo did not give me any
group and that he was under custodial investigation. She then told Col. information of the whereabouts of my husband. Col.
Kasim that her husband was a diabetic taking maintenance Ancanan told me that Sana ngayon alam mo na kung saan
medication, and asked that the Colonel relay to the persons holding ang kinalalagyan ng asawa mo. When I was in
him the need to give him his medication.[38] Zamboanga, I was thinking of dropping by the office of
Col. Ancanan, but I was hesitant to pay him a visit for the
On February 11, 2008, TASK FORCE TAGITIS submitted two reason that the Chief of Police of Jolo told me not to
narrative reports,[39] signed by the respondent, detailing her efforts to contact any AFP officials and he promised me that he can
locate her husband which led to her meetings with Col. Ancanan of the solve the case of my husband (Engr. Tagitis) within nine
Philippine Army and Col. Kasim of the PNP. In her narrative report days.
concerning her meeting with Col. Ancanan, the respondent
recounted, viz:[40] I appreciate the effort of Col. Ancanan on trying to solve
On November 11, 2007, we went to Zamboanga City with the case of my husband Engr. Morced Tagitis, yet failed to
my friend Mrs. Marydel Talbin. Our flight from Davao City do so.
is 9:00 oclock in the morning; we arrived at Zamboanga
Airport at around 10:00 oclock. We [were] fetched by the
two staffs of Col. Ancanan. We immediately proceed [sic] The respondent also narrated her encounter with Col. Kasim, as
to West Mindanao Command (WESTMINCOM). follows:[41]
On that same day, we had private conversation with Col. On November 7, 2007, I went to Land Bank of the
Ancanan. He interviewed me and got information about the Philippines, Bajada Branch, Davao City to meet Mr. Rudy
personal background of Engr. Morced N. Tagitis. After he Salvador. I told him that my husband, Engineer Morced
gathered all information, he revealed to us the contents of Tagitis was presumed to be abducted in Jolo, Sulu on
text messages they got from the cellular phone of the October 30, 2007. I asked him a favor to contact his
connections in the military in Jolo, Sulu where the respondent when she went to Zamboanga to see Col. Ancanan, and to
abduction of Engr. Tagitis took place. Mr. Salvador Davao City at Camp Katitipan to meet Col. Kasim.[42]
immediately called up Camp Katitipan located in Davao
City looking for high-ranking official who can help me In Zamboanga, Mrs. Talbin recounted that they met with Col.
gather reliable information behind the abduction of subject Ancanan, who told them that there was a report and that he showed
Engineer Tagitis. them a series of text messages from Tagitis cellular phone, which
showed that Tagitis and his daughter would meet in Manila on October
On that same day, Mr. Salvador and my friend, Anna 30, 2007.[43]
Mendoza, Executive Secretary, accompanied me to Camp
Katitipan to meet Col. Kasim. Mr. Salvador introduced me She further narrated that sometime on November 24, 2007, she went
to Col. Kasim and we had a short conversation. And he with the respondent together with two other companions, namely,
assured me that hell do the best he can to help me find my Salvacion Serrano and Mini Leong, to Camp Katitipan to talk to Col.
husband. Kasim.[44] The respondent asked Col. Kasim if he knew the exact
location of Engr. Tagitis. Col. Kasim told them that Tagitis was in good
After a few weeks, Mr. Salvador called me up informing me hands, although he was not certain whether he was with the PNP or
up informing me that I am to go to Camp Katitipan to meet with the Armed Forces of the Philippines (AFP). She further recounted
Col. Kasim for he has an urgent, confidential information to that based on the report Col. Kasim read in their presence, Tagitis was
reveal. under custodial investigation because he was being charged with
terrorism; Tagitis in fact had been under surveillance since January
On November 24, 2007, we went back to Camp Katitipan 2007 up to the time he was abducted when he was seen talking to
with my three friends. That was the time that Col. Kasim Omar Patik and a certain Santos of Bulacan, a Balik Islam charged
read to us the confidential report that Engr. Tagitis was with terrorism. Col. Kasim also told them that he could not give a copy
allegedly connected [with] different terrorist [groups], one of the report because it was a raw report. [45] She also related that the
of which he mentioned in the report was OMAR PATIK and Col. Kasim did not tell them exactly where Tagitis was being kept,
a certain SANTOS - a Balik Islam. although he mentioned Talipapao, Sulu. Prof., lalabas din yan.[50] Prof. Matli also
It is also said that Engr. Tagitis is carrying boxes of emphasized that despite what his January 4, 2008 affidavit indicated,
medicines for the injured terrorists as a supplier. These are [51]
he never told PS Supt. Pingay, or made any accusation, that Tagitis
the two information that I can still remember. It was took away money entrusted to him. [52] Prof. Matli confirmed, however,
written in a long bond paper with PNP Letterhead. It was that that he had received an e-mail report [53] from Nuraya Lackian of
not shown to us, yet Col. Kasim was the one who read it the Office of Muslim Affairs in Manila that the IDB was seeking
for us. assistance of the office in locating the funds of IDB scholars deposited
in Tagitis personal account.[54]
He asked a favor to me that Please dont quote my Name!
Because this is a raw report. He assured me that my On cross-examination by the respondents counsel, Prof. Matli testified
husband is alive and he is in the custody of the military for that his January 4, 2008 affidavit was already prepared when PS Supt.
custodial investigation. I told him to please take care of my Pingay asked him to sign it. [55] Prof Matli clarified that although he read
husband because he has aliments and he recently took the affidavit before signing it, he was not so much aware of [its]
insulin for he is a diabetic patient. contents.[56]

In my petition for writ of amparo, I emphasized the On February 11, 2008, the petitioners presented Col. Kasim to rebut
information that I got from Kasim. material portions of the respondents testimony, particularly the
On February 11, 2008, the respondent presented Mrs. Marydel Martin allegation that he had stated that Tagitis was in the custody of either
Talbin (Mrs. Talbin) to corroborate her testimony regarding her efforts the military or the PNP.[57] Col. Kasim categorically denied the
to locate her husband, in relation particularly with the information she statements made by the respondent in her narrative report,
received from Col. Kasim. Mrs. Talbin testified that she was with the specifically: (1) that Tagitis was seen carrying boxes of medicines as
supplier for the injured terrorists; (2) that Tagitis was under the
custody of the military, since he merely said to the respondent intelligence pinpointed the investigative arm of the PNP (CIDG) to be
that your husband is in good hands and is probably taken cared involved in the abduction, the missing-person case qualified as an
of by his armed abductors; and (3) that Tagitis was under custodial enforced disappearance. The conclusion that the CIDG was involved
investigation by the military, the PNP or the CIDG Zamboanga City. was based on the respondents testimony, corroborated by her
[58]
Col. Kasim emphasized that the informal letter he received from his companion, Mrs. Talbin. The CA noted that the information that the
informant in Sulu did not indicate that Tagitis was in the custody of the CIDG, as the police intelligence arm, was involved in Tagitis abduction
CIDG.[59] He also stressed that the information he provided to the came from no less than the military an independent agency of
respondent was merely a raw report sourced from barangay government. The CA thus greatly relied on the raw report from Col.
intelligence that still needed confirmation and follow-up as to its Kasims asset, pointing to the CIDGs involvement in Tagitis
veracity.[60] abduction. The CA held that raw reports from an asset carried great
On cross-examination, Col. Kasim testified that the information he weight in the intelligence world. It also labeled as suspect Col. Kasims
gave the respondent was given to him by his informant, who was a subsequent and belated retraction of his statement that the military,
civilian asset, through a letter which he considered as unofficial. [61] Col. the police, or the CIDG was involved in the abduction of Tagitis.
Kasim stressed that the letter was only meant for his consumption and
not for reading by others. [62] He testified further that he destroyed the The CA characterized as too farfetched and unbelievable and a bedlam
letter right after he read it to the respondent and her companions of speculation police theories painting the disappearance as intentional
because it was not important to him and also because the information on the part of Tagitis. He had no previous brushes with the law or any
it contained had no importance in relation with the abduction of record of overstepping the bounds of any trust regarding money
Tagitis.[63]He explained that he did not keep the letter because it did entrusted to him; no student of the IDB scholarship program ever
not contain any information regarding the whereabouts of Tagitis and came forward to complain that he or she did not get his or her
the person(s) responsible for his abduction. [64] stipend. The CA also found no basis for the police theory that Tagitis
was trying to escape from the clutches of his second wife, on the basis
In the same hearing on February 11, 2008, the petitioners also of the respondents testimony that Tagitis was a Muslim who could
presented Police Senior Superintendent Jose Volpane Pante (Col. have many wives under the Muslim faith, and that there was no issue
Pante), Chief of the CIDG-9, to disprove the respondents allegation at all when the latter divorced his first wife in order to marry the
that Tagitis was in the custody of CIDG-Zamboanga City. [65] Col. Pante second. Finally, the CA also ruled out kidnapping for ransom by the
clarified that the CIDG was the investigative arm of the PNP, and that Abu Sayyaf or by the ARMM paramilitary as the cause for Tagitis
the CIDG investigates and prosecutes all cases involving violations in disappearance, since the respondent, the police and the military noted
the Revised Penal Code particularly those considered as heinous that there was no acknowledgement of Tagitis abduction or demand
crimes.[66] Col. Pante further testified that the allegation that 9 RCIDU for payment of ransom the usual modus operandi of these terrorist
personnel were involved in the disappearance of Tagitis was baseless, groups.
since they did not conduct any operation in Jolo, Sulu before or after
Tagitis reported disappearance.[67] Col. Pante added that the four (4) Based on these considerations, the CA thus extended the privilege of
personnel assigned to the Sulu CIDT had no capability to conduct any the writ to Tagitis and his family, and directed the CIDG Chief, Col.
operation, since they were only assigned to investigate matters and to Jose Volpane Pante, PNP Chief Avelino I. Razon, TASK FORCE
monitor the terrorism situation.[68] He denied that his office conducted TAGITIS heads Gen. Joel Goltiao and Col. Ahiron Ajirim, and PACER
any surveillance on Tagitis prior to the latters disappearance. [69] Col. Chief Sr. Supt. Leonardo A. Espina to exert extraordinary diligence and
Pante further testified that his investigation of Tagitis disappearance efforts to protect the life, liberty and security of Tagitis, with the
was unsuccessful; the investigation was still facing a blank wall on the obligation to provide monthly reports of their actions to the CA. At the
whereabouts of Tagitis.[70] same time, the CA dismissed the petition against the then respondents
from the military, Lt. Gen Alexander Yano and Gen. Ruben Rafael,
THE CA RULING based on the finding that it was PNP-CIDG, not the military, that was
On March 7, 2008, the CA issued its decision [71] confirming that the involved.
disappearance of Tagitis was an enforced disappearance under the On March 31, 2008, the petitioners moved to reconsider the CA
United Nations (UN) Declaration on the Protection of All Persons from decision, but the CA denied the motion in its Resolution of April 9,
Enforced Disappearances.[72] The CA ruled that when military 2008.[73]
(c) The right to life, liberty and security of the
THE PETITION aggrieved party violated or threatened with
In this Rule 45 appeal questioning the CAs March 7, 2008 decision, the violation by an unlawful act or omission of the
petitioners mainly dispute the sufficiency in form and substance of respondent, and how such threat or violation is
the Amparo petition filed before the CA; the sufficiency of the legal committed with the attendant circumstances
remedies the respondent took before petitioning for the writ; the detailed in supporting affidavits;
finding that the rights to life, liberty and security of Tagitis had been
violated; the sufficiency of evidence supporting the conclusion that
Tagitis was abducted; the conclusion that the CIDG Zamboanga was
responsible for the abduction; and, generally, the ruling that the (d) The investigation conducted, if any, specifying
respondent discharged the burden of proving the allegations of the the names, personal circumstances, and addresses
petition by substantial evidence.[74] of the investigating authority or individuals, as well
as the manner and conduct of the investigation,
THE COURTS RULING together with any report;

We do not find the petition meritorious.

Sufficiency in Form and Substance (e) The actions and recourses taken by the
petitioner to determine the fate or whereabouts of
In questioning the sufficiency in form and substance of the the aggrieved party and the identity of the person
respondents Amparo petition, the petitioners contend that the petition responsible for the threat, act or omission; and
violated Section 5(c), (d), and (e) of the Amparo Rule. Specifically, the
petitioners allege that the respondent failed to:
The framers of the Amparo Rule never intended Section 5(c) to be
1) allege any act or omission the petitioners committed in
complete in every detail in stating the threatened or actual violation of a
violation of Tagitis rights to life, liberty and security;
victims rights. As in any other initiatory pleading, the pleader must of
2) allege in a complete manner how Tagitis was abducted, the
course state the ultimate facts constituting the cause of action, omitting
persons responsible for his disappearance, and the
the evidentiary details.[76] In an Amparo petition, however, this
respondents source of information;
requirement must be read in light of the nature and purpose of the
3) allege that the abduction was committed at the petitioners
proceeding, which addresses a situation of uncertainty; the petitioner may
instructions or with their consent;
not be able to describe with certainty how the victim exactly disappeared,
4) implead the members of CIDG regional office in Zamboanga
or who actually acted to kidnap, abduct or arrest him or her, or where the
alleged to have custody over her husband;
victim is detained, because these information may purposely be hidden or
5) attach the affidavits of witnesses to support her accusations;
covered up by those who caused the disappearance. In this type of
6) allege any action or inaction attributable to the petitioners in
situation, to require the level of specificity, detail and precision that the
the performance of their duties in the investigation of Tagitis
petitioners apparently want to read into the Amparo Rule is to make this
disappearance; and
Rule a token gesture of judicial concern for violations of the constitutional
7) specify what legally available efforts she took to determine
rights to life, liberty and security.
the fate or whereabouts of her husband.
To read the Rules of Court requirement on pleadings while
A petition for the Writ of Amparo shall be signed and verified and
addressing the unique Amparo situation, the test in reading the petition
shall allege, among others (in terms of the portions the petitioners cite): [75]
should be to determine whether it contains the details available to the
petitioner under the circumstances, while presenting a cause of action
showing a violation of the victims rights to life, liberty and security through
State or private party action. The petition should likewise be read in its
totality, rather than in terms of its isolated component parts, to determine Section 5(d) of the Amparo Rule requires that prior investigation of
if the required elements namely, of the disappearance, the State or private an alleged disappearance must have been made, specifying the manner
action, and the actual or threatened violations of the rights to life, liberty and results of the investigation.Effectively, this requirement seeks to
or security are present. establish at the earliest opportunity the level of diligence the public
authorities undertook in relation with the reported disappearance. [79]
In the present case, the petition amply recites in its paragraphs 4 We reject the petitioners argument that the respondents petition
to 11 the circumstances under which Tagitis suddenly dropped out of sight did not comply with the Section 5(d) requirements of the Amparo Rule, as
after engaging in normal activities, and thereafter was nowhere to be the petition specifies in its paragraph 11 that Kunnong and his companions
found despite efforts to locate him. The petition alleged, too, under its immediately reported Tagitis disappearance to the police authorities in
paragraph 7, in relation to paragraphs 15 and 16, that according to Jolo, Sulu as soon as they were relatively certain that he indeed had
reliable information,police operatives were the perpetrators of the disappeared.The police, however, gave them the ready answer that Tagitis
abduction. It also clearly alleged how Tagitis rights to life, liberty and could have been abducted by the Abu Sayyaf group or other anti-
security were violated when he was forcibly taken and boarded on a motor government groups. The respondent also alleged in paragraphs 17 and 18
vehicle by a couple of burly men believed to be police intelligence of her petition that she filed a complaint with the PNP Police Station in
operatives, and then taken into custody by the respondents police Cotobato and in Jolo, but she was told of an intriguing tale by the police
intelligence operatives since October 30, 2007, specifically by the CIDG, that her husband was having a good time with another woman. The
PNP Zamboanga City, x x x held against his will in an earnest attempt of disappearance was alleged to have been reported, too, to no less than the
the police to involve and connect [him] with different terrorist groups. [77] Governor of the ARMM, followed by the respondents personal inquiries that
yielded the factual bases for her petition.[80]
These allegations, in our view, properly pleaded ultimate facts
within the pleaders knowledge about Tagitis disappearance, the These allegations, to our mind, sufficiently specify that reports
participation by agents of the State in this disappearance, the failure of the have been made to the police authorities, and that investigations should
State to release Tagitis or to provide sufficient information about his have followed. That the petition did not state the manner and results of
whereabouts, as well as the actual violation of his right to liberty. Thus, the investigation that the Amparo Rule requires, but rather generally
the petition cannot be faulted for any failure in its statement of a cause of stated the inaction of the police, their failure to perform their duty to
action. investigate, or at the very least, their reported failed efforts, should not be
a reflection on the completeness of the petition. To require the respondent
If a defect can at all be attributed to the petition, this defect is its to elaborately specify the names, personal circumstances, and addresses
lack of supporting affidavit, as required by Section 5(c) of of the investigating authority, as well the manner and conduct of the
the Amparo Rule. Owing to the summary nature of the proceedings for the investigation is an overly strict interpretation of Section 5(d), given the
writ and to facilitate the resolution of the petition, the Amparo Rule respondents frustrations in securing an investigation with meaningful
incorporated the requirement for supporting affidavits, with the annotation results. Under these circumstances, we are more than satisfied that the
that these can be used as the affiants direct testimony. [78] This allegations of the petition on the investigations undertaken are sufficiently
requirement, however, should not be read as an absolute one that complete for purposes of bringing the petition forward.
necessarily leads to the dismissal of the petition if not strictly
followed. Where, as in this case, the petitioner has substantially complied Section 5(e) is in the Amparo Rule to prevent the use of a petition
with the requirement by submitting a verified petition sufficiently detailing that otherwise is not supported by sufficient allegations to constitute a
the facts relied upon, the strict need for the sworn statement that an proper cause of action as a means to fish for evidence. [81] The petitioners
affidavit represents is essentially fulfilled. We note that the failure to contend that the respondents petition did not specify what legally available
attach the required affidavits was fully cured when the respondent and her efforts were taken by the respondent, and that there was an undue haste
witness (Mrs. Talbin) personally testified in the CA hearings held on in the filing of the petition when, instead of cooperating with authorities,
January 7 and 17 and February 18, 2008 to swear to and flesh out the the respondent immediately invoked the Courts intervention.
allegations of the petition. Thus, even on this point, the petition cannot be
faulted. We do not see the respondents petition as the petitioners view it.
Section 5(e) merely requires that the Amparo petitioner (the operatives, specifically with the CIDG, PNP Zamboanga
respondent in the present case) allege the actions and recourses taken to City, being held against his will in an earnest attempt of
determine the fate or whereabouts of the aggrieved party and the identity the police to involve and connect Engr. Tagitis with the
of the person responsible for the threat, act or omission. The following different terrorist groups;
allegations of the respondents petition duly outlined the actions she had
taken and the frustrations she encountered, thus compelling her to file her xxxx
petition. 17. [The respondent] filed her complaint with the PNP Police
xxxx Station at the ARMM in Cotobato and in Jolo, as suggested
7. Soon after the student left the room, Engr. Tagitis went out of by her friends, seeking their help to find her husband, but
the pension house to take his early lunch but while out on [the respondents] request and pleadings failed to produce
the street, a couple of burly men believed to be police any positive results
intelligence operatives, forcibly took him and boarded the xxxx
latter on a motor vehicle then sped away without the 20. Lately, [respondent] was again advised by one of the
knowledge of his student, Arsimin Kunnong; [petitioners] to go to the ARMM Police Headquarters again
in Cotobato City and also to the different Police
xxxx Headquarters including the police headquarters in Davao
City, in Zamboanga City, in Jolo, and in Camp Crame,
10. When Kunnong could not locate Engr. Tagitis, the former Quezon City, and all these places have been visited by the
sought the help of another IDB scholar and reported the [respondent] in search for her husband, which entailed
matter to the local police agency; expenses for her trips to these places thereby resorting
her to borrowings and beggings [sic] for financial help
11. Arsimin Kunnong, including his friends and companions in Jolo, from friends and relatives only to try complying to the
exerted efforts in trying to locate the whereabouts of Engr. different suggestions of these police officers, despite of
Tagitis and when he reported the matter to the police which, her efforts produced no positive results up to the
authorities in Jolo, he was immediately given a ready present time;
answer that Engr. Tagitis could [have been] abducted by
the Abu Sayyaf group and other groups known to be xxxx
fighting against the government; 25. [The respondent] has exhausted all administrative
avenues and remedies but to no avail, and under the
12. Being scared with these suggestions and insinuations of the circumstances, [respondent] has no other plain, speedy
police officers, Kunnong reported the matter to the and adequate remedy to protect and get the release of
[respondent](wife of Engr. Tagitis) by phone and other subject Engr. Morced Tagitis from the illegal clutches of
responsible officers and coordinators of the IDB [the petitioners], their intelligence operatives and the like
Scholarship Programme in the Philippines who alerted the which are in total violation of the subjects human and
office of the Governor of ARMM who was then preparing to constitutional rights, except the issuance of a WRIT
attend the OIC meeting in Jeddah, Saudi Arabia; OF AMPARO.

13. [The respondent], on the other hand, approached Based on these considerations, we rule that the respondents
some of her co-employees with the Land Bank in Digos petition for the Writ of Amparo is sufficient in form and substance and that
branch, Digos City, Davao del Sur, who likewise sought the Court of Appeals had every reason to proceed with its consideration of
help from some of their friends in the military who could the case.
help them find/locate the whereabouts of her husband;

xxxx
15. According to reliable information received by the [respondent], The Desaparecidos
subject Engr. Tagitis is in the custody of police intelligence
The present case is one of first impression in the use and In the Philippines, enforced disappearances generally fall within the
application of the Rule on the Writ of Amparo in an enforced disappearance first two categories,[89] and 855 cases were recorded during the period of
situation. For a deeper appreciation of the application of this Rule to an martial law from 1972 until 1986. Of this number, 595 remained missing,
enforced disappearance situation, a brief look at the historical context of 132 surfaced alive and 127 were found dead. During former President
the writ and enforced disappearances would be very helpful. Corazon C. Aquinos term, 820 people were reported to have disappeared
and of these, 612 cases were documented. Of this number, 407 remain
The phenomenon of enforced disappearance arising from State missing, 108 surfaced alive and 97 were found dead. The number of
action first attracted notice in Adolf Hitlers Nact und Nebel Erlass or Night enforced disappearances dropped during former President Fidel V. Ramos
and Fog Decree of December 7, 1941. [82]The Third Reichs Night and Fog term when only 87 cases were reported, while the three-year term of
Program, a State policy, was directed at persons in occupied territories former President Joseph E. Estrada yielded 58 reported
endangering German security; they were transported secretly to Germany cases. KARAPATAN, a local non-governmental organization, reports that as
where they disappeared without a trace. In order to maximize the desired of March 31, 2008, the records show that there were a total of 193 victims
intimidating effect, the policy prohibited government officials from of enforced disappearance under incumbent President Gloria M. Arroyos
providing information about the fate of these targeted persons. [83] administration. The Commission on Human Rights records show a total of
636 verified cases of enforced disappearances from 1985 to 1993. Of this
In the mid-1970s, the phenomenon of enforced disappearances number, 406 remained missing, 92 surfaced alive, 62 were found dead,
resurfaced, shocking and outraging the world when individuals, numbering and 76 still have undetermined status.[90] Currently, the United Nations
anywhere from 6,000 to 24,000, were reported to have disappeared Working Group on Enforced or Involuntary Disappearance [91]reports 619
during the military regime in Argentina. Enforced disappearances spread in outstanding cases of enforced or involuntary disappearances covering the
Latin America, and the issue became an international concern when the period December 1, 2007 to November 30, 2008.[92]
world noted its widespread and systematic use by State security forces in
that continent under Operation Condor [84] and during the Dirty War[85] in Enforced Disappearances
the 1970s and 1980s. The escalation of the practice saw political activists Under Philippine Law
secretly arrested, tortured, and killed as part of governments counter-
insurgency campaigns. As this form of political brutality became routine The Amparo Rule expressly provides that the writ shall cover
elsewhere in the continent, the Latin American media standardized the extralegal killings and enforced disappearances or threats thereof. [93] We
term disappearance to describe the phenomenon. The victims of enforced note that although the writ specifically covers enforced disappearances,
disappearances were called the desaparecidos,[86]which literally means this concept is neither defined nor penalized in this jurisdiction. The
the disappeared ones.[87] In general, there are three different kinds of records of the Supreme Court Committee on the Revision of Rules
disappearance cases: (Committee) reveal that the drafters of the Amparo Rule initially
considered providing an elemental definition of the concept of enforced
1) those of people arrested without witnesses or disappearance:[94]
without positive identification of the arresting agents
and are never found again; JUSTICE MARTINEZ: I believe that first and foremost we
should come up or formulate a specific definition [for]
2) those of prisoners who are usually arrested without extrajudicial killings and enforced disappearances. From
an appropriate warrant and held in complete isolation that definition, then we can proceed to formulate the rules,
for weeks or months while their families are unable to definite rules concerning the same.
discover their whereabouts and the military authorities
deny having them in custody until they eventually CHIEF JUSTICE PUNO: As things stand, there is no law
reappear in one detention center or another; and penalizing extrajudicial killings and enforced
disappearances so initially also we have to [come up
3) those of victims of salvaging who have disappeared with] the nature of these extrajudicial killings and
until their lifeless bodies are later discovered. [88] enforced disappearances [to be covered by the Rule]
because our concept of killings and disappearances
will define the jurisdiction of the courts. So well have
to agree among ourselves about the nature of killings and directly compel the public authorities to act on actual or threatened
disappearances for instance, in other jurisdictions, the violations of constitutional rights. To state the obvious, judicial
rules only cover state actors. That is an element intervention can make a difference even if only procedurally in a situation
incorporated in their concept of extrajudicial killings and when the very same investigating public authorities may have had a hand
enforced disappearances. In other jurisdictions, the in the threatened or actual violations of constitutional rights.
concept includes acts and omissions not only of state
actors but also of non state actors. Well, more specifically Lest this Court intervention be misunderstood, we clarify once
in the case of the Philippines for instance, should these again that we do not rule on any issue of criminal culpability for the
rules include the killings, the disappearances which may be extrajudicial killing or enforced disappearance.This is an issue that requires
authored by let us say, the NPAs or the leftist criminal action before our criminal courts based on our existing penal
organizations and others. So, again we need to define the laws. Our intervention is in determining whether an enforced
nature of the extrajudicial killings and enforced disappearance has taken place and who is responsible or accountable for
disappearances that will be covered by these rules. this disappearance, and to define and impose the appropriate remedies to
[Emphasis supplied] [95] address it. The burden for the public authorities to discharge in these
situations, under the Rule on the Writ of Amparo, is twofold. The first is to
In the end, the Committee took cognizance of several bills filed in ensure that all efforts at disclosure and investigation are undertaken
the House of Representatives [96] and in the Senate[97] on extrajudicial under pain of indirect contempt from this Court when governmental efforts
killings and enforced disappearances, and resolved to do away with a clear are less than what the individual situations require. The second is to
textual definition of these terms in the Rule. The Committee instead address the disappearance, so that the life of the victim is preserved and
focused on the nature and scope of the concerns within its power to his or her liberty and security restored. In these senses, our orders and
address and provided the appropriate remedy therefor, mindful that an directives relative to the writ are continuing efforts that are not truly
elemental definition may intrude into the ongoing legislative efforts. [98] terminated until the extrajudicial killing or enforced disappearance is fully
addressed by the complete determination of the fate and the whereabouts
As the law now stands, extra-judicial killings and enforced of the victim, by the production of the disappeared person and the
disappearances in this jurisdiction are not crimes penalized separately restoration of his or her liberty and security, and, in the proper case, by
from the component criminal acts undertaken to carry out these killings the commencement of criminal action against the guilty parties.
and enforced disappearances and are now penalized under the Revised
Penal Code and special laws. [99] The simple reason is that the Legislature
has not spoken on the matter; the determination of what acts are criminal Enforced Disappearance
and what the corresponding penalty these criminal acts should carry are Under International Law
matters of substantive law that only the Legislature has the power to enact
under the countrys constitutional scheme and power structure. From the International Law perspective, involuntary or enforced
disappearance is considered a flagrant violation of human rights. [101] It
Even without the benefit of directly applicable substantive laws on does not only violate the right to life, liberty and security of
extra-judicial killings and enforced disappearances, however, the Supreme the desaparecido; it affects their families as well through the denial of
Court is not powerless to act under its own constitutional mandate to their right to information regarding the circumstances of the disappeared
promulgate rules concerning the protection and enforcement of family member. Thus, enforced disappearances have been said to be a
constitutional rights, pleading, practice and procedure in all courts, double form of torture, with doubly paralyzing impact for the victims, as
[100]
since extrajudicial killings and enforced disappearances, by their they are kept ignorant of their own fates, while family members are
nature and purpose, constitute State or private party violation of the deprived of knowing the whereabouts of their detained loved ones and
constitutional rights of individuals to life, liberty and security. Although the suffer as well the serious economic hardship and poverty that in most
Courts power is strictly procedural and as such does not diminish, increase cases follow the disappearance of the household breadwinner. [102]
or modify substantive rights, the legal protection that the Court can
provide can be very meaningful through the procedures it sets in The UN General Assembly first considered the issue of Disappeared
addressing extrajudicial killings and enforced disappearances. The Court, Persons in December 1978 under Resolution 33/173. The Resolution
through its procedural rules, can set the procedural standards and thereby expressed the General Assemblys deep concern arising from reports from
various parts of the world relating to enforced or involuntary that this right is non-derogable.[108] It provides that no one shall be
disappearances, and requested the UN Commission on Human Rights to subjected to enforced disappearance under any circumstances, be it a
consider the issue of enforced disappearances with a view to making state of war, internal political instability, or any other public emergency. It
appropriate recommendations.[103] obliges State Parties to codify enforced disappearance as an offense
punishable with appropriate penalties under their criminal law. [109] It also
In 1992, in response to the reality that the insidious practice of recognizes the right of relatives of the disappeared persons and of the
enforced disappearance had become a global phenomenon, the UN society as a whole to know the truth on the fate and whereabouts of the
General Assembly adopted the Declaration on the Protection of All disappeared and on the progress and results of the investigation.
Persons from Enforced Disappearance (Declaration).[104] This [110]
Lastly, it classifies enforced disappearance as a continuing offense,
Declaration, for the first time, provided in its third preambular clause a such that statutes of limitations shall not apply until the fate and
working description of enforced disappearance, as follows: whereabouts of the victim are established.[111]

Deeply concerned that in many countries, often


in a persistent manner, enforced disappearances occur, in Binding Effect of UN
the sense that persons are arrested, detained or Action on the Philippines
abducted against their will or otherwise deprived of
their liberty by officials of different branches or To date, the Philippines has neither signed nor ratified the Convention, so
levels of Government, or by organized groups or that the country is not yet committed to enact any law penalizing enforced
private individuals acting on behalf of, or with the disappearance as a crime. The absence of a specific penal law, however, is
support, direct or indirect, consent or acquiescence not a stumbling block for action from this Court, as heretofore mentioned;
of the Government, followed by a refusal to disclose underlying every enforced disappearance is a violation of the constitutional
the fate or whereabouts of the persons concerned or rights to life, liberty and security that the Supreme Court is mandated by
a refusal to acknowledge the deprivation of their the Constitution to protect through its rule-making powers.
liberty, which places such persons outside the protection
of the law. [Emphasis supplied] Separately from the Constitution (but still pursuant to its terms),
the Court is guided, in acting on Amparo cases, by the reality that the
Philippines is a member of the UN, bound by its Charter and by the various
Fourteen years after (or on December 20, 2006), the UN General conventions we signed and ratified, particularly the conventions touching
Assembly adopted the International Convention for the Protection of All on humans rights. Under the UN Charter, the Philippines pledged to
Persons from Enforced Disappearance (Convention).[105] The Convention promote universal respect for, and observance of, human rights and
was opened for signature in Paris, France on February 6, 2007. [106] Article fundamental freedoms for all without distinctions as to race, sex, language
2 of the Convention defined enforced disappearance as follows: or religion.[112] Although no universal agreement has been reached on the
precise extent of the human rights and fundamental freedoms guaranteed
For the purposes of this Convention, enforced to all by the Charter,[113] it was the UN itself that issued the Declaration on
disappearance is considered to be the arrest, detention, enforced disappearance, and this Declaration states: [114]
abduction or any other form of deprivation of liberty by
agents of the State or by persons or groups of persons Any act of enforced disappearance is an offence to
acting with the authorization, support or acquiescence of dignity. It is condemned as a denial of the purposes of
the State, followed by a refusal to acknowledge the the Charter of the United Nations and as a grave and
deprivation of liberty or by concealment of the fate or flagrant violation of human rights and fundamental
whereabouts of the disappeared person, which place such freedoms proclaimed in the Universal Declaration of
a person outside the protection of the law. [Emphasis Human Rights and reaffirmed and developed in
supplied] international instruments in this field. [Emphasis supplied]

The Convention is the first universal human rights instrument to assert As a matter of human right and fundamental freedom and as a policy
that there is a right not to be subject to enforced disappearance [107] and matter made in a UN Declaration, the ban on enforced disappearance
cannot but have its effects on the country, given our own adherence are indicative of the State practice and opinio juris requirements of
to generally accepted principles of international law as part of the law of international law.[121] We note the following in these respects:
the land.[115]
First, barely two years from the adoption of the Declaration, the
In the recent case of Pharmaceutical and Health Care Association Organization of American States (OAS) General Assembly adopted the
of the Philippines v. Duque III,[116] we held that: Inter-American Convention on Enforced Disappearance of Persons in June
1994.[122] State parties undertook under this Convention not to practice,
Under the 1987 Constitution, international law can permit, or tolerate the forced disappearance of persons, even in states of
become part of the sphere of domestic law either emergency or suspension of individual guarantees. [123] One of the key
by transformation or incorporation. The transformation provisions includes the States obligation to enact the crime of forced
method requires that an international law be transformed disappearance in their respective national criminal laws and to establish
into a domestic law through a constitutional mechanism jurisdiction over such cases when the crime was committed within their
such as local legislation. The incorporation method jurisdiction, when the victim is a national of that State, and when the
applies when, by mere constitutional declaration, alleged criminal is within its territory and it does not proceed to extradite
international law is deemed to have the force of him, which can be interpreted as establishing universal jurisdiction among
domestic law. [Emphasis supplied] the parties to the Inter-American Convention. [124] At present, Colombia,
Guatemala, Paraguay, Peru and Venezuela have enacted separate laws in
We characterized generally accepted principles of international law as accordance with the Inter-American Convention and have defined activities
norms of general or customary international law that are binding on all involving enforced disappearance to be criminal. [125]
states. We held further:[117]

[G]enerally accepted principles of international Second, in Europe, the European Convention on Human Rights has
law, by virtue of the incorporation clause of the no explicit provision dealing with the protection against enforced
Constitution, form part of the laws of the land even if they disappearance. The European Court of Human Rights (ECHR), however,
do not derive from treaty obligations. The classical has applied the Convention in a way that provides ample protection for the
formulation in international law sees underlying rights affected by enforced disappearance through the
those customary rules accepted as binding result Conventions Article 2 on the right to life; Article 3 on the prohibition of
from the combination [of] two elements: the torture; Article 5 on the right to liberty and security; Article 6, paragraph 1
established, widespread, and consistent practice on the on the right to a fair trial; and Article 13 on the right to an effective
part of States; and a psychological element known as remedy. A leading example demonstrating the protection afforded by the
the opinion juris sive necessitates (opinion as to law or European Convention is Kurt v. Turkey,[126] where the ECHR found a
necessity). Implicit in the latter element is a belief that violation of the right to liberty and security of the disappeared person
the practice in question is rendered obligatory by the when the applicants son disappeared after being taken into custody by
existence of a rule of law requiring it. [Emphasis in the Turkish forces in the Kurdish village of Agilli in November 1993. It further
original] found the applicant (the disappeared persons mother) to be a victim of a
violation of Article 3, as a result of the silence of the authorities and the
The most widely accepted statement of sources of international law inadequate character of the investigations undertaken. The ECHR also saw
today is Article 38(1) of the Statute of the International Court of Justice, the lack of any meaningful investigation by the State as a violation of
which provides that the Court shall apply international custom, as evidence Article 13.[127]
of a general practice accepted as law.[118] The material sources of custom
include State practice, State legislation, international and national judicial
decisions, recitals in treaties and other international instruments, a pattern Third, in the United States, the status of the prohibition on
of treaties in the same form, the practice of international organs, and enforced disappearance as part of customary international law is
resolutions relating to legal questions in the UN General Assembly. recognized in the most recent edition of Restatement of the Law: The
[119]
Sometimes referred to as evidence of international law, [120] these Third,[128] which provides that [a] State violates international law if, as a
sources identify the substance and content of the obligations of States and matter of State policy, it practices, encourages, or condones (3) the
murder or causing the disappearance of individuals. [129] We significantly Rome Statute has been incorporated in the statutes of other international
note that in a related matter that finds close identification with enforced and hybrid tribunals, including Sierra Leone Special Court, the Special
disappearance the matter of torture the United States Court of Appeals for Panels for Serious Crimes in Timor-Leste, and the Extraordinary Chambers
the Second Circuit Court held in Filartiga v. Pena-Irala[130] that the in the Courts of Cambodia.[134] In addition, the implementing legislation of
prohibition on torture had attained the status of customary international State Parties to the Rome Statute of the ICC has given rise to a number of
law. The court further elaborated on the significance of UN declarations, as national criminal provisions also covering enforced disappearance. [135]
follows:

These U.N. declarations are significant because While the Philippines is not yet formally bound by the terms of the
they specify with great precision the obligations of member Convention on enforced disappearance (or by the specific terms of the
nations under the Charter. Since their adoption, Rome Statute) and has not formally declared enforced disappearance as a
"(m)embers can no longer contend that they do not know specific crime, the above recital shows that enforced disappearance as
what human rights they promised in the Charter to a State practice has been repudiated by the international
promote. Moreover, a U.N. Declaration is, according to one community, so that the ban on it is now a generally accepted
authoritative definition, "a formal and solemn instrument, principle of international law, which we should consider a part of
suitable for rare occasions when principles of great and the law of the land, and which we should act upon to the extent
lasting importance are being enunciated. Accordingly, it already allowed under our laws and the international conventions
has been observed that the Universal Declaration of that bind us.
Human Rights "no longer fits into the dichotomy of binding The following civil or political rights under the Universal Declaration
treaty against non-binding pronouncement,' but is rather of Human Rights, the ICCPR and the International Convention on
an authoritative statement of the international Economic, Social and Cultural Rights (ICESR) may be infringed in the
community." Thus, a Declaration creates an expectation of course of a disappearance:[136]
adherence, and "insofar as the expectation is gradually
justified by State practice, a declaration may by custom
become recognized as laying down rules binding upon the
States." Indeed, several commentators have concluded 1) the right to recognition as a person before the law;
that the Universal Declaration has become, in toto, a part
of binding, customary international law. [Citations omitted] 2) the right to liberty and security of the person;
Fourth, in interpreting Article 2 (right to an effective domestic
3) the right not to be subjected to torture and other cruel,
remedy) of the International Convention on Civil and Political Rights
inhuman or degrading treatment or punishment;
(ICCPR), to which the Philippines is both a signatory and a State Party, the
UN Human Rights Committee, under the Office of the High Commissioner
4) the right to life, when the disappeared person is killed;
for Human Rights, has stated that the act of enforced disappearance
violates Articles 6 (right to life), 7 (prohibition on torture, cruel, inhuman 5) the right to an identity;
or degrading treatment or punishment) and 9 (right to liberty and security
of the person) of the ICCPR, and the act may also amount to a crime 6) the right to a fair trial and to judicial guarantees;
against humanity.[131]
7) the right to an effective remedy, including
reparation and compensation;
Fifth, Article 7, paragraph 1 of the 1998 Rome Statute establishing
the International Criminal Court (ICC) also covers enforced disappearances 8) the right to know the truth regarding the
insofar as they are defined as crimes against humanity, [132] i.e., crimes circumstances of a disappearance.
committed as part of a widespread or systematic attack against any
civilian population, with knowledge of the attack. While more than 100 9) the right to protection and assistance to the family;
countries have ratified the Rome Statute,[133] the Philippines is still merely
a signatory and has not yet ratified it. We note that Article 7(1) of the 10) the right to an adequate standard of living;
11) the right to health; and promptly, thoroughly and effectively through
independent and impartial bodies. A failure by a State
12) the right to education [Emphasis supplied] Party to investigate allegations of violations could in and of
itself give rise to a separate breach of the Covenant.
Cessation of an ongoing violation is an essential element of
the right to an effective remedy. [Emphasis supplied]
Article 2 of the ICCPR, which binds the Philippines as a state party,
provides:

Article 2 The UN Human Rights Committee further stated in the same


General Comment No. 31 that failure to investigate as well as failure to
3. Each State Party to the present Covenant undertakes: bring to justice the perpetrators of ICCPR violations could in and of itself
give rise to a separate breach of the Covenant, thus: [138]
(a) To ensure that any person whose rights or freedoms as
herein recognized are violated shall have an effective
remedy, notwithstanding that the violation has been
committed by persons acting in an official capacity; 18. Where the investigations referred to in paragraph 15
reveal violations of certain Covenant rights, States
(b) To ensure that any person claiming such a remedy Parties must ensure that those responsible are
shall have his right thereto determined by competent brought to justice. As with failure to investigate,
judicial, administrative or legislative authorities, or failure to bring to justice perpetrators of such
by any other competent authority provided for by the legal violations could in and of itself give rise to a
system of the State, and to develop the possibilities of separate breach of the Covenant. These obligations
judicial remedy; arise notably in respect of those violations
recognized as criminal under either domestic or
(c) To ensure that the competent authorities shall enforce international law, such as torture and similar cruel,
such remedies when granted. [Emphasis supplied] inhuman and degrading treatment (article 7), summary
and arbitrary killing (article 6) and enforced
disappearance (articles 7 and 9 and, frequently,
6). Indeed, the problem of impunity for these violations, a
In General Comment No. 31, the UN Human Rights Committee opined that
matter of sustained concern by the Committee, may well
the right to an effective remedy under Article 2 of the ICCPR includes the
be an important contributing element in the recurrence of
obligation of the State to investigate ICCPR violations promptly,
the violations. When committed as part of a widespread or
thoroughly, and effectively, viz:[137]
systematic attack on a civilian population, these violations
of the Covenant are crimes against humanity (see Rome
Statute of the International Criminal Court, article 7).
15. Article 2, paragraph 3, requires that in addition to [Emphasis supplied]
effective protection of Covenant rights, States Parties
must ensure that individuals also have accessible
and effective remedies to vindicate those rights The
In Secretary of National Defense v. Manalo,[139] this Court, in ruling
Committee attaches importance to States Parties'
that the right to security of persons is a guarantee of the protection of
establishing appropriate judicial and administrative
ones right by the government, held that:
mechanisms for addressing claims of rights violations
under domestic law Administrative mechanisms are
particularly required to give effect to the general
obligation to investigate allegations of violations
The right to security of person in this third sense is a account for his or her whereabouts. For this
corollary of the policy that the State guarantees full reason, Article 5 must be seen as requiring the
respect for human rights under Article II, Section 11 of the authorities to take effective measures to safeguard
1987 Constitution. As the government is the chief against the risk of disappearance and to conduct a
guarantor of order and security, the Constitutional prompt effective investigation into an arguable claim
guarantee of the rights to life, liberty and security of that a person has been taken into custody and has
person is rendered ineffective if government does not not been seen since. [Emphasis supplied]
afford protection to these rights especially when they are
under threat. Protection includes conducting effective
investigations, organization of the government These rulings effectively serve as the backdrop for the Rule on the
apparatus to extend protection to victims of Writ of Amparo, which the Court made effective on October 24,
extralegal killings or enforced disappearances (or 2007. Although the Amparo Rule still has gaps waiting to be filled through
threats thereof) and/or their families, and bringing substantive law, as evidenced primarily by the lack of a concrete definition
offenders to the bar of justice. The Inter-American of enforced disappearance, the materials cited above, among others,
Court of Human Rights stressed the importance of provide ample guidance and standards on how, through the
investigation in the Velasquez Rodriguez Case, viz: medium of the Amparo Rule, the Court can provide remedies and
protect the constitutional rights to life, liberty and security that
(The duty to investigate) must be underlie every enforced disappearance.
undertaken in a serious
manner and not as a mere formality Evidentiary Difficulties Posed
preordained to be ineffective. An by the Unique Nature of an
investigation must have an objective and Enforced Disappearance
be assumed by the State as its own
legal duty, not as a step taken by Before going into the issue of whether the respondent has
private interests that depends upon discharged the burden of proving the allegations of the petition for the
the initiative of the victim or his Writ of Amparo by the degree of proof required by the Amparo Rule, we
family or upon their offer of proof, without shall discuss briefly the unique evidentiary difficulties presented by
an effective search for the truth by the enforced disappearance cases; these difficulties form part of the setting
government. [Emphasis supplied] that the implementation of the Amparo Rule shall encounter.

These difficulties largely arise because the State itself the party
whose involvement is alleged investigates enforced disappearances. Past
Manalo significantly cited Kurt v. Turkey,[140] where the ECHR interpreted experiences in other jurisdictions show that the evidentiary difficulties are
the right to security not only as a prohibition on the State against generally threefold.
arbitrary deprivation of liberty, but also as the imposition of a positive
duty to afford protection to the right to liberty. The Court notably quoted First, there may be a deliberate concealment of the identities
the following ECHR ruling: of the direct perpetrators.[141] Experts note that abductors are well
organized, armed and usually members of the military or police forces,
thus:

[A]ny deprivation of liberty must not only have been


effected in conformity with the substantive and procedural The victim is generally arrested by the security forces or by
rules of national law but must equally be in keeping with persons acting under some form of governmental
the very purpose of Article 5, namely to protect the authority. In many countries the units that plan,
individual from arbitrariness... Having assumed control implement and execute the program are generally
over that individual, it is incumbent on the authorities to specialized, highly-secret bodies within the armed or
security forces. They are generally directed through a
separate, clandestine chain of command, but they have the Evidence and Burden of Proof in
necessary credentials to avoid or prevent any interference Enforced Disappearances Cases
by the "legal" police forces. These authorities take their
victims to secret detention centers where they subject
them to interrogation and torture without fear of judicial or Sections 13, 17 and 18 of the Amparo Rule define the nature of
other controls.[142] an Amparo proceeding and the degree and burden of proof the parties to
the case carry, as follows:

In addition, there are usually no witnesses to the crime; if there Section 13. Summary Hearing. The hearing on the petition
are, these witnesses are usually afraid to speak out publicly or to testify shall be summary. However, the court, justice or judge
on the disappearance out of fear for their own lives. [143] We have had may call for a preliminary conference to simplify the issues
occasion to note this difficulty in Secretary of Defense v. Manalo[144] when and determine the possibility of obtaining stipulations and
we acknowledged that where powerful military officers are implicated, the admissions from the parties.
hesitation of witnesses to surface and testify against them comes as no
surprise. xxxx
Section 17. Burden of Proof and Standard of Diligence
Required. The parties shall establish their claims
Second, deliberate concealment of pertinent evidence of the by substantial evidence.
disappearance is a distinct possibility; the central piece of evidence in an
enforced disappearance i.e., the corpus delicti or the victims body is The respondent who is a private individual must
usually concealed to effectively thwart the start of any investigation or the prove that ordinary diligence as required by applicable
progress of one that may have begun.[145] The problem for the victims laws, rules and regulations was observed in the
family is the States virtual monopoly of access to pertinent evidence . The performance of duty.
Inter-American Court of Human Rights (IACHR) observed in the landmark The respondent who is a public official or employee
case of Velasquez Rodriguez[146] that inherent to the practice of enforced must prove that extraordinary diligence as required by
disappearance is the deliberate use of the States power to destroy the applicable laws, rules and regulations was observed in the
pertinent evidence. The IACHR described the concealment as a clear performance of duty.
attempt by the State to commit the perfect crime.[147]
The respondent public official or employee cannot
invoke the presumption that official duty has been
regularly performed or evade responsibility or liability.
Third is the element of denial; in many cases, the State
authorities deliberately deny that the enforced disappearance ever Section 18. Judgment. If the allegations in the petition
occurred.[148] Deniability is central to the policy of enforced are proven by substantial evidence, the court
disappearances, as the absence of any proven disappearance makes it shall grant the privilege of the writ and such reliefs as
easier to escape the application of legal standards ensuring the victims may be proper and appropriate; otherwise, the privilege
human rights.[149] Experience shows that government officials typically shall be denied. [Emphasis supplied]
respond to requests for information about desaparecidos by saying that
they are not aware of any disappearance, that the missing people may
have fled the country, or that their names have merely been invented. [150]
These characteristics namely, of being summary and the use of
substantial evidence as the required level of proof (in contrast to the usual
These considerations are alive in our minds, as these are the
preponderance of evidence or proof beyond reasonable doubt in court
difficulties we confront, in one form or another, in our consideration of this
proceedings) reveal the clear intent of the framers of the Amparo Rule to
case.
have the equivalent of an administrative proceeding, albeit judicially not need to apply due to the summary nature of Amparo proceedings. We
conducted, in addressing Amparo situations. The standard of diligence said:
required the duty of public officials and employees to observe
extraordinary diligence point, too, to the extraordinary measures expected The remedy [of the writ of amparo] provides rapid
in the protection of constitutional rights and in the consequent handling judicial relief as it partakes of a summary proceeding that
and investigation of extra-judicial killings and enforced disappearance requires only substantial evidence to make the appropriate
cases. reliefs available to the petitioner; it is not an action to
determine criminal guilt requiring proof beyond reasonable
Thus, in these proceedings, the Amparo petitioner needs only to doubt, or liability for damages requiring preponderance of
properly comply with the substance and form requirements of a Writ evidence, or administrative responsibility requiring
of Amparo petition, as discussed above, and prove the allegations by substantial evidence that will require full and
substantial evidence. Once a rebuttable case has been proven, the exhaustive proceedings. [Emphasis supplied]
respondents must then respond and prove their defenses based on the Not to be forgotten in considering the evidentiary aspects
standard of diligence required. The rebuttable case, of course, must show of Amparo petitions are the unique difficulties presented by the nature of
that an enforced disappearance took place under circumstances showing a enforced disappearances, heretofore discussed, which difficulties this Court
violation of the victims constitutional rights to life, liberty or security, and must frontally meet if the Amparo Rule is to be given a chance to achieve
the failure on the part of the investigating authorities to appropriately its objectives. These evidentiary difficulties compel the Court to adopt
respond. standards appropriate and responsive to the circumstances, without
transgressing the due process requirements that underlie every
proceeding.
The landmark case of Ang Tibay v. Court of Industrial
Relations[151] provided the Court its first opportunity to define the In the seminal case of Velasquez Rodriguez,[153] the IACHR faced
substantial evidence required to arrive at a valid decision in administrative with a lack of direct evidence that the government of Honduras was
proceedings. To directly quote Ang Tibay: involved in Velasquez Rodriguez disappearance adopted a relaxed and
informal evidentiary standard, and established the rule that presumes
Substantial evidence is more than a mere governmental responsibility for a disappearance if it can be proven that
scintilla. It means such relevant evidence as a the government carries out a general practice of enforced disappearances
reasonable mind might accept as adequate to and the specific case can be linked to that practice. [154] The IACHR took
support a conclusion. [citations omitted] The statute note of the realistic fact that enforced disappearances could be proven
provides that the rules of evidence prevailing in courts of only through circumstantial or indirect evidence or by logical inference;
law and equity shall not be controlling. The obvious otherwise, it was impossible to prove that an individual had been made to
purpose of this and similar provisions is to free disappear. It held:
administrative boards from the compulsion of technical
rules so that the mere admission of matter which would be 130. The practice of international and domestic courts
deemed incompetent in judicial proceedings would not shows that direct evidence, whether testimonial or
invalidate the administrative order. [citations omitted] But documentary, is not the only type of evidence that may be
this assurance of a desirable flexibility in administrative legitimately considered in reaching a
procedure does not go so far as to justify orders without a decision. Circumstantial evidence, indicia, and
basis in evidence having rational probative force. presumptions may be considered, so long as they
[Emphasis supplied] lead to conclusions consistent with the facts.

131. Circumstantial or presumptive evidence is


In Secretary of Defense v. Manalo, which was the Courts first
[152] especially important in allegations of
petition for a Writ of Amparo, we recognized that the full and exhaustive disappearances, because this type of repression is
proceedings that the substantial evidence standard regularly requires do characterized by an attempt to suppress all
information about the kidnapping or the
whereabouts and fate of the victim. [Emphasis We note in this regard that the use of flexibility in the consideration of
supplied] evidence is not at all novel in the Philippine legal system. In child abuse
cases, Section 28 of the Rule on Examination of a Child Witness [157] is
In concluding that the disappearance of Manfredo Velsquez expressly recognized as an exception to the hearsay rule. This Rule allows
(Manfredo) was carried out by agents who acted under cover of public the admission of the hearsay testimony of a child describing any act or
authority, the IACHR relied on circumstantial evidence including the attempted act of sexual abuse in any criminal or non-criminal proceeding,
hearsay testimony of Zenaida Velsquez, the victims sister, who subject to certain prerequisites and the right of cross-examination by the
described Manfredos kidnapping on the basis of conversations she had adverse party. The admission of the statement is determined by the court
with witnesses who saw Manfredo kidnapped by men in civilian clothes in in light of specified subjective and objective considerations that provide
broad daylight. She also told the Court that a former Honduran military sufficient indicia of reliability of the child witness. [158] These requisites for
official had announced that Manfredo was kidnapped by a special military admission find their counterpart in the present case under the above-
squadron acting under orders of the Chief of the Armed Forces. [155] The described conditions for the exercise of flexibility in the consideration of
IACHR likewise considered the hearsay testimony of a second witness who evidence, including hearsay evidence, in extrajudicial killings and enforced
asserted that he had been told by a Honduran military officer about the disappearance cases.
disappearance, and a third witness who testified that he had spoken in
prison to a man who identified himself as Manfredo. [156] Assessment of the Evidence

The threshold question for our resolution is: was there an enforced
disappearance within the meaning of this term under the UN Declaration
Velasquez stresses the lesson that flexibility is necessary under the unique we have cited?
circumstances that enforced disappearance cases pose to the courts; to
have an effective remedy, the standard of evidence must be responsive to The Convention defines enforced disappearance as the arrest,
the evidentiary difficulties faced. On the one hand, we cannot be arbitrary detention, abduction or any other form of deprivation of liberty by agents
in the admission and appreciation of evidence, as arbitrariness entails of the State or by persons or groups of persons acting with the
violation of rights and cannot be used as an effective counter-measure; we authorization, support or acquiescence of the State, followed by a refusal
only compound the problem if a wrong is addressed by the commission of to acknowledge the deprivation of liberty or by concealment of the fate or
another wrong. On the other hand, we cannot be very strict in our whereabouts of the disappeared person, which place such a person outside
evidentiary rules and cannot consider evidence the way we do in the usual the protection of the law. [159] Under this definition, the elements that
criminal and civil cases; precisely, the proceedings before us are constitute enforced disappearance are essentially fourfold: [160]
administrative in nature where, as a rule, technical rules of evidence are
not strictly observed. Thus, while we must follow the substantial evidence (a) arrest, detention, abduction or any form of
rule, we must observe flexibility in considering the evidence we shall take deprivation of liberty;
into account.
(b) carried out by agents of the State or persons or
groups of persons acting with the authorization,
support or acquiescence of the State;
The fair and proper rule, to our mind, is to consider all the pieces of
(c) followed by a refusal to acknowledge the detention,
evidence adduced in their totality, and to consider any evidence otherwise
or a concealment of the fate of the disappeared
inadmissible under our usual rules to be admissible if it is consistent with
person; and
the admissible evidence adduced. In other words, we reduce our rules
to the most basic test of reason i.e., to the relevance of the
(d) placement of the disappeared person outside the
evidence to the issue at hand and its consistency with all other
protection of the law. [Emphasis supplied]
pieces of adduced evidence. Thus, even hearsay evidence can be
admitted if it satisfies this basic minimum test.
We find no direct evidence indicating how the victim actually
disappeared. The direct evidence at hand only shows that Tagitis went out
of the ASY Pension House after depositing his room key with the hotel
desk and was never seen nor heard of again. The undisputed conclusion,
however, from all concerned the petitioner, Tagitis colleagues and even
the police authorities is that Tagistis disappeared under mysterious A: Jemaah Islamiah, sir.
circumstances and was never seen again. The respondent injected the
causal element in her petition and testimony, as we shall discuss below.

We likewise find no direct evidence showing that operatives of PNP Q: Was there any information that was read to you during
CIDG Zamboanga abducted or arrested Tagitis. If at all, only the one of those visits of yours in that Camp?
respondents allegation that Tagistis was under CIDG Zamboanga custody
stands on record, but it is not supported by any other evidence, direct or
circumstantial.
A: Col. Casim did not furnish me a copy of his report
because he said those reports are highly
In her direct testimony, the respondent pointed to two sources of
confidential, sir.
information as her bases for her allegation that Tagistis had been placed
under government custody (in contrast with CIDG Zamboanga custody).
The first was an unnamed friend in Zamboanga (later identified as Col.
Ancanan), who occupied a high position in the military and who allegedly Q: Was it read to you then even though you were not
mentioned that Tagitis was in good hands. Nothing came out of this claim, furnished a copy?
as both the respondent herself and her witness, Mrs. Talbin, failed to
establish that Col. Ancanan gave them any information that Tagitis was in
government custody. Col. Ancanan, for his part, admitted the meeting with
the respondent but denied giving her any information about the A: Yes, sir. In front of us, my friends.
disappearance.

The more specific and productive source of information was


Col. Kasim, whom the respondent, together with her witness Mrs. Talbin, Q: And what was the content of that highly confidential report?
met in Camp Katitipan in Davao City. To quote the relevant portions of the
respondents testimony:

A: Those alleged activities of Engineer Tagitis, sir.


Q: Were you able to speak to other military officials [161]
[Emphasis supplied]
regarding the whereabouts of your husband
particularly those in charge of any records or She confirmed this testimony in her cross-examination:
investigation?

Q: You also mentioned that you went to Camp Katitipan in


Davao City?
A: I went to Camp Katitipan in Davao City. Then one
military officer, Col. Casim, told me that my
husband is being abducted [sic] because he is
under custodial investigation because he is A: Yes, maam.
allegedly parang liason ng J.I., sir.

Q: What is J.I.?
Q: And a certain Col. Kasim told you that your A: Mrs. Talbin, tapos yung dalawang friends nya from Mati
husband was abducted and under custodial City, Davao Oriental, maam.[162]
investigation?
xxxx

Q: When you were told that your husband is in good


A: Yes, maam. hands, what was your reaction and what did you
do?

Q: And you mentioned that he showed you a report?


A: May binasa kasi sya that my husband has
a parang meeting with other people na
parang mga terorista na mga tao. Tapos at the
A: Yes, maam. end of the report is [sic] under custodial
investigation. So I told him Colonel, my husband
is sick. He is diabetic at nagmemaintain yun ng
gamot. Pakisabi lang sa naghohold sa asawa ko na
Q: Were you able to read the contents of that report? bigyan siya ng gamot, maam.[163]

A: He did not furnish me a copy of those [sic] report xxxx


because those [sic] were highly
confidential. That is a military report, maam. Q: You mentioned that you received information that
Engineer Tagitis is being held by the CIDG in
Zamboanga, did you go to CIDG Zamboanga to
verify that information?
Q: But you were able to read the contents?

A: I did not go to CIDG Zamboanga. I went to Camp


A: No. But he read it in front of us, my friends, maam.
Karingal instead. Enough na yun
na effort ko because I know that they would deny
it, maam.[164]
Q: How many were you when you went to see Col. Kasim?

A: There were three of us, maam.


On February 11, 2008, the respondent presented Mrs. Talbin to
corroborate her testimony that her husband was abducted and held under
custodial investigation by the PNP-CIDG Zamboanga City, viz:
Q: Who were your companions?
Q: You said that you went to Camp Katitipan in Davao City
sometime November 24, 2007, who was with you
when you went there? Q: What information did you get from Col. Kasim during that time?

A: Mary Jean Tagitis, sir. A: The first time we met with [him] I asked him if he knew
of the exact location, if he can furnish us the
location of Engr. Tagitis. And he was reading this
report. He told us that Engr. Tagitis is in good
Q: Only the two of you? hands. He is with the military, but he is not
certain whether he is with the AFP or PNP. He
has this serious case. He was charged of
terrorism because he was under surveillance
A: No. We have some other companions. We were four at that from January 2007 up to the time that he was
time, sir. abducted. He told us that he was under
custodial investigation. As Ive said earlier, he
was seen under surveillance from January. He
was seen talking to Omar Patik, a certain
Q: Who were they?
Santos of Bulacan who is also a Balik Islam
and charged with terrorism. He was seen
carrying boxes of medicines. Then we asked
A: Salvacion Serrano, Mini Leong, Mrs. Tagitis and me, sir. him how long will he be in custodial
investigation. He said until we can get some
information. But he also told us that he cannot give
us that report because it was a raw report. It was
Q: Were you able to talk, see some other officials at Camp not official, sir.
Katitipan during that time?

A: Col. Kasim (PS Supt. Julasirim Ahadin Kasim) only, sir.


Q: You said that he was reading a report, was that report
in document form, in a piece of paper or was it in
the computer or what?
Q: Were you able to talk to him?

A: As far as I can see it, sir, it is written in white bond


A: Yes, sir. paper. I dont know if it was computerized but Im
certain that it was typewritten. Im not sure if it
used computer, fax or what, sir.

Q: The four of you?

Q: When he was reading it to you, was he reading it line


by line or he was reading in a summary form?
A: Yes, sir.
PNP or AFP is not worthy of belief, since Sr. Supt. Kasim is a high ranking
police officer who would certainly know that the PNP is not part of the
A: Sometimes he was glancing to the report and talking to us, sir. military.
[165]

Upon deeper consideration of these inconsistencies, however, what


appears clear to us is that the petitioners never really steadfastly disputed
or presented evidence to refute the credibility of the respondent and her
xxxx witness, Mrs. Talbin. The inconsistencies the petitioners point out relate,
more than anything else, to details that should not affect the credibility of
Q: Were you informed as to the place where he was being the respondent and Mrs. Talbin; the inconsistencies are not on material
kept during that time? points.[168] We note, for example, that these witnesses are lay people in so
far as military and police matters are concerned, and confusion between
the police and the military is not unusual. As a rule, minor inconsistencies
such as these indicate truthfulness rather than prevarication [169]and only
A: He did not tell us where he [Tagitis] was being
tend to strengthen their probative value, in contrast to testimonies from
kept. But he mentioned this Talipapao, Sulu,
various witnesses dovetailing on every detail; the latter cannot but
sir.
generate suspicion that the material circumstances they testified to were
integral parts of a well thought of and prefabricated story. [170]

Q: After that incident, what did you do if any? Based on these considerations and the unique evidentiary
situation in enforced disappearance cases, we hold it duly
established that Col. Kasim informed the respondent and her
friends, based on the informants letter, that Tagitis, reputedly a
A: We just left and as Ive mentioned, we just waited liaison for the JI and who had been under surveillance since
because that raw information that he was reading January 2007, was in good hands and under custodial investigation
to us [sic] after the custodial investigation, for complicity with the JI after he was seen talking to one Omar
Engineer Tagitis will be released. [Emphasis Patik and a certain Santos of Bulacan, a Balik Islam charged with
supplied][166] terrorism. The respondents and Mrs. Talbins testimonies cannot simply
be defeated by Col. Kasims plain denial and his claim that he had
destroyed his informants letter, the critical piece of evidence that supports
Col. Kasim never denied that he met with the respondent and her or negates the parties conflicting claims. Col. Kasims admitted destruction
friends, and that he provided them information based on the input of an of this letter effectively, a suppression of this evidence raises the
unnamed asset. He simply claimed in his testimony that the informal letter presumption that the letter, if produced, would be proof of what the
he received from his informant in Sulu did not indicate that Tagitis was in respondent claimed.[171] For brevity, we shall call the evidence of what Col.
the custody of the CIDG. He also stressed that the information he provided Kasim reported to the respondent to be the Kasim evidence.
the respondent was merely a raw report from barangay intelligence that
still needed confirmation and follow up as to its veracity. [167] Given this evidence, our next step is to decide whether we can
accept this evidence, in lieu of direct evidence, as proof that the
To be sure, the respondents and Mrs. Talbins testimonies were far disappearance of Tagitis was due to action with government participation,
from perfect, as the petitioners pointed out. The respondent mistakenly knowledge or consent and that he was held for custodial investigation. We
characterized Col. Kasim as a military officer who told her that her note in this regard that Col. Kasim was never quoted to have said that the
husband is being abducted because he is under custodial investigation custodial investigation was by the CIDG Zamboanga. The Kasim evidence
because he is allegedly parang liason ng J.I. The petitioners also noted only implies government intervention through the use of the term
that Mrs. Talbins testimony imputing certain statements to Sr. Supt. Kasim custodial investigation, and does not at all point to CIDG Zamboanga as
that Engr. Tagitis is with the military, but he is not certain whether it is the Tagitis custodian.
Strictly speaking, we are faced here with a classic case of hearsay these pieces of evidence, no other information exists in the records
evidence i.e., evidence whose probative value is not based on the personal relating to the personal circumstances of Tagitis.
knowledge of the witnesses (the respondent, Mrs. Talbin and Col. Kasim
himself) but on the knowledge of some other person not on the witness The actual disappearance of Tagitis is as murky as his personal
stand (the informant).[172] circumstances. While the Amparo petition recited that he was taken away
by burly men believed to be police intelligence operatives, no evidence
To say that this piece of evidence is incompetent and inadmissible whatsoever was introduced to support this allegation. Thus, the available
evidence of what it substantively states is to acknowledge as the direct evidence is that Tagitis was last seen at 12.30 p.m. of October 30,
petitioners effectively suggest that in the absence of any direct evidence, 2007 the day he arrived in Jolo and was never seen again.
we should simply dismiss the petition. To our mind, an immediate
dismissal for this reason is no different from a statement that The Kasim evidence assumes critical materiality given the dearth
the Amparo Rule despite its terms is ineffective, as it cannot allow for the of direct evidence on the above aspects of the case, as it supplies the
special evidentiary difficulties that are unavoidably present gaps that were never looked into and clarifiedby police investigation. It is
in Amparo situations, particularly in extrajudicial killings and enforced the evidence, too, that colors a simple missing person report into an
disappearances. The Amparo Rule was not promulgated with this intent or enforced disappearance case, as it injects the element of participation by
with the intent to make it a token gesture of concern for constitutional agents of the State and thus brings into question how the State reacted to
rights. It was promulgated to provide effective and timely remedies, using the disappearance.
and profiting from local and international experiences in extrajudicial
killings and enforced disappearances, as the situation may Denials on the part of the police authorities, and frustration on the
require. Consequently, we have no choice but to meet the evidentiary part of the respondent, characterize the attempts to locate Tagitis. Initially
difficulties inherent in enforced disappearances with the flexibility that in Jolo, the police informed Kunnong that Tagitis could have been taken by
these difficulties demand. the Abu Sayyaf or other groups fighting the government. No evidence was
ever offered on whether there was active Jolo police investigation and how
To give full meaning to our Constitution and the rights it protects, and why the Jolo police arrived at this conclusion. The respondents own
we hold that, as in Velasquez, we should at least take a close look at the inquiry in Jolo yielded the answer that he was not missing but was with
available evidence to determine the correct import of every piece of another woman somewhere. Again, no evidence exists that this
evidence even of those usually considered inadmissible under the general explanation was arrived at based on an investigation. As already related
rules of evidence taking into account the surrounding circumstances and above, the inquiry with Col. Ancanan in Zamboanga yielded ambivalent
the test of reason that we can use as basic minimum admissibility results not useful for evidentiary purposes. Thus, it was only the inquiry
requirement. In the present case, we should at least determine whether from Col. Kasim that yielded positive results. Col. Kasims story, however,
the Kasim evidence before us is relevant and meaningful to the confirmed only the fact of his custodial investigation (and, impliedly, his
disappearance of Tagistis and reasonably consistent with other evidence in arrest or abduction), without identifying his abductor/s or the party
the case. holding him in custody. The more significant part of Col. Kasims story is
that the abduction came after Tagitis was seen talking with Omar Patik
The evidence about Tagitis personal circumstances surrounded him and a certain Santos of Bulacan, a Balik Islam charged with terrorism. Mrs.
with an air of mystery. He was reputedly a consultant of the World Bank Talbin mentioned, too, that Tagitis was being held at Talipapao,
and a Senior Honorary Counselor for the IDB who attended a seminar in Sulu. None of the police agencies participating in the investigation ever
Zamboanga and thereafter proceded to Jolo for an overnight stay, pursued these leads. Notably, TASK FORCE TAGITIS to which this
indicated by his request to Kunnong for the purchase of a return ticket to information was relayed did not appear to have lifted a finger to pursue
Zamboanga the day after he arrived in Jolo. Nothing in the records these aspects of the case.
indicates the purpose of his overnight sojourn in Jolo. A colleague in the
IDB, Prof. Matli, early on informed the Jolo police that Tagitis may have More denials were manifested in the Returns on the writ to the CA
taken funds given to him in trust for IDB scholars. Prof Matli later on made by the petitioners. Then PNP Chief Gen. Avelino I. Razon merely
stated that he never accused Tagitis of taking away money held in trust, reported the directives he sent to the ARMM Regional Director and the
although he confirmed that the IDB was seeking assistance in locating Regional Chief of the CIDG on Tagitis, and these reports merely reiterated
funds of IDB scholars deposited in Tagitis personal account. Other than the open-ended initial report of the disappearance. The CIDG directed a
search in all of its divisions with negative results. These, to the PNP Chief, disappearance, starting from the initial response by the Jolo police to
constituted the exhaustion of all possible efforts. PNP-CIDG Chief General Kunnongs initial reports of the disappearance, to the responses made to
Edgardo M. Doromal, for his part, also reported negative results after the respondent when she herself reported and inquired about her
searching all divisions and departments [of the CIDG] for a person named husbands disappearance, and even at TASK FORCE TAGITIS itself.
Engr. Morced N. Tagitis . . . and after a diligent and thorough research,
records show that no such person is being detained in the CIDG or any of As the CA found through TASK FORCE TAGITIS, the investigation
its department or divisions. PNP-PACER Chief PS Supt. Leonardo A. Espina was at best haphazard since the authorities were looking for a man whose
and PNP PRO ARMM Regional Director PC Superintendent Joel R. Goltiao picture they initially did not even secure. The returns and reports made to
did no better in their affidavits-returns, as they essentially reported the the CA fared no better, as the CIDG efforts themselves were confined to
results of their directives to their units to search for Tagitis. searching for custodial records of Tagitis in their various departments and
divisions. To point out the obvious, if the abduction of Tagitis was a black
The extent to which the police authorities acted was fully tested operation because it was unrecorded or officially unauthorized, no record
when the CA constituted TASK FORCE TAGITIS, with specific directives on of custody would ever appear in the CIDG records; Tagitis, too, would not
what to do. The negative results reflected in the Returns on the writ were be detained in the usual police or CIDG detention places. In sum, none of
again replicated during the three hearings the CA scheduled. Aside from the reports on record contains any meaningful results or details on
the previously mentioned retraction that Prof. Matli made to correct his the depth and extent of the investigation made. To be sure, reports
accusation that Tagitis took money held in trust for students, PS Supt. of top police officials indicating the personnel and units they directed to
Ajirim reiterated in his testimony that the CIDG consistently denied any investigate can never constitute exhaustive and meaningful investigation,
knowledge or complicity in any abduction and said that there was no basis or equal detailed investigative reports of the activities undertaken to
to conclude that the CIDG or any police unit had anything to do with the search for Tagitis. Indisputably, the police authorities from the very
disappearance of Tagitis; he likewise considered it premature to conclude beginning failed to come up to the extraordinary diligence that
that Tagitis simply ran away with the money in his custody. As already the Amparo Rule requires.
noted above, the TASK FORCE notably did not pursue any investigation
about the personal circumstances of Tagitis, his background in relation to
the IDB and the background and activities of this Bank itself, and the CONCLUSIONS AND THE AMPARO REMEDY
reported sighting of Tagistis with terrorists and his alleged custody in
Talipapao, Sulu. No attempt appears to have ever been made to look into Based on these considerations, we conclude that Col. Kasims
the alleged IDB funds that Tagitis held in trust, or to tap any of the assets disclosure, made in an unguarded moment, unequivocally point to some
who are indispensable in investigations of this nature. These omissions government complicity in the disappearance.The consistent but unfounded
and negative results were aggravated by the CA findings that it was only denials and the haphazard investigations cannot but point to this
as late as January 28, 2008 or three months after the disappearance that conclusion. For why would the government and its officials engage in their
the police authorities requested for clear pictures of Tagitis. Col. Kasim chorus of concealment if the intent had not been to deny what they
could not attend the trial because his subpoena was not served, despite already knew of the disappearance? Would not an in-depth and thorough
the fact that he was designated as Ajirims replacement in the latters last investigation that at least credibly determined the fate of Tagitis be a
post.Thus, Col. Kasim was not then questioned. No investigation even an feather in the governments cap under the circumstances of the
internal one appeared to have been made to inquire into the identity of disappearance? From this perspective, the evidence and developments,
Col. Kasims asset and what he indeed wrote. particularly the Kasim evidence, already establish a concrete case of
enforced disappearance that the Amparo Rule covers. From the prism of
We glean from all these pieces of evidence and the UN Declaration, heretofore cited and quoted, [173] the evidence at hand
developments a consistency in the governments denial of any and the developments in this case confirm the fact of the enforced
complicity in the disappearance of Tagitis, disrupted only by the disappearance and government complicity, under a background of
report made by Col. Kasim to the respondent at Camp consistent and unfounded government denials and haphazard handling.
Katitipan. Even Col. Kasim, however, eventually denied that he ever The disappearance as well effectively placed Tagitis outside the protection
made the disclosure that Tagitis was under custodial investigation for of the law a situation that will subsist unless this Court acts.
complicity in terrorism. Another distinctive trait that runs through
these developments is the governments dismissive approach to the
This kind of fact situation and the conclusion reached are not directly intervened by assigning the investigation of Tagitis disappearance
without precedent in international enforced disappearance rulings. While exclusively to the NBI.
the facts are not exactly the same, the facts of this case run very close to
those of Timurtas v. Turkey,[174] a case decided by ECHR. The European Given their mandates, the PNP and PNP-CIDG officials and
tribunal in that case acted on the basis of the photocopy of a post- members were the ones who were remiss in their duties when the
operation report in finding that Abdulvahap Timurtas (Abdulvahap) was government completely failed to exercise the extral'>To fully enforce
abducted and later detained by agents (gendarmes) of the government of the Amparo remedy, we refer this case back to the CA for appropriate
Turkey. The victim's father in this case brought a claim against Turkey for proceedings directed at the monitoring of the PNP and the PNP-CIDG
numerous violations of the European Convention, including the right to life investigations and actions, and the validation of their results through
(Article 2) and the rights to liberty and security of a person (Article 5). The hearings the CA may deem appropriate to conduct. For purposes of these
applicant contended that on August 14, 1993, gendarmes apprehended his investigations, the PNP/PNP-CIDG shall initially present to the CA a plan of
son, Abdulvahap for being a leader of the Kurdish Workers Party (PKK) in action for further investigation, periodically reporting the detailed results
the Silopi region. The petition was filed in southeast Turkey nearly six and of its investigation to the CA for its consideration and action. On behalf of
one half years after the apprehension. According to the this Court, the CA shall pass upon: the need for the PNP and the PNP-CIDG
father, gendarmes first detained Abdulvahap and then transferred him to to make disclosures of matters known to them as indicated in this Decision
another detainment facility. Although there was no eyewitness and as further CA hearings may indicate; the petitioners submissions; the
evidence of the apprehension or subsequent detainment, the sufficiency of their investigative efforts; and submit to this Court
applicant presented evidence corroborating his version of events, a quarterly report containing its actions and recommendations, copy
including a photocopy of a post-operation report signed by the furnished the petitioners and the respondent, with the first report due at
commander of gendarme operations in Silopi, Turkey. The report the end of the first quarter counted from the finality of this Decision. The
included a description of Abdulvahap's arrest and the result of a PNP and the PNP-CIDG shall have one (1) full year to undertake their
subsequent interrogation during detention where he was accused of being investigation. The CA shall submit its full report for the consideration of
a leader of the PKK in the Silopi region. On this basis, Turkey was held this Court at the end of the 4 th quarter counted from the finality of this
responsible for Abdulvahaps enforced disappearance. Decision.

Following the lead of this Turkish experience - adjusted to WHEREFORE, premises considered, we DENY the petitioners
the Philippine legal setting and the Amparo remedy this Court has petition for review on certiorari for lack of merit, and AFFIRM the decision
established, as applied to the unique facts and developments of of the Court of Appeals dated March 7, 2008 under the following terms:
this case we believe and so hold that the government in general,
through the PNP and the PNP-CIDG, and in particular, the Chiefs of a. Recognition that the disappearance of Engineer
these organizations together with Col. Kasim, should be held fully Morced N. Tagitis is an enforced disappearance covered by
accountable for the enforced disappearance of Tagitis. the Rule on the Writ of Amparo;
b. Without any specific pronouncement on exact
The PNP and CIDG are accountable because Section 24 of Republic authorship and responsibility, declaring the government
Act No. 6975, otherwise known as the PNP Law, [175] specifies the PNP as (through the PNP and the PNP-CIDG) and Colonel Julasirim
the governmental office with the mandate to investigate and prevent Ahadin Kasim accountable for the enforced disappearance
crimes, effect the arrest of criminal offenders, bring offenders to justice of Engineer Morced N. Tagitis;
and assist in their prosecution. The PNP-CIDG, as Col. Jose Volpane Pante c. Confirmation of the validity of the Writ
(then Chief of CIDG Region 9) testified, is the investigative arm of the PNP of Amparo the Court of Appeals issued;
and is mandated to investigate and prosecute all cases involving violations d. Holding the PNP, through the PNP Chief, and the
of the Revised Penal Code, particularly those considered as heinous PNP-CIDG, through its Chief, directly responsible for the
crimes.[176] Under the PNP organizational structure, the PNP-CIDG is tasked disclosure of material facts known to the government and
to investigate all major crimes involving violations of the Revised Penal to their offices regarding the disappearance of Engineer
Code and operates against organized crime groups, unless the President Morced N. Tagitis, and for the conduct of proper
assigns the case exclusively to the National Bureau of Investigation (NBI). investigations using extraordinary diligence, with the
[177]
No indication exists in this case showing that the President ever
obligation to show investigation results acceptable to this
Court;
e. Ordering Colonel Julasirim Ahadin Kasim impleaded
in this case and holding him accountable with the
obligation to disclose information known to him and to his
assets in relation with the enforced disappearance of
Engineer Morced N. Tagitis;
f. Referring this case back to the Court of Appeals for
appropriate proceedings directed at the monitoring of the
PNP and PNP-CIDG investigations, actions and the
validation of their results; the PNP and the PNP-CIDG shall
initially present to the Court of Appeals a plan of action for
further investigation, periodically reporting their results to
the Court of Appeals for consideration and action;
g. Requiring the Court of Appeals to submit to this
Court a quarterly report with its recommendations, copy
furnished the incumbent PNP and PNP-CIDG Chiefs as
petitioners and the respondent, with the first report due at
the end of the first quarter counted from the finality of this
Decision;
h. The PNP and the PNP-CIDG shall have one (1) full
year to undertake their investigations; the Court of
Appeals shall submit its full report for the consideration of
this Court at the end of the 4 th quarter counted from the
finality of this Decision;

These directives and those of the Court of Appeals made pursuant


to this Decision shall be given to, and shall be directly enforceable against,
whoever may be the incumbent Chiefs of the Philippine National Police and
its Criminal Investigation and Detection Group, under pain of contempt
from this Court when the initiatives and efforts at disclosure and
investigation constitute less than the extraordinary diligence that the Rule
on the Writ of Amparo and the circumstances of this case demand. Given
the unique nature of Amparo cases and their varying attendant
circumstances, these directives particularly, the referral back to and
monitoring by the CA are specific to this case and are not standard
remedies that can be applied to every Amparo situation.

The dismissal of the Amparo petition with respect to General


Alexander Yano, Commanding General, Philippine Army, and General
Ruben Rafael, Chief, Anti-Terrorism Task Force Comet, Zamboanga City, is
hereby AFFIRMED.

SO ORDERED. SECOND DIVISION


[G.R. No. 139325. April 12, 2005] principle all too often leads to a result that may be morally correct, but
legally wrong.

Nonetheless, the application of the legal principles involved in this


case will comfort those who maintain that our substantive and procedural
PRISCILLA C. MIJARES, LORETTA ANN P. ROSALES, HILDA B. laws, for all their perceived ambiguity and susceptibility to myriad
NARCISO, SR. MARIANI DIMARANAN, SFIC, and JOEL C. interpretations, are inherently fair and just. The relief sought by the
LAMANGAN in their behalf and on behalf of the Class petitioners is expressly mandated by our laws and conforms to established
Plaintiffs in Class Action No. MDL 840, United States District legal principles. The granting of this petition for certiorari is warranted in
Court of Hawaii, petitioners, vs. HON. SANTIAGO JAVIER order to correct the legally infirm and unabashedly unjust ruling of the
RANADA, in his capacity as Presiding Judge of Branch 137, respondent judge.
Regional Trial Court, Makati City, and the ESTATE OF
FERDINAND E. MARCOS, through its court appointed legal The essential facts bear little elaboration. On 9 May 1991, a complaint
representatives in Class Action MDL 840, United States was filed with the United States District Court (US District Court), District
District Court of Hawaii, namely: Imelda R. Marcos and of Hawaii, against the Estate of former Philippine President Ferdinand E.
Ferdinand Marcos, Jr., respondents. Marcos (Marcos Estate). The action was brought forth by ten Filipino
citizens[2] who each alleged having suffered human rights abuses such as
DECISION arbitrary detention, torture and rape in the hands of police or military
forces during the Marcos regime. [3] The Alien Tort Act was invoked as basis
TINGA, J.: for the US District Courts jurisdiction over the complaint, as it involved a
suit by aliens for tortious violations of international law. [4] These plaintiffs
Our martial law experience bore strange unwanted fruits, and we brought the action on their own behalf and on behalf of a class of similarly
have yet to finish weeding out its bitter crop. While the restoration of situated individuals, particularly consisting of all current civilian citizens of
freedom and the fundamental structures and processes of democracy have the Philippines, their heirs and beneficiaries, who between 1972 and 1987
been much lauded, according to a significant number, the changes, were tortured, summarily executed or had disappeared while in the
however, have not sufficiently healed the colossal damage wrought under custody of military or paramilitary groups. Plaintiffs alleged that the class
the oppressive conditions of the martial law period. The cries of justice for consisted of approximately ten thousand (10,000) members; hence,
the tortured, the murdered, and the desaparecidos arouse outrage and joinder of all these persons was impracticable.
sympathy in the hearts of the fair-minded, yet the dispensation of the The institution of a class action suit was warranted under Rule 23(a)
appropriate relief due them cannot be extended through the same caprice and (b)(1)(B) of the US Federal Rules of Civil Procedure, the provisions of
or whim that characterized the ill-wind of martial rule. The damage done which were invoked by the plaintiffs. Subsequently, the US District Court
was not merely personal but institutional, and the proper rebuke to the certified the case as a class action and created three (3) sub-classes of
iniquitous past has to involve the award of reparations due within the torture, summary execution and disappearance victims. [5] Trial ensued,
confines of the restored rule of law. and subsequently a jury rendered a verdict and an award of compensatory
The petitioners in this case are prominent victims of human rights and exemplary damages in favor of the plaintiff class. Then, on 3 February
violations[1] who, deprived of the opportunity to directly confront the man 1995, the US District Court, presided by Judge Manuel L. Real, rendered a
who once held absolute rule over this country, have chosen to do battle Final Judgment (Final Judgment) awarding the plaintiff class a total of One
instead with the earthly representative, his estate. The clash has been for Billion Nine Hundred Sixty Four Million Five Thousand Eight Hundred Fifty
now interrupted by a trial court ruling, seemingly comported to legal logic, Nine Dollars and Ninety Cents ($1,964,005,859.90). The Final
that required the petitioners to pay a whopping filing fee of over Four Judgment was eventually affirmed by the US Court of Appeals for the
Hundred Seventy-Two Million Pesos (P472,000,000.00) in order that they Ninth Circuit, in a decision rendered on 17 December 1996. [6]
be able to enforce a judgment awarded them by a foreign court. There is On 20 May 1997, the present petitioners filed Complaint with the
an understandable temptation to cast the struggle within the simplistic Regional Trial Court, City of Makati (Makati RTC) for the enforcement of
confines of a morality tale, and to employ short-cuts to arrive at what the Final Judgment. They alleged that they are members of the plaintiff
might seem the desirable solution. But easy, reflexive resort to the equity class in whose favor the US District Court awarded damages. [7] They
argued that since the Marcos Estate failed to file a petition for certiorari Petitioners invoke Section 11, Article III of the Bill of Rights of the
with the US Supreme Court after the Ninth Circuit Court of Appeals had Constitution, which provides that Free access to the courts and quasi-
affirmed the Final Judgment, the decision of the US District Court had judicial bodies and adequate legal assistance shall not be denied to any
become final and executory, and hence should be recognized and enforced person by reason of poverty, a mandate which is essentially defeated by
in the Philippines, pursuant to Section 50, Rule 39 of the Rules of Court the required exorbitant filing fee. The adjudicated amount of the filing fee,
then in force.[8] as arrived at by the RTC, was characterized as indisputably unfair,
inequitable, and unjust.
On 5 February 1998, the Marcos Estate filed a motion to dismiss,
raising, among others, the non-payment of the correct filing fees. It The Commission on Human Rights (CHR) was permitted to intervene
alleged that petitioners had only paid Four Hundred Ten Pesos (P410.00) in this case.[12] It urged that the petition be granted and a judgment
as docket and filing fees, notwithstanding the fact that they sought to rendered, ordering the enforcement and execution of the District Court
enforce a monetary amount of damages in the amount of over Two and a judgment in accordance with Section 48, Rule 39 of the 1997 Rules of Civil
Quarter Billion US Dollars (US$2.25 Billion). The Marcos Estate cited Procedure. For the CHR, the Makati RTC erred in interpreting the action for
Supreme Court Circular No. 7, pertaining to the proper computation and the execution of a foreign judgment as a new case, in violation of the
payment of docket fees. In response, the petitioners claimed that an principle that once a case has been decided between the same parties in
action for the enforcement of a foreign judgment is not capable of one country on the same issue with finality, it can no longer be relitigated
pecuniary estimation; hence, a filing fee of only Four Hundred Ten Pesos again in another country.[13] The CHR likewise invokes the principle of
(P410.00) was proper, pursuant to Section 7(c) of Rule 141. [9] comity, and of vested rights.

On 9 September 1998, respondent Judge Santiago Javier Ranada [10] of The Courts disposition on the issue of filing fees will prove a useful
the Makati RTC issued the subject Order dismissing the complaint without jurisprudential guidepost for courts confronted with actions enforcing
prejudice. Respondent judge opined that contrary to the petitioners foreign judgments, particularly those lodged against an estate. There is no
submission, the subject matter of the complaint was indeed capable of basis for the issuance a limited pro hac vice ruling based on the special
pecuniary estimation, as it involved a judgment rendered by a foreign circumstances of the petitioners as victims of martial law, or on the
court ordering the payment of definite sums of money, allowing for easy emotionally-charged allegation of human rights abuses.
determination of the value of the foreign judgment. On that score, Section
7(a) of Rule 141 of the Rules of Civil Procedure would find application, and An examination of Rule 141 of the Rules of Court readily evinces that
the RTC estimated the proper amount of filing fees was approximately the respondent judge ignored the clear letter of the law when he
Four Hundred Seventy Two Million Pesos, which obviously had not been concluded that the filing fee be computed based on the total sum claimed
paid. or the stated value of the property in litigation.

Not surprisingly, petitioners filed a Motion for Reconsideration, which In dismissing the complaint, the respondent judge relied on Section
Judge Ranada denied in an Order dated 28 July 1999. From this denial, 7(a), Rule 141 as basis for the computation of the filing fee of over P472
petitioners filed a Petition for Certiorari under Rule 65 assailing the twin Million. The provision states:
orders of respondent judge. [11] They prayed for the annulment of the
questioned orders, and an order directing the reinstatement of Civil Case SEC. 7. Clerk of Regional Trial Court.-
No. 97-1052 and the conduct of appropriate proceedings thereon.
(a) For filing an action or a permissive counterclaim or
Petitioners submit that their action is incapable of pecuniary
money claim against an estate not based on judgment, or for
estimation as the subject matter of the suit is the enforcement of a foreign
filing with leave of court a third-party, fourth-party, etc.,
judgment, and not an action for the collection of a sum of money or
complaint, or a complaint in intervention, and for all clerical
recovery of damages. They also point out that to require the class plaintiffs
services in the same time, if the total sum claimed, exclusive of
to pay Four Hundred Seventy Two Million Pesos (P472,000,000.00) in filing
interest, or the started value of the property in litigation, is:
fees would negate and render inutile the liberal construction ordained by
the Rules of Court, as required by Section 6, Rule 1 of the Rules of Civil
Procedure, particularly the inexpensive disposition of every action. 1. Less than P 100,00.00 P 500.00
2. P 100,000.00 or more - P 800.00
but less than P 150,000.00 (b) For filing
3. P 150,000.00 or more but - P 1,000.00
less than P 200,000.00 1. Actions where the value
4. P 200,000.00 or more but of the subject matter
less than P 250,000.00 - P 1,500.00 cannot be estimated --- P 600.00
5. P 250,000.00 or more but
less than P 300,00.00 - P 1,750.00 2. Special civil actions except
6. P 300,000.00 or more but judicial foreclosure which
not more than P 400,000.00 - P 2,000.00 shall be governed by
7. P 350,000.00 or more but not paragraph (a) above --- P 600.00
more than P400,000.00 - P 2,250.00
8. For each P 1,000.00 in excess of 3. All other actions not
involving property --- P 600.00
P 400,000.00 - P 10.00

... In a real action, the assessed value of the property, or if there is none, the
estimated value, thereof shall be alleged by the claimant and shall be the
basis in computing the fees.
(Emphasis supplied)
It is worth noting that the provision also provides that in real actions,
Obviously, the above-quoted provision covers, on one hand, ordinary the assessed value or estimated value of the property shall be alleged by
actions, permissive counterclaims, third-party, etc. complaints and the claimant and shall be the basis in computing the fees. Yet again, this
complaints-in-interventions, and on the other, money claims against provision does not apply in the case at bar. A real action is one where the
estates which are not based on judgment. Thus, the relevant question for plaintiff seeks the recovery of real property or an action affecting title to or
purposes of the present petition is whether the action filed with the lower recovery of possession of real property. [16] Neither the complaint nor the
court is a money claim against an estate not based on judgment. award of damages adjudicated by the US District Court involves any real
Petitioners complaint may have been lodged against an estate, but it property of the Marcos Estate.
is clearly based on a judgment, the Final Judgment of the US District Thus, respondent judge was in clear and serious error when he
Court. The provision does not make any distinction between a local concluded that the filing fees should be computed on the basis of the
judgment and a foreign judgment, and where the law does not distinguish, schematic table of Section 7(a), as the action involved pertains to a claim
we shall not distinguish. against an estate based on judgment. What provision, if any, then should
A reading of Section 7 in its entirety reveals several instances wherein apply in determining the filing fees for an action to enforce a foreign
the filing fee is computed on the basis of the amount of the relief sought, judgment?
or on the value of the property in litigation. The filing fee for requests for To resolve this question, a proper understanding is required on the
extrajudicial foreclosure of mortgage is based on the amount of nature and effects of a foreign judgment in this jurisdiction.
indebtedness or the mortgagees claim.[14] In special proceedings involving
properties such as for the allowance of wills, the filing fee is again based The rules of comity, utility and convenience of nations have
on the value of the property. [15] The aforecited rules evidently have no established a usage among civilized states by which final judgments of
application to petitioners complaint. foreign courts of competent jurisdiction are reciprocally respected and
rendered efficacious under certain conditions that may vary in different
Petitioners rely on Section 7(b), particularly the proviso on actions countries.[17] This principle was prominently affirmed in the leading
where the value of the subject matter cannot be estimated. The provision American case of Hilton v. Guyot[18] and expressly recognized in our
reads in full: jurisprudence beginning with Ingenholl v. Walter E. Olsen & Co.[19] The
conditions required by the Philippines for recognition and enforcement of a
SEC. 7. Clerk of Regional Trial Court.- foreign judgment were originally contained in Section 311 of the Code of
Civil Procedure, which was taken from the California Code of Civil there is no question that the filing of a civil complaint is an appropriate
Procedure which, in turn, was derived from the California Act of March 11, measure for such purpose. A civil action is one by which a party sues
1872.[20]Remarkably, the procedural rule now outlined in Section 48, Rule another for the enforcement or protection of a right, [29] and clearly an
39 of the Rules of Civil Procedure has remained unchanged down to the action to enforce a foreign judgment is in essence a vindication of a right
last word in nearly a century. Section 48 states: prescinding either from a conclusive judgment upon title or the
presumptive evidence of a right. [30] Absent perhaps a statutory grant of
SEC. 48. Effect of foreign judgments. The effect of a judgment of a jurisdiction to a quasi-judicial body, the claim for enforcement of judgment
tribunal of a foreign country, having jurisdiction to pronounce the must be brought before the regular courts. [31]
judgment is as follows: There are distinctions, nuanced but discernible, between the cause of
action arising from the enforcement of a foreign judgment, and that
(a) In case of a judgment upon a specific thing, the judgment is conclusive arising from the facts or allegations that occasioned the foreign judgment.
upon the title to the thing; They may pertain to the same set of facts, but there is an essential
difference in the right-duty correlatives that are sought to be vindicated.
(b) In case of a judgment against a person, the judgment is presumptive For example, in a complaint for damages against a tortfeasor, the cause of
evidence of a right as between the parties and their successors in interest action emanates from the violation of the right of the complainant through
by a subsequent title; the act or omission of the respondent. On the other hand, in a complaint
for the enforcement of a foreign judgment awarding damages from the
same tortfeasor, for the violation of the same right through the same
In either case, the judgment or final order may be repelled by evidence of
manner of action, the cause of action derives not from the tortious act but
a want of jurisdiction, want of notice to the party, collusion, fraud, or clear
from the foreign judgment itself.
mistake of law or fact.
More importantly, the matters for proof are different. Using the above
There is an evident distinction between a foreign judgment in an example, the complainant will have to establish before the court the
action in rem and one in personam. For an action in rem, the foreign tortious act or omission committed by the tortfeasor, who in turn is
judgment is deemed conclusive upon the title to the thing, while in an allowed to rebut these factual allegations or prove extenuating
action in personam, the foreign judgment is presumptive, and not circumstances. Extensive litigation is thus conducted on the facts, and
conclusive, of a right as between the parties and their successors in from there the right to and amount of damages are assessed. On the other
interest by a subsequent title. [21] However, in both cases, the foreign hand, in an action to enforce a foreign judgment, the matter left for proof
judgment is susceptible to impeachment in our local courts on the grounds is the foreign judgment itself, and not the facts from which it prescinds.
of want of jurisdiction or notice to the party, [22] collusion, fraud,[23] or clear
As stated in Section 48, Rule 39, the actionable issues are generally
mistake of law or fact.[24] Thus, the party aggrieved by the foreign
restricted to a review of jurisdiction of the foreign court, the service of
judgment is entitled to defend against the enforcement of such decision in
personal notice, collusion, fraud, or mistake of fact or law. The limitations
the local forum. It is essential that there should be an opportunity to
on review is in consonance with a strong and pervasive policy in all legal
challenge the foreign judgment, in order for the court in this jurisdiction to
systems to limit repetitive litigation on claims and issues. [32] Otherwise
properly determine its efficacy.[25]
known as the policy of preclusion, it seeks to protect party expectations
It is clear then that it is usually necessary for an action to be filed in resulting from previous litigation, to safeguard against the harassment of
order to enforce a foreign judgment [26], even if such judgment has defendants, to insure that the task of courts not be increased by never-
conclusive effect as in the case of in rem actions, if only for the purpose of ending litigation of the same disputes, and in a larger sense to promote
allowing the losing party an opportunity to challenge the foreign judgment, what Lord Coke in the Ferrers Case of 1599 stated to be the goal of all
and in order for the court to properly determine its efficacy. law: rest and quietness. [33] If every judgment of a foreign court were
[27]
Consequently, the party attacking a foreign judgment has the burden of reviewable on the merits, the plaintiff would be forced back on his/her
overcoming the presumption of its validity. [28] original cause of action, rendering immaterial the previously concluded
litigation.[34]
The rules are silent as to what initiatory procedure must be
undertaken in order to enforce a foreign judgment in the Philippines. But
Petitioners appreciate this distinction, and rely upon it to support the in Singsong and Raymundo actually derives, but which incorporates this
proposition that the subject matter of the complaintthe enforcement of a additional nuance omitted in the latter cases:
foreign judgmentis incapable of pecuniary estimation. Admittedly the
proposition, as it applies in this case, is counter-intuitive, and thus xxx However, where the basic issue is something other than the right to
deserves strict scrutiny. For in all practical intents and purposes, the recover a sum of money, where the money claim is purely incidental to, or
matter at hand is capable of pecuniary estimation, down to the last cent. a consequence of, the principal relief sought, like in suits to have the
In the assailed Order, the respondent judge pounced upon this point defendant perform his part of the contract (specific performance)
without equivocation: and in actions for support, or for annulment of judgment or to
foreclose a mortgage, this Court has considered such actions as cases
The Rules use the term where the value of the subject matter cannot be where the subject of the litigation may not be estimated in terms of
estimated. The subject matter of the present case is the judgment money, and are cognizable exclusively by courts of first instance. [37]
rendered by the foreign court ordering defendant to pay plaintiffs definite
sums of money, as and for compensatory damages. The Court finds that Petitioners go on to add that among the actions the Court has
the value of the foreign judgment can be estimated; indeed, it can even be recognized as being incapable of pecuniary estimation include legality of
easily determined. The Court is not minded to distinguish between the conveyances and money deposits,[38] validity of a mortgage,[39] the right to
enforcement of a judgment and the amount of said judgment, and support,[40] validity of documents,[41] rescission of contracts,[42] specific
separate the two, for purposes of determining the correct filing fees. performance,[43] and validity or annulment of judgments.[44] It is urged that
Similarly, a plaintiff suing on promissory note for P1 million cannot be an action for enforcement of a foreign judgment belongs to the same
allowed to pay only P400 filing fees (sic), on the reasoning that the subject class.
matter of his suit is not the P1 million, but the enforcement of the
promissory note, and that the value of such enforcement cannot be This is an intriguing argument, but ultimately it is self-evident that
estimated.[35] while the subject matter of the action is undoubtedly the enforcement of a
foreign judgment, the effect of a providential award would be the
The jurisprudential standard in gauging whether the subject matter of adjudication of a sum of money. Perhaps in theory, such an action is
an action is capable of pecuniary estimation is well-entrenched. The primarily for the enforcement of the foreign judgment, but there is a
Marcos Estate cites Singsong v. Isabela Sawmill and Raymundo v. Court of certain obtuseness to that sort of argument since there is no denying that
Appeals, which ruled: the enforcement of the foreign judgment will necessarily result in the
award of a definite sum of money.
[I]n determining whether an action is one the subject matter of which is But before we insist upon this conclusion past beyond the point of
not capable of pecuniary estimation this Court has adopted the criterion of reckoning, we must examine its possible ramifications. Petitioners raise
first ascertaining the nature of the principal action or remedy sought. If it the point that a declaration that an action for enforcement of foreign
is primarily for the recovery of a sum of money, the claim is considered judgment may be capable of pecuniary estimation might lead to an
capable of pecuniary estimation, and whether jurisdiction is in the instance wherein a first level court such as the Municipal Trial Court would
municipal courts or in the courts of first instance would depend on the have jurisdiction to enforce a foreign judgment. But under the statute
amount of the claim. However, where the basic issue is something other defining the jurisdiction of first level courts, B.P. 129, such courts are not
than the right to recover a sum of money, where the money claim is vested with jurisdiction over actions for the enforcement of foreign
purely incidental to, or a consequence of, the principal relief sought, this judgments.
Court has considered such actions as cases where the subject of the
litigation may not be estimated in terms of money, and are cognizable Sec. 33. Jurisdiction of Metropolitan Trial Courts, Municipal Trial Courts
exclusively by courts of first instance (now Regional Trial Courts). and Municipal Circuit Trial Courts in civil cases. Metropolitan Trial Courts,
Municipal Trial Courts, and Municipal Circuit Trial Courts shall exercise:
On the other hand, petitioners cite the ponencia of Justice JBL Reyes
in Lapitan v. Scandia,[36] from which the rule (1) Exclusive original jurisdiction over civil actions and probate
proceedings, testate and intestate, including the grant of
provisional remedies in proper cases, where the value of the xxx
personal property, estate, or amount of the demand does not
exceed One hundred thousand pesos (P100,000.00) or, in Metro (6) In all cases not within the exclusive jurisdiction of any court, tribunal,
Manila where such personal property, estate, or amount of the person or body exercising jurisdiction or any court, tribunal, person or
demand does not exceed Two hundred thousand pesos body exercising judicial or quasi-judicial functions.
(P200,000.00) exclusive of interest damages of whatever kind,
attorney's fees, litigation expenses, and costs, the amount of
Thus, we are comfortable in asserting the obvious, that the complaint
which must be specifically alleged: Provided, That where there
to enforce the US District Court judgment is one capable of pecuniary
are several claims or causes of action between the same or
estimation. But at the same time, it is also an action based on judgment
different parties, embodied in the same complaint, the amount of
against an estate, thus placing it beyond the ambit of Section 7(a) of Rule
the demand shall be the totality of the claims in all the causes of
141. What provision then governs the proper computation of the filing fees
action, irrespective of whether the causes of action arose out of
over the instant complaint? For this case and other similarly situated
the same or different transactions;
instances, we find that it is covered by Section 7(b)(3), involving as it
(2) Exclusive original jurisdiction over cases of forcible entry and does, other actions not involving property.
unlawful detainer: Provided, That when, in such cases, the
Notably, the amount paid as docket fees by the petitioners on the
defendant raises the question of ownership in his pleadings and
premise that it was an action incapable of pecuniary estimation
the question of possession cannot be resolved without deciding
corresponds to the same amount required for other actions not involving
the issue of ownership, the issue of ownership shall be resolved
property. The petitioners thus paid the correct amount of filing fees, and it
only to determine the issue of possession.
was a grave abuse of discretion for respondent judge to have applied
(3) Exclusive original jurisdiction in all civil actions which involve instead a clearly inapplicable rule and dismissed the complaint.
title to, or possession of, real property, or any interest therein
There is another consideration of supreme relevance in this case, one
where the assessed value of the property or interest therein does
which should disabuse the notion that the doctrine affirmed in this decision
not exceed Twenty thousand pesos (P20,000.00) or, in civil
is grounded solely on the letter of the procedural rule. We earlier adverted
actions in Metro Manila, where such assessed value does not
to the the internationally recognized policy of preclusion, [46] as well as the
exceed Fifty thousand pesos (P50,000.00) exclusive of interest,
principles of comity, utility and convenience of nations [47] as the basis for
damages of whatever kind, attorney's fees, litigation expenses
the evolution of the rule calling for the recognition and enforcement of
and costs: Provided, That value of such property shall be
foreign judgments. The US Supreme Court in Hilton v. Guyot[48] relied
determined by the assessed value of the adjacent lots. [45]
heavily on the concept of comity, as especially derived from the landmark
Section 33 of B.P. 129 refers to instances wherein the cause of action treatise of Justice Story in his Commentaries on the Conflict of Laws of
or subject matter pertains to an assertion of rights and interests over 1834.[49] Yet the notion of comity has since been criticized as one of dim
property or a sum of money. But as earlier pointed out, the subject matter contours[50] or suffering from a number of fallacies. [51] Other conceptual
of an action to enforce a foreign judgment is the foreign judgment itself, bases for the recognition of foreign judgments have evolved such as the
and the cause of action arising from the adjudication of such judgment. vested rights theory or the modern doctrine of obligation. [52]

An examination of Section 19(6), B.P. 129 reveals that the instant There have been attempts to codify through treaties or multilateral
complaint for enforcement of a foreign judgment, even if capable of agreements the standards for the recognition and enforcement of foreign
pecuniary estimation, would fall under the jurisdiction of the Regional Trial judgments, but these have not borne fruition. The members of the
Courts, thus negating the fears of the petitioners. Indeed, an examination European Common Market accede to the Judgments Convention, signed in
of the provision indicates that it can be relied upon as jurisdictional basis 1978, which eliminates as to participating countries all of such obstacles to
with respect to actions for enforcement of foreign judgments, provided recognition such as reciprocity and rvision au fond.[53] The most ambitious
that no other court or office is vested jurisdiction over such complaint: of these attempts is the Convention on the Recognition and Enforcement
of Foreign Judgments in Civil and Commercial Matters, prepared in 1966
Sec. 19. Jurisdiction in civil cases. Regional Trial Courts shall exercise by the Hague Conference of International Law. [54] While it has not received
exclusive original jurisdiction: the ratifications needed to have it take effect, [55] it is recognized as
representing current scholarly thought on the topic. [56] Neither the recognition of judgments serves as an umbrella for a variety of concerns in
Philippines nor the United States are signatories to the Convention. international practice which may lead to a denial of recognition. [62]

Yet even if there is no unanimity as to the applicable theory behind The viability of the public policy defense against the enforcement of a
the recognition and enforcement of foreign judgments or a universal treaty foreign judgment has been recognized in this jurisdiction. [63] This defense
rendering it obligatory force, there is consensus that the viability of such allows for the application of local standards in reviewing the foreign
recognition and enforcement is essential. Steiner and Vagts note: judgment, especially when such judgment creates only a presumptive
right, as it does in cases wherein the judgment is against a person. [64] The
. . . The notion of unconnected bodies of national law on private defense is also recognized within the international sphere, as many civil
international law, each following a quite separate path, is not one law nations adhere to a broad public policy exception which may result in a
conducive to the growth of a transnational community encouraging travel denial of recognition when the foreign court, in the light of the choice-of-
and commerce among its members. There is a contemporary resurgence law rules of the recognizing court, applied the wrong law to the case.
of writing stressing the identity or similarity of the values that systems of
[65]
The public policy defense can safeguard against possible abuses to the
public and private international law seek to further a community interest in easy resort to offshore litigation if it can be demonstrated that the original
common, or at least reasonable, rules on these matters in national legal claim is noxious to our constitutional values.
systems. And such generic principles as reciprocity play an important role There is no obligatory rule derived from treaties or conventions that
in both fields.[57] requires the Philippines to recognize foreign judgments, or allow a
procedure for the enforcement thereof. However, generally accepted
Salonga, whose treatise on private international law is of worldwide principles of international law, by virtue of the incorporation clause of the
renown, points out: Constitution, form part of the laws of the land even if they do not derive
from treaty obligations.[66] The classical formulation in international law
Whatever be the theory as to the basis for recognizing foreign judgments, sees those customary rules accepted as binding result from the
there can be little dispute that the end is to protect the reasonable combination two elements: the established, widespread, and consistent
expectations and demands of the parties. Where the parties have practice on the part of States; and a psychological element known as
submitted a matter for adjudication in the court of one state, and the opinion juris sive necessitates (opinion as to law or necessity). Implicit
proceedings there are not tainted with irregularity, they may fairly be in the latter element is a belief that the practice in question is rendered
expected to submit, within the state or elsewhere, to the enforcement of obligatory by the existence of a rule of law requiring it. [67]
the judgment issued by the court. [58]
While the definite conceptual parameters of the recognition and
enforcement of foreign judgments have not been authoritatively
There is also consensus as to the requisites for recognition of a established, the Court can assert with certainty that such an undertaking
foreign judgment and the defenses against the enforcement thereof. As is among those generally accepted principles of international law. [68] As
earlier discussed, the exceptions enumerated in Section 48, Rule 39 have earlier demonstrated, there is a widespread practice among states
remain unchanged since the time they were adapted in this jurisdiction accepting in principle the need for such recognition and enforcement,
from long standing American rules. The requisites and exceptions as albeit subject to limitations of varying degrees. The fact that there is no
delineated under Section 48 are but a restatement of generally accepted binding universal treaty governing the practice is not indicative of a
principles of international law. Section 98 of The Restatement, Second, widespread rejection of the principle, but only a disagreement as to the
Conflict of Laws, states that a valid judgment rendered in a foreign nation imposable specific rules governing the procedure for recognition and
after a fair trial in a contested proceeding will be recognized in the United enforcement.
States, and on its face, the term valid brings into play requirements such
notions as valid jurisdiction over the subject matter and parties. Aside from the widespread practice, it is indubitable that the
[59]
Similarly, the notion that fraud or collusion may preclude the procedure for recognition and enforcement is embodied in the rules of law,
enforcement of a foreign judgment finds affirmation with foreign whether statutory or jurisprudential, adopted in various foreign
jurisprudence and commentators,[60] as well as the doctrine that the jurisdictions. In the Philippines, this is evidenced primarily by Section 48,
foreign judgment must not constitute a clear mistake of law or fact. [61] And Rule 39 of the Rules of Court which has existed in its current form since
finally, it has been recognized that public policy as a defense to the the early 1900s. Certainly, the Philippine legal system has long ago
accepted into its jurisprudence and procedural rules the viability of an enforcement of a foreign judgment is the foreign judgment itself, and not
action for enforcement of foreign judgment, as well as the requisites for the right-duty correlatives that resulted in the foreign judgment. In this
such valid enforcement, as derived from internationally accepted particular circumstance, given that the complaint is lodged against an
doctrines. Again, there may be distinctions as to the rules adopted by each estate and is based on the US District Courts Final Judgment, this foreign
particular state,[69] but they all prescind from the premise that there is a judgment may, for purposes of classification under the governing
rule of law obliging states to allow for, however generally, the recognition procedural rule, be deemed as subsumed under Section 7(b)(3) of Rule
and enforcement of a foreign judgment. The bare principle, to our mind, 141, i.e., within the class of all other actions not involving property. Thus,
has attained the status of opinio juris in international practice. only the blanket filing fee of minimal amount is required.

This is a significant proposition, as it acknowledges that the procedure Finally, petitioners also invoke Section 11, Article III of the
and requisites outlined in Section 48, Rule 39 derive their efficacy not Constitution, which states that [F]ree access to the courts and quasi-
merely from the procedural rule, but by virtue of the incorporation clause judicial bodies and adequate legal assistance shall not be denied to any
of the Constitution. Rules of procedure are promulgated by the Supreme person by reason of poverty. Since the provision is among the guarantees
Court,[70] and could very well be abrogated or revised by the high court ensured by the Bill of Rights, it certainly gives rise to a demandable right.
itself. Yet the Supreme Court is obliged, as are all State components, to However, now is not the occasion to elaborate on the parameters of this
obey the laws of the land, including generally accepted principles of constitutional right. Given our preceding discussion, it is not necessary to
international law which form part thereof, such as those ensuring the utilize this provision in order to grant the relief sought by the petitioners.
qualified recognition and enforcement of foreign judgments. [71] It is axiomatic that the constitutionality of an act will not be resolved by
the courts if the controversy can be settled on other grounds [73] or unless
Thus, relative to the enforcement of foreign judgments in the the resolution thereof is indispensable for the determination of the case. [74]
Philippines, it emerges that there is a general right recognized within our
body of laws, and affirmed by the Constitution, to seek recognition and One more word. It bears noting that Section 48, Rule 39
enforcement of foreign judgments, as well as a right to defend against acknowledges that the Final Judgment is not conclusive yet, but
such enforcement on the grounds of want of jurisdiction, want of notice to presumptive evidence of a right of the petitioners against the Marcos
the party, collusion, fraud, or clear mistake of law or fact. Estate. Moreover, the Marcos Estate is not precluded to present evidence,
if any, of want of jurisdiction, want of notice to the party, collusion, fraud,
The preclusion of an action for enforcement of a foreign judgment in or clear mistake of law or fact. This ruling, decisive as it is on the question
this country merely due to an exhorbitant assessment of docket fees is of filing fees and no other, does not render verdict on the enforceability of
alien to generally accepted practices and principles in international law. the Final Judgment before the courts under the jurisdiction of the
Indeed, there are grave concerns in conditioning the amount of the filing Philippines, or for that matter any other issue which may legitimately be
fee on the pecuniary award or the value of the property subject of the presented before the trial court. Such issues are to be litigated before the
foreign decision. Such pecuniary award will almost certainly be in foreign trial court, but within the confines of the matters for proof as laid down in
denomination, computed in accordance with the applicable laws and Section 48, Rule 39. On the other hand, the speedy resolution of this claim
standards of the forum.[72] The vagaries of inflation, as well as the relative by the trial court is encouraged, and contumacious delay of the decision on
low-income capacity of the Filipino, to date may very well translate into an the merits will not be brooked by this Court.
award virtually unenforceable in this country, despite its integral validity, if
the docket fees for the enforcement thereof were predicated on the WHEREFORE, the petition is GRANTED. The assailed orders are
amount of the award sought to be enforced. The theory adopted by NULLIFIED and SET ASIDE, and a new order REINSTATING Civil Case No.
respondent judge and the Marcos Estate may even lead to absurdities, 97-1052 is hereby issued. No costs.
such as if applied to an award involving real property situated in places
such as the United States or Scandinavia where real property values are SO ORDERED.
inexorably high. We cannot very well require that the filing fee be Puno, (Chairman), Austria-Martinez, Callejo, Sr., and Chico-Nazario,
computed based on the value of the foreign property as determined by the JJ., concur.
standards of the country where it is located.

As crafted, Rule 141 of the Rules of Civil Procedure avoids


unreasonableness, as it recognizes that the subject matter of an action for
custody over petitioner was passed on by Edgardo to his relatives,
EN BANC Emiliano Militar (Emiliano) and his wife. Three days after, 6 September
1968, Emiliano reported and registered petitioner as a foundling with the
March 8, 2016 Office of the Civil Registrar of Iloilo City (OCR-Iloilo). In her Foundling
Certificate and Certificate of Live Birth, the petitioner was given the name
"Mary Grace Natividad Contreras Militar." 1
G.R. No. 221697

When petitioner was five (5) years old, celebrity spouses Ronald Allan
MARY GRACE NATIVIDAD S. POE-LLAMANZARES, Petitioners,
Kelley Poe (a.k.a. Fenando Poe, Jr.) and Jesusa Sonora Poe (a.k.a. Susan
vs.
Roces) filed a petition for her adoption with the Municipal Trial Court (MTC)
COMELEC AND ESTRELLA C. ELAMPARO Respondents.
of San Juan City. On 13 May 1974, the trial court granted their petition
and ordered that petitioner's name be changed from "Mary Grace
x-----------------------x Natividad Contreras Militar" to "Mary Grace Natividad Sonora Poe."
Although necessary notations were made by OCR-Iloilo on petitioner's
G.R. No. 221698-700 foundling certificate reflecting the court decreed adoption, 2 the petitioner's
adoptive mother discovered only sometime in the second half of 2005 that
MARY GRACE NATIVIDAD S. POE-LLAMANZARES, Petitioners, the lawyer who handled petitioner's adoption failed to secure from the
vs. OCR-Iloilo a new Certificate of Live Birth indicating petitioner's new name
COMELEC, FRANCISCO S. TATAD, ANTONIO P. CONTRERAS AND and the name of her adoptive parents. 3 Without delay, petitioner's mother
AMADO D. VALDEZ Respondents. executed an affidavit attesting to the lawyer's omission which she
submitted to the OCR-Iloilo. On 4 May 2006, OCR-Iloilo issued a new
Certificate of Live Birth in the name of Mary Grace Natividad Sonora Poe. 4
DECISION
Having reached the age of eighteen (18) years in 1986, petitioner
PEREZ, J.: registered as a voter with the local COMELEC Office in San Juan City. On
13 December 1986, she received her COMELEC Voter's Identification Card
Before the Court are two consolidated petitions under Rule 64 in relation to for Precinct No. 196 in Greenhills, San Juan, Metro Manila. 5
Rule 65 of the Rules of Court with extremely urgent application for an ex
parte issuance of temporary restraining order/status quo ante order On 4 April 1988, petitioner applied for and was issued Philippine Passport
and/or writ of preliminary injunction assailing the following: (1) 1 No. F9272876 by the Department of Foreign Affairs (DFA). Subsequently,
December 2015 Resolution of the Commission on Elections (COMELEC) on 5 April 1993 and 19 May 1998, she renewed her Philippine passport
Second Division; (2) 23 December 2015 Resolution of the COMELEC En and respectively secured Philippine Passport Nos. L881511 and
Banc, in SPA No. 15-001 (DC); (3) 11 December 2015 Resolution of the DD156616.7
COMELEC First Division; and ( 4) 23 December 2015 Resolution of the
COMELEC En Banc, in SPA No. 15-002 (DC), SPA No. 15-007 (DC) and SPA
No. 15-139 (DC) for having been issued without jurisdiction or with grave Initially, the petitioner enrolled and pursued a degree in Development
abuse of discretion amounting to lack or excess of jurisdiction. Studies at the University of the Philippines8 but she opted to continue her
studies abroad and left for the United States of America (U.S.) in 1988.
Petitioner graduated in 1991 from Boston College in Chestnuts Hill,
The Facts Massachusetts where she earned her Bachelor of Arts degree in Political
Studies.9
Mary Grace Natividad S. Poe-Llamanzares (petitioner) was found
abandoned as a newborn infant in the Parish Church of Jaro, Iloilo by a On 27 July 1991, petitioner married Teodoro Misael Daniel V. Llamanzares
certain Edgardo Militar (Edgardo) on 3 September 1968. Parental care and (Llamanzares), a citizen of both the Philippines and the U.S., at Sanctuario
de San Jose Parish in San Juan City. 10 Desirous of being with her husband The petitioner and her children briefly stayed at her mother's place until
who was then based in the U.S., the couple flew back to the U.S. two days she and her husband purchased a condominium unit with a parking slot at
after the wedding ceremony or on 29 July 1991. 11 One Wilson Place Condominium in San Juan City in the second half of
2005.27 The corresponding Condominium Certificates of Title covering the
While in the U.S., the petitioner gave birth to her eldest child Brian Daniel unit and parking slot were issued by the Register of Deeds of San Juan
(Brian) on 16 April 1992.12 Her two daughters Hanna MacKenzie (Hanna) City to petitioner and her husband on 20 February 2006. 28 Meanwhile, her
and Jesusa Anika (Anika) were both born in the Philippines on 10 July children of school age began attending Philippine private schools.
1998 and 5 June 2004, respectively. 13
On 14 February 2006, the petitioner made a quick trip to the U.S. to
On 18 October 2001, petitioner became a naturalized American supervise the disposal of some of the family's remaining household
citizen. 14 She obtained U.S. Passport No. 017037793 on 19 December belongings.29 She travelled back to the Philippines on 11 March 2006. 30
2001. 15
In late March 2006, petitioner's husband officially informed the U.S. Postal
On 8 April 2004, the petitioner came back to the Philippines together with Service of the family's change and abandonment of their address in the
Hanna to support her father's candidacy for President in the May 2004 U.S.31 The family home was eventually sold on 27 April 2006. 32 Petitioner's
elections. It was during this time that she gave birth to her youngest husband resigned from his job in the U.S. in April 2006, arrived in the
daughter Anika. She returned to the U.S. with her two daughters on 8 July country on 4 May 2006 and started working for a major Philippine
2004. 16 company in July 2006.33

After a few months, specifically on 13 December 2004, petitioner rushed In early 2006, petitioner and her husband acquired a 509-square meter lot
back to the Philippines upon learning of her father's deteriorating medical in Corinthian Hills, Quezon City where they built their family home 34 and to
condition. 17 Her father slipped into a coma and eventually expired. The this day, is where the couple and their children have been residing. 35 A
petitioner stayed in the country until 3 February 2005 to take care of her Transfer Certificate of Title covering said property was issued in the
father's funeral arrangements as well as to assist in the settlement of his couple's name by the Register of Deeds of Quezon City on 1June 2006.
estate.18
On 7 July 2006, petitioner took her Oath of Allegiance to the Republic of
According to the petitioner, the untimely demise of her father was a severe the Philippines pursuant to Republic Act (R.A.) No. 9225 or the Citizenship
blow to her entire family. In her earnest desire to be with her grieving Retention and Re-acquisition Act of 2003.36 Under the same Act, she filed
mother, the petitioner and her husband decided to move and reside with the Bureau of Immigration (BI) a sworn petition to reacquire
permanently in the Philippines sometime in the first quarter of 2005. 19 The Philippine citizenship together with petitions for derivative citizenship on
couple began preparing for their resettlement including notification of their behalf of her three minor children on 10 July 2006. 37 As can be gathered
children's schools that they will be transferring to Philippine schools for the from its 18 July 2006 Order, the BI acted favorably on petitioner's
next semester;20coordination with property movers for the relocation of petitions and declared that she is deemed to have reacquired her
their household goods, furniture and cars from the U.S. to the Philippine citizenship while her children are considered as citizens of the
Philippines;21 and inquiry with Philippine authorities as to the proper Philippines.38 Consequently, the BI issued Identification Certificates (ICs) in
procedure to be followed in bringing their pet dog into the country. 22 As petitioner's name and in the names of her three (3) children. 39
early as 2004, the petitioner already quit her job in the U.S. 23
Again, petitioner registered as a voter of Barangay Santa Lucia, San Juan
Finally, petitioner came home to the Philippines on 24 May 2005 24 and City on 31 August 2006.40 She also secured from the DFA a new Philippine
without delay, secured a Tax Identification Number from the Bureau of Passport bearing the No. XX4731999.41 This passport was renewed on 18
Internal Revenue. Her three (3) children immediately followed 25 while her March 2014 and she was issued Philippine Passport No. EC0588861 by the
husband was forced to stay in the U.S. to complete pending projects as DFA.42
well as to arrange the sale of their family home there. 26
On 6 October 2010, President Benigno S. Aquino III appointed petitioner Petitioner's filing of her COC for President in the upcoming elections
as Chairperson of the Movie and Television Review and Classification Board triggered the filing of several COMELEC cases against her which were the
(MTRCB).43 Before assuming her post, petitioner executed an "Affidavit of subject of these consolidated cases.
Renunciation of Allegiance to the United States of America and
Renunciation of American Citizenship" before a notary public in Pasig City Origin of Petition for Certiorari in G.R. No. 221697
on 20 October 2010,44 in satisfaction of the legal requisites stated in
Section 5 of R.A. No. 9225.45 The following day, 21 October 2010
A day after petitioner filed her COC for President, Estrella Elamparo
petitioner submitted the said affidavit to the BI46 and took her oath of
(Elamparo) filed a petition to deny due course or cancel said COC which
office as Chairperson of the MTRCB.47 From then on, petitioner stopped
was docketed as SPA No. 15-001 (DC) and raffled to the COMELEC Second
using her American passport. 48
Division.59She is convinced that the COMELEC has jurisdiction over her
petition.60 Essentially, Elamparo's contention is that petitioner committed
On 12 July 2011, the petitioner executed before the Vice Consul of the material misrepresentation when she stated in her COC that she is a
U.S. Embassy in Manila an "Oath/Affirmation of Renunciation of Nationality natural-born Filipino citizen and that she is a resident of the Philippines for
of the United States."49 On that day, she accomplished a sworn at least ten (10) years and eleven (11) months up to the day before the 9
questionnaire before the U.S. Vice Consul wherein she stated that she had May 2016 Elections.61
taken her oath as MTRCB Chairperson on 21 October 2010 with the intent,
among others, of relinquishing her American citizenship. 50 In the same
On the issue of citizenship, Elamparo argued that petitioner cannot be
questionnaire, the petitioner stated that she had resided outside of the
considered as a natural-born Filipino on account of the fact that she was a
U.S., specifically in the Philippines, from 3 September 1968 to 29 July
foundling.62 Elamparo claimed that international law does not confer
1991 and from May 2005 to present. 51
natural-born status and Filipino citizenship on foundlings. 63 Following this
line of reasoning, petitioner is not qualified to apply for reacquisition of
On 9 December 2011, the U.S. Vice Consul issued to petitioner a Filipino citizenship under R.A. No. 9225 for she is not a natural-born
"Certificate of Loss of Nationality of the United States" effective 21 October Filipino citizen to begin with.64Even assuming arguendo that petitioner was
2010.52 a natural-born Filipino, she is deemed to have lost that status when she
became a naturalized American citizen.65 According to Elamparo, natural-
On 2 October 2012, the petitioner filed with the COMELEC her Certificate born citizenship must be continuous from birth. 66
of Candidacy (COC) for Senator for the 2013 Elections wherein she
answered "6 years and 6 months" to the question "Period of residence in On the matter of petitioner's residency, Elamparo pointed out that
the Philippines before May 13, 2013."53 Petitioner obtained the highest petitioner was bound by the sworn declaration she made in her 2012 COC
number of votes and was proclaimed Senator on 16 May 2013. 54 for Senator wherein she indicated that she had resided in the country for
only six ( 6) years and six ( 6) months as of May 2013 Elections. Elamparo
On 19 December 2013, petitioner obtained Philippine Diplomatic Passport likewise insisted that assuming arguendo that petitioner is qualified to
No. DE0004530. 55 regain her natural-born status under R.A. No. 9225, she still fell short of
the ten-year residency requirement of the Constitution as her residence
On 15 October 2015, petitioner filed her COC for the Presidency for the could only be counted at the earliest from July 2006, when she reacquired
May 2016 Elections. 56 In her COC, the petitioner declared that she is a Philippine citizenship under the said Act. Also on the assumption that
natural-born citizen and that her residence in the Philippines up to the day petitioner is qualified to reacquire lost Philippine Citizenship, Elamparo is of
before 9 May 2016 would be ten (10) years and eleven (11) months the belief that she failed to reestablish her domicile in the Philippines. 67
counted from 24 May 2005.57 The petitioner attached to her COC an
"Affidavit Affirming Renunciation of U.S.A. Citizenship" subscribed and Petitioner seasonably filed her Answer wherein she countered that:
sworn to before a notary public in Quezon City on 14 October 2015. 58
(1) the COMELEC did not have jurisdiction over Elamparo's petition
as it was actually a petition for quo warranto which could only be
filed if Grace Poe wins in the Presidential elections, and that the
Department of Justice (DOJ) has primary jurisdiction to revoke the i. Elamparo's petition is merely an action to usurp the
BI's July 18, 2006 Order; sovereign right of the Filipino people to decide a purely
political question, that is, should she serve as the country's
(2) the petition failed to state a cause of action because it did not next leader.68
contain allegations which, if hypothetically admitted, would make
false the statement in her COC that she is a natural-born Filipino After the parties submitted their respective Memoranda, the petition was
citizen nor was there any allegation that there was a willful or deemed submitted for resolution.
deliberate intent to misrepresent on her part;
On 1 December 2015, the COMELEC Second Division promulgated a
(3) she did not make any material misrepresentation in the COC Resolution finding that petitioner's COC, filed for the purpose of running
regarding her citizenship and residency qualifications for: for the President of the Republic of the Philippines in the 9 May 2016
National and Local Elections, contained material representations which are
a. the 1934 Constitutional Convention deliberations show false. The fallo of the aforesaid Resolution reads:
that foundlings were considered citizens;
WHEREFORE, in view of all the foregoing considerations, the instant
b. foundlings are presumed under international law to have Petition to Deny Due Course to or Cancel Certificate of Candidacy is
been born of citizens of the place where they are found; hereby GRANTED. Accordingly, the Certificate of Candidacy for President
of the Republic of the Philippines in the May 9, 2016 National and Local
Elections filed by respondent Mary Grace Natividad Sonora Poe
c. she reacquired her natural-born Philippine citizenship
Llamanzares is hereby CANCELLED.69
under the provisions of R.A. No. 9225;

Motion for Reconsideration of the 1 December 2015 Resolution was filed by


d. she executed a sworn renunciation of her American
petitioner which the COMELEC En Banc resolved in its 23 December 2015
citizenship prior to the filing of her COC for President in the
Resolution by denying the same.70
May 9, 2016 Elections and that the same is in full force
and effect and has not been withdrawn or recanted;
Origin of Petition for Certiorari in G.R. Nos. 221698-700
e. the burden was on Elamparo in proving that she did not
possess natural-born status; This case stemmed from three (3) separate petitions filed by Francisco S.
Tatad (Tatad), Antonio P. Contreras (Contreras) and Amado D. Valdez
(Valdez) against petitioner before the COMELEC which were consolidated
f. residence is a matter of evidence and that she
and raffled to its First Division.
reestablished her domicile in the Philippines as early as
May 24, 2005;
In his petition to disqualify petitioner under Rule 25 of the COMELEC Rules
of Procedure,71 docketed as SPA No. 15-002 (DC), Tatad alleged that
g. she could reestablish residence even before she
petitioner lacks the requisite residency and citizenship to qualify her for
reacquired natural-born citizenship under R.A. No. 9225;
the Presidency.72

h. statement regarding the period of residence in her 2012


Tatad theorized that since the Philippines adheres to the principle of jus
COC for Senator was an honest mistake, not binding and
sanguinis, persons of unknown parentage, particularly foundlings, cannot
should give way to evidence on her true date of
be considered natural-born Filipino citizens since blood relationship is
reacquisition of domicile;
determinative of natural-born status.73 Tatad invoked the rule of statutory
construction that what is not included is excluded. He averred that the fact
that foundlings were not expressly included in the categories of citizens in
the 193 5 Constitution is indicative of the framers' intent to exclude should be cancelled on the ground that she did not possess the ten-year
them.74 Therefore, the burden lies on petitioner to prove that she is a period of residency required for said candidacy and that she made false
natural-born citizen.75 entry in her COC when she stated that she is a legal resident of the
Philippines for ten (10) years and eleven (11) months by 9 May
Neither can petitioner seek refuge under international conventions or 2016.86 Contreras contended that the reckoning period for computing
treaties to support her claim that foundlings have a nationality. 76 According petitioner's residency in the Philippines should be from 18 July 2006, the
to Tatad, international conventions and treaties are not self-executory and date when her petition to reacquire Philippine citizenship was approved by
that local legislations are necessary in order to give effect to treaty the BI.87 He asserted that petitioner's physical presence in the country
obligations assumed by the Philippines.77 He also stressed that there is no before 18 July 2006 could not be valid evidence of reacquisition of her
standard state practice that automatically confers natural-born status to Philippine domicile since she was then living here as an American citizen
foundlings.78 and as such, she was governed by the Philippine immigration laws. 88

Similar to Elamparo's argument, Tatad claimed that petitioner cannot avail In her defense, petitioner raised the following arguments:
of the option to reacquire Philippine citizenship under R.A. No. 9225
because it only applies to former natural-born citizens and petitioner was First, Tatad's petition should be dismissed outright for failure to state a
not as she was a foundling.79 cause of action. His petition did not invoke grounds proper for a
disqualification case as enumerated under Sections 12 and 68 of the
Referring to petitioner's COC for Senator, Tatad concluded that she did not Omnibus Election Code.89 Instead, Tatad completely relied on the alleged
comply with the ten (10) year residency requirement. 80 Tatad opined that lack of residency and natural-born status of petitioner which are not
petitioner acquired her domicile in Quezon City only from the time she among the recognized grounds for the disqualification of a candidate to an
renounced her American citizenship which was sometime in 2010 or elective office.90
2011.81 Additionally, Tatad questioned petitioner's lack of intention to
abandon her U.S. domicile as evinced by the fact that her husband stayed Second, the petitions filed against her are basically petitions for quo
thereat and her frequent trips to the U.S.82 warranto as they focus on establishing her ineligibility for the
Presidency.91 A petition for quo warranto falls within the exclusive
In support of his petition to deny due course or cancel the COC of jurisdiction of the Presidential Electoral Tribunal (PET) and not the
petitioner, docketed as SPA No. 15-139 (DC), Valdez alleged that her COMELEC.92
repatriation under R.A. No. 9225 did not bestow upon her the status of a
natural-born citizen.83 He advanced the view that former natural-born Third, the burden to prove that she is not a natural-born Filipino citizen is
citizens who are repatriated under the said Act reacquires only their on the respondents.93 Otherwise stated, she has a presumption in her
Philippine citizenship and will not revert to their original status as natural- favor that she is a natural-born citizen of this country.
born citizens.84
Fourth, customary international law dictates that foundlings are entitled to
He further argued that petitioner's own admission in her COC for Senator a nationality and are presumed to be citizens of the country where they
that she had only been a resident of the Philippines for at least six (6) are found.94 Consequently, the petitioner is considered as a natural-born
years and six (6) months prior to the 13 May 2013 Elections operates citizen of the Philippines.95
against her. Valdez rejected petitioner's claim that she could have validly
reestablished her domicile in the Philippines prior to her reacquisition of
Philippine citizenship. In effect, his position was that petitioner did not
meet the ten (10) year residency requirement for President.

Unlike the previous COMELEC cases filed against petitioner, Contreras'


petition,85 docketed as SPA No. 15-007 (DC), limited the attack to the
residency issue. He claimed that petitioner's 2015 COC for President
Fifth, she claimed that as a natural-born citizen, she has every right to be COMELEC En Banc issued a Resolution denying petitioner's motion for
repatriated under R.A. No. 9225 or the right to reacquire her natural-born reconsideration.
status.96 Moreover, the official acts of the Philippine Government enjoy the
presumption of regularity, to wit: the issuance of the 18 July 2006 Order Alarmed by the adverse rulings of the COMELEC, petitioner instituted the
of the BI declaring her as natural-born citizen, her appointment as MTRCB present petitions for certiorari with urgent prayer for the issuance of an ex
Chair and the issuance of the decree of adoption of San Juan RTC. 97 She parte temporary restraining order/status quo ante order and/or writ of
believed that all these acts reinforced her position that she is a natural- preliminary injunction. On 28 December 2015, temporary restraining
born citizen of the Philippines.98 orders were issued by the Court enjoining the COMELEC and its
representatives from implementing the assailed COMELEC Resolutions until
Sixth, she maintained that as early as the first quarter of 2005, she further orders from the Court. The Court also ordered the consolidation of
started reestablishing her domicile of choice in the Philippines as the two petitions filed by petitioner in its Resolution of 12 January 2016.
demonstrated by her children's resettlement and schooling in the country, Thereafter, oral arguments were held in these cases.
purchase of a condominium unit in San Juan City and the construction of
their family home in Corinthian Hills.99 The Court GRANTS the petition of Mary Grace Natividad S. Poe-
Llamanzares and to ANNUL and SET ASIDE the:
Seventh, she insisted that she could legally reestablish her domicile of
choice in the Philippines even before she renounced her American 1. Resolution dated 1 December 2015 rendered through its Second
citizenship as long as the three determinants for a change of domicile are Division, in SPA No. 15-001 (DC), entitled Estrella C. Elamparo,
complied with.100She reasoned out that there was no requirement that petitioner, vs. Mary Grace Natividad Sonora Poe-Llamanzares.
renunciation of foreign citizenship is a prerequisite for the acquisition of a
new domicile of choice.101
2. Resolution dated 11 December 2015, rendered through its First
Division, in the consolidated cases SPA No. 15-002 (DC)
Eighth, she reiterated that the period appearing in the residency portion of entitled Francisco S. Tatad, petitioner, vs. Mary Grace Natividad
her COC for Senator was a mistake made in good faith. 102 Sonora Poe-Llamanzares, respondent; SPA No. 15-007 (DC)
entitled Antonio P. Contreras, petitioner, vs. Mary Grace Natividad
In a Resolution103 promulgated on 11 December 2015, the COMELEC First Sonora Poe-Llamanzares, respondent; and SPA No. 15-139 (DC)
Division ruled that petitioner is not a natural-born citizen, that she failed to entitled Amado D. Valdez, petitioner, v. Mary Grace Natividad
complete the ten (10) year residency requirement, and that she Sonora Poe-Llamanzares, respondent.
committed material misrepresentation in her COC when she declared
therein that she has been a resident of the Philippines for a period of ten 3. Resolution dated 23 December 2015 of the Commission En
(10) years and eleven (11) months as of the day of the elections on 9 May Banc, upholding the 1 December 2015 Resolution of the Second
2016. The COMELEC First Division concluded that she is not qualified for Division.
the elective position of President of the Republic of the Philippines. The
dispositive portion of said Resolution reads:
4. Resolution dated 23 December 2015 of the Commission En
Banc, upholding the 11 December 2015 Resolution of the First
WHEREFORE, premises considered, the Commission RESOLVED, as it Division.
hereby RESOLVES, to GRANT the Petitions and cancel the Certificate of
Candidacy of MARY GRACE NATIVIDAD SONORA POE-
The procedure and the conclusions from which the questioned Resolutions
LLAMANZARES for the elective position of President of the Republic of the
emanated are tainted with grave abuse of discretion amounting to lack of
Philippines in connection with the 9 May 2016 Synchronized Local and
jurisdiction. The petitioner is a QUALIFIED CANDIDATE for President in the
National Elections.
9 May 2016 National Elections.

Petitioner filed a motion for reconsideration seeking a reversal of the


COMELEC First Division's Resolution. On 23 December 2015, the
The issue before the COMELEC is whether or not the COC of petitioner (5) Register, after sufficient publication, political parties,
should be denied due course or cancelled "on the exclusive ground" that organizations, or coalitions which, in addition to other
she made in the certificate a false material representation. The exclusivity requirements, must present their platform or program of
of the ground should hedge in the discretion of the COMELEC and restrain government; and accredit citizens' arms of the Commission
it from going into the issue of the qualifications of the candidate for the on Elections. Religious denominations and sects shall not
position, if, as in this case, such issue is yet undecided or undetermined by be registered. Those which seek to achieve their goals
the proper authority. The COMELEC cannot itself, in the same cancellation through violence or unlawful means, or refuse to uphold
case, decide the qualification or lack thereof of the candidate. and adhere to this Constitution, or which are supported by
any foreign government shall likewise be refused
We rely, first of all, on the Constitution of our Republic, particularly its registration.
provisions in Article IX, C, Section 2:
Financial contributions from foreign governments and their
Section 2. The Commission on Elections shall exercise the following agencies to political parties, organizations, coalitions, or
powers and functions: candidates related to elections constitute interference in
national affairs, and, when accepted, shall be an additional
ground for the cancellation of their registration with the
(1) Enforce and administer all laws and regulations relative
Commission, in addition to other penalties that may be
to the conduct of an election, plebiscite, initiative,
prescribed by law.
referendum, and recall.

(6) File, upon a verified complaint, or on its own initiative,


(2) Exercise exclusive original jurisdiction over all contests
petitions in court for inclusion or exclusion of voters;
relating to the elections, returns, and qualifications of all
investigate and, where appropriate, prosecute cases of
elective regional, provincial, and city officials, and
violations of election laws, including acts or omissions
appellate jurisdiction over all contests involving elective
constituting election frauds, offenses, and malpractices.
municipal officials decided by trial courts of general
jurisdiction, or involving elective barangay officials decided
by trial courts of limited jurisdiction. (7) Recommend to the Congress effective measures to
minimize election spending, including limitation of places
where propaganda materials shall be posted, and to
Decisions, final orders, or rulings of the Commission on
prevent and penalize all forms of election frauds, offenses,
election contests involving elective municipal and barangay
malpractices, and nuisance candidacies.
offices shall be final, executory, and not appealable.

(8) Recommend to the President the removal of any officer


(3) Decide, except those involving the right to vote, all
or employee it has deputized, or the imposition of any
questions affecting elections, including determination of
other disciplinary action, for violation or disregard of, or
the number and location of polling places, appointment of
disobedience to its directive, order, or decision.
election officials and inspectors, and registration of voters.

(9) Submit to the President and the Congress a


(4) Deputize, with the concurrence of the President, law
comprehensive report on the conduct of each election,
enforcement agencies and instrumentalities of the
plebiscite, initiative, referendum, or recall.
Government, including the Armed Forces of the Philippines,
for the exclusive purpose of ensuring free, orderly, honest,
peaceful, and credible elections. Not any one of the enumerated powers approximate the exactitude of the
provisions of Article VI, Section 17 of the same basic law stating that:
The Senate and the House of Representatives shall each have an The lack of provision for declaring the ineligibility of candidates, however,
Electoral Tribunal which shall be the sole judge of all contests cannot be supplied by a mere rule. Such an act is equivalent to the
relating to the election, returns, and qualifications of their creation of a cause of action which is a substantive matter which the
respective Members. Each Electoral Tribunal shall be composed of COMELEC, in the exercise of its rule-making power under Art. IX, A, §6 of
nine Members, three of whom shall be Justices of the Supreme the Constitution, cannot do it. It is noteworthy that the Constitution
Court to be designated by the Chief Justice, and the remaining six withholds from the COMELEC even the power to decide cases involving the
shall be Members of the Senate or the House of Representatives, right to vote, which essentially involves an inquiry into qualifications based
as the case may be, who shall be chosen on the basis of on age, residence and citizenship of voters. [Art. IX, C, §2(3)]
proportional representation from the political parties and the
parties or organizations registered under the party-list system The assimilation in Rule 25 of the COMELEC rules of grounds for
represented therein. The senior Justice in the Electoral Tribunal ineligibility into grounds for disqualification is contrary to the evident
shall be its Chairman. intention of the law. For not only in their grounds but also in their
consequences are proceedings for "disqualification" different from those for
or of the last paragraph of Article VII, Section 4 which provides that: a declaration of "ineligibility." "Disqualification" proceedings, as already
stated, are based on grounds specified in § 12 and §68 of the Omnibus
The Supreme Court, sitting en banc, shall be the sole judge of all Election Code and in §40 of the Local Government Code and are for the
contests relating to the election, returns, and qualifications of the purpose of barring an individual from becoming a candidate or from
President or Vice-President, and may promulgate its rules for the continuing as a candidate for public office. In a word, their purpose is
purpose. to eliminate a candidate from the race either from the start or during its
progress. "Ineligibility," on the other hand, refers to the lack of the
qualifications prescribed in the Constitution or the statutes for holding
The tribunals which have jurisdiction over the question of the qualifications
public office and the purpose of the proceedings for declaration of
of the President, the Vice-President, Senators and the Members of the
ineligibility is to remove the incumbent from office.
House of Representatives was made clear by the Constitution. There is no
such provision for candidates for these positions.
Consequently, that an individual possesses the qualifications for a public
office does not imply that he is not disqualified from becoming a candidate
Can the COMELEC be such judge?
or continuing as a candidate for a public office and vice versa. We have
this sort of dichotomy in our Naturalization Law. (C.A. No. 473) That an
The opinion of Justice Vicente V. Mendoza in Romualdez-Marcos v. alien has the qualifications prescribed in §2 of the Law does not imply that
Commission on Elections,104 which was affirmatively cited in the En he does not suffer from any of [the] disqualifications provided in §4.
Banc decision in Fermin v. COMELEC105 is our guide. The citation
in Fermin reads:
Before we get derailed by the distinction as to grounds and the
consequences of the respective proceedings, the importance of the opinion
Apparently realizing the lack of an authorized proceeding for declaring the is in its statement that "the lack of provision for declaring the ineligibility
ineligibility of candidates, the COMELEC amended its rules on February 15, of candidates, however, cannot be supplied by a mere rule". Justice
1993 so as to provide in Rule 25 § 1, the following: Mendoza lectured in Romualdez-Marcos that:

Grounds for disqualification. - Any candidate who does not Three reasons may be cited to explain the absence of an authorized
possess all the qualifications of a candidate as provided for proceeding for determining before election the qualifications of a
by the Constitution or by existing law or who commits any candidate.
act declared by law to be grounds for disqualification may
be disqualified from continuing as a candidate.
First is the fact that unless a candidate wins and is proclaimed elected,
there is no necessity for determining his eligibility for the office. In
contrast, whether an individual should be disqualified as a candidate for
acts constituting election offenses (e.g., vote buying, over spending, Grounds. - Any candidate who, in action or protest in which he is a party,
commission of prohibited acts) is a prejudicial question which should be is declared by final decision of a competent court, guilty of, or found by
determined lest he wins because of the very acts for which his the Commission to be suffering from any disqualification provided by law
disqualification is being sought. That is why it is provided that if the or the Constitution.
grounds for disqualification are established, a candidate will not be voted
for; if he has been voted for, the votes in his favor will not be counted; A Petition to Disqualify a Candidate invoking grounds for a Petition to Deny
and if for some reason he has been voted for and he has won, either he to or Cancel a Certificate of Candidacy or Petition to Declare a Candidate
will not be proclaimed or his proclamation will be set aside. as a Nuisance Candidate, or a combination thereof, shall be summarily
dismissed.
Second is the fact that the determination of a candidates'
eligibility, e.g., his citizenship or, as in this case, his domicile, may take a Clearly, the amendment done in 2012 is an acceptance of the reality of
long time to make, extending beyond the beginning of the term of the absence of an authorized proceeding for determining before election the
office. This is amply demonstrated in the companion case (G.R. No. qualifications of candidate. Such that, as presently required, to disqualify a
120265, Agapito A. Aquino v. COMELEC) where the determination of candidate there must be a declaration by a final judgment of a competent
Aquino's residence was still pending in the COMELEC even after the court that the candidate sought to be disqualified "is guilty of or found by
elections of May 8, 1995. This is contrary to the summary character the Commission to be suffering from any disqualification provided by law
proceedings relating to certificates of candidacy. That is why the law or the Constitution."
makes the receipt of certificates of candidacy a ministerial duty of the
COMELEC and its officers. The law is satisfied if candidates state in their
Insofar as the qualification of a candidate is concerned, Rule 25 and Rule
certificates of candidacy that they are eligible for the position which they
23 are flipsides of one to the other. Both do not allow, are not
seek to fill, leaving the determination of their qualifications to be made
authorizations, are not vestment of jurisdiction, for the COMELEC to
after the election and only in the event they are elected. Only in cases
determine the qualification of a candidate. The facts of qualification must
involving charges of false representations made in certificates of candidacy
beforehand be established in a prior proceeding before an authority
is the COMELEC given jurisdiction.
properly vested with jurisdiction. The prior determination of qualification
may be by statute, by executive order or by a judgment of a competent
Third is the policy underlying the prohibition against pre-proclamation court or tribunal.
cases in elections for President, Vice President, Senators and members of
the House of Representatives. (R.A. No. 7166, § 15) The purpose is to
If a candidate cannot be disqualified without a prior finding that he or she
preserve the prerogatives of the House of Representatives Electoral
is suffering from a disqualification "provided by law or the Constitution,"
Tribunal and the other Tribunals as "sole judges" under the Constitution of
neither can the certificate of candidacy be cancelled or denied due course
the election, returns and qualifications of members of Congress of the
on grounds of false representations regarding his or her qualifications,
President and Vice President, as the case may be. 106
without a prior authoritative finding that he or she is not qualified, such
prior authority being the necessary measure by which the falsity of the
To be sure, the authoritativeness of the Romualdez pronouncements as representation can be found. The only exception that can be conceded are
reiterated in Fermin, led to the amendment through COMELEC Resolution self-evident facts of unquestioned or unquestionable veracity and judicial
No. 9523, on 25 September 2012 of its Rule 25. This, the 15 confessions. Such are, anyway, bases equivalent to prior decisions against
February1993 version of Rule 25, which states that: which the falsity of representation can be determined.

Grounds for disqualification. -Any candidate who does not possess all the The need for a predicate finding or final pronouncement in a proceeding
qualifications of a candidate as provided for by the Constitution or by under Rule 23 that deals with, as in this case, alleged false representations
existing law or who commits any act declared by law to be grounds for regarding the candidate's citizenship and residence, forced the COMELEC
disqualification may be disqualified from continuing as a candidate. 107 to rule essentially that since foundlings108 are not mentioned in the
enumeration of citizens under the 1935 Constitution, 109 they then cannot
was in the 2012 rendition, drastically changed to: be citizens. As the COMELEC stated in oral arguments, when petitioner
admitted that she is a foundling, she said it all. This borders on bigotry. there were 210,349 Filipino males and 886 male aliens, or 99.58%. In
Oddly, in an effort at tolerance, the COMELEC, after saying that it cannot 1970, there were 270,299 Filipino females versus 1, 190 female aliens,
rule that herein petitioner possesses blood relationship with a Filipino or 99.56%. That same year, there were 245,740 Filipino males as against
citizen when "it is certain that such relationship is indemonstrable," only 1,165 male aliens or 99.53%. COMELEC did not dispute these
proceeded to say that "she now has the burden to present evidence to figures. Notably, Commissioner Arthur Lim admitted, during the oral
prove her natural filiation with a Filipino parent." arguments, that at the time petitioner was found in 1968, the majority of
the population in Iloilo was Filipino.112
The fact is that petitioner's blood relationship with a Filipino citizen is
DEMONSTRABLE. Other circumstantial evidence of the nationality of petitioner's parents are
the fact that she was abandoned as an infant in a Roman Catholic Church
At the outset, it must be noted that presumptions regarding paternity is in Iloilo City.1âwphi1 She also has typical Filipino features: height, flat
neither unknown nor unaccepted in Philippine Law. The Family Code of the nasal bridge, straight black hair, almond shaped eyes and an oval face.
Philippines has a whole chapter on Paternity and Filiation. 110 That said,
there is more than sufficient evider1ce that petitioner has Filipino parents There is a disputable presumption that things have happened according to
and is therefore a natural-born Filipino. Parenthetically, the burden of the ordinary course of nature and the ordinary habits of life. 113 All of the
proof was on private respondents to show that petitioner is not a Filipino foregoing evidence, that a person with typical Filipino features is
citizen. The private respondents should have shown that both of abandoned in Catholic Church in a municipality where the population of the
petitioner's parents were aliens. Her admission that she is a foundling did Philippines is overwhelmingly Filipinos such that there would be more than
not shift the burden to her because such status did not exclude the a 99% chance that a child born in the province would be a Filipino, would
possibility that her parents were Filipinos, especially as in this case where indicate more than ample probability if not statistical certainty, that
there is a high probability, if not certainty, that her parents are Filipinos. petitioner's parents are Filipinos. That probability and the evidence on
which it is based are admissible under Rule 128, Section 4 of the Revised
The factual issue is not who the parents of petitioner are, as their Rules on Evidence.
identities are unknown, but whether such parents are Filipinos. Under
Section 4, Rule 128: To assume otherwise is to accept the absurd, if not the virtually
impossible, as the norm. In the words of the Solicitor General:
Sect. 4. Relevancy, collateral matters - Evidence must have such a relation
to the fact in issue as to induce belief in its existence or no-existence. Second. It is contrary to common sense because foreigners do not come to
Evidence on collateral matters shall not be allowed, except when it tends the Philippines so they can get pregnant and leave their newborn babies
in any reasonable degree to establish the probability of improbability of behind. We do not face a situation where the probability is such that every
the fact in issue. foundling would have a 50% chance of being a Filipino and a 50% chance
of being a foreigner. We need to frame our questions properly. What are
The Solicitor General offered official statistics from the Philippine Statistics the chances that the parents of anyone born in the Philippines would be
Authority (PSA)111 that from 1965 to 1975, the total number of foreigners foreigners? Almost zero. What are the chances that the parents of anyone
born in the Philippines was 15,986 while the total number of Filipinos born born in the Philippines would be Filipinos? 99.9%.
in the country was 10,558,278. The statistical probability that any child
born in the Philippines in that decade is natural-born Filipino According to the Philippine Statistics Authority, from 2010 to 2014, on a
was 99.83%. For her part, petitioner presented census statistics for Iloilo yearly average, there were 1,766,046 children born in the Philippines to
Province for 1960 and 1970, also from the PSA. In 1960, there were Filipino parents, as opposed to 1,301 children in the Philippines of foreign
962,532 Filipinos and 4,734 foreigners in the province; 99.62% of the parents. Thus, for that sample period, the ratio of non-Filipino children to
population were Filipinos. In 1970, the figures were 1,162,669 Filipinos natural born Filipino children is 1:1357. This means that the statistical
and 5,304 foreigners, or 99.55%. Also presented were figures for the probability that any child born in the Philippines would be a natural born
child producing ages (15-49). In 1960, there were 230,528 female Filipino is 99.93%.
Filipinos as against 730 female foreigners or 99.68%. In the same year,
From 1965 to 1975, the total number of foreigners born in the Philippines ratifying the Constitution were guided mainly by the explanation
is 15,986 while the total number of Filipinos born in the Philippines is offered by the framers.115
15,558,278. For this period, the ratio of non-Filipino children is 1:661. This
means that the statistical probability that any child born in the Philippines As pointed out by petitioner as well as the Solicitor General, the
on that decade would be a natural born Filipino is 99.83%. deliberations of the 1934 Constitutional Convention show that the framers
intended foundlings to be covered by the enumeration. The following
We can invite statisticians and social anthropologists to crunch the exchange is recorded:
numbers for us, but I am confident that the statistical probability that a
child born in the Philippines would be a natural born Filipino will not be Sr. Rafols: For an amendment. I propose that after subsection 2, the
affected by whether or not the parents are known. If at all, the likelihood following is inserted: "The natural children of a foreign father and a Filipino
that a foundling would have a Filipino parent might even be higher than mother not recognized by the father.
99.9%. Filipinos abandon their children out of poverty or perhaps, shame.
We do not imagine foreigners abandoning their children here in the
xxxx
Philippines thinking those infants would have better economic
opportunities or believing that this country is a tropical paradise suitable
for raising abandoned children. I certainly doubt whether a foreign couple President:
has ever considered their child excess baggage that is best left behind. [We] would like to request a clarification from the proponent of the
amendment. The gentleman refers to natural children or to any kind of
illegitimate children?
To deny full Filipino citizenship to all foundlings and render them stateless
just because there may be a theoretical chance that one among the
thousands of these foundlings might be the child of not just one, but two, Sr. Rafols:
foreigners is downright discriminatory, irrational, and unjust. It just To all kinds of illegitimate children. It also includes natural children of
doesn't make any sense. Given the statistical certainty - 99.9% - that any unknown parentage, natural or illegitimate children of unknown parents.
child born in the Philippines would be a natural born citizen, a decision
denying foundlings such status is effectively a denial of their birthright. Sr. Montinola:
There is no reason why this Honorable Court should use an improbable For clarification. The gentleman said "of unknown parents." Current codes
hypothetical to sacrifice the fundamental political rights of an entire class consider them Filipino, that is, I refer to the Spanish Code wherein all
of human beings. Your Honor, constitutional interpretation and the use of children of unknown parentage born in Spanish territory are considered
common sense are not separate disciplines. Spaniards, because the presumption is that a child of unknown parentage
is the son of a Spaniard. This may be applied in the Philippines in that a
As a matter of law, foundlings are as a class, natural-born citizens. While child of unknown parentage born in the Philippines is deemed to be
the 1935 Constitution's enumeration is silent as to foundlings, there is no Filipino, and there is no need ...
restrictive language which would definitely exclude foundlings either.
Because of silence and ambiguity in the enumeration with respect to Sr. Rafols:
foundlings, there is a need to examine the intent of the framers. In Nitafan There is a need, because we are relating the conditions that are [required]
v. Commissioner of Internal Revenue,114 this Court held that: to be Filipino.

The ascertainment of that intent is but in keeping with the Sr. Montinola:
fundamental principle of constitutional construction that the intent But that is the interpretation of the law, therefore, there is no [more] need
of the framers of the organic law and of the people adopting it for amendment.
should be given effect. The primary task in constitutional
construction is to ascertain and thereafter assure the realization of
the purpose of the framers and of the people in the adoption of the
Constitution. It may also be safely assumed that the people in
Sr. Rafols: During the debates on this provision, Delegate Rafols presented an
The amendment should read thus: amendment to include as Filipino citizens the illegitimate children
"Natural or illegitimate of a foreign father and a Filipino mother recognized with a foreign father of a mother who was a citizen of the
by one, or the children of unknown parentage." Philippines, and also foundlings; but this amendment was defeated
primarily because the Convention believed that the cases,
Sr. Briones: being too few to warrant the inclusion of a provision in the
The amendment [should] mean children born in the Philippines of Constitution to apply to them, should be governed by statutory
unknown parentage. legislation. Moreover, it was believed that the rules of international
law were already clear to the effect that illegitimate children
followed the citizenship of the mother, and that foundlings followed
Sr. Rafols:
the nationality of the place where they were found,
The son of a Filipina to a Foreigner, although this [person] does not
thereby making unnecessary the inclusion in the Constitution of
recognize the child, is not unknown.
the proposed amendment.

President:
This explanation was likewise the position of the Solicitor General during
Does the gentleman accept the amendment or not?
the 16 February 2016 Oral Arguments:

Sr. Rafols:
We all know that the Rafols proposal was rejected. But note that what was
I do not accept the amendment because the amendment would exclude
declined was the proposal for a textual and explicit recognition of
the children of a Filipina with a foreigner who does not recognize the child.
foundlings as Filipinos. And so, the way to explain the constitutional
Their parentage is not unknown and I think those of overseas Filipino
silence is by saying that it was the view of Montinola and Roxas which
mother and father [whom the latter] does not recognize, should also be
prevailed that there is no more need to expressly declare foundlings as
considered as Filipinos.
Filipinos.

President:
Obviously, it doesn't matter whether Montinola's or Roxas' views were
The question in order is the amendment to the amendment from the
legally correct. Framers of a constitution can constitutionalize rules based
Gentleman from Cebu, Mr. Briones.
on assumptions that are imperfect or even wrong. They can even overturn
existing rules. This is basic. What matters here is that Montinola and
Sr. Busion: Roxas were able to convince their colleagues in the convention that there
Mr. President, don't you think it would be better to leave this matter in the is no more need to expressly declare foundlings as Filipinos because they
hands of the Legislature? are already impliedly so recognized.

Sr. Roxas: In other words, the constitutional silence is fully explained in terms of
Mr. President, my humble opinion is that these cases are few and far in linguistic efficiency and the avoidance of redundancy. The policy is clear: it
between, that the constitution need [not] refer to them. By international is to recognize foundlings, as a class, as Filipinos under Art. IV, Section 1
law the principle that children or people born in a country of unknown (3) of the 1935 Constitution. This inclusive policy is carried over into the
parents are citizens in this nation is recognized, and it is not necessary to 1973 and 1987 Constitution. It is appropriate to invoke a famous scholar
include a provision on the subject exhaustively. 116 as he was paraphrased by Chief Justice Fernando: the constitution is not
silently silent, it is silently vocal. 118
Though the Rafols amendment was not carried out, it was not because
there was any objection to the notion that persons of "unknown The Solicitor General makes the further point that the framers "worked to
parentage" are not citizens but only because their number was not enough create a just and humane society," that "they were reasonable patriots
to merit specific mention. Such was the account, 117 cited by petitioner, of and that it would be unfair to impute upon them a discriminatory intent
delegate and constitution law author Jose Aruego who said: against foundlings." He exhorts that, given the grave implications of the
argument that foundlings are not natural-born Filipinos, the Court must Recent legislation is more direct. R.A. No. 8043 entitled "An Act
search the records of the 1935, 1973 and 1987 Constitutions "for an Establishing the Rules to Govern the Inter-Country Adoption of Filipino
express intention to deny foundlings the status of Filipinos. The burden is Children and For Other Purposes" (otherwise known as the "Inter-Country
on those who wish to use the constitution to discriminate against Adoption Act of 1995"), R.A. No. 8552, entitled "An Act Establishing the
foundlings to show that the constitution really intended to take this path to Rules and Policies on the Adoption of Filipino Children and For Other
the dark side and inflict this across the board marginalization." Purposes" (otherwise known as the Domestic Adoption Act of 1998) and
this Court's A.M. No. 02-6-02-SC or the "Rule on Adoption," all expressly
We find no such intent or language permitting discrimination against refer to "Filipino children" and include foundlings as among Filipino children
foundlings. On the contrary, all three Constitutions guarantee the basic who may be adopted.
right to equal protection of the laws. All exhort the State to render social
justice. Of special consideration are several provisions in the present It has been argued that the process to determine that the child is a
charter: Article II, Section 11 which provides that the "State values the foundling leading to the issuance of a foundling certificate under these
dignity of every human person and guarantees full respect for human laws and the issuance of said certificate are acts to acquire or perfect
rights," Article XIII, Section 1 which mandates Congress to "give highest Philippine citizenship which make the foundling a naturalized Filipino at
priority to the enactment of measures that protect and enhance the right best. This is erroneous. Under Article IV, Section 2 "Natural-born citizens
of all the people to human dignity, reduce social, economic, and political are those who are citizens of the Philippines from birth without having to
inequalities x x x" and Article XV, Section 3 which requires the State to perform any act to acquire or perfect their Philippine citizenship." In the
defend the "right of children to assistance, including proper care and first place, "having to perform an act" means that the act must be
nutrition, and special protection from all forms of neglect, abuse, cruelty, personally done by the citizen. In this instance, the determination of
exploitation, and other conditions prejudicial to their development." foundling status is done not by the child but by the
Certainly, these provisions contradict an intent to discriminate against authorities.121 Secondly, the object of the process is the determination of
foundlings on account of their unfortunate status. the whereabouts of the parents, not the citizenship of the child. Lastly, the
process is certainly not analogous to naturalization proceedings to acquire
Domestic laws on adoption also support the principle that foundlings are Philippine citizenship, or the election of such citizenship by one born of an
Filipinos. These laws do not provide that adoption confers citizenship upon alien father and a Filipino mother under the 1935 Constitution, which is an
the adoptee. Rather, the adoptee must be a Filipino in the first place to be act to perfect it.
adopted. The most basic of such laws is Article 15 of the Civil Code which
provides that "[l]aws relating to family rights, duties, status, conditions, In this instance, such issue is moot because there is no dispute that
legal capacity of persons are binding on citizens of the Philippines even petitioner is a foundling, as evidenced by a Foundling Certificate issued in
though living abroad." Adoption deals with status, and a Philippine her favor.122 The Decree of Adoption issued on 13 May 1974, which
adoption court will have jurisdiction only if the adoptee is a Filipino. In Ellis approved petitioner's adoption by Jesusa Sonora Poe and Ronald Allan
and Ellis v. Republic,119 a child left by an unidentified mother was sought to Kelley Poe, expressly refers to Emiliano and his wife, Rosario Militar, as her
be adopted by aliens. This Court said: "foundling parents," hence effectively affirming petitioner's status as a
foundling.123
In this connection, it should be noted that this is a proceedings in
rem, which no court may entertain unless it has jurisdiction, not only over Foundlings are likewise citizens under international law. Under the 1987
the subject matter of the case and over the parties, but also over the Constitution, an international law can become part of the sphere of
res, which is the personal status of Baby Rose as well as that of petitioners domestic law either by transformation or incorporation. The transformation
herein. Our Civil Code (Art. 15) adheres to the theory that jurisdiction over method requires that an international law be transformed into a domestic
the status of a natural person is determined by the latter's nationality. law through a constitutional mechanism such as local legislation. 124 On the
Pursuant to this theory, we have jurisdiction over the status of Baby Rose, other hand, generally accepted principles of international law, by virtue of
she being a citizen of the Philippines, but not over the status of the the incorporation clause of the Constitution, form part of the laws of the
petitioners, who are foreigners.120 (Underlining supplied) land even if they do not derive from treaty obligations. Generally accepted
principles of international law include international custom as evidence of a
general practice accepted as law, and general principles of law recognized
by civilized nations.125 International customary rules are accepted as In 1986, the country also ratified the 1966 International Covenant on Civil
binding as a result from the combination of two elements: the established, and Political Rights (ICCPR). Article 24 thereof provide for the right
widespread, and consistent practice on the part of States; and a of every child "to acquire a nationality:"
psychological element known as the opinionjuris sive necessitates (opinion
as to law or necessity). Implicit in the latter element is a belief that the Article 24
practice in question is rendered obligatory by the existence of a rule of law
requiring it.126 "General principles of law recognized by civilized nations"
1. Every child shall have, without any discrimination as to race, colour,
are principles "established by a process of reasoning" or judicial logic,
sex, language, religion, national or social origin, property or birth, the
based on principles which are "basic to legal systems generally," 127 such as
right, to such measures of protection as are required by his status as a
"general principles of equity, i.e., the general principles of fairness and
minor, on the part of his family, society and the State.
justice," and the "general principle against discrimination" which is
embodied in the "Universal Declaration of Human Rights, the International
Covenant on Economic, Social and Cultural Rights, the International 2. Every child shall be registered immediately after birth and shall have a
Convention on the Elimination of All Forms of Racial Discrimination, the name.
Convention Against Discrimination in Education, the Convention (No. 111)
Concerning Discrimination in Respect of Employment and 3. Every child has the right to acquire a nationality.
Occupation."128 These are the same core principles which underlie the
Philippine Constitution itself, as embodied in the due process and equal The common thread of the UDHR, UNCRC and ICCPR is to obligate the
protection clauses of the Bill of Rights. 129 Philippines to grant nationality from birth and ensure that no child is
stateless. This grant of nationality must be at the time of birth, and it
Universal Declaration of Human Rights ("UDHR") has been interpreted by cannot be accomplished by the application of our present naturalization
this Court as part of the generally accepted principles of international law laws, Commonwealth Act No. 473, as amended, and R.A. No. 9139, both
and binding on the State.130 Article 15 thereof states: of which require the applicant to be at least eighteen (18) years old.

1. Everyone has the right to a nationality. The principles found in two conventions, while yet unratified by the
Philippines, are generally accepted principles of international law. The first
2. No one shall be arbitrarily deprived of his nationality nor denied is Article 14 of the 1930 Hague Convention on Certain Questions Relating
the right to change his nationality. to the Conflict of Nationality Laws under which a foundling is presumed to
have the "nationality of the country of birth," to wit:
The Philippines has also ratified the UN Convention on the Rights of the
Child (UNCRC). Article 7 of the UNCRC imposes the following obligations Article 14
on our country:
A child whose parents are both unknown shall have the nationality of the
Article 7 country of birth. If the child's parentage is established, its nationality shall
be determined by the rules applicable in cases where the parentage is
known.
1. The child shall be registered immediately after birth and shall have the
right from birth to a name, the right to acquire a nationality and as far as
possible, the right to know and be cared for by his or her parents. A foundling is, until the contrary is proved, presumed to have been born
on the territory of the State in which it was found. (Underlining supplied)
2. States Parties shall ensure the implementation of these rights in
accordance with their national law and their obligations under the relevant The second is the principle that a foundling is presumed born of citizens of
international instruments in this field, in particular where the child would the country where he is found, contained in Article 2 of the 1961 United
otherwise be stateless. Nations Convention on the Reduction of Statelessness:
Article 2 Rights and which are "basic to legal systems generally," 136 support the
notion that the right against enforced disappearances and the recognition
A foundling found in the territory of a Contracting State shall, in the of foreign judgments, were correctly considered as "generally accepted
absence of proof to the contrary, be considered to have been born within principles of international law" under the incorporation clause.
the territory of parents possessing the nationality of that State.
Petitioner's evidence137 shows that at least sixty countries in Asia, North
That the Philippines is not a party to the 1930 Hague Convention nor to and South America, and Europe have passed legislation recognizing
the 1961 Convention on the Reduction of Statelessness does not mean foundlings as its citizen. Forty-two (42) of those countries follow the jus
that their principles are not binding. While the Philippines is not a party to sanguinis regime. Of the sixty, only thirty-three (33) are parties to the
the 1930 Hague Convention, it is a signatory to the Universal Declaration 1961 Convention on Statelessness; twenty-six (26) are not signatories to
on Human Rights, Article 15(1) ofwhich131effectively affirms Article 14 of the Convention. Also, the Chief Justice, at the 2 February 2016 Oral
the 1930 Hague Convention. Article 2 of the 1961 "United Nations Arguments pointed out that in 166 out of 189 countries surveyed (or
Convention on the Reduction of Statelessness" merely "gives effect" to 87.83%), foundlings are recognized as citizens. These circumstances,
Article 15(1) of the UDHR.132 In Razon v. Tagitis, 133 this Court noted that including the practice of jus sanguinis countries, show that it is a generally
the Philippines had not signed or ratified the "International Convention for accepted principle of international law to presume foundlings as having
the Protection of All Persons from Enforced Disappearance." Yet, we ruled been born of nationals of the country in which the foundling is found.
that the proscription against enforced disappearances in the said
convention was nonetheless binding as a "generally accepted principle of Current legislation reveals the adherence of the Philippines to this
international law." Razon v. Tagitis is likewise notable for declaring the ban generally accepted principle of international law. In particular, R.A. No.
as a generally accepted principle of international law although the 8552, R.A. No. 8042 and this Court's Rules on Adoption, expressly refer to
convention had been ratified by only sixteen states and had not even come "Filipino children." In all of them, foundlings are among the Filipino
into force and which needed the ratification of a minimum of twenty children who could be adopted. Likewise, it has been pointed that the DFA
states. Additionally, as petitioner points out, the Court was content with issues passports to foundlings. Passports are by law, issued only to
the practice of international and regional state organs, regional state citizens. This shows that even the executive department, acting through
practice in Latin America, and State Practice in the United States. the DFA, considers foundlings as Philippine citizens.

Another case where the number of ratifying countries was not Adopting these legal principles from the 1930 Hague Convention and the
determinative is Mijares v. Ranada, 134 where only four countries had 1961 Convention on Statelessness is rational and reasonable and
"either ratified or acceded to"135 the 1966 "Convention on the Recognition consistent with the jus sanguinis regime in our Constitution. The
and Enforcement of Foreign Judgments in Civil and Commercial Matters" presumption of natural-born citizenship of foundlings stems from the
when the case was decided in 2005. The Court also pointed out that that presumption that their parents are nationals of the Philippines. As the
nine member countries of the European Common Market had acceded to empirical data provided by the PSA show, that presumption is at more
the Judgments Convention. The Court also cited U.S. laws and than 99% and is a virtual certainty.
jurisprudence on recognition of foreign judgments. In all, only the
practices of fourteen countries were considered and yet, there was In sum, all of the international law conventions and instruments on the
pronouncement that recognition of foreign judgments was widespread matter of nationality of foundlings were designed to address the plight of a
practice. defenseless class which suffers from a misfortune not of their own making.
We cannot be restrictive as to their application if we are a country which
Our approach in Razon and Mijares effectively takes into account the fact calls itself civilized and a member of the community of nations. The
that "generally accepted principles of international law" are based not only Solicitor General's warning in his opening statement is relevant:
on international custom, but also on "general principles of law recognized
by civilized nations," as the phrase is understood in Article 38.1 paragraph .... the total effect of those documents is to signify to this Honorable Court
(c) of the ICJ Statute. Justice, fairness, equity and the policy against that those treaties and conventions were drafted because the world
discrimination, which are fundamental principles underlying the Bill of community is concerned that the situation of foundlings renders them
legally invisible. It would be tragically ironic if this Honorable Court ended More importantly, COMELEC's position that natural-born status must be
up using the international instruments which seek to protect and uplift continuous was already rejected in Bengson III v. HRET145 where the
foundlings a tool to deny them political status or to accord them second- phrase "from birth" was clarified to mean at the time of birth: "A person
class citizenship.138 who at the time of his birth, is a citizen of a particular country, is a
natural-born citizen thereof." Neither is "repatriation" an act to "acquire or
The COMELEC also ruled139 that petitioner's repatriation in July 2006 under perfect" one's citizenship. In Bengson III v. HRET, this Court pointed out
the provisions of R.A. No. 9225 did not result in the reacquisition of that there are only two types of citizens under the 1987 Constitution:
natural-born citizenship. The COMELEC reasoned that since the applicant natural-born citizen and naturalized, and that there is no third category for
must perform an act, what is reacquired is not "natural-born" citizenship repatriated citizens:
but only plain "Philippine citizenship."
It is apparent from the enumeration of who are citizens under the present
The COMELEC's rule arrogantly disregards consistent jurisprudence on the Constitution that there are only two classes of citizens: (1) those who are
matter of repatriation statutes in general and of R.A. No. 9225 in natural-born and (2) those who are naturalized in accordance with law. A
particular. citizen who is not a naturalized Filipino, ie., did not have to undergo the
process of naturalization to obtain Philippine citizenship, necessarily is a
natural-born Filipino. Noteworthy is the absence in said enumeration of a
In the seminal case of Bengson Ill v. HRET, 140
repatriation was explained
separate category for persons who, after losing Philippine citizenship,
as follows:
subsequently reacquire it. The reason therefor is clear: as to such persons,
they would either be natural-born or naturalized depending on the reasons
Moreover, repatriation results in the recovery of the original nationality. for the loss of their citizenship and the mode prescribed by the applicable
This means that a naturalized Filipino who lost his citizenship will be law for the reacquisition thereof. As respondent Cruz was not required by
restored to his prior status as a naturalized Filipino citizen. On the other law to go through naturalization proceedings in order to reacquire his
hand, if he was originally a natural-born citizen before he lost his citizenship, he is perforce a natural-born Filipino. As such, he possessed all
Philippine citizenship, he will be restored to his former status as a natural- the necessary qualifications to be elected as member of the House of
born Filipino. Representatives.146

R.A. No. 9225 is a repatriation statute and has been described as such in The COMELEC cannot reverse a judicial precedent. That is reserved to this
several cases. They include Sobejana-Condon v. COMELEC141 where we Court. And while we may always revisit a doctrine, a new rule reversing
described it as an "abbreviated repatriation process that restores one's standing doctrine cannot be retroactively applied. In Morales v. Court of
Filipino citizenship x x x." Also included is Parreno v. Commission on Appeals and Jejomar Erwin S. Binay, Jr.,147 where we decreed reversed the
Audit,142 which cited Tabasa v. Court of Appeals,143where we said that condonation doctrine, we cautioned that it "should be prospective in
"[t]he repatriation of the former Filipino will allow him to recover his application for the reason that judicial decisions applying or interpreting
natural-born citizenship. Parreno v. Commission on Audit144 is categorical the laws of the Constitution, until reversed, shall form part of the legal
that "if petitioner reacquires his Filipino citizenship (under R.A. No. 9225), system of the Philippines." This Court also said that "while the future may
he will ... recover his natural-born citizenship." ultimately uncover a doctrine's error, it should be, as a general rule,
recognized as good law prior to its abandonment. Consequently, the
The COMELEC construed the phrase "from birth" in the definition of natural people's reliance thereupon should be respected."148
citizens as implying "that natural-born citizenship must begin at birth and
remain uninterrupted and continuous from birth." R.A. No. 9225 was Lastly, it was repeatedly pointed out during the oral arguments that
obviously passed in line with Congress' sole prerogative to determine how petitioner committed a falsehood when she put in the spaces for "born to"
citizenship may be lost or reacquired. Congress saw it fit to decree that in her application for repatriation under R.A. No. 9225 the names of her
natural-born citizenship may be reacquired even if it had been once lost. It adoptive parents, and this misled the BI to presume that she was a
is not for the COMELEC to disagree with the Congress' determination. natural-born Filipino. It has been contended that the data required were
the names of her biological parents which are precisely unknown.
This position disregards one important fact - petitioner was legally domicile.152 To successfully effect a change of domicile, one must
adopted. One of the effects of adoption is "to sever all legal ties between demonstrate an actual removal or an actual change of domicile; a bona
the biological parents and the adoptee, except when the biological parent fide intention of abandoning the former place of residence and establishing
is the spouse of the adoptee."149 Under R.A. No. 8552, petitioner was also a new one and definite acts which correspond with the purpose. In other
entitled to an amended birth certificate "attesting to the fact that the words, there must basically be animus manendi coupled with animus non
adoptee is the child of the adopter(s)" and which certificate "shall not bear revertendi. The purpose to remain in or at the domicile of choice must be
any notation that it is an amended issue." 150 That law also requires that for an indefinite period of time; the change of residence must be
"[a]ll records, books, and papers relating to the adoption cases in the files voluntary; and the residence at the place chosen for the new domicile
of the court, the Department [of Social Welfare and Development], or any must be actual.153
other agency or institution participating in the adoption proceedings shall
be kept strictly confidential."151 The law therefore allows petitioner to state Petitioner presented voluminous evidence showing that she and her family
that her adoptive parents were her birth parents as that was what would abandoned their U.S. domicile and relocated to the Philippines for good.
be stated in her birth certificate anyway. And given the policy of strict These evidence include petitioner's former U.S. passport showing her
confidentiality of adoption records, petitioner was not obligated to disclose arrival on 24 May 2005 and her return to the Philippines every time she
that she was an adoptee. travelled abroad; e-mail correspondences starting in March 2005 to
September 2006 with a freight company to arrange for the shipment of
Clearly, to avoid a direct ruling on the qualifications of petitioner, which it their household items weighing about 28,000 pounds to the Philippines; e-
cannot make in the same case for cancellation of COC, it resorted to mail with the Philippine Bureau of Animal Industry inquiring how to ship
opinionatedness which is, moreover, erroneous. The whole process their dog to the Philippines; school records of her children showing
undertaken by COMELEC is wrapped in grave abuse of discretion. enrollment in Philippine schools starting June 2005 and for succeeding
years; tax identification card for petitioner issued on July 2005; titles for
On Residence condominium and parking slot issued in February 2006 and their
corresponding tax declarations issued in April 2006; receipts dated 23
February 2005 from the Salvation Army in the U.S. acknowledging
The tainted process was repeated in disposing of the issue of whether or
donation of items from petitioner's family; March 2006 e-mail to the U.S.
not petitioner committed false material representation when she stated in
Postal Service confirming request for change of address; final statement
her COC that she has before and until 9 May 2016 been a resident of the
from the First American Title Insurance Company showing sale of their
Philippines for ten (10) years and eleven (11) months.
U.S. home on 27 April 2006; 12 July 2011 filled-up questionnaire
submitted to the U.S. Embassy where petitioner indicated that she had
Petitioner's claim that she will have been a resident for ten (10) years and been a Philippine resident since May 2005; affidavit from Jesusa Sonora
eleven (11) months on the day before the 2016 elections, is true. Poe (attesting to the return of petitioner on 24 May 2005 and that she and
her family stayed with affiant until the condominium was purchased); and
The Constitution requires presidential candidates to have ten (10) years' Affidavit from petitioner's husband (confirming that the spouses jointly
residence in the Philippines before the day of the elections. Since the decided to relocate to the Philippines in 2005 and that he stayed behind in
forthcoming elections will be held on 9 May 2016, petitioner must have the U.S. only to finish some work and to sell the family home).
been a resident of the Philippines prior to 9 May 2016 for ten (10) years.
In answer to the requested information of "Period of Residence in the The foregoing evidence were undisputed and the facts were even listed by
Philippines up to the day before May 09, 2016," she put in "10 years 11 the COMELEC, particularly in its Resolution in the Tatad, Contreras and
months" which according to her pleadings in these cases corresponds to a Valdez cases.
beginning date of 25 May 2005 when she returned for good from the U.S.
However, the COMELEC refused to consider that petitioner's domicile had
When petitioner immigrated to the U.S. in 1991, she lost her original been timely changed as of 24 May 2005. At the oral arguments, COMELEC
domicile, which is the Philippines. There are three requisites to acquire a Commissioner Arthur Lim conceded the presence of the first two
new domicile: 1. Residence or bodily presence in a new locality; 2. an requisites, namely, physical presence and animus manendi, but
intention to remain there; and 3. an intention to abandon the old maintained there was no animus non-revertendi.154 The COMELEC
disregarded the import of all the evidence presented by petitioner on the constructing a residence here, returning to the Philippines after all trips
basis of the position that the earliest date that petitioner could have abroad, her husband getting employed here). Indeed, coupled with her
started residence in the Philippines was in July 2006 when her application eventual application to reacquire Philippine citizenship and her family's
under R.A. No. 9225 was approved by the BI. In this regard, COMELEC actual continuous stay in the Philippines over the years, it is clear that
relied on Coquilla v. COMELEC,155 Japzon v. COMELEC156 and Caballero v. when petitioner returned on 24 May 2005 it was for good.
COMELEC. 157 During the oral arguments, the private respondents also
added Reyes v. COMELEC.158 Respondents contend that these cases decree In this connection, the COMELEC also took it against petitioner that she
that the stay of an alien former Filipino cannot be counted until he/she had entered the Philippines visa-free as a balikbayan. A closer look at R.A.
obtains a permanent resident visa or reacquires Philippine citizenship, a No. 6768 as amended, otherwise known as the "An Act Instituting a
visa-free entry under a balikbayan stamp being insufficient. Since Balikbayan Program," shows that there is no overriding intent to
petitioner was still an American (without any resident visa) until her treat balikbayans as temporary visitors who must leave after one year.
reacquisition of citizenship under R.A. No. 9225, her stay from 24 May Included in the law is a former Filipino who has been naturalized abroad
2005 to 7 July 2006 cannot be counted. and "comes or returns to the Philippines." 163 The law institutes
a balikbayan program "providing the opportunity to avail of the necessary
But as the petitioner pointed out, the facts in these four cases are very training to enable the balikbayan to become economically self-reliant
different from her situation. In Coquilla v. COMELEC,159 the only evidence members of society upon their return to the country" 164in line with the
presented was a community tax certificate secured by the candidate and government's "reintegration program."165 Obviously, balikbayans are not
his declaration that he would be running in the elections. Japzon v. ordinary transients.
COMELEC160 did not involve a candidate who wanted to count residence
prior to his reacquisition of Philippine citizenship. With the Court decreeing Given the law's express policy to facilitate the return of a balikbayan and
that residence is distinct from citizenship, the issue there was whether the help him reintegrate into society, it would be an unduly harsh conclusion
candidate's acts after reacquisition sufficed to establish residence. to say in absolute terms that the balikbayan must leave after one year.
In Caballero v. COMELEC, 161 the candidate admitted that his place of work That visa-free period is obviously granted him to allow him to re-establish
was abroad and that he only visited during his frequent vacations. his life and reintegrate himself into the community before he attends to
In Reyes v. COMELEC,162 the candidate was found to be an American the necessary formal and legal requirements of repatriation. And that is
citizen who had not even reacquired Philippine citizenship under R.A. No. exactly what petitioner did - she reestablished life here by enrolling her
9225 or had renounced her U.S. citizenship. She was disqualified on the children and buying property while awaiting the return of her husband and
citizenship issue. On residence, the only proof she offered was a seven- then applying for repatriation shortly thereafter.
month stint as provincial officer. The COMELEC, quoted with approval by
this Court, said that "such fact alone is not sufficient to prove her one-year
No case similar to petitioner's, where the former Filipino's evidence of
residency."
change in domicile is extensive and overwhelming, has as yet been
decided by the Court. Petitioner's evidence of residence is unprecedented.
It is obvious that because of the sparse evidence on residence in the four There is no judicial precedent that comes close to the facts of residence of
cases cited by the respondents, the Court had no choice but to hold that petitioner. There is no indication in Coquilla v. COMELEC,166 and the other
residence could be counted only from acquisition of a permanent resident cases cited by the respondents that the Court intended to have its rulings
visa or from reacquisition of Philippine citizenship. In contrast, the there apply to a situation where the facts are different. Surely, the issue of
evidence of petitioner is overwhelming and taken together leads to no residence has been decided particularly on the facts-of-the case basis.
other conclusion that she decided to permanently abandon her U.S.
residence (selling the house, taking the children from U.S. schools, getting
quotes from the freight company, notifying the U.S. Post Office of the
abandonment of their address in the U.S., donating excess items to the
Salvation Army, her husband resigning from U.S. employment right after
selling the U.S. house) and permanently relocate to the Philippines and
actually re-established her residence here on 24 May 2005 (securing T.I.N,
enrolling her children in Philippine schools, buying property here,
To avoid the logical conclusion pointed out by the evidence of residence of The COMELEC, by its own admission, disregarded the evidence that
petitioner, the COMELEC ruled that petitioner's claim of residence of ten petitioner actually and physically returned here on 24 May 2005 not
(10) years and eleven (11) months by 9 May 2016 in her 2015 COC was because it was false, but only because COMELEC took the position that
false because she put six ( 6) years and six ( 6) months as "period of domicile could be established only from petitioner's repatriation under R.A.
residence before May 13, 2013" in her 2012 COC for Senator. Thus, No. 9225 in July 2006. However, it does not take away the fact that in
according to the COMELEC, she started being a Philippine resident only in reality, petitioner had returned from the U.S. and was here to stay
November 2006. In doing so, the COMELEC automatically assumed as true permanently, on 24 May 2005. When she claimed to have been a resident
the statement in the 2012 COC and the 2015 COC as false. for ten (10) years and eleven (11) months, she could do so in good faith.

As explained by petitioner in her verified pleadings, she misunderstood the For another, it could not be said that petitioner was attempting to hide
date required in the 2013 COC as the period of residence as of the day she anything. As already stated, a petition for quo warranto had been filed
submitted that COC in 2012. She said that she reckoned residency from against her with the SET as early as August 2015. The event from which
April-May 2006 which was the period when the U.S. house was sold and the COMELEC pegged the commencement of residence, petitioner's
her husband returned to the Philippines. In that regard, she was advised repatriation in July 2006 under R.A. No. 9225, was an established fact to
by her lawyers in 2015 that residence could be counted from 25 May 2005. repeat, for purposes of her senatorial candidacy.

Petitioner's explanation that she misunderstood the query in 2012 (period Notably, on the statement of residence of six (6) years and six (6) months
of residence before 13 May 2013) as inquiring about residence as of the in the 2012 COC, petitioner recounted that this was first brought up in the
time she submitted the COC, is bolstered by the change which the media on 2 June 2015 by Rep. Tobias Tiangco of the United Nationalist
COMELEC itself introduced in the 2015 COC which is now "period of Alliance. Petitioner appears to have answered the issue immediately, also
residence in the Philippines up to the day before May 09, 2016." The in the press. Respondents have not disputed petitioner's evidence on this
COMELEC would not have revised the query if it did not acknowledge that point. From that time therefore when Rep. Tiangco discussed it in the
the first version was vague. media, the stated period of residence in the 2012 COC and the
circumstances that surrounded the statement were already matters of
That petitioner could have reckoned residence from a date earlier than the public record and were not hidden.
sale of her U.S. house and the return of her husband is plausible given the
evidence that she had returned a year before. Such evidence, to repeat, Petitioner likewise proved that the 2012 COC was also brought up in the
would include her passport and the school records of her children. SET petition for quo warranto. Her Verified Answer, which was filed on 1
September 2015, admitted that she made a mistake in the 2012 COC
It was grave abuse of discretion for the COMELEC to treat the 2012 COC when she put in six ( 6) years and six ( 6) months as she misunderstood
as a binding and conclusive admission against petitioner. It could be given the question and could have truthfully indicated a longer period. Her
in evidence against her, yes, but it was by no means conclusive. There is answer in the SET case was a matter of public record. Therefore, when
precedent after all where a candidate's mistake as to period of residence petitioner accomplished her COC for President on 15 October 2015, she
made in a COC was overcome by evidence. In Romualdez-Marcos v. could not be said to have been attempting to hide her erroneous
COMELEC,167 the candidate mistakenly put seven (7) months as her period statement in her 2012 COC for Senator which was expressly mentioned in
of residence where the required period was a minimum of one year. We her Verified Answer.
said that "[i]t is the fact of residence, not a statement in a certificate of
candidacy which ought to be decisive in determining whether or not an The facts now, if not stretched to distortion, do not show or even hint at
individual has satisfied the constitutions residency qualification an intention to hide the 2012 statement and have it covered by the 2015
requirement." The COMELEC ought to have looked at the evidence representation. Petitioner, moreover, has on her side this Court's
presented and see if petitioner was telling the truth that she was in the pronouncement that:
Philippines from 24 May 2005. Had the COMELEC done its duty, it would
have seen that the 2012 COC and the 2015 COC both correctly stated Concededly, a candidate's disqualification to run for public office does not
the pertinent period of residency. necessarily constitute material misrepresentation which is the sole ground
for denying due course to, and for the cancellation of, a COC. Further, as Meanwhile [petitioner] and her children lived with her mother in San Juan
already discussed, the candidate's misrepresentation in his COC must not City. [Petitioner] enrolled Brian in Beacon School in Taguig City in 2005
only refer to a material fact (eligibility and qualifications for elective and Hanna in Assumption College in Makati City in 2005. Anika was
office), but should evince a deliberate intent to mislead, misinform or hide enrolled in Learning Connection in San Juan in 2007, when she was
a fact which would otherwise render a candidate ineligible. It must be already old enough to go to school.
made with an intention to deceive the electorate as to one's qualifications
to run for public office.168 In the second half of 2005, [petitioner] and her husband acquired Unit 7F
of One Wilson Place Condominium in San Juan. [Petitioner] and her family
In sum, the COMELEC, with the same posture of infallibilism, virtually lived in Unit 7F until the construction of their family home in Corinthian
ignored a good number of evidenced dates all of which can evince animus Hills was completed.
manendi to the Philippines and animus non revertedi to the United States
of America. The veracity of the events of coming and staying home was as Sometime in the second half of 2005, [petitioner's] mother discovered that
much as dismissed as inconsequential, the focus having been fixed at the her former lawyer who handled [petitioner's] adoption in 1974 failed to
petitioner's "sworn declaration in her COC for Senator" which the secure from the Office of the Civil Registrar of Iloilo a new Certificate of
COMELEC said "amounts to a declaration and therefore an admission that Live Birth indicating [petitioner's] new name and stating that her parents
her residence in the Philippines only commence sometime in November are "Ronald Allan K. Poe" and "Jesusa L. Sonora."
2006"; such that "based on this declaration, [petitioner] fails to meet the
residency requirement for President." This conclusion, as already shown,
In February 2006, [petitioner] travelled briefly to the US in order to
ignores the standing jurisprudence that it is the fact of residence, not the
supervise the disposal of some of the family's remaining household
statement of the person that determines residence for purposes of
belongings.1a\^/phi1 [Petitioner] returned to the Philippines on 11 March
compliance with the constitutional requirement of residency for election as
2006.
President. It ignores the easily researched matter that cases on questions
of residency have been decided favorably for the candidate on the basis of
facts of residence far less in number, weight and substance than that In late March 2006, [petitioner's] husband informed the United States
presented by petitioner.169 It ignores, above all else, what we consider as a Postal Service of the family's abandonment of their address in the US.
primary reason why petitioner cannot be bound by her declaration in her
COC for Senator which declaration was not even considered by the SET as The family home in the US was sole on 27 April 2006.
an issue against her eligibility for Senator. When petitioner made the
declaration in her COC for Senator that she has been a resident for a In April 2006, [petitioner's] husband resigned from his work in the US. He
period of six (6) years and six (6) months counted up to the 13 May 2013 returned to the Philippines on 4 May 2006 and began working for a
Elections, she naturally had as reference the residency requirements for Philippine company in July 2006.
election as Senator which was satisfied by her declared years of residence.
It was uncontested during the oral arguments before us that at the time
the declaration for Senator was made, petitioner did not have as yet any In early 2006, [petitioner] and her husband acquired a vacant lot in
intention to vie for the Presidency in 2016 and that the general public was Corinthian Hills, where they eventually built their family home. 170
never made aware by petitioner, by word or action, that she would run for
President in 2016. Presidential candidacy has a length-of-residence In light of all these, it was arbitrary for the COMELEC to satisfy its
different from that of a senatorial candidacy. There are facts of residence intention to let the case fall under the exclusive ground of false
other than that which was mentioned in the COC for Senator. Such other representation, to consider no other date than that mentioned by
facts of residence have never been proven to be false, and these, to petitioner in her COC for Senator.
repeat include:
All put together, in the matter of the citizenship and residence of petitioner
[Petitioner] returned to the Philippines on 24 May 2005. (petitioner's] for her candidacy as President of the Republic, the questioned Resolutions
husband however stayed in the USA to finish pending projects and arrange of the COMELEC in Division and En Banc are, one and all, deadly diseased
the sale of their family home. with grave abuse of discretion from root to fruits.
WHEREFORE, the petition is GRANTED. The Resolutions, to wit:

1. dated 1 December 2015 rendered through the COMELEC Second


Division, in SPA No. 15-001 (DC), entitled Estrella C. Elamparo, petitioner,
vs. Mary Grace Natividad Sonora Poe-Llamanzares, respondent, stating
that:

[T]he Certificate of Candidacy for President of the Republic of the


Philippines in the May 9, 2016 National and Local Elections filed by
EN BANC
respondent Mary Grace Natividad Sonora Poe-Llamanzares is hereby
GRANTED.
[G.R. No. 139465. January 18, 2000]
2. dated 11 December 2015, rendered through the COMELEC First
Division, in the consolidated cases SPA No. 15-002 (DC) SECRETARY OF JUSTICE, petitioner, vs. HON. RALPH C. LANTION,
entitled Francisco S. Tatad, petitioner, vs. Mary Grace Natividad Sonora Presiding Judge, Regional Trial Court of Manila, Branch 25, and
Poe-Llamanzares, respondent; SPA No. 15-007 (DC) entitled Antonio P. MARK B. JIMENEZ, respondents.
Contreras, petitioner, vs. Mary Grace Natividad Sonora Poe-Llamanzares,
respondent; and SPA No. 15-139 (DC) entitled Amado D. Valdez, DECISION
petitioner, v. Mary Grace Natividad Sonora Poe-Llamanzares, respondent;
stating that: MELO, J.:

WHEREFORE, premises considered, the Commission RESOLVED, as it The individual citizen is but a speck of particle or molecule vis--vis the vast
hereby RESOLVES, to GRANT the petitions and cancel the Certificate of and overwhelming powers of government. His only guarantee against
Candidacy of MARY GRACE NATIVIDAD SONORA POE-LLAMANZARES for oppression and tyranny are his fundamental liberties under the Bill of
the elective position of President of the Republic of the Philippines in Rights which shield him in times of need. The Court is now called to decide
connection with the 9 May 2016 Synchronized Local and National Elections. whether to uphold a citizens basic due process rights, or the governments
ironclad duties under a treaty. The bugle sounds and this Court must once
3. dated 23 December 2015 of the COMELEC En Banc, upholding the 1 again act as the faithful guardian of the fundamental writ.
December 2015 Resolution of the Second Division stating that:
The petition at our doorstep is cast against the following factual backdrop:
WHEREFORE, premises considered, the Commission RESOLVED, as it
hereby RESOLVES, to DENY the Verified Motion for Reconsideration of On January 13, 1977, then President Ferdinand E. Marcos issued
SENATOR MARY GRACE NATIVIDAD SONORA POE-LLAMANZARES. The Presidential Decree No. 1069 "Prescribing the Procedure for the Extradition
Resolution dated 11 December 2015 of the Commission First Division is of Persons Who Have Committed Crimes in a Foreign Country". The Decree
AFFIRMED. is founded on: the doctrine of incorporation under the Constitution; the
mutual concern for the suppression of crime both in the state where it was
4. dated 23 December 2015 of the COMELEC En Banc, upholding the 11 committed and the state where the criminal may have escaped; the
December 2015 Resolution of the First Division. extradition treaty with the Republic of Indonesia and the intention of the
Philippines to enter into similar treaties with other interested countries;
are hereby ANNULED and SET ASIDE. Petitioner MARY GRACE and the need for rules to guide the executive department and the courts in
NATIVIDAD SONORA POE-LLAMANZARES is DECLARED QUALIFIED to be the proper implementation of said treaties.
a candidate for President in the National and Local Elections of 9 May
2016.
On November 13, 1994, then Secretary of Justice Franklin M. Drilon, case pursuant to Section 5(1) of Presidential Decree No. 1069.
representing the Government of the Republic of the Philippines, signed in Accordingly, the panel began with the "technical evaluation and
Manila the "Extradition Treaty Between the Government of the Republic of assessment" of the extradition request and the documents in support
the Philippines and the Government of the United States of America" thereof. The panel found that the "official English translation of some
(hereinafter referred to as the RP-US Extradition Treaty). The Senate, by documents in Spanish were not attached to the request and that there are
way of Resolution No. 11, expressed its concurrence in the ratification of some other matters that needed to be addressed" (p. 15,
said treaty. It also expressed its concurrence in the Diplomatic Notes Rollo).
correcting Paragraph (5)(a), Article 7 thereof (on the admissibility of the
documents accompanying an extradition request upon certification by the Pending evaluation of the aforestated extradition documents, private
principal diplomatic or consular officer of the requested state resident in respondent, through counsel, wrote a letter dated July 1, 1999 addressed
the Requesting State). to petitioner requesting copies of the official extradition request from the
U. S. Government, as well as all documents and papers submitted
On June 18, 1999, the Department of Justice received from the therewith; and that he be given ample time to comment on the request
Department of Foreign Affairs U. S. Note Verbale No. 0522 containing a after he shall have received copies of the requested papers. Private
request for the extradition of private respondent Mark Jimenez to the respondent also requested that the proceedings on the matter be held in
United States. Attached to the Note Verbale were the Grand Jury abeyance in the meantime.
Indictment, the warrant of arrest issued by the U.S. District Court,
Southern District of Florida, and other supporting documents for said Later, private respondent requested that preliminarily, he be given at least
extradition. Based on the papers submitted, private respondent appears to a copy of, or access to, the request of the United States Government, and
be charged in the United States with violation of the following provisions of after receiving a copy of the Diplomatic Note, a period of time to amplify
the United States Code (USC): on his request.

A)......18 USC 371 (Conspiracy to commit offense or to In response to private respondents July 1, 1999 letter, petitioner, in a
defraud the United States; two [2] counts; Maximum reply-letter dated July 13, 1999 (but received by private respondent only
Penalty 5 years on each count); on August 4, 1999), denied the foregoing requests for the following
reasons:
B)......26 USC 7201 (Attempt to evade or defeat tax; four
[4] counts; Maximum Penalty 5 years on each count); 1. We find it premature to furnish you with copies of the
extradition request and supporting documents from the
C)......18 USC 1343 (Fraud by wire, radio, or television; United States Government, pending evaluation by this
two [2] counts; Maximum Penalty 5 years on each count); Department of the sufficiency of the extradition documents
submitted in accordance with the provisions of the
D)......18 USC 1001 (False statement or entries; six [6] extradition treaty and our extradition law. Article 7 of the
counts; Maximum Penalty 5 years on each count); Extradition Treaty between the Philippines and the United
States enumerates the documentary requirements and
establishes the procedures under which the documents
E)......2 USC 441f (Election contributions in name of
submitted shall be received and admitted as evidence.
another; thirty-three [33] counts; Maximum Penalty less
Evidentiary requirements under our domestic law are also
than one year).
set forth in Section 4 of P.D. No. 1069.

(p. 14, Rollo.)


Evaluation by this Department of the aforementioned
documents is not a preliminary investigation nor akin to
On the same day, petitioner issued Department Order No. 249 designating preliminary investigation of criminal cases. We merely
and authorizing a panel of attorneys to take charge of and to handle the determine whether the procedures and requirements under
the relevant law and treaty have been complied with by Such was the state of affairs when, on August 6, 1999, private respondent
the Requesting Government. The constitutionally filed with the Regional Trial Court of the National Capital Judicial Region a
guaranteed rights of the accused in all criminal petition against the Secretary of Justice, the Secretary of Foreign Affairs,
prosecutions are therefore not available. and the Director of the National Bureau of Investigation, for mandamus (to
compel herein petitioner to furnish private respondent the extradition
It is only after the filing of the petition for extradition when documents, to give him access thereto, and to afford him an opportunity
the person sought to be extradited will be furnished by the to comment on, or oppose, the extradition request, and thereafter to
court with copies of the petition, request and extradition evaluate the request impartially, fairly and objectively); certiorari (to set
documents and this Department will not pose any aside herein petitioners letter dated July 13, 1999); and prohibition (to
objection to a request for ample time to evaluate said restrain petitioner from considering the extradition request and from filing
documents. an extradition petition in court; and to enjoin the Secretary of Foreign
Affairs and the Director of the NBI from performing any act directed to the
extradition of private respondent to the United States), with an application
2. The formal request for extradition of the United States
for the issuance of a temporary restraining order and a writ of preliminary
contains grand jury information and documents obtained
injunction (pp. 104-105, Rollo).
through grand jury process covered by strict secrecy rules
under United States law. The United States had to secure
orders from the concerned District Courts authorizing the The aforementioned petition was docketed as Civil Case No. 99-94684 and
United States to disclose certain grand jury information to thereafter raffled to Branch 25 of said regional trial court stationed in
Philippine government and law enforcement personnel for Manila which is presided over by the Honorable Ralph C. Lantion.
the purpose of extradition of Mr. Jimenez. Any further
disclosure of the said information is not authorized by the After due notice to the parties, the case was heard on August 9, 1999.
United States District Courts. In this particular extradition Petitioner, who appeared in his own behalf, moved that he be given ample
request the United States Government requested the time to file a memorandum, but the same was denied.
Philippine Government to prevent unauthorized disclosure
of the subject information. This Departments denial of your On August 10, 1999, respondent judge issued an order dated the previous
request is consistent with Article 7 of the RP-US Extradition day, disposing:
Treaty which provides that the Philippine Government
must represent the interests of the United States in any
WHEREFORE, this Court hereby Orders the respondents,
proceedings arising out of a request for extradition. The
namely: the Secretary of Justice, the Secretary of Foreign
Department of Justice under P.D. No. 1069 is the counsel
Affairs and the Director of the National Bureau of
of the foreign governments in all extradition requests.
Investigation, their agents and/or representatives to
maintain the status quo by refraining from committing the
3. This Department is not in a position to hold in abeyance acts complained of; from conducting further proceedings in
proceedings in connection with an extradition request. connection with the request of the United States
Article 26 of the Vienna Convention on the Law of Treaties, Government for the extradition of the petitioner; from
to which we are a party provides that "[E]very treaty in filing the corresponding Petition with a Regional Trial
force is binding upon the parties to it and must be court; and from performing any act directed to the
performed by them in good faith". Extradition is a tool of extradition of the petitioner to the United States, for a
criminal law enforcement and to be effective, requests for period of twenty (20) days from service on respondents of
extradition or surrender of accused or convicted persons this Order, pursuant to Section 5, Rule 58 of the 1997
must be processed expeditiously. Rules of Court.

(pp. 77-78, Rollo.) The hearing as to whether or not this Court shall issue the
preliminary injunction, as agreed upon by the counsels for
the parties herein, is set on August 17, 1999 at 9:00 IV.
oclock in the morning. The respondents are, likewise,
ordered to file their written comment and/or opposition to PRIVATE RESPONDENT HAS NO RIGHT IN ESSE THAT
the issuance of a Preliminary Injunction on or before said NEEDS PROTECTION AND ENFORCEMENT, AND WILL NOT
date. SUFFER ANY IRREPARABLE INJURY.

SO ORDERED. (pp. 19-20, Rollo.)

(pp. 110-111, Rollo.) On August 17, 1999, the Court required private respondent to file his
comment. Also issued, as prayed for, was a temporary restraining order
Forthwith, petitioner initiated the instant proceedings, arguing that: (TRO) providing:

PUBLIC RESPONDENT ACTED WITHOUT OR IN EXCESS OF NOW, THEREFORE, effective immediately and continuing
JURISDICTION OR WITH GRAVE ABUSE OF DISCRETION until further orders from this Court, You, Respondent
AMOUNTING TO LACK OR EXCESS OF JURISDICTION IN Judge Ralph C. Lantion, your agents, representatives or
ISSUING THE TEMPORARY RESTRAINING ORDER any person or persons acting in your place or stead are
BECAUSE:  hereby ORDERED to CEASE and DESIST from enforcing the
assailed order dated August 9, 1999 issued by public
I. respondent in Civil Case No. 99-94684.

BY ORDERING HEREIN PETITIONER TO REFRAIN FROM GIVEN by the Honorable HILARIO G. DAVIDE, JR., Chief
COMMITTING THE ACTS COMPLAINED OF, I. E., TO Justice, Supreme Court of the Philippines, this 17th day of
DESIST FROM REFUSING PRIVATE RESPONDENT ACCESS August 1999.
TO THE OFFICIAL EXTRADITION REQUEST AND
DOCUMENTS AND FROM DENYING PRIVATE RESPONDENT (pp. 120-121, Rollo.)
AN OPPORTUNITY TO FILE A COMMENT ON, OR
OPPOSITION TO, THE REQUEST, THE MAIN PRAYER FOR A The case was heard on oral argument on August 31, 1999, after which the
WRIT OF MANDAMUS IN THE PETITION FOR MANDAMUS, parties, as directed, filed their respective memoranda.
CERTIORARI AND PROHIBITION WAS, IN EFFECT,
GRANTED SO AS TO CONSTITUTE AN ADJUDICATION ON
From the pleadings of the opposing parties, both procedural and
THE MERITS OF THE MANDAMUS ISSUES;
substantive issues are patent. However, a review of these issues as well as
the extensive arguments of both parties, compel us to delineate the focal
II. point raised by the pleadings: During the evaluation stage of the
extradition proceedings, is private respondent entitled to the two basic due
PETITIONER WAS UNQUALIFIEDLY PREVENTED FROM process rights of notice and hearing? An affirmative answer would
PERFORMING LEGAL DUTIES UNDER THE EXTRADITION necessarily render the proceedings at the trial court, moot and academic
TREATY AND THE PHILIPPINE EXTRADITION LAW; (the issues of which are substantially the same as those before us now),
while a negative resolution would call for the immediate lifting of the TRO
III. issued by this Court dated August 24, 1999, thus allowing petitioner to
fast-track the process leading to the filing of the extradition petition with
the proper regional trial court. Corollarily, in the event that private
THE PETITION FOR (MANDAMUS), CERTIORARI AND
respondent is adjudged entitled to basic due process rights at the
PROHIBITION IS, ON ITS FACE, FORMALLY AND
evaluation stage of the extradition proceedings, would this entitlement
SUBSTANTIALLY DEFICIENT; AND
constitute a breach of the legal commitments and obligations of the 3. The text of the applicable law or a statement of the contents of said
Philippine Government under the RP-US Extradition Treaty? And assuming law, and the designation or description of the offense by the law, sufficient
that the result would indeed be a breach, is there any conflict between for evaluation of the request; and
private respondents basic due process rights and the provisions of the RP-
US Extradition Treaty? 4. Such other documents or information in support of the request.

The issues having transcendental importance, the Court has elected to go (Section 4, Presidential Decree No. 1069.)
directly into the substantive merits of the case, brushing aside peripheral
procedural matters which concern the proceedings in Civil Case No. 99-
Section 5 of the Presidential Decree, which sets forth the duty of the
94684, particularly the propriety of the filing of the petition therein, and of
Secretary of Foreign Affairs, pertinently provides:
the issuance of the TRO of August 17, 1999 by the trial
court.
. . . (1) Unless it appears to the Secretary of Foreign
Affairs that the request fails to meet the requirements of
To be sure, the issues call for a review of the extradition procedure. The
this law and the relevant treaty or convention, he shall
RP-US Extradition Treaty which was executed only on November 13, 1994,
forward the request together with the related documents
ushered into force the implementing provisions of Presidential Decree No.
to the Secretary of Justice, who shall immediately
1069, also called as the Philippine Extradition Law. Section 2(a) thereof
designate and authorize an attorney in his office to take
defines extradition as "the removal of an accused from the Philippines with
charge of the case.
the object of placing him at the disposal of foreign authorities to enable
the requesting state or government to hold him in connection with any
criminal investigation directed against him or the execution of a penalty The above provision shows only too clearly that the executive authority
imposed on him under the penal or criminal law of the requesting state or given the task of evaluating the sufficiency of the request and the
government." The portions of the Decree relevant to the instant case supporting documents is the Secretary of Foreign Affairs. What then is the
which involves a charged and not convicted individual, are abstracted as coverage of this task?
follows:
In accordance with Paragraphs 2 and 3, Article 7 of the RP-US Extradition
The Extradition Request Treaty, the executive authority must ascertain whether or not the request
is supported by:
The request is made by the Foreign Diplomat of the Requesting State,
addressed to the Secretary of Foreign Affairs, and shall be accompanied 1. Documents, statements, or other types of information which describe
by: the identity and probable location of the person sought;

1. The original or an authentic copy of the criminal charge and the warrant 2. A statement of the facts of the offense and the procedural history of the
of arrest issued by the authority of the Requesting State having case;
jurisdiction over the matter, or some other instruments having equivalent
legal force; 3. A statement of the provisions of the law describing the essential
elements of the offense for which extradition is requested;
2. A recital of the acts for which extradition is requested, with the fullest
particulars as to the name and identity of the accused, his whereabouts in 4. A statement of the provisions of law describing the punishment for the
the Philippines, if known, the acts or omissions complained of, and the offense;
time and place of the commission of these acts;
5. A statement of the provisions of the law describing any time limit on the
prosecution or the execution of punishment for the offense;
6. Documents, statements, or other types of information specified in the prospective extraditee to appear and to answer the petition on the day
paragraph 3 or paragraph 4 of said Article, as applicable. and hour fixed in the order. The judge may issue a warrant of arrest if it
appears that the immediate arrest and temporary detention of the accused
(Paragraph 2, Article 7, Presidential Decree No. 1069.) will best serve the ends of justice (Paragraph [1], Section 6, ibid.),
particularly to prevent the flight of the prospective extraditee.
7. Such evidence as, according to the law of the Requested State, would
provide probable cause for his arrest and committal for trial if the offense The Extradition Hearing
had been committed there;
The Extradition Law does not specifically indicate whether the extradition
8. A copy of the warrant or order of arrest issued by a judge or other proceeding is criminal, civil, or a special proceeding. Nevertheless,
competent authority; and Paragraph [1], Section 9 thereof provides that in the hearing of the
extradition petition, the provisions of the Rules of Court, insofar as
practicable and not inconsistent with the summary nature of the
9. A copy of the charging document.
proceedings, shall apply. During the hearing, Section 8 of the Decree
provides that the attorney having charge of the case may, upon
(Paragraph 3, ibid.) application by the Requesting State, represent the latter throughout the
proceedings.
The executive authority (Secretary of Foreign Affairs) must also see to it
that the accompanying documents received in support of the request had Upon conclusion of the hearing, the court shall render a decision granting
been certified by the principal diplomatic or consular officer of the the extradition and giving the reasons therefor upon a showing of the
Requested State resident in the Requesting State (Embassy Note No. 052 existence of a prima facie case, or dismiss the petition (Section 10, ibid.).
from U. S. Embassy; Embassy Note No. 951309 from the Department of Said decision is appealable to the Court of Appeals, whose decision shall
Foreign Affairs). be final and immediately executory (Section 12, ibid.). The provisions of
the Rules of Court governing appeal in criminal cases in the Court of
In this light, Paragraph 3, Article 3 of the Treaty provides that Appeals shall apply in the aforementioned appeal, except for the required
"[e]xtradition shall not be granted if the executive authority of the 15-day period to file brief (Section 13, ibid.).
Requested State determines that the request is politically motivated, or
that the offense is a military offense which is not punishable under non- The trial court determines whether or not the offense mentioned in the
military penal legislation." petition is extraditable based on the application of the dual criminality rule
and other conditions mentioned in Article 2 of the RP-US Extradition
The Extradition Petition Treaty. The trial court also determines whether or not the offense for
which extradition is requested is a political one (Paragraph [1], Article 3,
Upon a finding made by the Secretary of Foreign Affairs that the RP-US Extradition Treaty).
extradition request and its supporting documents are sufficient and
complete in form and substance, he shall deliver the same to the With the foregoing abstract of the extradition proceedings as backdrop,
Secretary of Justice, who shall immediately designate and authorize an the following query presents itself: What is the nature of the role of the
attorney in his office to take charge of the case (Paragraph [1], Section 5, Department of Justice at the evaluation stage of the extradition
P. D. No. 1069). The lawyer designated shall then file a written petition proceedings?
with the proper regional trial court of the province or city, with a prayer
that the court take the extradition request under consideration (Paragraph A strict observance of the Extradition Law indicates that the only duty of
[2], ibid.). the Secretary of Justice is to file the extradition petition after the request
and all the supporting papers are forwarded to him by the Secretary of
The presiding judge of the regional trial court, upon receipt of the petition Foreign Affairs. It is the latter official who is authorized to evaluate the
for extradition, shall, as soon as practicable, issue an order summoning extradition papers, to assure their sufficiency, and under Paragraph [3],
Article 3 of the Treaty, to determine whether or not the request is requirements of law. The Secretary of Justice, eminent as he is in the field
politically motivated, or that the offense is a military offense which is not of law, could not privately review the papers all by himself. He had to
punishable under non-military penal legislation. Ipso facto, as expressly officially constitute a panel of attorneys. How then could the DFA Secretary
provided in Paragraph [1], Section 5 of the Extradition Law, the Secretary or his undersecretary, in less than one day, make the more authoritative
of Justice has the ministerial duty of filing the extradition papers. determination?

However, looking at the factual milieu of the case before us, it would The evaluation process, just like the extradition proceedings proper,
appear that there was failure to abide by the provisions of Presidential belongs to a class by itself. It is sui generis. It is not a criminal
Decree No. 1069. For while it is true that the extradition request was investigation, but it is also erroneous to say that it is purely an exercise of
delivered to the Department of Foreign Affairs on June 17, 1999, the ministerial functions. At such stage, the executive authority has the
following day or less than 24 hours later, the Department of Justice power: (a) to make a technical assessment of the completeness and
received the request, apparently without the Department of Foreign Affairs sufficiency of the extradition papers; (b) to outrightly deny the request if
discharging its duty of thoroughly evaluating the same and its on its face and on the face of the supporting documents the crimes
accompanying documents. The statement of an assistant secretary at the indicated are not extraditable; and (c) to make a determination whether or
Department of Foreign Affairs that his Department, in this regard, is not the request is politically motivated, or that the offense is a military one
merely acting as a post office, for which reason he simply forwarded the which is not punishable under non-military penal legislation (tsn, August
request to the Department of Justice, indicates the magnitude of the error 31, 1999, pp. 28-29; Article 2 & and Paragraph [3], Article 3, RP-US
of the Department of Foreign Affairs in taking lightly its responsibilities. Extradition Treaty). Hence, said process may be characterized as an
Thereafter, the Department of Justice took it upon itself to determine the investigative or inquisitorial process in contrast to a proceeding conducted
completeness of the documents and to evaluate the same to find out in the exercise of an administrative bodys quasi-judicial power.
whether they comply with the requirements laid down in the Extradition
Law and the RP-US Extradition Treaty. Petitioner ratiocinates in this In administrative law, a quasi-judicial proceeding involves: (a) taking and
connection that although the Department of Justice had no obligation to evaluation of evidence; (b) determining facts based upon the evidence
evaluate the extradition documents, the Department also had to go over presented; and (c) rendering an order or decision supported by the facts
them so as to be able to prepare an extradition petition (tsn, August 31, proved (De Leon, Administrative Law: Text and Cases, 1993 ed., p. 198,
1999, pp. 24-25). Notably, it was also at this stage where private citing Morgan vs. United States, 304 U.S. 1). Inquisitorial power, which is
respondent insisted on the following: (1) the right to be furnished the also known as examining or investigatory power, is one of the
request and the supporting papers; (2) the right to be heard which determinative powers of an administrative body which better enables it to
consists in having a reasonable period of time to oppose the request, and exercise its quasi-judicial authority (Cruz, Phil. Administrative Law, 1996
to present evidence in support of the opposition; and (3) that the ed., p. 26). This power allows the administrative body to inspect the
evaluation proceedings be held in abeyance pending the filing of private records and premises, and investigate the activities, of persons or entities
respondent's opposition to the request. coming under its jurisdiction (Ibid., p. 27), or to require disclosure of
information by means of accounts, records, reports, testimony of
The two Departments seem to have misread the scope of their duties and witnesses, production of documents, or otherwise (De Leon, op. cit., p.
authority, one abdicating its powers and the other enlarging its 64).
commission. The Department of Foreign Affairs, moreover, has, through
the Solicitor General, filed a manifestation that it is adopting the instant The power of investigation consists in gathering, organizing, and analyzing
petition as its own, indirectly conveying the message that if it were to evidence, which is a useful aid or tool in an administrative agencys
evaluate the extradition request, it would not allow private respondent to performance of its rule-making or quasi-judicial functions. Notably,
participate in the process of evaluation. investigation is indispensable to prosecution.

Plainly then, the record cannot support the presumption of regularity that In Ruperto v. Torres (100 Phil. 1098 [1957], unreported), the Court had
the Department of Foreign Affairs thoroughly reviewed the extradition occasion to rule on the functions of an investigatory body with the sole
request and supporting documents and that it arrived at a well-founded power of investigation. It does not exercise judicial functions and its power
judgment that the request and its annexed documents satisfy the
is limited to investigating the facts and making findings in respect thereto. Clearly, there is an impending threat to a prospective extraditees liberty as
The Court laid down the test of determining whether an administrative early as during the evaluation stage. It is not only an imagined threat to
body is exercising judicial functions or merely investigatory functions: his liberty, but a very imminent one.
Adjudication signifies the exercise of power and authority to adjudicate
upon the rights and obligations of the parties before it. Hence, if the only Because of these possible consequences, we conclude that the evaluation
purpose for investigation is to evaluate evidence submitted before it based process is akin to an administrative agency conducting an investigative
on the facts and circumstances presented to it, and if the agency is not proceeding, the consequences of which are essentially criminal since such
authorized to make a final pronouncement affecting the parties, then there technical assessment sets off or commences the procedure for, and
is an absence of judicial discretion and judgment. ultimately, the deprivation of liberty of a prospective extraditee. As
described by petitioner himself, this is a "tool" for criminal law
The above description in Ruperto applies to an administrative body enforcement (p. 78, Rollo). In essence, therefore, the evaluation process
authorized to evaluate extradition documents. The body has no power to partakes of the nature of a criminal investigation. In a number of cases,
adjudicate in regard to the rights and obligations of both the Requesting we had occasion to make available to a respondent in an administrative
State and the prospective extraditee. Its only power is to determine case or investigation certain constitutional rights that are ordinarily
whether the papers comply with the requirements of the law and the available only in criminal prosecutions. Further, as pointed out by Mr.
treaty and, therefore, sufficient to be the basis of an extradition petition. Justice Mendoza during the oral arguments, there are rights formerly
Such finding is thus merely initial and not final. The body has no power to available only at the trial stage that had been advanced to an earlier stage
determine whether or not the extradition should be effected. That is the in the proceedings, such as the right to counsel and the right against self-
role of the court. The bodys power is limited to an initial finding of whether incrimination (tsn, August 31, 1999, p. 135; Escobedo vs. Illinois, 378
or not the extradition petition can be filed in court. U.S. 478; Gideon vs. Wainwright, 372 U.S. 335; Miranda vs. Arizona, 384
U.S. 436).
It is to be noted, however, that in contrast to ordinary investigations, the
evaluation procedure is characterized by certain peculiarities. Primarily, it In Pascual v. Board of Medical Examiners (28 SCRA 344 [1969]), we held
sets into motion the wheels of the extradition process. Ultimately, it may that the right against self-incrimination under Section 17, Article III of the
result in the deprivation of liberty of the prospective extraditee. This 1987 Constitution which is ordinarily available only in criminal
deprivation can be effected at two stages: First, the provisional arrest of prosecutions, extends to administrative proceedings which possess a
the prospective extraditee pending the submission of the request. This is criminal or penal aspect, such as an administrative investigation of a
so because the Treaty provides that in case of urgency, a contracting party licensed physician who is charged with immorality, which could result in
may request the provisional arrest of the person sought pending his loss of the privilege to practice medicine if found guilty. The Court,
presentation of the request (Paragraph [1], Article 9, RP-US Extradition citing the earlier case of Cabal vs. Kapunan (6 SCRA 1059 [1962]),
Treaty), but he shall be automatically discharged after 60 days if no pointed out that the revocation of ones license as a medical practitioner, is
request is submitted (Paragraph 4). Presidential Decree No. 1069 provides an even greater deprivation than forfeiture of property.
for a shorter period of 20 days after which the arrested person could be
discharged (Section 20[d]). Logically, although the Extradition Law is Cabal vs. Kapunan (supra) involved an administrative charge of
silent on this respect, the provisions only mean that once a request is unexplained wealth against a respondent which was filed under Republic
forwarded to the Requested State, the prospective extraditee may be Act No. 1379, or the Anti-Graft Law. Again, we therein ruled that since the
continuously detained, or if not, subsequently rearrested (Paragraph [5], investigation may result in forfeiture of property, the administrative
Article 9, RP-US Extradition Treaty), for he will only be discharged if no proceedings are deemed criminal or penal, and such forfeiture partakes
request is submitted. Practically, the purpose of this detention is to the nature of a penalty. There is also the earlier case of Almeda, Sr. vs.
prevent his possible flight from the Requested State. Second, the Perez (5 SCRA 970 [1962]), where the Court, citing American
temporary arrest of the prospective extraditee during the pendency of the jurisprudence, laid down the test to determine whether a proceeding is
extradition petition in court (Section 6, Presidential Decree No. 1069). civil or criminal: If the proceeding is under a statute such that if an
indictment is presented the forfeiture can be included in the criminal case,
such proceeding is criminal in nature, although it may be civil in form; and
where it must be gathered from the statute that the action is meant to be
criminal in its nature, it cannot be considered as civil. If, however, the The due process clauses in the American and Philippine Constitutions are
proceeding does not involve the conviction of the wrongdoer for the not only worded in exactly identical language and terminology, but more
offense charged, the proceeding is civil in nature. importantly, they are alike in what their respective Supreme Courts have
expounded as the spirit with which the provisions are informed and
The cases mentioned above refer to an impending threat of deprivation of impressed, the elasticity in their interpretation, their dynamic and resilient
ones property or property right. No less is this true, but even more so in character which make them capable of meeting every modern problem,
the case before us, involving as it does the possible deprivation of liberty, and their having been designed from earliest time to the present to meet
which, based on the hierarchy of constitutionally protected rights, is placed the exigencies of an undefined and expanding future. The requirements of
second only to life itself and enjoys precedence over property, for while due process are interpreted in both the United States and the Philippines
forfeited property can be returned or replaced, the time spent in as not denying to the law the capacity for progress and improvement.
incarceration is irretrievable and beyond recompense. Toward this effect and in order to avoid the confines of a legal straitjacket,
the courts instead prefer to have the meaning of the due process clause
"gradually ascertained by the process of inclusion and exclusion in the
By comparison, a favorable action in an extradition request exposes a
course of the decisions of cases as they arise" (Twining vs. New Jersey,
person to eventual extradition to a foreign country, thus saliently
211 U.S. 78). Capsulized, it refers to "the embodiment of the sporting idea
exhibiting the criminal or penal aspect of the process. In this sense, the
of fair play" (Ermita-Malate Hotel and Motel Owners Association vs. City
evaluation procedure is akin to a preliminary investigation since both
Mayor of Manila, 20 SCRA 849 [1967]). It relates to certain immutable
procedures may have the same result the arrest and imprisonment of the
principles of justice which inhere in the very idea of free government
respondent or the person charged. Similar to the evaluation stage of
(Holden vs. Hardy, 169 U.S. 366).
extradition proceedings, a preliminary investigation, which may result in
the filing of an information against the respondent, can possibly lead to his
arrest, and to the deprivation of his liberty. Due process is comprised of two components substantive due process
which requires the intrinsic validity of the law in interfering with the rights
of the person to his life, liberty, or property, and procedural due process
Petitioners reliance on Wright vs. Court of Appeals (235 SCRA 241 [1992])
which consists of the two basic rights of notice and hearing, as well as the
(p. 8, Petitioners Memorandum) that the extradition treaty is neither a
guarantee of being heard by an impartial and competent tribunal (Cruz,
piece of criminal legislation nor a criminal procedural statute is not well-
Constitutional Law, 1993 Ed., pp. 102-106).
taken. Wright is not authority for petitioners conclusion that his
preliminary processing is not akin to a preliminary investigation. The
characterization of a treaty in Wright was in reference to the applicability True to the mandate of the due process clause, the basic rights of notice
of the prohibition against an ex post facto law. It had nothing to do with and hearing pervade not only in criminal and civil proceedings, but in
the denial of the right to notice, information, and hearing. administrative proceedings as well. Non-observance of these rights will
invalidate the proceedings. Individuals are entitled to be notified of any
pending case affecting their interests, and upon notice, they may claim the
As early as 1884, the United States Supreme Court ruled that "any legal
right to appear therein and present their side and to refute the position of
proceeding enforced by public authority, whether sanctioned by age or
the opposing parties (Cruz, Phil. Administrative Law, 1996 ed., p. 64).
custom, or newly devised in the discretion of the legislative power, in
furtherance of the general public good, which regards and preserves these
principles of liberty and justice, must be held to be due process of law" In a preliminary investigation which is an administrative investigatory
(Hurtado vs. California, 110 U.S. 516). Compliance with due process proceeding, Section 3, Rule 112 of the Rules of Court guarantees the
requirements cannot be deemed non-compliance with treaty respondents basic due process rights, granting him the right to be
commitments. furnished a copy of the complaint, the affidavits, and other supporting
documents, and the right to submit counter-affidavits and other supporting
documents within ten days from receipt thereof. Moreover, the respondent
The United States and the Philippines share a mutual concern about the
shall have the right to examine all other evidence submitted by the
suppression and punishment of crime in their respective jurisdictions. At
complainant.
the same time, both States accord common due process protection to their
respective citizens.
These twin rights may, however, be considered dispensable in certain the demanding state before some court or magistrate (35 C.J.S. 406-407).
instances, such as: The extradition documents are then filed with the governor of the asylum
state, and must contain such papers and documents prescribed by statute,
1. In proceedings where there is an urgent need for immediate action, like which essentially include a copy of the instrument charging the person
the summary abatement of a nuisance per se (Article 704, Civil Code), the demanded with a crime, such as an indictment or an affidavit made before
preventive suspension of a public servant facing administrative charges a magistrate. Statutory requirements with respect to said charging
(Section 63, Local Government Code, B. P. Blg. 337), the padlocking of instrument or papers are mandatory since said papers are necessary in
filthy restaurants or theaters showing obscene movies or like order to confer jurisdiction on the governor of the asylum state to effect
establishments which are immediate threats to public health and decency, the extradition (35 C.J.S. 408-410). A statutory provision requiring
and the cancellation of a passport of a person sought for criminal duplicate copies of the indictment, information, affidavit, or
prosecution; judgment of conviction or sentence and other instruments
accompanying the demand or requisitions be furnished and
delivered to the fugitive or his attorney is directory. However, the
2. Where there is tentativeness of administrative action, that is, where the
right being such a basic one has been held to be a right mandatory
respondent is not precluded from enjoying the right to notice and hearing
on demand (Ibid., p. 410, citing Ex parte Moore, 256 S.W. 2d 103, 158
at a later time without prejudice to the person affected, such as the
Tex. Cr. 407 and Ex parte Tucker, Cr., 324, S.W.2d
summary distraint and levy of the property of a delinquent taxpayer, and
853).
the replacement of a temporary appointee; and

In international proceedings, extradition treaties generally provide for the


3. Where the twin rights have previously been offered but the right to
presentation to the executive authority of the Requested State of a
exercise them had not been claimed.
requisition or demand for the return of the alleged offender, and the
designation of the particular officer having authority to act in behalf of the
Applying the above principles to the case at bar, the query may be asked: demanding nation (31A Am Jur 2d 815).
Does the evaluation stage of the extradition proceedings fall under any of
the described situations mentioned above?
In petitioners memorandum filed on September 15, 1999, he attached
thereto a letter dated September 13, 1999 from the Criminal Division of
Let us take a brief look at the nature of American extradition proceedings the U.S. Department of Justice, summarizing the U.S. extradition
which are quite noteworthy considering that the subject treaty involves the procedures and principles, which are basically governed by a combination
U.S. Government. of treaties (with special reference to the RP-US Extradition Treaty), federal
statutes, and judicial decisions, to wit:
American jurisprudence distinguishes between interstate rendition or
extradition which is based on the Extradition Clause in the U.S. 1. All requests for extradition are transmitted through the diplomatic
Constitution (Art. IV, 2 cl 2), and international extradition proceedings. In channel. In urgent cases, requests for the provisional arrest of an
interstate rendition or extradition, the governor of the asylum state has individual may be made directly by the Philippine Department of Justice to
the duty to deliver the fugitive to the demanding state. The Extradition the U.S. Department of Justice, and vice-versa. In the event of a
Clause and the implementing statute are given a liberal construction to provisional arrest, a formal request for extradition is transmitted
carry out their manifest purpose, which is to effect the return as swiftly as subsequently through the diplomatic channel.
possible of persons for trial to the state in which they have been charged
with crime (31A Am Jur 2d 754-755). In order to achieve extradition of an
2. The Department of State forwards the incoming Philippine extradition
alleged fugitive, the requisition papers or the demand must be in proper
request to the Department of Justice. Before doing so, the Department of
form, and all the elements or jurisdictional facts essential to the
State prepares a declaration confirming that a formal request has been
extradition must appear on the face of the papers, such as the allegation
made, that the treaty is in full force and effect, that under Article 17
that the person demanded was in the demanding state at the time the
thereof the parties provide reciprocal legal representation in extradition
offense charged was committed, and that the person demanded is charged
proceedings, that the offenses are covered as extraditable offenses under
with the commission of the crime or that prosecution has been begun in
Article 2 thereof, and that the documents have been authenticated in the beginning, and, in the person of the Secretary of State, the power to
accordance with the federal statute that ensures admissibility at any act or not to act on the courts determination of extraditability. In the
subsequent extradition hearing. Philippine setting, it is the Department of Foreign Affairs which should
make the initial evaluation of the request, and having satisfied itself on the
3. A judge or magistrate judge is authorized to issue a warrant for the points earlier mentioned (see pp. 10-12), then forwards the request to the
arrest of the prospective extraditee (18 U.S.C. 3184). Said judge or Department of Justice for the preparation and filing of the petition for
magistrate is authorized to hold a hearing to consider the evidence offered extradition. Sadly, however, the Department of Foreign Affairs, in the
in support of the extradition request (Ibid.) instant case, perfunctorily turned over the request to the Department of
Justice which has taken over the task of evaluating the request as well as
thereafter, if so warranted, preparing, filing, and prosecuting the petition
4. At the hearing, the court must determine whether the person arrested
for extradition.
is extraditable to the foreign country. The court must also determine that
(a) it has jurisdiction over the defendant and jurisdiction to conduct the
hearing; (b) the defendant is being sought for offenses for which the Private respondent asks what prejudice will be caused to the U.S.
applicable treaty permits extradition; and (c) there is probable cause to Government should the person sought to be extradited be given due
believe that the defendant is the person sought and that he committed the process rights by the Philippines in the evaluation stage. He emphasizes
offenses charged (Ibid.) that petitioners primary concern is the possible delay in the evaluation
process.
5. The judge or magistrate judge is vested with jurisdiction to certify
extraditability after having received a "complaint made under oath, We agree with private respondents citation of an American Supreme Court
charging any person found within his jurisdiction" with having committed ruling:
any of the crimes provided for by the governing treaty in the country
requesting extradition (Ibid.) [In this regard, it is noted that a long line of The establishment of prompt efficacious procedures to
American decisions pronounce that international extradition proceedings achieve legitimate state ends is a proper state interest
partake of the character of a preliminary examination before a committing worthy of cognizance in constitutional adjudication. But the
magistrate, rather than a trial of the guilt or innocence of the alleged Constitution recognizes higher values than speed and
fugitive (31A Am Jur 2d 826).] efficiency. Indeed, one might fairly say of the Bill of Rights
in general, and the Due Process Clause, in particular, that
6. If the court decides that the elements necessary for extradition are they were designed to protect the fragile values of a
present, it incorporates its determinations in factual findings and vulnerable citizenry from the overbearing concern for
conclusions of law and certifies the persons extraditability. The court then efficiency and efficacy that may characterize praiseworthy
forwards this certification of extraditability to the Department of State for government officials no less, and perhaps more, than
disposition by the Secretary of State. The ultimate decision whether to mediocre ones.
surrender an individual rests with the Secretary of State (18 U.S.C. 3186).
(Stanley vs. Illinois, 404 U.S. 645, 656)
7. The subject of an extradition request may not litigate questions
concerning the motives of the requesting government in seeking his The United States, no doubt, shares the same interest as
extradition. However, a person facing extradition may present whatever the Philippine Government that no right that of liberty
information he deems relevant to the Secretary of State, who makes the secured not only by the Bills of Rights of the Philippines
final determination whether to surrender an individual to the foreign Constitution but of the United States as well, is sacrificed
government concerned. at the altar of expediency.

From the foregoing, it may be observed that in the United States, (pp. 40-41, Private Respondents
extradition begins and ends with one entity the Department of State which Memorandum.)
has the power to evaluate the request and the extradition documents in
In the Philippine context, this Courts ruling is invoked: In this light, it should be observed that the Department of Justice exerted
notable efforts in assuring compliance with the requirements of the law
One of the basic principles of the democratic system is that and the treaty since it even informed the U.S. Government of certain
where the rights of the individual are concerned, the end problems in the extradition papers (such as those that are in Spanish and
does not justify the means. It is not enough that there be without the official English translation, and those that are not properly
a valid objective; it is also necessary that the means authenticated). In fact, petitioner even admits that consultation meetings
employed to pursue it be in keeping with the Constitution. are still supposed to take place between the lawyers in his Department
Mere expediency will not excuse constitutional shortcuts. and those from the U.S. Justice Department. With the meticulous nature of
There is no question that not even the strongest moral the evaluation, which cannot just be completed in an abbreviated period of
conviction or the most urgent public need, subject only to time due to its intricacies, how then can we say that it is a proceeding that
a few notable exceptions, will excuse the bypassing of an urgently necessitates immediate and prompt action where notice and
individuals rights. It is no exaggeration to say that a hearing can be dispensed with?
person invoking a right guaranteed under Article III of the
Constitution is a majority of one even as against the rest Worthy of inquiry is the issue of whether or not there is tentativeness of
of the nation who would deny him that right (Association administrative action. Is private respondent precluded from enjoying the
of Small Landowners in the Philippines, Inc. vs. Secretary right to notice and hearing at a later time without prejudice to him? Here
of Agrarian Reform, 175 SCRA 343, 375-376 [1989]). lies the peculiarity and deviant characteristic of the evaluation procedure.
On one hand, there is yet no extraditee, but ironically on the other, it
There can be no dispute over petitioners argument that extradition is a results in an administrative determination which, if adverse to the person
tool of criminal law enforcement. To be effective, requests for extradition involved, may cause his immediate incarceration. The grant of the request
or the surrender of accused or convicted persons must be processed shall lead to the filing of the extradition petition in court. The "accused"
expeditiously. Nevertheless, accelerated or fast-tracked proceedings and (as Section 2[c] of Presidential Decree No. 1069 calls him), faces the
adherence to fair procedures are, however, not always incompatible. They threat of arrest, not only after the extradition petition is filed in court, but
do not always clash in discord. Summary does not mean precipitous haste. even during the evaluation proceeding itself by virtue of the provisional
It does not carry a disregard of the basic principles inherent in "ordered arrest allowed under the treaty and the implementing law. The prejudice
liberty." to the "accused" is thus blatant and manifest.

Is there really an urgent need for immediate action at the evaluation Plainly, the notice and hearing requirements of administrative due process
stage? At that point, there is no extraditee yet in the strict sense of the cannot be dispensed with and shelved aside.
word. Extradition may or may not occur. In interstate extradition, the
governor of the asylum state may not, in the absence of mandatory Apart from the due process clause of the Constitution, private respondent
statute, be compelled to act favorably (37 C.J.S. 387) since after a close likewise invokes Section 7 of Article III which reads:
evaluation of the extradition papers, he may hold that federal and
statutory requirements, which are significantly jurisdictional, have not Sec. 7. The right of the people to information on matters
been met (31 Am Jur 2d 819). Similarly, under an extradition treaty, the of public concern shall be recognized. Access to official
executive authority of the requested state has the power to deny the records, and to documents and papers pertaining to official
behest from the requesting state. Accordingly, if after a careful acts, transactions, or decisions, as well as to government
examination of the extradition documents the Secretary of Foreign Affairs research data used as basis for policy development, shall
finds that the request fails to meet the requirements of the law and the be afforded the citizen, subject to such limitations as may
treaty, he shall not forward the request to the Department of Justice for be provided by law.
the filing of the extradition petition since non-compliance with the
aforesaid requirements will not vest our government with jurisdiction to
The above provision guarantees political rights which are available to
effect the extradition.
citizens of the Philippines, namely: (1) the right to information on matters
of public concern, and (2) the corollary right of access to official records
and documents. The general right guaranteed by said provision is the right In the case at bar, the papers requested by private respondent pertain to
to information on matters of public concern. In its implementation, the official government action from the U. S. Government. No official action
right of access to official records is likewise conferred. These cognate or from our country has yet been taken. Moreover, the papers have some
related rights are "subject to limitations as may be provided by law" relation to matters of foreign relations with the U. S. Government.
(Bernas, The 1987 Phil. Constitution A Reviewer-Primer, 1997 ed., p. 104) Consequently, if a third party invokes this constitutional provision, stating
and rely on the premise that ultimately it is an informed and critical public that the extradition papers are matters of public concern since they may
opinion which alone can protect the values of democratic government result in the extradition of a Filipino, we are afraid that the balance must
(Ibid.). be tilted, at such particular time, in favor of the interests necessary for the
proper functioning of the government. During the evaluation procedure, no
Petitioner argues that the matters covered by private respondents letter- official governmental action of our own government has as yet been done;
request dated July 1, 1999 do not fall under the guarantee of the hence the invocation of the right is premature. Later, and in contrast,
foregoing provision since the matters contained in the documents records of the extradition hearing would already fall under matters of
requested are not of public concern. On the other hand, private public concern, because our government by then shall have already made
respondent argues that the distinction between matters vested with public an official decision to grant the extradition request. The extradition of a
interest and matters which are of purely private interest only becomes fellow Filipino would be forthcoming.
material when a third person, who is not directly affected by the matters
requested, invokes the right to information. However, if the person We now pass upon the final issue pertinent to the subject matter of the
invoking the right is the one directly affected thereby, his right to instant controversy: Would private respondents entitlement to notice and
information becomes absolute. hearing during the evaluation stage of the proceedings constitute a breach
of the legal duties of the Philippine Government under the RP-Extradition
The concept of matters of public concern escapes exact definition. Strictly Treaty? Assuming the answer is in the affirmative, is there really a conflict
speaking, every act of a public officer in the conduct of the governmental between the treaty and the due process clause in the Constitution?
process is a matter of public concern (Bernas, The 1987 Constitution of the
Republic of the Philippines, 1996 ed., p. 336). This concept embraces a First and foremost, let us categorically say that this is not the proper time
broad spectrum of subjects which the public may want to know, either to pass upon the constitutionality of the provisions of the RP-US
because these directly affect their lives or simply because such matters Extradition Treaty nor the Extradition Law implementing the same. We
arouse the interest of an ordinary citizen (Legaspi v. Civil Service limit ourselves only to the effect of the grant of the basic rights of notice
Commission, 150 SCRA 530 [1987]). Hence, the real party in interest is and hearing to private respondent on foreign relations.
the people and any citizen has "standing".
The rule of pacta sunt servanda, one of the oldest and most fundamental
When the individual himself is involved in official government action maxims of international law, requires the parties to a treaty to keep their
because said action has a direct bearing on his life, and may either cause agreement therein in good faith. The observance of our country's legal
him some kind of deprivation or injury, he actually invokes the basic right duties under a treaty is also compelled by Section 2, Article II of the
to be notified under Section 1 of the Bill of Rights and not exactly the right Constitution which provides that "[t]he Philippines renounces war as an
to information on matters of public concern. As to an accused in a criminal instrument of national policy, adopts the generally accepted principles of
proceeding, he invokes Section 14, particularly the right to be informed of international law as part of the law of the land, and adheres to the policy
the nature and cause of the accusation against him. of peace, equality, justice, freedom, cooperation and amity with all
nations." Under the doctrine of incorporation, rules of international law
The right to information is implemented by the right of access to form part of the law of the land and no further legislative action is needed
information within the control of the government (Bernas, The 1987 to make such rules applicable in the domestic sphere (Salonga & Yap,
Constitution of the Republic of the Philippines, 1996 ed., p. 337). Such Public International Law, 1992 ed., p. 12).
information may be contained in official records, and in documents and
papers pertaining to official acts, transactions, or decisions. The doctrine of incorporation is applied whenever municipal tribunals (or
local courts) are confronted with situations in which there appears to be a
conflict between a rule of international law and the provisions of the In the absence of a law or principle of law, we must apply the rules of fair
constitution or statute of the local state. Efforts should first be exerted to play. An application of the basic twin due process rights of notice and
harmonize them, so as to give effect to both since it is to be presumed hearing will not go against the treaty or the implementing law. Neither the
that municipal law was enacted with proper regard for the generally Treaty nor the Extradition Law precludes these rights from a prospective
accepted principles of international law in observance of the Incorporation extraditee. Similarly, American jurisprudence and procedures on
Clause in the above-cited constitutional provision (Cruz, Philippine Political extradition pose no proscription. In fact, in interstate extradition
Law, 1996 ed., p. 55). In a situation, however, where the conflict is proceedings as explained above, the prospective extraditee may even
irreconcilable and a choice has to be made between a rule of international request for copies of the extradition documents from the governor of the
law and municipal law, jurisprudence dictates that municipal law should be asylum state, and if he does, his right to be supplied the same becomes a
upheld by the municipal courts (Ichong vs. Hernandez, 101 Phil. 1155 demandable right (35 C.J.S. 410).
[1957]; Gonzales vs. Hechanova, 9 SCRA 230 [1963]; In re: Garcia, 2
SCRA 984 [1961]) for the reason that such courts are organs of municipal Petitioner contends that the United States requested the Philippine
law and are accordingly bound by it in all circumstances (Salonga & Yap, Government to prevent unauthorized disclosure of confidential information.
op. cit., p. 13). The fact that international law has been made part of the Hence, the secrecy surrounding the action of the Department of Justice
law of the land does not pertain to or imply the primacy of international Panel of Attorneys. The confidentiality argument is, however, overturned
law over national or municipal law in the municipal sphere. The doctrine of by petitioners revelation that everything it refuses to make available at
incorporation, as applied in most countries, decrees that rules of this stage would be obtainable during trial. The Department of Justice
international law are given equal standing with, but are not superior to, states that the U.S. District Court concerned has authorized the disclosure
national legislative enactments. Accordingly, the principle lex posterior of certain grand jury information. If the information is truly confidential,
derogat priori takes effect a treaty may repeal a statute and a statute may the veil of secrecy cannot be lifted at any stage of the extradition
repeal a treaty. In states where the constitution is the highest law of the proceedings. Not even during trial.
land, such as the Republic of the Philippines, both statutes and treaties
may be invalidated if they are in conflict with the constitution
A libertarian approach is thus called for under the premises.
(Ibid.).

One will search in vain the RP-US Extradition Treaty, the Extradition Law,
In the case at bar, is there really a conflict between international law and
as well as American jurisprudence and procedures on extradition, for any
municipal or national law? En contrario, these two components of the law
prohibition against the conferment of the two basic due process rights of
of the land are not pitted against each other. There is no occasion to
notice and hearing during the evaluation stage of the extradition
choose which of the two should be upheld. Instead, we see a void in the
proceedings. We have to consider similar situations in jurisprudence for an
provisions of the RP-US Extradition Treaty, as implemented by Presidential
application by analogy.
Decree No. 1069, as regards the basic due process rights of a prospective
extraditee at the evaluation stage of extradition proceedings. From the
procedures earlier abstracted, after the filing of the extradition petition Earlier, we stated that there are similarities between the evaluation
and during the judicial determination of the propriety of extradition, the process and a preliminary investigation since both procedures may result
rights of notice and hearing are clearly granted to the prospective in the arrest of the respondent or the prospective extraditee. In the
extraditee. However, prior thereto, the law is silent as to these rights. evaluation process, a provisional arrest is even allowed by the Treaty and
Reference to the U.S. extradition procedures also manifests this silence. the Extradition Law (Article 9, RP-US Extradition Treaty; Sec. 20,
Presidential Decree No. 1069). Following petitioners theory, because there
is no provision of its availability, does this imply that for a period of time,
Petitioner interprets this silence as unavailability of these rights.
the privilege of the writ of habeas corpus is suspended, despite Section 15,
Consequently, he describes the evaluation procedure as an "ex
Article III of the Constitution which states that "[t]he privilege of the writ
parte technical assessment" of the sufficiency of the extradition request
of habeas corpus shall not be suspended except in cases of invasion or
and the supporting documents.
rebellion when the public safety requires it"? Petitioners theory would also
infer that bail is not available during the arrest of the prospective
We disagree. extraditee when the extradition petition has already been filed in court
since Presidential Decree No. 1069 does not provide therefor, ... [I]t is clear to us that what the opening sentence of
notwithstanding Section 13, Article III of the Constitution which provides Section 40 is saying is that an employee may be removed
that "[a]ll persons, except those charged with offenses punishable or dismissed even without formal investigation, in certain
by reclusion perpetua when evidence of guilt is strong, shall, before instances. It is equally clear to us that an employee must
conviction, be bailable by sufficient sureties, or be released on be informed of the charges preferred against him, and that
recognizance as may be provided by law. The right to bail shall not be the normal way by which the employee is so informed is
impaired even when the privilege of the writ of habeas corpus is by furnishing him with a copy of the charges against him.
suspended " Can petitioner validly argue that since these contraventions This is a basic procedural requirement that a statute
are by virtue of a treaty and hence affecting foreign relations, the cannot dispense with and still remain consistent with the
aforestated guarantees in the Bill of Rights could thus be subservient constitutional provision on due process. The second
thereto? minimum requirement is that the employee charged with
some misfeasance or malfeasance must have a reasonable
The basic principles of administrative law instruct us that "the essence of opportunity to present his side of the matter, that is to
due process in administrative proceedings is an opportunity to explain say, his defenses against the charges levelled against him
ones side or an opportunity to seek reconsideration of the actions or ruling and to present evidence in support of his
complained of (Mirano vs. NLRC, 270 SCRA 96 [1997]; Padilla vs. NLRC, defenses. 
273 SCRA 457 [1997]; PLDT vs. NLRC, 276 SCRA 1 [1997]; Helpmate,
Inc. vs. NLRC, 276 SCRA 315 [1997]; Aquinas School vs. Magnaye, 278 (at p. 671)
SCRA 602 [1997]; Jamer vs. NLRC, 278 SCRA 632 [1997]). In essence,
procedural due process refers to the method or manner by which the law Said summary dismissal proceedings are also non-litigious in nature, yet
is enforced (Corona vs. United Harbor Pilots Association of the Phils., 283 we upheld the due process rights of the respondent.
SCRA 31 [1997]). This Court will not tolerate the least disregard of
constitutional guarantees in the enforcement of a law or treaty. Petitioners
In the case at bar, private respondent does not only face a clear and
fears that the Requesting State may have valid objections to the
present danger of loss of property or employment, but of liberty itself,
Requested States non-performance of its commitments under the
which may eventually lead to his forcible banishment to a foreign land. The
Extradition Treaty are insubstantial and should not be given paramount
convergence of petitioners favorable action on the extradition request and
consideration.
the deprivation of private respondents liberty is easily comprehensible.

How then do we implement the RP-US Extradition Treaty? Do we limit


We have ruled time and again that this Courts equity jurisdiction, which is
ourselves to the four corners of Presidential Decree No. 1069?
aptly described as "justice outside legality," may be availed of only in the
absence of, and never against, statutory law or judicial pronouncements
Of analogous application are the rulings in Government Service Insurance (Smith Bell & Co., Inc. vs. Court of Appeals, 267 SCRA 530 [1997]; David-
System vs. Court of Appeals (201 SCRA 661 [1991]) and Go vs. National Chan vs. Court of Appeals, 268 SCRA 677 [1997]). The constitutional issue
Police Commission (271 SCRA 447 [1997]) where we ruled that in in the case at bar does not even call for "justice outside legality," since
summary proceedings under Presidential Decree No. 807 (Providing for the private respondents due process rights, although not guaranteed by
Organization of the Civil Service Commission in Accordance with Provisions statute or by treaty, are protected by constitutional guarantees. We would
of the Constitution, Prescribing its Powers and Functions and for Other not be true to the organic law of the land if we choose strict construction
Purposes), and Presidential Decree No. 971 (Providing Legal Assistance for over guarantees against the deprivation of liberty. That would not be in
Members of the Integrated National Police who may be charged for keeping with the principles of democracy on which our Constitution is
Service-Connected Offenses and Improving the Disciplinary System in the premised.
Integrated National Police, Appropriating Funds Therefor and for other
purposes), as amended by Presidential Decree No. 1707, although
Verily, as one traverses treacherous waters of conflicting and opposing
summary dismissals may be effected without the necessity of a formal
currents of liberty and government authority, he must ever hold the oar of
investigation, the minimum requirements of due process still operate. As
freedom in the stronger arm, lest an errant and wayward course be laid.
held in GSIS vs. Court of Appeals:
WHEREFORE, in view of the foregoing premises, the instant petition is
hereby DISMISSED for lack of merit. Petitioner is ordered to furnish
private respondent copies of the extradition request and its supporting
papers, and to grant him a reasonable period within which to file his
comment with supporting evidence. The incidents in Civil Case No. 99-
94684 having been rendered moot and academic by this decision, the
same is hereby ordered dismissed.

SO ORDERED. 6/2/00 2:12 PM


Republic of the Philippines One unavoidable consequence of everyone having the freedom to choose is that
Supreme Court others may make different choices choices we would not make for ourselves,
Baguio City choices we may disapprove of, even choices that may shock or offend or anger
us. However, choices are not to be legally prohibited merely because they are
EN BANC different, and the right to disagree and debate about important questions of
public policy is a core value protected by our Bill of Rights. Indeed, our
ANG LADLAD LGBT PARTY G.R. No. 190582 democracy is built on genuine recognition of, and respect for, diversity and
represented herein by its Chair, difference in opinion.
DANTON REMOTO, Since ancient times, society has grappled with deep disagreements about
Petitioner, Present: the definitions and demands of morality. In many cases, where moral convictions
are concerned, harmony among those theoretically opposed is an insurmountable
PUNO, C. J., goal. Yet herein lies the paradox philosophical justifications about what is moral
CARPIO, are indispensable and yet at the same time powerless to create agreement.This
CORONA, Court recognizes, however, that practical solutions are preferable to ideological
CARPIO MORALES, stalemates; accommodation is better than intransigence; reason more worthy
VELASCO, JR., than rhetoric. This will allow persons of diverse viewpoints to live together, if not
NACHURA, harmoniously, then, at least, civilly.
LEONARDO-DE CASTRO,
- versus - BRION, Factual Background
PERALTA,
BERSAMIN, This is a Petition for Certiorari under Rule 65 of the Rules of Court, with
DEL CASTILLO, an application for a writ of preliminary mandatory injunction, filed by Ang
ABAD, Ladlad LGBT Party (Ang Ladlad) against the Resolutions of the Commission on
VILLARAMA, JR., Elections (COMELEC) dated November 11, 2009[2] (the First Assailed Resolution)
PEREZ, and and December 16, 2009[3] (the Second Assailed Resolution) in SPP No. 09-228
MENDOZA, JJ. (PL) (collectively, the Assailed Resolutions). The case has its roots in the
COMELECs refusal to accredit Ang Ladlad as a party-list organization under
COMMISSION ON ELECTIONS, Promulgated: Republic Act (RA) No. 7941, otherwise known as the Party-List System Act.[4]
Respondent. April 8, 2010
x---------------------------------------------------- Ang Ladlad is an organization composed of men and women who identify
----x themselves as lesbians, gays, bisexuals, or trans-gendered individuals (LGBTs).
Incorporated in 2003, Ang Ladlad first applied for registration with the COMELEC
DECISION in 2006. The application for accreditation was denied on the ground that the
organization had no substantial membership base. On August 17, 2009, Ang
DEL CASTILLO, J.: Ladlad again filed a Petition[5] for registration with the COMELEC.

... [F]reedom to differ is not limited to things that do not matter Before the COMELEC, petitioner argued that the LGBT community is a
much. That would be a mere shadow of freedom. The test of its marginalized and under-represented sector that is particularly disadvantaged
substance is the right to differ as to things that touch the heart because of their sexual orientation and gender identity; that LGBTs are victims of
of the existing order. exclusion, discrimination, and violence; that because of negative societal
attitudes, LGBTs are constrained to hide their sexual orientation; and that Ang
Justice Robert A. Jackson Ladladcomplied with the 8-point guidelines enunciated by this Court in Ang
West Virginia State Board of Education v. Bagong Bayani-OFW Labor Party v. Commission on Elections.[6] Ang Ladlad laid
Barnette[1] out its national membership base consisting of individual members and
organizational supporters, and outlined its platform of governance.[7]
On November 11, 2009, after admitting the petitioners evidence, the The ANG LADLAD apparently advocates sexual
COMELEC (Second Division) dismissed the Petition on moral grounds, stating immorality as indicated in the Petitions par. 6F:
that: Consensual partnerships or relationships by
x x x This Petition is dismissible on moral grounds. gays and lesbians who are already of age. It is
Petitioner defines the Filipino Lesbian, Gay, Bisexual and further indicated in par. 24 of the Petition
Transgender (LGBT) Community, thus: which waves for the record: In 2007, Men
Having Sex with Men or MSMs in
x x x a marginalized and under-represented the Philippines were estimated as 670,000
sector that is particularly disadvantaged (Genesis 19 is the history
because of their sexual orientation and gender of Sodom and Gomorrah).
identity.
and proceeded to define sexual orientation as that which: Laws are deemed incorporated in every
contract, permit, license, relationship, or
x x x refers to a persons capacity for profound accreditation. Hence, pertinent provisions of
emotional, affectional and sexual attraction to, the Civil Code and the Revised Penal Code are
and intimate and sexual relations with, deemed part of the requirement to be
individuals of a different gender, of the same complied with for accreditation.
gender, or more than one gender.
ANG LADLAD collides with Article 695 of the
This definition of the LGBT sector makes it crystal clear that Civil Code which defines nuisance as Any act,
petitioner tolerates immorality which offends religious beliefs. In omission, establishment, business, condition of
Romans 1:26, 27, Paul wrote: property, or anything else which x x x (3)
shocks, defies; or disregards decency or
For this cause God gave them up into vile morality x x x
affections, for even their women did change
the natural use into that which is against It also collides with Article 1306 of the Civil
nature: And likewise also the men, leaving the Code: The contracting parties may establish
natural use of the woman, burned in their lust such stipulations, clauses, terms and conditions
one toward another; men with men working as they may deem convenient, provided they
that which is unseemly, and receiving in are not contrary to law, morals, good customs,
themselves that recompense of their error public order or public policy. Art 1409 of the
which was meet. Civil Code provides that Contracts whose
cause, object or purpose is contrary to
In the Koran, the hereunder verses are pertinent: law, morals, good customs, public order or
public policy are inexistent and void from the
For ye practice your lusts on men in preference beginning.
to women ye are indeed a people transgressing
beyond bounds. (7.81) And we rained down on Finally to safeguard the morality of the Filipino community, the
them a shower (of brimstone): Then see what Revised Penal Code, as amended, penalizes Immoral doctrines,
was the end of those who indulged in sin and obscene publications and exhibitions and indecent shows as
crime! (7:84) He said: O my Lord! Help Thou follows:
me against people who do mischief (29:30).
Art. 201. Immoral doctrines, obscene
As correctly pointed out by the Law Department in its Comment publications and exhibitions, and indecent
dated October 2, 2008: shows. The penalty of prision mayor or a fine
ranging from six thousand to twelve thousand
pesos, or both such imprisonment and fine, Section 13, Article II of the Constitution to protect our youth
shall be imposed upon: from moral and spiritual degradation.[8]

1. Those who shall publicly expound or


proclaim doctrines openly contrary to public When Ang Ladlad sought reconsideration,[9] three commissioners voted
morals; to overturn the First Assailed Resolution (Commissioners Gregorio Y. Larrazabal,
Rene V. Sarmiento, and Armando Velasco), while three commissioners voted to
2. (a) The authors of obscene literature, deny Ang Ladlads Motion for Reconsideration (Commissioners Nicodemo T.
published with their knowledge in any form; Ferrer, Lucenito N. Tagle, and Elias R. Yusoph). The COMELEC Chairman,
the editors publishing such literature; and the breaking the tie and speaking for the majority in his Separate Opinion, upheld the
owners/operators of the establishment selling First Assailed Resolution, stating that:
the same;
I. The Spirit of Republic Act No. 7941
(b) Those who, in theaters, fairs,
cinematographs or any other place, exhibit Ladlad is applying for accreditation as a sectoral party in the
indecent or immoral plays, scenes, acts or party-list system. Even assuming that it has properly proven its
shows, it being understood that the obscene under-representation and marginalization, it cannot be said
literature or indecent or immoral plays, scenes, that Ladlads expressed sexual orientations per se would benefit
acts or shows, whether live or in film, which the nation as a whole.
are prescribed by virtue hereof, shall include
those which: (1) glorify criminals or condone Section 2 of the party-list law unequivocally states that the
crimes; (2) serve no other purpose but to purpose of the party-list system of electing congressional
satisfy the market for violence, lust or representatives is to enable Filipino citizens belonging to
pornography; (3) offend any race or religion; marginalized and under-represented sectors, organizations and
(4) tend to abet traffic in and use of prohibited parties, and who lack well-defined political constituencies but
drugs; and (5) are contrary to law, public who could contribute to the formulation and enactment of
order, morals, good customs, established appropriate legislation that will benefit the nation as a whole, to
policies, lawful orders, decrees and edicts. become members of the House of Representatives.

3. Those who shall sell, give away or exhibit If entry into the party-list system would depend only on the
films, prints, engravings, sculpture or literature ability of an organization to represent its constituencies, then all
which are offensive to morals. representative organizations would have found themselves into
the party-list race. But that is not the intention of the framers of
Petitioner should likewise be denied accreditation not only for the law. The party-list system is not a tool to advocate tolerance
advocating immoral doctrines but likewise for not being truthful and acceptance of misunderstood persons or groups of persons.
when it said that it or any of its nominees/party-list Rather, the party-list system is a tool for the realization of
representatives have not violated or failed to comply with laws, aspirations of marginalized individuals whose interests
rules, or regulations relating to the elections. are also the nations only that their interests have not been
brought to the attention of the nation because of their under
Furthermore, should this Commission grant the petition, we will be representation. Until the time comes when Ladlad is able
exposing our youth to an environment that does not conform to to justify that having mixed sexual orientations and
the teachings of our faith. Lehman Strauss, a famous bible transgender identities is beneficial to the nation, its
teacher and writer in the U.S.A. said in one article that older application for accreditation under the party-list system
practicing homosexuals are a threat to the youth. As an agency will remain just that.
of the government, ours too is the States avowed duty under
II. No substantial differentiation
Having Sex with Men or MSMs in the Philippines were estimated
In the United States, whose equal protection doctrine pervades as 670,000. Moreoever, Article 694 of the Civil Code defines
Philippine jurisprudence, courts do not recognize lesbians, gays, nuisance as any act, omission x x x or anything else x x x which
homosexuals, and bisexuals (LGBT) as a special class of shocks, defies or disregards decency or morality x x x. These
individuals. x x x Significantly, it has also been held that are all unlawful.[10]
homosexuality is not a constitutionally protected fundamental
right, and that nothing in the U.S. Constitution discloses a
comparable intent to protect or promote the social or legal On January 4, 2010, Ang Ladlad filed this Petition, praying that the Court
equality of homosexual relations, as in the case of race or annul the Assailed Resolutions and direct the COMELEC to grant Ang
religion or belief. Ladlads application for accreditation. Ang Ladladalso sought the issuance ex
parte of a preliminary mandatory injunction against the COMELEC, which had
xxxx previously announced that it would begin printing the final ballots for the May
2010 elections by January 25, 2010.
Thus, even if societys understanding, tolerance, and acceptance
of LGBTs is elevated, there can be no denying On January 6, 2010, we ordered the Office of the Solicitor General (OSG)
that Ladlad constituencies are still males and females, and they to file its Comment on behalf of COMELEC not later than 12:00 noon of January
will remain either male or female protected by the same 11, 2010.[11] Instead of filing a Comment, however, the OSG filed a Motion for
Bill of Rights that applies to all citizens alike. Extension, requesting that it be given until January 16, 2010 to Comment.
[12]
Somewhat surprisingly, the OSG later filed a Comment in support of
xxxx petitioners application.[13] Thus, in order to give COMELEC the opportunity to fully
ventilate its position, we required it to file its own comment. [14] The COMELEC,
IV. Public Morals through its Law Department, filed its Comment on February 2, 2010.[15]

x x x There is no question about not imposing In the meantime, due to the urgency of the petition, we issued a
on Ladlad Christian or Muslim religious practices. Neither is temporary restraining order on January 12, 2010, effective immediately and
there any attempt to any particular religious groups moral rules continuing until further orders from this Court, directing the COMELEC to cease
on Ladlad. Rather, what are being adopted as moral parameters and desist from implementing the Assailed Resolutions.[16]
and precepts are generally accepted public morals. They are
possibly religious-based, but as a society, the Philippines Also, on January 13, 2010, the Commission on Human Rights (CHR) filed
cannot ignore its more than 500 years of Muslim and a Motion to Intervene or to Appear as Amicus Curiae, attaching thereto
Christian upbringing, such that some moral precepts its Comment-in-Intervention.[17] The CHR opined that the denial of Ang
espoused by said religions have sipped [sic] into society Ladlads petition on moral grounds violated the standards and principles of the
and these are not publicly accepted moral norms. Constitution, the Universal Declaration of Human Rights (UDHR), and the
International Covenant on Civil and Political Rights (ICCPR). On January 19,
V. Legal Provisions 2010, we granted the CHRs motion to intervene.

But above morality and social norms, they have become part of On January 26, 2010, Epifanio D. Salonga, Jr. filed his Motion to
the law of the land. Article 201 of the Revised Penal Code Intervene[18] which motion was granted on February 2, 2010.[19]
imposes the penalty of prision mayor upon Those who shall
publicly expound or proclaim doctrines openly contrary to public The Parties Arguments
morals. It penalizes immoral doctrines, obscene publications
and exhibition and indecent shows. Ang Ladlad apparently falls Ang Ladlad argued that the denial of accreditation, insofar as it justified
under these legal provisions. This is clear from its Petitions the exclusion by using religious dogma, violated the constitutional guarantees
paragraph 6F: Consensual partnerships or relationships by gays against the establishment of religion.Petitioner also claimed that the Assailed
and lesbians who are already of age It is further indicated in Resolutions contravened its constitutional rights to privacy, freedom of speech
par. 24 of the Petition which waves for the record: In 2007, Men and assembly, and equal protection of laws, as well as constituted violations of
the Philippines international obligations against discrimination based on sexual and affiliate organizations. The COMELEC claims that upon verification by its field
orientation. personnel, it was shown that save for a few isolated places in the country,
petitioner does not exist in almost all provinces in the country.[21]
The OSG concurred with Ang Ladlads petition and argued that the COMELEC This argument that petitioner made untruthful statements in its petition
erred in denying petitioners application for registration since there was no basis when it alleged its national existence is a new one; previously, the COMELEC
for COMELECs allegations of immorality. It also opined that LGBTs have their own claimed that petitioner was not being truthful when it said that it or any of its
special interests and concerns which should have been recognized by the nominees/party-list representatives have not violated or failed to comply with
COMELEC as a separate classification. However, insofar as the purported laws, rules, or regulations relating to the elections. Nowhere was this ground for
violations of petitioners freedom of speech, expression, and assembly were denial of petitioners accreditation mentioned or even alluded to in the Assailed
concerned, the OSG maintained that there had been no restrictions on these Resolutions. This, in itself, is quite curious, considering that the reports of
rights. petitioners alleged non-existence were already available to the COMELEC prior to
the issuance of the First Assailed Resolution. At best, this is irregular procedure;
In its Comment, the COMELEC reiterated that petitioner does not have a at worst, a belated afterthought, a change in respondents theory, and a serious
concrete and genuine national political agenda to benefit the nation and that the violation of petitioners right to procedural due process.
petition was validly dismissed on moral grounds. It also argued for the first
time that the LGBT sector is not among the sectors enumerated by the Nonetheless, we find that there has been no misrepresentation. A
Constitution and RA 7941, and that petitioner made untruthful statements in its cursory perusal of Ang Ladlads initial petition shows that it never claimed to exist
petition when it alleged its national existence contrary to actual verification in each province of the Philippines. Rather, petitioner alleged that the LGBT
reports by COMELECs field personnel. community in the Philippines was estimated to constitute at least 670,000
persons; that it had 16,100 affiliates and members around the country, and
Our Ruling 4,044 members in its electronic discussion group.[22] Ang Ladlad also represented
itself to be a national LGBT umbrella organization with affiliates around
We grant the petition. the Philippines composed of the following LGBT networks:

Compliance with the Abra Gay Association


Requirements of the Aklan Butterfly Brigade (ABB) Aklan
Constitution and Republic Albay Gay Association
Act No. 7941 Arts Center of Cabanatuan City Nueva Ecija
Boys Legion Metro Manila
Cagayan de Oro People Like Us (CDO PLUS)
The COMELEC denied Ang Ladlads application for registration on the Cant Live in the Closet, Inc. (CLIC)
ground that the LGBT sector is neither enumerated in the Constitution and RA Metro Manila
7941, nor is it associated with or related to any of the sectors in the enumeration. Cebu Pride Cebu City
Circle of Friends
Respondent mistakenly opines that our ruling in Ang Bagong Dipolog Gay Association Zamboanga del Norte
Bayani stands for the proposition that only those sectors specifically enumerated Gay, Bisexual, & Transgender Youth
in the law or related to said sectors (labor, peasant, fisherfolk, urban poor, Association (GABAY)
indigenous cultural communities, elderly, handicapped, women, youth, veterans, Gay and Lesbian Activists Network for Gender
overseas workers, and professionals) may be registered under the party-list Equality (GALANG) Metro Manila
system. As we explicitly ruled in Ang Bagong Bayani-OFW Labor Party v. Gay Mens Support Group (GMSG) Metro
Commission on Elections,[20] the enumeration of marginalized and under- Manila
represented sectors is not exclusive. The crucial element is not whether a sector Gay United for Peace and Solidarity (GUPS)
is specifically enumerated, but whether a particular organization complies with the Lanao del Norte
requirements of the Constitution and RA 7941. Iloilo City Gay Association Iloilo City
Respondent also argues that Ang Ladlad made untruthful statements in Kabulig Writers Group Camarines Sur
its petition when it alleged that it had nationwide existence through its members Lesbian Advocates Philippines, Inc. (LEAP)
LUMINA Baguio City neutrality in religious matters.[24] Clearly, governmental reliance on religious
Marikina Gay Association Metro Manila justification is inconsistent with this policy of neutrality. [25] We thus find that it was
Metropolitan Community Church (MCC) grave violation of the non-establishment clause for the COMELEC to utilize the
Metro Manila Bible and the Koran to justify the exclusion of Ang Ladlad.
Naga City Gay Association Naga City
ONE BACARDI Rather than relying on religious belief, the legitimacy of the Assailed
Order of St. Aelred (OSAe) Metro Manila Resolutions should depend, instead, on whether the COMELEC is able to advance
PUP LAKAN some justification for its rulings beyond mere conformity to religious
RADAR PRIDEWEAR doctrine. Otherwise stated, government must act for secular purposes and in
Rainbow Rights Project (R-Rights), Inc. ways that have primarily secular effects. As we held in Estrada v. Escritor:[26]
Metro Manila
San Jose del Monte Gay Association Bulacan x x x The morality referred to in the law is public and necessarily
Sining Kayumanggi Royal Family Rizal secular, not religious as the dissent of Mr. Justice Carpio holds.
Society of Transexual Women of "Religious teachings as expressed in public debate may influence
the Philippines (STRAP) Metro Manila the civil public order but public moral disputes may be resolved
Soul Jive Antipolo, Rizal only on grounds articulable in secular terms." Otherwise, if
The Link Davao City government relies upon religious beliefs in formulating public
Tayabas Gay Association Quezon policies and morals, the resulting policies and morals would
Womens Bisexual Network Metro Manila require conformity to what some might regard as religious
Zamboanga Gay programs or agenda. The non-believers would therefore be
Association Zamboanga City[23] compelled to conform to a standard of conduct buttressed by a
religious belief, i.e., to a "compelled religion," anathema to
religious freedom. Likewise, if government based its actions
Since the COMELEC only searched for the names ANG LADLAD LGBT upon religious beliefs, it would tacitly approve or endorse that
or LADLAD LGBT, it is no surprise that they found that petitioner had no presence belief and thereby also tacitly disapprove contrary religious or
in any of these regions. In fact, if COMELECs findings are to be believed, non-religious views that would not support the policy. As a
petitioner does not even exist in Quezon City, which is registered as Ang result, government will not provide full religious freedom for all
Ladlads principal place of business. its citizens, or even make it appear that those whose beliefs are
disapproved are second-class citizens.
Against this backdrop, we find that Ang Ladlad has sufficiently In other words, government action, including its proscription of
demonstrated its compliance with the legal requirements for accreditation. immorality as expressed in criminal law like concubinage, must
Indeed, aside from COMELECs moral objection and the belated allegation of non- have a secular purpose. That is, the government proscribes this
existence, nowhere in the records has the respondent ever found/ruled that Ang conduct because it is "detrimental (or dangerous) to those
Ladlad is not qualified to register as a party-list organization under any of the conditions upon which depend the existence and progress of
requisites under RA 7941 or the guidelines in Ang Bagong Bayani. The difference, human society" and not because the conduct is proscribed by
COMELEC claims, lies in Ang Ladlads morality, or lack thereof. the beliefs of one religion or the other. Although admittedly,
moral judgments based on religion might have a compelling
Religion as the Basis for influence on those engaged in public deliberations over what
Refusal to Accept Ang actions would be considered a moral disapprobation punishable
Ladlads Petition for by law. After all, they might also be adherents of a religion and
Registration thus have religious opinions and moral codes with a compelling
influence on them; the human mind endeavors to regulate the
temporal and spiritual institutions of society in a uniform
Our Constitution provides in Article III, Section 5 that [n]o law shall be manner, harmonizing earth with heaven. Succinctly put, a law
made respecting an establishment of religion, or prohibiting the free exercise could be religious or Kantian or Aquinian or utilitarian in its
thereof. At bottom, what our non-establishment clause calls for is government deepest roots, but it must have an articulable and discernible
secular purpose and justification to pass scrutiny of the religion
clauses. x x x Recognizing the religious nature of the Filipinos x x x A person may be sexually attracted to a person of the
and the elevating influence of religion in society, however, the same gender, of a different gender, or more than one gender,
Philippine constitution's religion clauses prescribe not a strict but but mere attraction does not translate to immoral acts. There is
a benevolent neutrality. Benevolent neutrality recognizes that a great divide between thought and action. Reduction ad
government must pursue its secular goals and interests but at absurdum. If immoral thoughts could be penalized, COMELEC
the same time strive to uphold religious liberty to the greatest would have its hands full of disqualification cases against both
extent possible within flexible constitutional limits. Thus, the straights and the gays. Certainly this is not the intendment
although the morality contemplated by laws is secular, of the law.[31]
benevolent neutrality could allow for accommodation of morality
based on religion, provided it does not offend compelling state
interests.[27] Respondent has failed to explain what societal ills are sought to be
prevented, or why special protection is required for the youth. Neither has the
COMELEC condescended to justify its position that petitioners admission into the
Public Morals as a Ground party-list system would be so harmful as to irreparably damage the moral fabric
to Deny Ang Ladlads of society. We, of course, do not suggest that the state is wholly without authority
Petition for Registration to regulate matters concerning morality, sexuality, and sexual relations, and we
recognize that the government will and should continue to restrict behavior
considered detrimental to society. Nonetheless, we cannot countenance
Respondent suggests that although the moral condemnation of advocates who, undoubtedly with the loftiest of intentions, situate morality on one
homosexuality and homosexual conduct may be religion-based, it has long been end of an argument or another, without bothering to go through the rigors of
transplanted into generally accepted public morals.The COMELEC argues: legal reasoning and explanation. In this, the notion of morality is robbed of all
value. Clearly then, the bare invocation of morality will not remove an issue from
Petitioners accreditation was denied not necessarily because our scrutiny.
their group consists of LGBTs but because of the danger it poses
to the people especially the youth. Once it is recognized by the We also find the COMELECs reference to purported violations of our penal
government, a sector which believes that there is nothing wrong and civil laws flimsy, at best; disingenuous, at worst. Article 694 of the Civil Code
in having sexual relations with individuals of the same gender is defines a nuisance as any act, omission, establishment, condition of property, or
a bad example. It will bring down the standard of morals we anything else which shocks, defies, or disregards decency or morality, the
cherish in our civilized society. Any society without a set of remedies for which are a prosecution under the Revised Penal Code or any local
moral precepts is in danger of losing its own existence.[28] ordinance, a civil action, or abatement without judicial proceedings. [32] A violation
of Article 201 of the Revised Penal Code, on the other hand, requires proof
beyond reasonable doubt to support a criminal conviction. It hardly needs to be
We are not blind to the fact that, through the years, homosexual emphasized that mere allegation of violation of laws is not proof, and a mere
conduct, and perhaps homosexuals themselves, have borne the brunt of societal blanket invocation of public morals cannot replace the institution of civil or
disapproval. It is not difficult to imagine the reasons behind this censure religious criminal proceedings and a judicial determination of liability or culpability.
beliefs, convictions about the preservation of marriage, family, and procreation, As such, we hold that moral disapproval, without more, is not a sufficient
even dislike or distrust of homosexuals themselves and their perceived governmental interest to justify exclusion of homosexuals from participation in
lifestyle.Nonetheless, we recall that the Philippines has not seen fit to criminalize the party-list system. The denial of Ang Ladlads registration on purely moral
homosexual conduct. Evidently, therefore, these generally accepted public morals grounds amounts more to a statement of dislike and disapproval of homosexuals,
have not been convincingly transplanted into the realm of law.[29] rather than a tool to further any substantial public interest. Respondents blanket
justifications give rise to the inevitable conclusion that the COMELEC targets
The Assailed Resolutions have not identified any specific overt immoral act homosexuals themselves as a class, not because of any particular morally
performed by Ang Ladlad. Even the OSG agrees that there should have been a reprehensible act. It is this selective targeting that implicates our equal protection
finding by the COMELEC that the groups members have committed or are clause.
committing immoral acts.[30] The OSG argues:
Equal Protection sufficient evidence to this effect, and it is simply unnecessary to make such a
ruling today. Petitioner itself has merely demanded that it be recognized under
Despite the absolutism of Article III, Section 1 of our Constitution, which the same basis as all other groups similarly situated, and that the COMELEC
provides nor shall any person be denied equal protection of the laws, courts have made an unwarranted and impermissible classification not justified by the
never interpreted the provision as an absolute prohibition on circumstances of the case.
classification. Equality, said Aristotle, consists in the same treatment of similar
persons.[33] The equal protection clause guarantees that no person or class of Freedom of Expression
persons shall be deprived of the same protection of laws which is enjoyed by and Association
other persons or other classes in the same place and in like circumstances.[34]
Under our system of laws, every group has the right to promote its
Recent jurisprudence has affirmed that if a law neither burdens a fundamental agenda and attempt to persuade society of the validity of its position through
right nor targets a suspect class, we will uphold the classification as long as it normal democratic means.[39] It is in the public square that deeply held
bears a rational relationship to some legitimate government end. [35] In Central convictions and differing opinions should be distilled and deliberated upon. As we
Bank Employees Association, Inc. v. Banko Sentral ng Pilipinas,[36] we declared held in Estrada v. Escritor:[40]
that [i]n our jurisdiction, the standard of analysis of equal protection challenges x
x x have followed the rational basis test, coupled with a deferential attitude to In a democracy, this common agreement on political and moral
legislative classifications and a reluctance to invalidate a law unless there is a ideas is distilled in the public square. Where citizens are free,
showing of a clear and unequivocal breach of the Constitution.[37] every opinion, every prejudice, every aspiration, and every
moral discernment has access to the public square where people
The COMELEC posits that the majority of the Philippine population deliberate the order of their life together. Citizens are the
considers homosexual conduct as immoral and unacceptable, and this constitutes bearers of opinion, including opinion shaped by, or espousing
sufficient reason to disqualify the petitioner. Unfortunately for the respondent, the religious belief, and these citizens have equal access to the
Philippine electorate has expressed no such belief. No law exists to criminalize public square. In this representative democracy, the state is
homosexual behavior or expressions or parties about homosexual prohibited from determining which convictions and moral
behavior. Indeed, even if we were to assume that public opinion is as the judgments may be proposed for public deliberation. Through a
COMELEC describes it, the asserted state interest here that is, moral disapproval constitutionally designed process, the people deliberate and
of an unpopular minority is not a legitimate state interest that is sufficient to decide. Majority rule is a necessary principle in this democratic
satisfy rational basis review under the equal protection clause. The COMELECs governance. Thus, when public deliberation on moral judgments
differentiation, and its unsubstantiated claim that Ang Ladlad cannot contribute to is finally crystallized into law, the laws will largely reflect the
the formulation of legislation that would benefit the nation, furthers no legitimate beliefs and preferences of the majority, i.e., the mainstream or
state interest other than disapproval of or dislike for a disfavored group. median groups. Nevertheless, in the very act of adopting and
accepting a constitution and the limits it specifies including
From the standpoint of the political process, the lesbian, gay, bisexual, protection of religious freedom "not only for a minority, however
and transgender have the same interest in participating in the party-list system small not only for a majority, however large but for each of us"
on the same basis as other political parties similarly situated. State intrusion in the majority imposes upon itself a self-denying ordinance. It
this case is equally burdensome. Hence, laws of general application should apply promises not to do what it otherwise could do: to ride
with equal force to LGBTs, and they deserve to participate in the party-list system roughshod over the dissenting minorities.
on the same basis as other marginalized and under-represented sectors.

It bears stressing that our finding that COMELECs act of differentiating Freedom of expression constitutes one of the essential foundations of a
LGBTs from heterosexuals insofar as the party-list system is concerned does not democratic society, and this freedom applies not only to those that are favorably
imply that any other law distinguishing between heterosexuals and homosexuals received but also to those that offend, shock, or disturb. Any restriction imposed
under different circumstances would similarly fail. We disagree with the OSGs in this sphere must be proportionate to the legitimate aim pursued. Absent any
position that homosexuals are a class in themselves for the purposes of the equal compelling state interest, it is not for the COMELEC or this Court to impose its
protection clause.[38] We are not prepared to single out homosexuals as a views on the populace. Otherwise stated, the COMELEC is certainly not free to
separate class meriting special or differentiated treatment. We have not received
interfere with speech for no better reason than promoting an approved message entitled to hold and express that view. However, as far as this Court is concerned,
or discouraging a disfavored one. our democracy precludes using the religious or moral views of one part of the
community to exclude from consideration the values of other members of the
This position gains even more force if one considers that homosexual community.
conduct is not illegal in this country. It follows that both expressions concerning
ones homosexuality and the activity of forming a political association that Of course, none of this suggests the impending arrival of a golden age for gay
supports LGBT individuals are protected as well. rights litigants. It well may be that this Decision will only serve to highlight the
Other jurisdictions have gone so far as to categorically rule that even discrepancy between the rigid constitutional analysis of this Court and the more
overwhelming public perception that homosexual conduct violates public morality complex moral sentiments of Filipinos. We do not suggest that public opinion,
does not justify criminalizing same-sex conduct.[41] European and United Nations even at its most liberal, reflect a clear-cut strong consensus favorable to gay
judicial decisions have ruled in favor of gay rights claimants on both privacy and rights claims and we neither attempt nor expect to affect individual perceptions of
equality grounds, citing general privacy and equal protection provisions in foreign homosexuality through this Decision.
and international texts.[42] To the extent that there is much to learn from other
jurisdictions that have reflected on the issues we face here, such jurisprudence is The OSG argues that since there has been neither prior restraint nor subsequent
certainly illuminating. These foreign authorities, while not formally binding on punishment imposed on Ang Ladlad, and its members have not been deprived of
Philippine courts, may nevertheless have persuasive influence on the Courts their right to voluntarily associate, then there has been no restriction on their
analysis. freedom of expression or association. The OSG argues that:

In the area of freedom of expression, for instance, United States courts There was no utterance restricted, no publication censored, or
have ruled that existing free speech doctrines protect gay and lesbian rights to any assembly denied. [COMELEC] simply exercised its authority
expressive conduct. In order to justify the prohibition of a particular expression of to review and verify the qualifications of petitioner as a sectoral
opinion, public institutions must show that their actions were caused by party applying to participate in the party-list system. This lawful
something more than a mere desire to avoid the discomfort and unpleasantness exercise of duty cannot be said to be a transgression of Section
that always accompany an unpopular viewpoint.[43] 4, Article III of the Constitution.

With respect to freedom of association for the advancement of ideas and xxxx
beliefs, in Europe, with its vibrant human rights tradition, the European Court of
Human Rights (ECHR) has repeatedly stated that a political party may campaign A denial of the petition for registration x x x does not deprive the
for a change in the law or the constitutional structures of a state if it uses legal members of the petitioner to freely take part in the conduct of
and democratic means and the changes it proposes are consistent with elections. Their right to vote will not be hampered by said denial.
democratic principles. The ECHR has emphasized that political ideas that In fact, the right to vote is a constitutionally-guaranteed right
challenge the existing order and whose realization is advocated by peaceful which cannot be limited.
means must be afforded a proper opportunity of expression through the exercise
of the right of association, even if such ideas may seem shocking or unacceptable As to its right to be elected in a genuine periodic election,
to the authorities or the majority of the population.[44] A political group should not petitioner contends that the denial of Ang Ladlads petition has
be hindered solely because it seeks to publicly debate controversial political issues the clear and immediate effect of limiting, if not outrightly
in order to find solutions capable of satisfying everyone concerned. [45] Only if a nullifying the capacity of its members to fully and equally
political party incites violence or puts forward policies that are incompatible with participate in public life through engagement in the party list
democracy does it fall outside the protection of the freedom of association elections.
guarantee.[46]
This argument is puerile. The holding of a public office is
We do not doubt that a number of our citizens may believe that not a right but a privilege subject to limitations imposed by law.
homosexual conduct is distasteful, offensive, or even defiant. They are entitled to x x x[47]
hold and express that view. On the other hand, LGBTs and their supporters, in all The OSG fails to recall that petitioner has, in fact, established its
likelihood, believe with equal fervor that relationships between individuals of the qualifications to participate in the party-list system, and as advanced by the OSG
same sex are morally equivalent to heterosexual relationships. They, too, are itself the moral objection offered by the COMELEC was not a limitation imposed
by law. To the extent, therefore, that the petitioner has been precluded, because The UDHR provides:
of COMELECs action, from publicly expressing its views as a political party and
participating on an equal basis in the political process with other equally-qualified Article 21.
party-list candidates, we find that there has, indeed, been a transgression of
petitioners fundamental rights. (1) Everyone has the right to take part in the
government of his country, directly or through freely chosen
Non-Discrimination and representatives.
International Law Likewise, the ICCPR states:

Article 25
In an age that has seen international law evolve geometrically in scope Every citizen shall have the right and the opportunity,
and promise, international human rights law, in particular, has grown dynamically without any of the distinctions mentioned in article 2 and
in its attempt to bring about a more just and humane world order. For individuals without unreasonable restrictions:
and groups struggling with inadequate structural and governmental support,
international human rights norms are particularly significant, and should be (a) To take part in the conduct of public affairs, directly
effectively enforced in domestic legal systems so that such norms may become or through freely chosen representatives;
actual, rather than ideal, standards of conduct.
(b) To vote and to be elected at genuine periodic
Our Decision today is fully in accord with our international obligations to elections which shall be by universal and equal suffrage and
protect and promote human rights. In particular, we explicitly recognize the shall be held by secret ballot, guaranteeing the free expression
principle of non-discrimination as it relates to the right to electoral participation, of the will of the electors;
enunciated in the UDHR and the ICCPR.
(c) To have access, on general terms of equality, to
The principle of non-discrimination is laid out in Article 26 of the ICCPR, as public service in his country.
follows:

Article 26 As stated by the CHR in its Comment-in-Intervention, the scope of the right to
electoral participation is elaborated by the Human Rights Committee in its
All persons are equal before the law and are entitled without any General Comment No. 25 (Participation in Public Affairs and the Right to Vote) as
discrimination to the equal protection of the law. In this respect, follows:
the law shall prohibit any discrimination and guarantee to all
persons equal and effective protection against discrimination on 1. Article 25 of the Covenant recognizes and protects
any ground such as race, colour, sex, language, religion, political the right of every citizen to take part in the conduct of public
or other opinion, national or social origin, property, birth or other affairs, the right to vote and to be elected and the right to have
status. access to public service. Whatever form of constitution or
government is in force, the Covenant requires States to adopt
such legislative and other measures as may be necessary to
In this context, the principle of non-discrimination requires that laws of ensure that citizens have an effective opportunity to enjoy the
general application relating to elections be applied equally to all persons, rights it protects. Article 25 lies at the core of democratic
regardless of sexual orientation. Although sexual orientation is not specifically government based on the consent of the people and in
enumerated as a status or ratio for discrimination in Article 26 of the ICCPR, the conformity with the principles of the Covenant.
ICCPR Human Rights Committee has opined that the reference to sex in Article
26 should be construed to include sexual orientation.[48] Additionally, a variety of xxxx
United Nations bodies have declared discrimination on the basis of sexual
orientation to be prohibited under various international agreements.[49] 15. The effective implementation of the right and the
opportunity to stand for elective office ensures that persons
entitled to vote have a free choice of candidates. Any restrictions
on the right to stand for election, such as minimum age, must As a final note, we cannot help but observe that the social issues
be justifiable on objective and reasonable criteria. Persons who presented by this case are emotionally charged, societal attitudes are in flux,
are otherwise eligible to stand for election should not be even the psychiatric and religious communities are divided in opinion. This Courts
excluded by unreasonable or discriminatory requirements such role is not to impose its own view of acceptable behavior. Rather, it is to apply the
as education, residence or descent, or by reason of political Constitution and laws as best as it can, uninfluenced by public opinion, and
affiliation. No person should suffer discrimination or confident in the knowledge that our democracy is resilient enough to withstand
disadvantage of any kind because of that person's candidacy. vigorous debate.
States parties should indicate and explain the legislative
provisions which exclude any group or category of persons from WHEREFORE, the Petition is hereby GRANTED. The Resolutions of the
elective office.[50] Commission on Elections dated November 11, 2009 and December 16, 2009 in
SPP No. 09-228 (PL) are hereby SET ASIDE. The Commission on Elections is
We stress, however, that although this Court stands willing to assume directed to GRANT petitioners application for party-list accreditation.
the responsibility of giving effect to the Philippines international law obligations, SO ORDERED.
the blanket invocation of international law is not the panacea for all social ills. We
refer now to the petitioners invocation of the Yogyakarta Principles (the EN BANC
Application of International Human Rights Law In Relation to Sexual Orientation
and Gender Identity),[51] which petitioner declares to reflect binding principles of
international law.
[G.R. No. 141284. August 15, 2000]
At this time, we are not prepared to declare that these Yogyakarta
Principles contain norms that are obligatory on the Philippines. There are
declarations and obligations outlined in said Principles which are not reflective of
the current state of international law, and do not find basis in any of the sources
INTEGRATED BAR OF THE PHILIPPINES, petitioner, vs. HON.
of international law enumerated under Article 38(1) of the Statute of the
RONALDO B. ZAMORA, GEN. PANFILO M. LACSON, GEN.
International Court of Justice.[52] Petitioner has not undertaken any objective and
EDGAR B. AGLIPAY, and GEN. ANGELO REYES, respondents.
rigorous analysis of these alleged principles of international law to ascertain their
true status.
DECISION
We also hasten to add that not everything that society or a certain
KAPUNAN, J.:
segment of society wants or demands is automatically a human right. This is not
an arbitrary human intervention that may be added to or subtracted from at
will. It is unfortunate that much of what passes for human rights today is a much At bar is a special civil action for certiorari and prohibition with prayer
broader context of needs that identifies many social desires as rights in order to for issuance of a temporary restraining order seeking to nullify on
further claims that international law obliges states to sanction these innovations. constitutional grounds the order of President Joseph Ejercito Estrada
This has the effect of diluting real human rights, and is a result of the notion that commanding the deployment of the Philippine Marines (the Marines) to
if wants are couched in rights language, then they are no longer controversial. join the Philippine National Police (the PNP) in visibility patrols around the
metropolis.
Using even the most liberal of lenses, these Yogyakarta Principles,
In view of the alarming increase in violent crimes in Metro Manila, like
consisting of a declaration formulated by various international law professors, are
robberies, kidnappings and carnappings, the President, in a verbal
at best de lege ferenda and do not constitute binding obligations on the
directive, ordered the PNP and the Marines to conduct joint visibility
Philippines. Indeed, so much of contemporary international law is characterized
patrols for the purpose of crime prevention and suppression. The Secretary
by the soft law nomenclature, i.e., international law is full of principles that
of National Defense, the Chief of Staff of the Armed Forces of the
promote international cooperation, harmony, and respect for human rights, most
Philippines (the AFP), the Chief of the PNP and the Secretary of the
of which amount to no more than well-meaning desires, without the support of
Interior and Local Government were tasked to execute and implement the
either State practice or opinio juris.[53]
said order. In compliance with the presidential mandate, the PNP Chief, 4. MISSION:
through Police Chief Superintendent Edgar B. Aglipay, formulated Letter of
Instruction 02/2000[1] (the LOI) which detailed the manner by which the The PNP NCRPO will organize a provisional Task Force to conduct joint
joint visibility patrols, called Task Force Tulungan, would be conducted. NCRPO-PM visibility patrols to keep Metro Manila streets crime-free,
[2]
Task Force Tulungan was placed under the leadership of the Police Chief through a sustained street patrolling to minimize or eradicate all forms of
of Metro Manila. high-profile crimes especially those perpetrated by organized crime
Subsequently, the President confirmed his previous directive on the syndicates whose members include those that are well-trained, disciplined
deployment of the Marines in a Memorandum, dated 24 January 2000, and well-armed active or former PNP/Military personnel.
addressed to the Chief of Staff of the AFP and the PNP Chief. [3] In the
Memorandum, the President expressed his desire to improve the peace 5. CONCEPT IN JOINT VISIBILITY PATROL OPERATIONS:
and order situation in Metro Manila through a more effective crime
prevention program including increased police patrols. [4] The President a. The visibility patrols shall be conducted jointly by the NCRPO [National
further stated that to heighten police visibility in the metropolis, Capital Regional Police Office] and the Philippine Marines to curb
augmentation from the AFP is necessary. [5] Invoking his powers as criminality in Metro Manila and to preserve the internal security of the
Commander-in-Chief under Section 18, Article VII of the Constitution, the state against insurgents and other serious threat to national security,
President directed the AFP Chief of Staff and PNP Chief to coordinate with although the primary responsibility over Internal Security Operations still
each other for the proper deployment and utilization of the Marines to rests upon the AFP.
assist the PNP in preventing or suppressing criminal or lawless violence.
[6]
Finally, the President declared that the services of the Marines in the
b. The principle of integration of efforts shall be applied to eradicate all
anti-crime campaign are merely temporary in nature and for a reasonable
forms of high-profile crimes perpetrated by organized crime syndicates
period only, until such time when the situation shall have improved. [7]
operating in Metro Manila. This concept requires the military and police to
The LOI explains the concept of the PNP-Philippine Marines joint work cohesively and unify efforts to ensure a focused, effective and
visibility patrols as follows: holistic approach in addressing crime prevention. Along this line, the role
of the military and police aside from neutralizing crime syndicates is to
xxx bring a wholesome atmosphere wherein delivery of basic services to the
people and development is achieved. Hand-in-hand with this joint NCRPO-
2. PURPOSE: Philippine Marines visibility patrols, local Police Units are responsible for
the maintenance of peace and order in their locality.
The Joint Implementing Police Visibility Patrols between the PNP NCRPO
and the Philippine Marines partnership in the conduct of visibility patrols in c. To ensure the effective implementation of this project, a provisional
Metro Manila for the suppression of crime prevention and other serious Task Force TULUNGAN shall be organized to provide the mechanism,
threats to national security. structure, and procedures for the integrated planning, coordinating,
monitoring and assessing the security situation.
3. SITUATION:
xxx.[8]
Criminal incidents in Metro Manila have been perpetrated not only by The selected areas of deployment under the LOI are: Monumento
ordinary criminals but also by organized syndicates whose members Circle, North Edsa (SM City), Araneta Shopping Center, Greenhills, SM
include active and former police/military personnel whose training, skill, Megamall, Makati Commercial Center, LRT/MRT Stations and the NAIA and
discipline and firepower prove well-above the present capability of the Domestic Airport.[9]
local police alone to handle. The deployment of a joint PNP NCRPO-
Philippine Marines in the conduct of police visibility patrol in urban areas On 17 January 2000, the Integrated Bar of the Philippines (the IBP)
will reduce the incidence of crimes specially those perpetrated by active or filed the instant petition to annul LOI 02/2000 and to declare the
former police/military personnel. deployment of the Philippine Marines, null and void and unconstitutional,
arguing that:
I The issues raised in the present petition are: (1) Whether or not
petitioner has legal standing; (2) Whether or not the Presidents factual
THE DEPLOYMENT OF THE PHILIPPINE MARINES IN METRO MANILA IS determination of the necessity of calling the armed forces is subject to
VIOLATIVE OF THE CONSTITUTION, IN THAT: judicial review; and, (3) Whether or not the calling of the armed forces to
assist the PNP in joint visibility patrols violates the constitutional provisions
on civilian supremacy over the military and the civilian character of the
A) NO EMERGENCY SITUATION OBTAINS IN METRO MANILA AS WOULD
PNP.
JUSTIFY, EVEN ONLY REMOTELY, THE DEPLOYMENT OF SOLDIERS FOR
LAW ENFORCEMENT WORK; HENCE, SAID DEPLOYMENT IS IN The petition has no merit.
DEROGATION OF ARTICLE II, SECTION 3 OF THE CONSTITUTION;
First, petitioner failed to sufficiently show that it is in possession of
the requisites of standing to raise the issues in the petition. Second, the
B) SAID DEPLOYMENT CONSTITUTES AN INSIDIOUS INCURSION BY THE
President did not commit grave abuse of discretion amounting to lack or
MILITARY IN A CIVILIAN FUNCTION OF GOVERNMENT (LAW
excess of jurisdiction nor did he commit a violation of the civilian
ENFORCEMENT) IN DEROGATION OF ARTICLE XVI, SECTION 5 (4), OF THE
supremacy clause of the Constitution.
CONSTITUTION;
The power of judicial review is set forth in Section 1, Article VIII of
C) SAID DEPLOYMENT CREATES A DANGEROUS TENDENCY TO RELY ON the Constitution, to wit:
THE MILITARY TO PERFORM THE CIVILIAN FUNCTIONS OF THE
GOVERNMENT. Section 1. The judicial power shall be vested in one Supreme Court and in
such lower courts as may be established by law.
II
Judicial power includes the duty of the courts of justice to settle actual
IN MILITARIZING LAW ENFORCEMENT IN METRO MANILA, THE controversies involving rights which are legally demandable and
ADMINISTRATION IS UNWITTINGLY MAKING THE MILITARY MORE enforceable, and to determine whether or not there has been grave abuse
POWERFUL THAN WHAT IT SHOULD REALLY BE UNDER THE of discretion amounting to lack or excess of jurisdiction on the part of any
CONSTITUTION.[10] branch or instrumentality of the Government.

Asserting itself as the official organization of Filipino lawyers tasked When questions of constitutional significance are raised, the Court can
with the bounden duty to uphold the rule of law and the Constitution, the exercise its power of judicial review only if the following requisites are
IBP questions the validity of the deployment and utilization of the Marines complied with, namely: (1) the existence of an actual and appropriate
to assist the PNP in law enforcement. case; (2) a personal and substantial interest of the party raising the
constitutional question; (3) the exercise of judicial review is pleaded at the
Without granting due course to the petition, the Court in a Resolution, earliest opportunity; and (4) the constitutional question is the lis mota of
dated 25 January 2000, required the Solicitor General to file his
[11]
the case.[12]
Comment on the petition. On 8 February 2000, the Solicitor General
submitted his Comment.

The Solicitor General vigorously defends the constitutionality of the The IBP has not sufficiently complied with the requisites of
act of the President in deploying the Marines, contending, among others, standing in this case.
that petitioner has no legal standing; that the question of deployment of
the Marines is not proper for judicial scrutiny since the same involves a
political question; that the organization and conduct of police visibility Legal standing or locus standi has been defined as a personal and
patrols, which feature the team-up of one police officer and one Philippine substantial interest in the case such that the party has sustained or will
Marine soldier, does not violate the civilian supremacy clause in the sustain direct injury as a result of the governmental act that is being
Constitution. challenged.[13] The term interest means a material interest, an interest in
issue affected by the decree, as distinguished from mere interest in the not a few cases, the Court has adopted a liberal attitude on the locus
question involved, or a mere incidental interest. [14] The gist of the question standi of a petitioner where the petitioner is able to craft an issue of
of standing is whether a party alleges such personal stake in the outcome transcendental significance to the people. [17] Thus, when the issues raised
of the controversy as to assure that concrete adverseness which sharpens are of paramount importance to the public, the Court may brush aside
the presentation of issues upon which the court depends for illumination of technicalities of procedure.[18] In this case, a reading of the petition shows
difficult constitutional questions.[15] that the IBP has advanced constitutional issues which deserve the
attention of this Court in view of their seriousness, novelty and weight as
In the case at bar, the IBP primarily anchors its standing on its precedents. Moreover, because peace and order are under constant threat
alleged responsibility to uphold the rule of law and the Constitution. Apart and lawless violence occurs in increasing tempo, undoubtedly aggravated
from this declaration, however, the IBP asserts no other basis in support of by the Mindanao insurgency problem, the legal controversy raised in the
its locus standi. The mere invocation by the IBP of its duty to preserve the petition almost certainly will not go away. It will stare us in the face
rule of law and nothing more, while undoubtedly true, is not sufficient to again. It, therefore, behooves the Court to relax the rules on standing and
clothe it with standing in this case. This is too general an interest which is to resolve the issue now, rather than later.
shared by other groups and the whole citizenry. Based on the standards
above-stated, the IBP has failed to present a specific and substantial
interest in the resolution of the case. Its fundamental purpose which,
The President did not commit grave abuse of discretion in calling out the Marines.
under Section 2, Rule 139-A of the Rules of Court, is to elevate the
standards of the law profession and to improve the administration of
justice is alien to, and cannot be affected by the deployment of the
Marines. It should also be noted that the interest of the National President In the case at bar, the bone of contention concerns the factual
of the IBP who signed the petition, is his alone, absent a formal board determination of the President of the necessity of calling the armed forces,
resolution authorizing him to file the present action. To be sure, members particularly the Marines, to aid the PNP in visibility patrols. In this regard,
of the BAR, those in the judiciary included, have varying opinions on the the IBP admits that the deployment of the military personnel falls under
issue. Moreover, the IBP, assuming that it has duly authorized the National the Commander-in-Chief powers of the President as stated in Section 18,
President to file the petition, has not shown any specific injury which it has Article VII of the Constitution, specifically, the power to call out the armed
suffered or may suffer by virtue of the questioned governmental forces to prevent or suppress lawless violence, invasion or rebellion. What
act. Indeed, none of its members, whom the IBP purportedly represents, the IBP questions, however, is the basis for the calling of the Marines
has sustained any form of injury as a result of the operation of the joint under the aforestated provision. According to the IBP, no emergency exists
visibility patrols. Neither is it alleged that any of its members has been that would justify the need for the calling of the military to assist the
arrested or that their civil liberties have been violated by the deployment police force. It contends that no lawless violence, invasion or rebellion
of the Marines. What the IBP projects as injurious is the supposed exist to warrant the calling of the Marines. Thus, the IBP prays that this
militarization of law enforcement which might threaten Philippine Court review the sufficiency of the factual basis for said troop [Marine]
democratic institutions and may cause more harm than good in the long deployment.[19]
run. Not only is the presumed injury not personal in character, it is
likewise too vague, highly speculative and uncertain to satisfy the The Solicitor General, on the other hand, contends that the issue
requirement of standing. Since petitioner has not successfully established pertaining to the necessity of calling the armed forces is not proper for
a direct and personal injury as a consequence of the questioned act, it judicial scrutiny since it involves a political question and the resolution of
does not possess the personality to assail the validity of the deployment of factual issues which are beyond the review powers of this Court.
the Marines. This Court, however, does not categorically rule that the IBP As framed by the parties, the underlying issues are the scope of
has absolutely no standing to raise constitutional issues now or in the presidential powers and limits, and the extent of judicial review. But, while
future. The IBP must, by way of allegations and proof, satisfy this Court this Court gives considerable weight to the parties formulation of the
that it has sufficient stake to obtain judicial resolution of the controversy. issues, the resolution of the controversy may warrant a creative approach
Having stated the foregoing, it must be emphasized that this Court that goes beyond the narrow confines of the issues raised. Thus, while the
has the discretion to take cognizance of a suit which does not satisfy the parties are in agreement that the power exercised by the President is the
requirement of legal standing when paramount interest is involved. [16] In power to call out the armed forces, the Court is of the view that the power
involved may be no more than the maintenance of peace and order and
promotion of the general welfare.[20] For one, the realities on the ground do with the workings of another co-equal branch unless the case shows a
not show that there exist a state of warfare, widespread civil unrest or clear need for the courts to step in to uphold the law and the Constitution.
anarchy. Secondly, the full brunt of the military is not brought upon the
citizenry, a point discussed in the latter part of this decision. In the words As Taada v. Cuenco[23] puts it, political questions refer to those
of the late Justice Irene Cortes in Marcos v. Manglapus: questions which, under the Constitution, are to be decided by the people in
their sovereign capacity, or in regard to which full discretionary authority
has been delegated to the legislative or executive branch of government.
More particularly, this case calls for the exercise of the Presidents powers Thus, if an issue is clearly identified by the text of the Constitution as
as protector of the peace. [Rossiter, The American Presidency]. The power matters for discretionary action by a particular branch of government or to
of the President to keep the peace is not limited merely to exercising the the people themselves then it is held to be a political question. In the
commander-in-chief powers in times of emergency or to leading the State classic formulation of Justice Brennan in Baker v. Carr,[24] [p]rominent on
against external and internal threats to its existence. The President is not the surface of any case held to involve a political question is found a
only clothed with extraordinary powers in times of emergency, but is also textually demonstrable constitutional commitment of the issue to a
tasked with attending to the day-to-day problems of maintaining peace coordinate political department; or a lack of judicially discoverable and
and order and ensuring domestic tranquility in times when no foreign foe manageable standards for resolving it; or the impossibility of deciding
appears on the horizon. Wide discretion, within the bounds of law, in without an initial policy determination of a kind clearly for nonjudicial
fulfilling presidential duties in times of peace is not in any way diminished discretion; or the impossibility of a courts undertaking independent
by the relative want of an emergency specified in the commander-in-chief resolution without expressing lack of the respect due coordinate branches
provision. For in making the President commander-in-chief the of government; or an unusual need for unquestioning adherence to a
enumeration of powers that follow cannot be said to exclude the political decision already made; or the potentiality of embarassment from
Presidents exercising as Commander-in-Chief powers short of the calling of multifarious pronouncements by various departments on the one question.
the armed forces, or suspending the privilege of the writ of habeas
corpus or declaring martial law, in order to keep the peace, and maintain The 1987 Constitution expands the concept of judicial review by
public order and security. providing that (T)he Judicial power shall be vested in one Supreme Court
and in such lower courts as may be established by law.Judicial power
xxx[21] includes the duty of the courts of justice to settle actual controversies
involving rights which are legally demandable and enforceable, and to
Nonetheless, even if it is conceded that the power involved is the determine whether or not there has been a grave abuse of discretion
Presidents power to call out the armed forces to prevent or suppress amounting to lack or excess of jurisdiction on the part of any branch or
lawless violence, invasion or rebellion, the resolution of the controversy instrumentality of the Government.[25] Under this definition, the Court
will reach a similar result. cannot agree with the Solicitor General that the issue involved is a political
question beyond the jurisdiction of this Court to review. When the grant of
We now address the Solicitor Generals argument that the issue
power is qualified, conditional or subject to limitations, the issue of
involved is not susceptible to review by the judiciary because it involves a
whether the prescribed qualifications or conditions have been met or the
political question, and thus, not justiciable.
limitations respected, is justiciable - the problem being one of legality or
As a general proposition, a controversy is justiciable if it refers to a validity, not its wisdom.[26] Moreover, the jurisdiction to delimit
matter which is appropriate for court review. [22] It pertains to issues which constitutional boundaries has been given to this Court. [27] When political
are inherently susceptible of being decided on grounds recognized by law. questions are involved, the Constitution limits the determination as to
Nevertheless, the Court does not automatically assume jurisdiction over whether or not there has been a grave abuse of discretion amounting to
actual constitutional cases brought before it even in instances that are ripe lack or excess of jurisdiction on the part of the official whose action is
for resolution. One class of cases wherein the Court hesitates to rule on being questioned.[28]
are political questions. The reason is that political questions are concerned
By grave abuse of discretion is meant simply capricious or whimsical
with issues dependent upon the wisdom, not the legality, of a particular
exercise of judgment that is patent and gross as to amount to an evasion
act or measure being assailed. Moreover, the political question being a
of positive duty or a virtual refusal to perform a duty enjoined by law, or
function of the separation of powers, the courts will not normally interfere
to act at all in contemplation of law, as where the power is exercised in an
arbitrary and despotic manner by reason of passion or hostility. [29] Under
this definition, a court is without power to directly decide matters over xxx
which full discretionary authority has been delegated. But while this Court
has no power to substitute its judgment for that of Congress or of the The full discretionary power of the President to determine the factual
President, it may look into the question of whether such exercise has been basis for the exercise of the calling out power is also implied and further
made in grave abuse of discretion. [30] A showing that plenary power is reinforced in the rest of Section 18, Article VII which reads, thus:
granted either department of government, may not be an obstacle to xxx
judicial inquiry, for the improvident exercise or abuse thereof may give
rise to justiciable controversy.[31]
Within forty-eight hours from the proclamation of martial law or the
When the President calls the armed forces to prevent or suppress suspension of the privilege of the writ of habeas corpus, the President shall
lawless violence, invasion or rebellion, he necessarily exercises a submit a report in person or in writing to the Congress.The Congress,
discretionary power solely vested in his wisdom. This is clear from the voting jointly, by a vote of at least a majority of all its Members in regular
intent of the framers and from the text of the Constitution itself. The or special session, may revoke such proclamation or suspension, which
Court, thus, cannot be called upon to overrule the Presidents wisdom or revocation shall not be set aside by the President. Upon the initiative of
substitute its own. However, this does not prevent an examination of the President, the Congress may, in the same manner, extend such
whether such power was exercised within permissible constitutional limits proclamation or suspension for a period to be determined by the Congress,
or whether it was exercised in a manner constituting grave abuse of if the invasion or rebellion shall persist and public safety requires it.
discretion. In view of the constitutional intent to give the President full
discretionary power to determine the necessity of calling out the armed The Congress, if not in session, shall within twenty-four hours following
forces, it is incumbent upon the petitioner to show that the Presidents such proclamation or suspension, convene in accordance with its rules
decision is totally bereft of factual basis. The present petition fails to without need of a call.
discharge such heavy burden as there is no evidence to support the
assertion that there exist no justification for calling out the armed
The Supreme Court may review, in an appropriate proceeding filed by any
forces. There is, likewise, no evidence to support the proposition that
citizen, the sufficiency of the factual basis of the proclamation of martial
grave abuse was committed because the power to call was exercised in
law or the suspension of the privilege of the writ or the extension thereof,
such a manner as to violate the constitutional provision on civilian
and must promulgate its decision thereon within thirty days from its filing.
supremacy over the military. In the performance of this Courts duty of
purposeful hesitation[32] before declaring an act of another branch as
unconstitutional, only where such grave abuse of discretion is clearly A state of martial law does not suspend the operation of the Constitution,
shown shall the Court interfere with the Presidents judgment. To doubt is nor supplant the functioning of the civil courts or legislative assemblies,
to sustain. nor authorize the conferment of jurisdiction on military courts and
agencies over civilians where civil courts are able to function, nor
There is a clear textual commitment under the Constitution to bestow automatically suspend the privilege of the writ.
on the President full discretionary power to call out the armed forces and
to determine the necessity for the exercise of such power. Section 18, The suspension of the privilege of the writ shall apply only to persons
Article VII of the Constitution, which embodies the powers of the President judicially charged for rebellion or offenses inherent in or directly connected
as Commander-in-Chief, provides in part: with invasion.

The President shall be the Commander-in-Chief of all armed forces of the During the suspension of the privilege of the writ, any person thus
Philippines and whenever it becomes necessary, he may call out such arrested or detained shall be judicially charged within three days,
armed forces to prevent or suppress lawless violence, invasion or otherwise he shall be released.
rebellion. In case of invasion or rebellion, when the public safety requires
it, he may, for a period not exceeding sixty days, suspend the privilege of
the writ of habeas corpus, or place the Philippines or any part thereof Under the foregoing provisions, Congress may revoke such
under martial law. proclamation or suspension and the Court may review the sufficiency of
the factual basis thereof. However, there is no such equivalent provision
dealing with the revocation or review of the Presidents action to call out MR. REGALADO. That does not require any concurrence by the legislature
the armed forces. The distinction places the calling out power in a different nor is it subject to judicial review.[34]
category from the power to declare martial law and the power to suspend
the privilege of the writ of habeas corpus, otherwise, the framers of the The reason for the difference in the treatment of the aforementioned
Constitution would have simply lumped together the three powers and powers highlights the intent to grant the President the widest leeway and
provided for their revocation and review without any broadest discretion in using the power to call out because it is considered
qualification. Expressio unius est exclusio alterius. Where the terms are as the lesser and more benign power compared to the power to suspend
expressly limited to certain matters, it may not, by interpretation or the privilege of the writ of habeas corpus and the power to impose martial
construction, be extended to other matters. [33] That the intent of the law, both of which involve the curtailment and suppression of certain basic
Constitution is exactly what its letter says, i.e., that the power to call is civil rights and individual freedoms, and thus necessitating safeguards by
fully discretionary to the President, is extant in the deliberation of the Congress and review by this Court.
Constitutional Commission, to wit:
Moreover, under Section 18, Article VII of the Constitution, in the
FR. BERNAS. It will not make any difference. I may add that there is a exercise of the power to suspend the privilege of the writ of habeas
graduated power of the President as Commander-in-Chief. First, he can corpus or to impose martial law, two conditions must concur: (1) there
call out such Armed Forces as may be necessary to suppress lawless must be an actual invasion or rebellion and, (2) public safety must require
violence; then he can suspend the privilege of the writ of habeas corpus, it. These conditions are not required in the case of the power to call out
then he can impose martial law. This is a graduated sequence. the armed forces. The only criterion is that whenever it becomes
necessary, the President may call the armed forces to prevent or suppress
lawless violence, invasion or rebellion." The implication is that the
When he judges that it is necessary to impose martial law or suspend the President is given full discretion and wide latitude in the exercise of the
privilege of the writ of habeas corpus, his judgment is subject to power to call as compared to the two other powers.
review. We are making it subject to review by the Supreme Court and
subject to concurrence by the National Assembly. But when he exercises If the petitioner fails, by way of proof, to support the assertion that
this lesser power of calling on the Armed Forces, when he says it is the President acted without factual basis, then this Court cannot undertake
necessary, it is my opinion that his judgment cannot be reviewed by an independent investigation beyond the pleadings.The factual necessity of
anybody. calling out the armed forces is not easily quantifiable and cannot be
objectively established since matters considered for satisfying the same is
xxx a combination of several factors which are not always accessible to the
courts. Besides the absence of textual standards that the court may use to
judge necessity, information necessary to arrive at such judgment might
FR. BERNAS. Let me just add that when we only have imminent danger,
also prove unmanageable for the courts. Certain pertinent information
the matter can be handled by the first sentence: The President may call
might be difficult to verify, or wholly unavailable to the courts. In many
out such armed forces to prevent or suppress lawless violence, invasion or
instances, the evidence upon which the President might decide that there
rebellion. So we feel that that is sufficient for handling imminent danger.
is a need to call out the armed forces may be of a nature not constituting
technical proof.
MR. DE LOS REYES. So actually, if a President feels that there is imminent
danger, the matter can be handled by the First Sentence: The On the other hand, the President as Commander-in-Chief has a vast
President....may call out such Armed Forces to prevent or suppress lawless intelligence network to gather information, some of which may be
violence, invasion or rebellion. So we feel that that is sufficient for classified as highly confidential or affecting the security of the state. In the
handling imminent danger, of invasion or rebellion, instead of imposing exercise of the power to call, on-the-spot decisions may be imperatively
martial law or suspending the writ of habeas corpus, he must necessarily necessary in emergency situations to avert great loss of human lives and
have to call the Armed Forces of the Philippines as their Commander-in- mass destruction of property.Indeed, the decision to call out the military to
Chief. Is that the idea? prevent or suppress lawless violence must be done swiftly and decisively if
it were to have any effect at all. Such a scenario is not farfetched when we
consider the present situation in Mindanao, where the insurgency problem
could spill over the other parts of the country. The determination of the Police Chief is the overall leader of the PNP-Philippine Marines joint
necessity for the calling out power if subjected to unfettered judicial visibility patrols.[37] Under the LOI, the police forces are tasked to brief or
scrutiny could be a veritable prescription for disaster, as such power may orient the soldiers on police patrol procedures. [38] It is their responsibility
be unduly straitjacketed by an injunction or a temporary restraining order to direct and manage the deployment of the Marines. [39] It is, likewise,
every time it is exercised. their duty to provide the necessary equipment to the Marines and render
logistical support to these soldiers. [40] In view of the foregoing, it cannot be
Thus, it is the unclouded intent of the Constitution to vest upon the properly argued that military authority is supreme over civilian
President, as Commander-in-Chief of the Armed Forces, full discretion to authority. Moreover, the deployment of the Marines to assist the PNP does
call forth the military when in his judgment it is necessary to do so in not unmake the civilian character of the police force. Neither does it
order to prevent or suppress lawless violence, invasion or rebellion. Unless amount to an insidious incursion of the military in the task of law
the petitioner can show that the exercise of such discretion was gravely enforcement in violation of Section 5(4), Article XVI of the Constitution. [41]
abused, the Presidents exercise of judgment deserves to be accorded
respect from this Court. In this regard, it is not correct to say that General Angelo Reyes,
Chief of Staff of the AFP, by his alleged involvement in civilian law
The President has already determined the necessity and factual basis enforcement, has been virtually appointed to a civilian post in derogation
for calling the armed forces. In his Memorandum, he categorically asserted of the aforecited provision. The real authority in these operations, as
that, [V]iolent crimes like bank/store robberies, holdups, kidnappings and stated in the LOI, is lodged with the head of a civilian institution, the PNP,
carnappings continue to occur in Metro Manila... [35] We do not doubt the and not with the military. Such being the case, it does not matter whether
veracity of the Presidents assessment of the situation, especially in the the AFP Chief actually participates in the Task Force Tulungan since he
light of present developments. The Court takes judicial notice of the recent does not exercise any authority or control over the same. Since none of
bombings perpetrated by lawless elements in the shopping malls, public the Marines was incorporated or enlisted as members of the PNP, there
utilities, and other public places. These are among the areas of can be no appointment to civilian position to speak of. Hence, the
deployment described in the LOI 2000. Considering all these facts, we hold deployment of the Marines in the joint visibility patrols does not destroy
that the President has sufficient factual basis to call for military aid in law the civilian character of the PNP.
enforcement and in the exercise of this constitutional power.
Considering the above circumstances, the Marines render nothing
more than assistance required in conducting the patrols. As such, there
can be no insidious incursion of the military in civilian affairs nor can there
The deployment of the Marines does not violate the civilian be a violation of the civilian supremacy clause in the Constitution.
supremacy clause nor does it infringe the civilian character of the
police force. It is worth mentioning that military assistance to civilian authorities in
various forms persists in Philippine jurisdiction. The Philippine experience
reveals that it is not averse to requesting the assistance of the military in
Prescinding from its argument that no emergency situation exists to the implementation and execution of certain traditionally civil functions. As
justify the calling of the Marines, the IBP asserts that by the deployment of correctly pointed out by the Solicitor General, some of the multifarious
the Marines, the civilian task of law enforcement is militarized in violation activities wherein military aid has been rendered, exemplifying the
of Section 3, Article II[36] of the Constitution. activities that bring both the civilian and the military together in a
relationship of cooperation, are:
We disagree. The deployment of the Marines does not constitute a
breach of the civilian supremacy clause. The calling of the Marines in this 1. Elections;[42]
case constitutes permissible use of military assets for civilian law
enforcement. The participation of the Marines in the conduct of joint 2. Administration of the Philippine National Red Cross; [43]
visibility patrols is appropriately circumscribed. The limited participation of 3. Relief and rescue operations during calamities and disasters;
the Marines is evident in the provisions of the LOI itself, which sufficiently [44]

provides the metes and bounds of the Marines authority. It is noteworthy


that the local police forces are the ones in charge of the visibility patrols at 4. Amateur sports promotion and development; [45]
all times, the real authority belonging to the PNP. In fact, the Metro Manila
5. Development of the culture and the arts; [46] be fined not more than $10,000 or imprisoned not more than two years, or
both.[62]
6. Conservation of natural resources; [47]

7. Implementation of the agrarian reform program;[48] To determine whether there is a violation of the Posse Comitatus Act
in the use of military personnel, the US courts [63] apply the
8. Enforcement of customs laws; [49]
following standards, to wit:
9. Composite civilian-military law enforcement activities; [50]
Were Army or Air Force personnel used by the civilian law enforcement
10. Conduct of licensure examinations;[51] officers at Wounded Knee in such a manner that the military personnel
11. Conduct of nationwide tests for elementary and high school subjected the citizens to the exercise of military power which was
students;[52] regulatory, proscriptive, or compulsory[64] George Washington Law Review,
pp. 404-433 (1986), which discusses the four divergent standards for
12. Anti-drug enforcement activities;[53] assessing acceptable involvement of military personnel in civil law
enforcement. See likewise HONORED IN THE BREECH: PRESIDENTIAL
13. Sanitary inspections;[54] AUTHORITY TO EXECUTE THE LAWS WITH MILITARY FORCE, 83 Yale Law
14. Conduct of census work;[55] Journal, pp. 130-152, 1973. 64 in nature, either presently or
prospectively?
15. Administration of the Civil Aeronautics Board; [56]

16. Assistance in installation of weather forecasting devices; [57] xxx

17. Peace and order policy formulation in local government units. When this concept is transplanted into the present legal context, we take it
[58]
to mean that military involvement, even when not expressly authorized by
the Constitution or a statute, does not violate the Posse Comitatus Act
This unquestionably constitutes a gloss on executive power resulting
unless it actually regulates, forbids or compels some conduct on the part
from a systematic, unbroken, executive practice, long pursued to the
of those claiming relief. A mere threat of some future injury would be
knowledge of Congress and, yet, never before questioned. [59] What we
insufficient. (emphasis supplied)
have here is mutual support and cooperation between the military
and civilian authorities, not derogation of civilian supremacy.
Even if the Court were to apply the above rigid standards to the
In the United States, where a long tradition of suspicion and hostility present case to determine whether there is permissible use of the military
towards the use of military force for domestic purposes has persisted, in civilian law enforcement, the conclusion is inevitable that no violation of
[60]
and whose Constitution, unlike ours, does not expressly provide for the the civilian supremacy clause in the Constitution is committed. On this
power to call, the use of military personnel by civilian law enforcement point, the Court agrees with the observation of the Solicitor General:
officers is allowed under circumstances similar to those surrounding the
present deployment of the Philippine Marines. Under the Posse
3. The designation of tasks in Annex A[65] does not constitute the
Comitatus Act[61] of the US, the use of the military in civilian law
exercise of regulatory, proscriptive, or compulsory military
enforcement is generally prohibited, except in certain allowable
power. First, the soldiers do not control or direct the operation. This is
circumstances. A provision of the Act states:
evident from Nos. 6,[66] 8(k)[67] and 9(a)[68] of Annex A. These soldiers,
second, also have no power to prohibit or condemn. In No. 9(d)[69] of
1385. Use of Army and Air Force as posse comitatus Annex A, all arrested persons are brought to the nearest police
stations for proper disposition. And last, these soldiers apply no
Whoever, except in cases and under circumstances expressly authorized coercive force. The materials or equipment issued to them, as shown
by the Constitution or Act of Congress, willfully uses any part of the Army in No. 8(c)[70] of Annex A, are all low impact and defensive in
or the Air Force as posse comitatus or otherwise to execute the laws shall character. The conclusion is that there being no exercise of
regulatory, proscriptive or compulsory military power, the deployment
of a handful of Philippine Marines constitutes no impermissible use of scrutiny. If the attempt succeeded, it would have diminished the
military power for civilian law enforcement. [71] power of judicial review and weakened the checking authority of
this Court over the Chief Executive when he exercises his
It appears that the present petition is anchored on fear that once the commander-in-chief powers. The attempt should remind us of the
armed forces are deployed, the military will gain ascendancy, and thus tragedy that befell the country when this Court sought refuge in
place in peril our cherished liberties. Such apprehensions, however, are the political question doctrine and forfeited its most important role
unfounded. The power to call the armed forces is just that - calling out the as protector of the civil and political rights of our people. The
armed forces. Unless, petitioner IBP can show, which it has not, that in the ongoing conflict in Mindanao may worsen and can force the Chief
deployment of the Marines, the President has violated the fundamental Executive to resort to the use of his greater commander-in-chief
law, exceeded his authority or jeopardized the civil liberties of the people, powers, hence, this Court should be extra cautious in assaying
this Court is not inclined to overrule the Presidents determination of the similar attempts. A laid back posture may not sit well with our
factual basis for the calling of the Marines to prevent or suppress lawless people considering that the 1987 Constitution strengthened the
violence. checking powers of this Court and expanded its jurisdiction
precisely to stop any act constituting xxx grave abuse of
One last point. Since the institution of the joint visibility patrol in jurisdiction xxx on the part of any branch or instrumentality of the
January, 2000, not a single citizen has complained that his political or civil Government.1
rights have been violated as a result of the deployment of the Marines. It
was precisely to safeguard peace, tranquility and the civil liberties of the The importance of the issue at bar includes this humble separate
people that the joint visibility patrol was conceived. Freedom and opinion. We can best perceive the different intersecting dimensions of the
democracy will be in full bloom only when people feel secure in their political question doctrine by viewing them from the broader canvass of
homes and in the streets, not when the shadows of violence and anarchy history. Political questions are defined as those questions which under the
constantly lurk in their midst. Constitution, are to be decided by the people in their sovereign capacity,
or in regard to which full discretionary authority has been delegated to the
WHEREFORE, premises considered, the petition is hereby legislative or executive branch of government. 2 They have two aspects:
DISMISSED. (1) those matters that are to be exercised by the people in their primary
political capacity and (2) matters which have been specifically delegated to
SO ORDERED. some other department or particular office of the government, with
Davide, Jr., C.J., Melo, Purisima, Pardo, Buena, Gonzaga-Reyes, discretionary power to act. 3 The exercise of the discretionary power of the
Ynares-Santiago, and De Leon, Jr., JJ., concur. legislative or executive branch of government was often the area where
Bellosillo, J., on official leave. the Court had to wrestle with the political question doctrine. 4
Puno, J., see separate opinion. A brief review of some of our case law will thus give us a sharper
Vitug, J., see separate opinion. perspective of the political question doctrine. This question confronted the
Mendoza, J., see concurring and dissenting opinion. Court as early as 1905 in the case of Barcelon v. Baker.5 The Governor-
Panganiban, J., in the result. General of the Philippine Islands, pursuant to a resolution of the Philippine
Quisumbing, J., joins the opinion of J. Mendoza. Commission, suspended the privilege of the writ of habeas corpus in Cavite
and Batangas based on a finding of open insurrection in said
provinces. Felix Barcelon, who was detained by constabulary officers in
SEPARATE OPINION Batangas, filed a petition for the issuance of a writ of habeas corpus
alleging that there was no open insurrection in Batangas. The issue to
resolve was whether or not the judicial department may investigate the
PUNO, J.: facts upon which the legislative (the Philippine Commission) and executive
(the Governor-General) branches of government acted in suspending the
privilege of the writ.
If the case at bar is significant, it is because of the government
attempt to foist the political question doctrine to shield an executive The Court ruled that under our form of government, one department
act done in the exercise of the commander-in-chief powers from judicial has no authority to inquire into the acts of another, which acts are
performed within the discretion of the other department. 6Surveying In the 1947 case of Mabanag v. Lopez-Vito,14 three Senators and
American law and jurisprudence, it held that whenever a statute gives eight representatives who were proclaimed elected by Comelec were not
discretionary power to any person, to be exercised by him upon his own allowed by Congress to take part in the voting for the passage of the
opinion of certain facts, the statute constitutes him the sole judge of the Parity amendment to the Constitution. If their votes had been counted, the
existence of those facts.7 Since the Philippine Bill of 1902 empowered the affirmative votes in favor of the proposed amendment would have been
Philippine Commission and the Governor-General to suspend the privilege short of the necessary three-fourths vote in either House of Congress to
of the writ of habeas corpus, this power is exclusively within the pass the amendment. The amendment was eventually submitted to the
discretion of the legislative and executive branches of government. The people for ratification. The Court declined to intervene and held that a
exercise of this discretion is conclusive upon the courts. 8 proposal to amend the Constitution is a highly political function performed
by Congress in its sovereign legislative capacity. 15
The Court further held that once a determination is made by the
executive and legislative departments that the conditions justifying the In the 1955 case of Arnault v. Balagtas,16 petitioner, a private
assailed acts exists, it will presume that the conditions continue until the citizen, assailed the legality of his detention ordered by the Senate for his
same authority decide that they no longer exist. 9 It adopted refusal to answer questions put to him by members of one of its
the rationale that the executive branch, thru its civil and military investigating committees. This Court refused to order his release holding
branches, are better situated to obtain information about peace and that the process by which a contumacious witness is dealt with by the
order from every corner of the nation, in contrast with the judicial legislature is a necessary concomitant of the legislative process and the
department, with its very limited machinery. 10 The seed of the political legislature's exercise of its discretionary authority is not subject to judicial
question doctrine was thus planted in Philippine soil. interference.

The doctrine barring judicial review because of the political In the 1960 case of Osmena v. Pendatun,17 the Court followed the
question doctrine was next applied to the internal affairs of the traditional line. Congressman Sergio Osmena, Jr. was suspended by the
legislature. The Court refused to interfere in the legislative exercise of House of Representatives for serious disorderly behavior for making a
disciplinary power over its own members. In the 1924 case privilege speech imputing "malicious charges" against the President of the
of Alejandrino v. Quezon,11 Alejandrino, who was appointed Senator by Philippines. Osmena, Jr. invoked the power of review of this Court but the
the Governor-General, was declared by Senate Resolution as guilty of Court once more did not interfere with Congress' power to discipline its
disorderly conduct for assaulting another Senator in the course of a members.
debate, and was suspended from office for one year. Senator Alejandrino
filed a petition for mandamus and injunction to compel the Senate to The contours of the political question doctrine have always been
reinstate him. The Court held that under the Jones Law, the power of the tricky. To be sure, the Court did not always stay its hand whenever the
Senate to punish its members for disorderly behavior does not authorize it doctrine is invoked. In the 1949 case of Avelino v. Cuenco,18 Senate
to suspend an appointive member from the exercise of his office. While the President Jose Avelino, who was deposed and replaced, questioned his
Court found that the suspension was illegal, it refused to issue the writ of successor's title claiming that the latter had been elected without a
mandamus on the ground that "the Supreme Court does not possess the quorum. The petition was initially dismissed on the ground that the
power of coercion to make the Philippine Senate take any particular selection of Senate President was an internal matter and not subject to
action. [T]he Philippine Legislature or any branch thereof cannot be judicial review.19 On reconsideration, however, the Court ruled that it could
directly controlled in the exercise of their legislative powers by any judicial assume jurisdiction over the controversy in light of subsequent events
process."12 justifying intervention among which was the existence of a
quorum.20 Though the petition was ultimately dismissed, the Court
The issue revisited the Court twenty-two (22) years later. In 1946, declared respondent Cuenco as the legally elected Senate President.
in Vera v. Avelino,13 three senators-elect who had been prevented from
taking their oaths of office by a Senate resolution repaired to this Court to In the 1957 case of Tanada v. Cuenco,21 the Court assumed
compel their colleagues to allow them to occupy their seats contending jurisdiction over a dispute involving the formation and composition of the
that only the Electoral Tribunal had jurisdiction over contests relating to Senate Electoral Tribunal. It rejected the Solicitor General's claim that the
their election, returns and qualifications. Again, the Court refused to dispute involved a political question. Instead, it declared that the Senate is
intervene citing Alejandrino and affirmed the inherent right of the not clothed with "full discretionary authority" in the choice of members of
legislature to determine who shall be admitted to its membership. the Senate Electoral Tribunal and the exercise of its power thereon is
subject to constitutional limitations which are mandatory in nature. 22 It The Court explained that the power to amend the Constitution or to
held that under the Constitution, the membership of the Senate Electoral propose amendments thereto is not included in the general grant of
Tribunal was designed to insure the exercise of judicial impartiality in the legislative powers to Congress. As a constituent assembly, the members of
disposition of election contests affecting members of the lawmaking Congress derive their authority from the fundamental law and they do not
body.23 The Court then nullified the election to the Senate Electoral have the final say on whether their acts are within or beyond constitutional
Tribunal made by Senators belonging to the party having the largest limits.28 This ruling was reiterated in Tolentino which held that acts of a
number of votes of two of their party members but purporting to act on constitutional convention called for the purpose of proposing amendments
behalf of the party having the second highest number of votes. to the Constitution are at par with acts of Congress acting as a constituent
assembly.29
In the 1962 case of Cunanan v. Tan, Jr.,24 the Court passed
judgment on whether Congress had formed the Commission on In sum, this Court brushed aside the political question doctrine
Appointments in accordance with the Constitution and found that it did and assumed jurisdiction whenever it found constitutionally-
not. It declared that the Commission on Appointments is a creature of the imposed limits on the exercise of powers conferred upon the
Constitution and its power does not come from Congress but from the Legislature.30
Constitution.
The Court hewed to the same line as regards the exercise of
The 1967 case of Gonzales v. Comelec25 and the 1971 case Executive power. Thus, the respect accorded executive discretion was
of Tolentino v. Comelec26 abandoned Mabanag v. Lopez-Vito. The observed in Severino v. Governor-General,31 where it was held that the
question of whether or not Congress, acting as a constituent assembly in Governor-General, as head of the executive department, could not be
proposing amendments to the Constitution violates the Constitution was compelled by mandamus to call a special election in the town of Silay for
held to be a justiciable and not a political issue. In Gonzales, the Court the purpose of electing a municipal president. Mandamus and injunction
ruled: could not lie to enforce or restrain a duty which is discretionary. It was
held that when the Legislature conferred upon the Governor-General
"It is true that in Mabanag v. Lopez-Vito, this Court characterizing the powers and duties, it did so for the reason that he was in a better position
issue submitted thereto as a political one, declined to pass upon the to know the needs of the country than any other member of the executive
question whether or not a given number of votes cast in Congress in favor department, and with full confidence that he will perform such duties as
of a proposed amendment to the Constitution-which was being submitted his best judgment dictates.32
to the people for ratification-satisfied the three-fourths vote requirement Similarly, in Abueva v. Wood,33 the Court held that the Governor-
of the fundamental law. The force of this precedent has been weakened, General could not be compelled by mandamus to produce certain vouchers
however, by Suanes v. Chief Accountant of the Senate, Avelino v. Cuenco, showing the various expenditures of the Independence Commission. Under
Tanada v. Cuenco, and Macias v. Commission on Elections. In the first, we the principle of separation of powers, it ruled that it was not intended by
held that the officers and employees of the Senate Electoral Tribunal are the Constitution that one branch of government could encroach upon the
under its supervision and control, not of that of the Senate President, as field of duty of the other. Each department has an exclusive field within
claimed by the latter; in the second, this Court proceeded to determine the which it can perform its part within certain discretionary limits. 34 It
number of Senators necessary for a quorum in the Senate; in the third, we observed that "the executive and legislative departments of government
nullified the election, by Senators belonging to the party having the largest are frequently called upon to deal with what are known as political
number of votes in said chamber, purporting to act on behalf of the party questions, with which the judicial department of government has no
having the second largest number of votes therein, of two (2) Senators intervention. In all such questions, the courts uniformly refused to
belonging to the first party, as members, for the second party, of the intervene for the purpose of directing or controlling the actions of the
Senate Electoral Tribunal; and in the fourth, we declared unconstitutional other department; such questions being many times reserved to those
an act of Congress purporting to apportion the representative districts for departments in the organic law of the state." 35
the House of Representatives upon the ground that the apportionment had
not been made as may be possible according to the number of inhabitants In Forties v. Tiaco,36 the Court also refused to take cognizance of a
of each province. Thus, we rejected the theory, advanced in these four case enjoining the Chief Executive from deporting an obnoxious alien
cases, that the issues therein raised were political questions the whose continued presence in the Philippines was found by him to be
determination of which is beyond judicial review. 27 injurious to the public interest. It noted that sudden and unexpected
conditions may arise, growing out of the presence of untrustworthy aliens, or imminent danger thereof; and (2) when the public safety requires it, in
which demand immediate action. The President's inherent power to deport any of which events the same may be suspended wherever during such
undesirable aliens is universally denominated as political, and this power period the necessity for the suspension shall exist. The extent of the power
continues to exist for the preservation of the peace and domestic which may be inquired into by courts is defined by these limitations. 43
tranquility of the nation.37
On the vital issue of how the Court may inquire into the President's
In Manalang v. Quitoriano,38 the Court also declined to interfere in exercise of power, it ruled that the function of the Court is not to supplant
the exercise of the President's appointing power. It held that the but merely to check the Executive; to ascertain whether the President has
appointing power is the exclusive prerogative of the President, upon which gone beyond the constitutional limits of his jurisdiction, not to exercise the
no limitations may be imposed by Congress, except those resulting from power vested in him or to determine the wisdom of his act. Judicial inquiry
the need of securing concurrence of the Commission on Appointments and is confined to the question of whether the President did not act
from the exercise of the limited legislative power to prescribe qualifications arbitrarily.44 Using this yardstick, the Court found that the President did
to a given appointive office. not.

We now come to the exercise by the President of his powers as The emergency period of the 1970's flooded the Court with cases
Commander-in-Chief vis-a-vis the political question doctrine. In the which raised the political question defense. The issue divided the Court
1940's, this Court has held that as Commander-in-Chief of the Armed down the middle. Javellana v. Executive Secretary45showed that while
Forces, the President has the power to determine whether war, in the legal a majority of the Court held that the issue of whether or not the 1973
sense, still continues or has terminated. It ruled that it is within the Constitution had been ratified in accordance with the 1935 Constitution
province of the political department and not of the judicial department of was justiciable, a majority also ruled that the decisive issue of whether the
government to determine when war is at end.39 1973 Constitution had come into force and effect, with or without
constitutional ratification, was a political question. 46
In 1952, the Court decided the landmark case of Montenegro v.
Castaneda.40 President Quirino suspended the privilege of the writ of The validity of the declaration of martial law by then President Marcos
habeas corpus for persons detained or to be detained for crimes of was next litigated before the Court. In Aquino, Jr. v. Enrile,47 it upheld
sedition, insurrection or rebellion. The Court, citing Barcelon, declared the President's declaration of martial law. On whether the validity of the
that the authority to decide whether the exigency has arisen requiring the imposition of martial law was a political or justiciable question, the Court
suspension of the privilege belongs to the President and his decision was almost evenly divided. One-half embraced the political question
is final and conclusive on the courts.41 position and the other half subscribed to the justiciable position in
Lansang. Those adhering to the political question doctrine used different
Barcelon was the ruling case law until the 1971 case of Lansang v. methods of approach to it.48
Garcia came.42 Lansang reversed the previous cases and held that the
suspension of the privilege of the writ of habeas corpus was not a political In 1983, the Lansang ruling was weakened by the Court in Garcia-
question. According to the Court, the weight of Barcelon was diluted by Padilla v. Enrile.49 The petitioners therein were arrested and detained by
two factors: (1) it relied heavily on Martin v. Mott, which involved the the Philippine Constabulary by virtue of a Presidential Commitment Order
U.S. President's power to call out the militia which is a much broader (PCO). Petitioners sought the issuance of a writ of habeas corpus. The
power than suspension of the privilege of the writ; and (2) the privilege Court found that the PCO had the function of validating a person's
was suspended by the American Governor-General whose act, as detention for any of the offenses covered in Proclamation No. 2045 which
representative of the sovereign affecting the freedom of its subjects, could continued in force the suspension of the privilege of the writ of habeas
not be equated with that of the President of the Philippines dealing with corpus. It held that the issuance of the PCO by the President was not
the freedom of the sovereign Filipino people. subject to judicial inquiry.50 It went further by declaring that there was a
need to re-examine Lansang with a view to reverting to Barcelon and
The Court declared that the power to suspend the privilege of Montenegro. It observed that in times of war or national emergency, the
the writ of habeas corpus is neither absolute nor unqualified President must be given absolute control for the very life of the nation and
because the Constitution sets limits on the exercise of executive government is in great peril. The President, it intoned, is answerable only
discretion on the matter. These limits are: (1) that the privilege must to his conscience, the people, and God.51
not be suspended except only in cases of invasion, insurrection or rebellion
But barely six (6) days after Garcia-Padilla, the Court "Sec. 18. The President shall be the Commander-in-Chief of all
promulgated Morales, Jr. v. Enrile52 reiterating Lansang. It held that by armed forces of the Philippines and whenever it becomes
the power of judicial review, the Court must inquire into every phase and necessary, he may call out such armed forces to prevent or
aspect of a person's detention from the moment he was taken into custody suppress lawless violence, invasion or rebellion. In case of
up to the moment the court passes upon the merits of the petition. Only invasion or rebellion, when the public safety requires it, he may,
after such a scrutiny can the court satisfy itself that the due process clause for a period not exceeding sixty days, suspend the privilege of the
of the Constitution has been met.53 writ of habeas corpus or place the Philippines or any part thereof
under martial law. Within forty-eight hours from the proclamation of
It is now history that the improper reliance by the Court on the martial law or the suspension of the privilege of the writ of habeas corpus,
political question doctrine eroded the people's faith in its capacity the President shall submit a report in person or in writing to Congress. The
to check abuses committed by the then Executive in the exercise of Congress, voting jointly, by a vote of at least a majority of all its Members
his commander-in-chief powers, particularly violations against in regular or special session, may revoke such proclamation or suspension,
human rights. The refusal of courts to be pro-active in the exercise which revocation shall not be set aside by the President. Upon the
of its checking power drove the people to the streets to resort to initiative of the President, the Congress may, in the same manner, extend
extralegal remedies. They gave birth to EDSA. such proclamation or suspension for a period to be determined by
Two lessons were not lost to the members of the Constitutional Congress, if the invasion or rebellion shall persist and public safety
Commission that drafted the 1987 Constitution. The first was the need to requires it.
grant this Court the express power to review the exercise of the powers as
commander-in-chief by the President and deny it of any discretion to The Congress, if not in session, shall, within twenty-four hours following
decline its exercise. The second was the need to compel the Court to such proclamation or suspension, convene in accordance with its rules
be pro-active by expanding its jurisdiction and, thus, reject its laid back without need of a call.
stance against acts constituting grave abuse of discretion on the part of
any branch or instrumentality of government. Then Chief Justice Roberto The Supreme Court may review, in an appropriate proceeding filed
Concepcion, a member of the Constitutional Commission, worked for the by any citizen, the sufficiency of the factual basis of the
insertion of the second paragraph of Section 1, Article VIII in the draft proclamation of martial law or the suspension of the privilege of
Constitution,54 which reads: the writ or the extension thereof, and must promulgate its decision
thereon within thirty days from its filing.
"Sec. 1. x x x.
x x x."
Judicial power includes the duty of the courts of justice to settle actual
controversies involving rights which are legally demandable and It is clear from the foregoing that the President, as Commander-in-
enforceable, and to determine whether or not there has been a Chief of the armed forces of the Philippines, may call out the
grave abuse of discretion amounting to lack or excess of armed forces subject to two conditions: (1) whenever it becomes
jurisdiction on the part of any branch or instrumentality of the necessary; and (2) to prevent or suppress lawless violence,
Government." invasion or rebellion. Undeniably, these conditions lay down
the sine qua requirement for the exercise of the power and the
The language of the provision clearly gives the Court the power to strike objective sought to be attained by the exercise of the power. They
down acts amounting to grave abuse of discretion of both the legislative define the constitutional parameters of the calling out
and executive branches of government. power. Whether or not there is compliance with these parameters
is a justiciable issue and is not a political question.
We should interpret Section 18, Article VII of the 1987 Constitution in
light of our constitutional history. The provision states: I am not unaware that in the deliberations of the Constitutional
Commission, Commissioner Bernas opined that the President's exercise of
the "calling out power," unlike the suspension of the privilege of the writ of
habeas corpus and the declaration of martial law, is not a justiciable issue and misleading to push the political question doctrine too far, is apropos.
but a political question and therefore not subject to judicial review. It will not be complementary to the Court if it handcuffs itself to
helplessness when a grievously injured citizen seeks relief from a palpably
It must be borne in mind, however, that while a member's opinion unwarranted use of presidential or military power, especially when the
expressed on the floor of the Constitutional Convention is valuable, it is question at issue falls in the penumbra between the "political" and the
not necessarily expressive of the people's intent. 55 The proceedings of the "justiciable. "58
Convention are less conclusive on the proper construction of the
fundamental law than are legislative proceedings of the proper We should not water down the ruling that deciding whether a matter
construction of a statute, for in the latter case it is the intent of the has been committed by the Constitution to another branch of government,
legislature the courts seek, while in the former, courts seek to arrive at the or whether the action of that branch exceeds whatever authority has been
intent of the people through the discussions and deliberations of their committed, is a delicate exercise in constitutional interpretation, and is
representatives.56 The conventional wisdom is that the Constitution does a responsibility of the Court as ultimate interpreter of the
not derive its force from the convention which framed it, but from the fundamental law.59 When private justiciable rights are involved in a suit,
people who ratified it, the intent to be arrived at is that of the people. 57 the Court must not refuse to assume jurisdiction even though
questions of extreme political importance are necessarily involved. 60 Every
It is true that the third paragraph of Section 18, Article VII of officer under a constitutional government must act according to law and
the 1987 Constitution expressly gives the Court the power to subject to the controlling power of the people, acting through the courts,
review the sufficiency of the factual bases used by the President in as well as through the executive and legislative. One department is just as
the suspension of the privilege of the writ of habeas corpus and representative of the other, and the judiciary is the department which is
the declaration of martial law. It does not follow, however, that charged with the special duty of determining the limitations which the law
just because the same provision did not grant to this Court the places upon all official action. 61 This historic role of the Court is the
power to review the exercise of the calling out power by the foundation stone of a government of laws and not of men. 62
President, ergo, this Court cannot pass upon the validity of its
exercise. I join the Decision in its result.

Given the light of our constitutional history, this express grant


of power merely means that the Court cannot decline the exercise
of its power because of the political question doctrine as it did in
the past. In fine, the express grant simply stresses the mandatory
duty of this Court to check the exercise of the commander-in-chief
powers of the President. It eliminated the discretion of the Court
not to wield its power of review thru the use of the political
question doctrine.

It may be conceded that the calling out power may be a "lesser


power" compared to the power to suspend the privilege of the writ of
habeas corpus and the power to declare martial law. Even then, its
exercise cannot be left to the absolute discretion of the Chief Executive as
Commander-in-Chief of the armed forces, as its impact on the rights of our
people protected by the Constitution cannot be downgraded. We cannot
hold that acts of the commander-in-chief cannot be reviewed on the
ground that they have lesser impact on the civil and political rights of our
people. The exercise of the calling out power may be "benign" in the case
at bar but may not be so in future cases.

The counsel of Mr. Chief Justice Enrique M. Fernando, in his


Dissenting and Concurring Opinion in Lansang that it would be dangerous
Republic of the Philippines In this instance, the validity of the National Defense Law, under which the
SUPREME COURT accused were sentenced, is impugned on the ground that it is
Manila unconstitutional. Section 2, Article II of the Constitution of the Philippines
provides as follows:
EN BANC
G.R. No. L-45892 July 13, 1938 SEC. 2. The defense of the state is a prime duty of government,
THE PEOPLE OF THE PHILIPPINES, plaintiff-appellee, and in the fulfillment of this duty all citizens may be required by
vs. law to render personal military or civil service.
TRANQUILINO LAGMAN, defendant-appellant.
----------------------------- The National Defense Law, in so far as it establishes compulsory military
G.R. No. L-45893 July 13, 1938 service, does not go against this constitutional provision but is, on the
THE PEOPLE OF THE PHILIPPINES, plaintiff-appellee, contrary, in faithful compliance therewith. The duty of the Government to
vs. defend the State cannot be performed except through an army. To leave
PRIMITIVO DE SOSA, defendant-appellant. the organization of an army to the will of the citizens would be to make
this duty of the Government excusable should there be no sufficient men
Severino P. Izon for appellants. who volunteer to enlist therein.1ªvvphïl.nët
Office of the Solicitor-General Tuason for appellee.
In the United States the courts have held in a series of decisions that the
AVANCEÑA, J.: compulsory military service adopted by reason of the civil war and the
world war does not violate the Constitution, because the power to
In these two cases (G.R. Nos. L-45892 and 45893), the appellants establish it is derived from that granted to Congress to declare war and to
Tranquilino and Primitivo de Sosa are charged with a violation of section organize and maintain an army. This is so because the right of the
60 of Commonwealth Act No. 1, known as the National Defense Law. It is Government to require compulsory military service is a consequence of its
alleged that these two appellants, being Filipinos and having reached the duty to defend the State and is reciprocal with its duty to defend the life,
age of twenty years in 1936, willfully and unlawfully refused to register in liberty, and property of the citizen. In the case of Jacobson vs.
the military service between the 1st and 7th of April of said year, Massachusetts (197 U.S., 11; 25 Sup. Ct. Rep., 385), it was said that,
notwithstanding the fact that they had been required to do so. The without violating the Constitution, a person may be compelled by force, if
evidence shows that these two appellants were duly notified by the need be, against his will, against his pecuniary interests, and even against
corresponding authorities to appear before the Acceptance Board in order his religious or political convictions, to take his place in the ranks of the
to register for military service in accordance with law, and that the said army of his country, and risk the chance of being shot down in its defense.
appellants, in spite of these notices, had not registered up to the date of In the case of United States vs. Olson (253 Fed., 233), it was also said
the filing of the information. that this is not deprivation of property without due process of law,
because, in its just sense, there is no right of property to an office or
employment.
The appellants do not deny these facts, but they allege in defense that
they have not registered in the military service because Primitivo de Sosa
is fatherless and has a mother and a brother eight years old to support, The circumstance that these decisions refer to laws enacted by reason on
and Tranquilino Lagman also has a father to support, has no military the actual existence of war does not make our case any different,
learnings, and does not wish to kill or be killed. inasmuch as, in the last analysis, what justifies compulsory military service
is the defense of the State, whether actual or whether in preparation to
make it more effective, in case of need. The circumstance that the
Each of these appellants was sentenced by the Court of First Instance to
appellants have dependent families to support does not excuse them from
one month and one day of imprisonment, with the costs.
their duty to present themselves before the Acceptance Board because, if
such circumstance exists, they can ask for determent in complying with
their duty and, at all events, they can obtain the proper pecuniary On March 7, 2000 President Joseph E. Estrada signed into law Republic Act
allowance to attend to these family responsibilities (secs. 65 and 69 of (R.A.) 8762, also known as the Retail Trade Liberalization Act of 2000. It
Commonwealth Act No. 1). expressly repealed R.A. 1180, which absolutely prohibited foreign
nationals from engaging in the retail trade business. R.A. 8762 now allows
The appealed judgment rendered in these two cases is affirmed, with the them to do so under four categories:
costs to the appellants. So ordered.

Villa-Real, Imperial, Diaz, Laurel and Concepcion, JJ., concur.

Republic of the Philippines Category A Less than Exclusively for Filipino citizens and
SUPREME COURT US$2,500,000.00 corporations wholly owned by
Manila Filipino citizens.

Category B US$2,500,000.00 up but For the first two years of R.A. 8762’s
EN BANC less than effectivity, foreign ownership is
US$7,500,000.00 allowed up to 60%. After the two-
G.R. No. 143855 September 21, 2010 year period, 100% foreign equity
shall be allowed.
REPRESENTATIVES GERARDO S. ESPINA, ORLANDO FUA, JR.,
Category C US$7,500,000.00 or more May be wholly owned by foreigners.
PROSPERO AMATONG, ROBERT ACE S. BARBERS, RAUL M. Foreign investments for establishing
GONZALES, PROSPERO PICHAY, JUAN MIGUEL ZUBIRI and
a store in Categories B and C shall
FRANKLIN BAUTISTA,Petitioners, not be less than the equivalent in
vs.
Philippine Pesos of US$830,000.00.
HON. RONALDO ZAMORA, JR. (Executive Secretary), HON. MAR
ROXAS (Secretary of Trade and Industry), HON. FELIPE MEDALLA Category D US$250,000.00 per store May be wholly owned by foreigners.
(Secretary of National Economic and Development Authority), of foreign enterprises
GOV. RAFAEL BUENAVENTURA (Bangko Sentral ng Pilipinas) and specializing in high-end or
HON. LILIA BAUTISTA (Chairman, Securities and Exchange luxury products
Commission), Respondents.
R.A. 8762 also allows natural-born Filipino citizens, who had lost their
citizenship and now reside in the Philippines, to engage in the retail trade
DECISION business with the same rights as Filipino citizens.

ABAD, J.: On October 11, 2000 petitioners ***Magtanggol T. Gunigundo I, Michael


T. Defensor, Gerardo S. Espina, Benjamin S. Lim, Orlando Fua, Jr.,
This case calls upon the Court to exercise its power of judicial review and Prospero Amatong, Sergio Apostol, Robert Ace S. Barbers, Enrique Garcia,
determine the constitutionality of the Retail Trade Liberalization Act of Jr., Raul M. Gonzales, Jaime Jacob, Apolinario Lozada, Jr., Leonardo
2000, which has been assailed as in breach of the constitutional mandate Montemayor, Ma. Elena Palma-Gil, Prospero Pichay, Juan Miguel Zubiri and
for the development of a self-reliant and independent national economy Franklin Bautista, all members of the House of Representatives, filed the
effectively controlled by Filipinos. present petition, assailing the constitutionality of R.A. 8762 on the
following grounds:
The Facts and the Case
First, the law runs afoul of Sections 9, 19, and 20 of Article II of
the Constitution which enjoins the State to place the national
economy under the control of Filipinos to achieve equal distribution
of opportunities, promote industrialization and full employment, Fourth, the Constitution mandates the regulation but not the
and protect Filipino enterprise against unfair competition and trade prohibition of foreign investments. It directs Congress to reserve
policies. to Filipino citizens certain areas of investments upon the
recommendation of the NEDA and when the national interest so
Second, the implementation of R.A. 8762 would lead to alien dictates. But the Constitution leaves to the discretion of the
control of the retail trade, which taken together with alien Congress whether or not to make such reservation. It does not
dominance of other areas of business, would result in the loss of prohibit Congress from enacting laws allowing the entry of
effective Filipino control of the economy. foreigners into certain industries not reserved by the Constitution
to Filipino citizens.
Third, foreign retailers like Walmart and K-Mart would crush
Filipino retailers and sari-sari store vendors, destroy self- The Issues Presented
employment, and bring about more unemployment.
Simplified, the case presents two issues:
Fourth, the World Bank-International Monetary Fund had
improperly imposed the passage of R.A. 8762 on the government 1. Whether or not petitioner lawmakers have the legal standing to
as a condition for the release of certain loans. challenge the constitutionality of R.A. 8762; and

Fifth, there is a clear and present danger that the law would 2. Whether or not R.A. 8762 is unconstitutional.
promote monopolies or combinations in restraint of trade.
The Court’s Ruling
Respondents Executive Secretary Ronaldo Zamora, Jr., Trade and Industry
Secretary Mar Roxas, National Economic and Development Authority One. The long settled rule is that he who challenges the validity of a law
(NEDA) Secretary Felipe Medalla, Bangko Sentral ng Pilipinas Gov. Rafael must have a standing to do so.1 Legal standing or locus standi refers to the
Buenaventura, and Securities and Exchange Commission Chairman Lilia right of a party to come to a court of justice and make such a challenge.
Bautista countered that: More particularly, standing refers to his personal and substantial interest in
that he has suffered or will suffer direct injury as a result of the passage of
First, petitioners have no legal standing to file the petition. They that law.2 To put it another way, he must show that he has been or is
cannot invoke the fact that they are taxpayers since R.A. 8762 about to be denied some right or privilege to which he is lawfully entitled
does not involve the disbursement of public funds. Nor can they or that he is about to be subjected to some burdens or penalties by reason
invoke the fact that they are members of Congress since they of the law he complains of.3
made no claim that the law infringes on their right as legislators.
Here, there is no clear showing that the implementation of the Retail Trade
Second, the petition does not involve any justiciable controversy. Liberalization Act prejudices petitioners or inflicts damages on them, either
Petitioners of course claim that, as members of Congress, they as taxpayers4 or as legislators.5 Still the Court will resolve the question
represent the small retail vendors in their respective districts but they raise since the rule on standing can be relaxed for nontraditional
the petition does not allege that the subject law violates the rights plaintiffs like ordinary citizens, taxpayers, and legislators when as in this
of those vendors. case the public interest so requires or the matter is of transcendental
importance, of overarching significance to society, or of paramount public
Third, petitioners have failed to overcome the presumption of interest.6
constitutionality of R.A. 8762. Indeed, they could not specify how
the new law violates the constitutional provisions they cite. Two. Petitioners mainly argue that R.A. 8762 violates the mandate of the
Sections 9, 19, and 20 of Article II of the Constitution are not self- 1987 Constitution for the State to develop a self-reliant and independent
executing provisions that are judicially demandable. national economy effectively controlled by Filipinos. They invoke the
provisions of the Declaration of Principles and State Policies under Article Section 13. The State shall pursue a trade policy that serves the general
II of the 1987 Constitution, which read as follows: welfare and utilizes all forms and arrangements of exchange on the basis
of equality and reciprocity.
Section 9. The State shall promote a just and dynamic social order that
will ensure the prosperity and independence of the nation and free the But, as the Court explained in Tañada v. Angara,7 the provisions of Article
people from poverty through policies that provide adequate social services, II of the 1987 Constitution, the declarations of principles and state
promote full employment, a rising standard of living, and an improved policies, are not self-executing. Legislative failure to pursue such policies
quality of life for all. cannot give rise to a cause of action in the courts.

xxxx The Court further explained in Tañada that Article XII of the 1987
Constitution lays down the ideals of economic nationalism: (1) by
Section 19. The State shall develop a self-reliant and independent national expressing preference in favor of qualified Filipinos in the grant of rights,
economy effectively controlled by Filipinos. privileges and concessions covering the national economy and patrimony
and in the use of Filipino labor, domestic materials and locally-produced
goods; (2) by mandating the State to adopt measures that help make
Section 20. The State recognizes the indispensable role of the private
them competitive; and (3) by requiring the State to develop a self-reliant
sector, encourages private enterprise, and provides incentives to needed
and independent national economy effectively controlled by
investments.
Filipinos.8ten.lihpwal

Petitioners also invoke the provisions of the National Economy and


In other words, while Section 19, Article II of the 1987 Constitution
Patrimony under Article XII of the 1987 Constitution, which reads:
requires the development of a self-reliant and independent national
economy effectively controlled by Filipino entrepreneurs, it does not
Section 10. The Congress shall, upon recommendation of the economic impose a policy of Filipino monopoly of the economic environment. The
and planning agency, when the national interest dictates, reserve to objective is simply to prohibit foreign powers or interests from
citizens of the Philippines or to corporations or associations at least sixty maneuvering our economic policies and ensure that Filipinos are given
per centum of whose capital is owned by such citizens, or such higher preference in all areas of development.
percentage as Congress may prescribe, certain areas of investments. The
Congress shall enact measures that will encourage the formation and
Indeed, the 1987 Constitution takes into account the realities of the
operation of enterprises whose capital is wholly owned by Filipinos.
outside world as it requires the pursuit of a trade policy that serves the
general welfare and utilizes all forms and arrangements of exchange on
In the grant of rights, privileges, and concessions covering the national the basis of equality and reciprocity; and speaks of industries which are
economy and patrimony, the State shall give preference to qualified competitive in both domestic and foreign markets as well as of the
Filipinos. protection of Filipino enterprises against unfair foreign competition and
trade practices. Thus, while the Constitution mandates a bias in favor of
The State shall regulate and exercise authority over foreign investments Filipino goods, services, labor and enterprises, it also recognizes the need
within its national jurisdiction and in accordance with its national goals and for business exchange with the rest of the world on the bases of equality
priorities. and reciprocity and limits protection of Filipino enterprises only against
foreign competition and trade practices that are unfair. 9
xxxx
In other words, the 1987 Constitution does not rule out the entry of
Section 12. The State shall promote the preferential use of Filipino labor, foreign investments, goods, and services. While it does not encourage
domestic materials and locally produced goods, and adopt measures that their unlimited entry into the country, it does not prohibit them either. In
help make them competitive. fact, it allows an exchange on the basis of equality and reciprocity,
frowning only on foreign competition that is unfair. 10 The key, as in all
economies in the world, is to strike a balance between protecting local First, aliens can only engage in retail trade business subject to the
businesses and allowing the entry of foreign investments and categories above-enumerated; Second, only nationals from, or juridical
services.1avvphi1 entities formed or incorporated in countries which allow the entry of
Filipino retailers shall be allowed to engage in retail trade business;
More importantly, Section 10, Article XII of the 1987 Constitution gives and Third, qualified foreign retailers shall not be allowed to engage in
Congress the discretion to reserve to Filipinos certain areas of investments certain retailing activities outside their accredited stores through the use of
upon the recommendation of the NEDA and when the national interest mobile or rolling stores or carts, the use of sales representatives, door-to-
requires. Thus, Congress can determine what policy to pass and when to door selling, restaurants and sari-sari stores and such other similar
pass it depending on the economic exigencies. It can enact laws allowing retailing activities.
the entry of foreigners into certain industries not reserved by the
Constitution to Filipino citizens. In this case, Congress has decided to open In sum, petitioners have not shown how the retail trade liberalization has
certain areas of the retail trade business to foreign investments instead of prejudiced and can prejudice the local small and medium enterprises since
reserving them exclusively to Filipino citizens. The NEDA has not opposed its implementation about a decade ago.
such policy.
WHEREFORE, the Court DISMISSES the petition for lack of merit. No
The control and regulation of trade in the interest of the public welfare is costs.
of course an exercise of the police power of the State. A person’s right to
property, whether he is a Filipino citizen or foreign national, cannot be SO ORDERED.
taken from him without due process of law. In 1954, Congress enacted the
Retail Trade Nationalization Act or R.A. 1180 that restricts the retail
business to Filipino citizens. In denying the petition assailing the validity of
such Act for violation of the foreigner’s right to substantive due process of
law, the Supreme Court held that the law constituted a valid exercise of
police power.11 The State had an interest in preventing alien control of the
retail trade and R.A. 1180 was reasonably related to that purpose. That
law is not arbitrary.

Here, to the extent that R.A. 8762, the Retail Trade Liberalization Act,
lessens the restraint on the foreigners’ right to property or to engage in an
ordinarily lawful business, it cannot be said that the law amounts to a
denial of the Filipinos’ right to property and to due process of law. Filipinos
continue to have the right to engage in the kinds of retail business to
which the law in question has permitted the entry of foreign investors.

Certainly, it is not within the province of the Court to inquire into the
wisdom of R.A. 8762 save when it blatantly violates the Constitution. But
as the Court has said, there is no showing that the law has contravened
any constitutional mandate. The Court is not convinced that the
implementation of R.A. 8762 would eventually lead to alien control of the
retail trade business. Petitioners have not mustered any concrete and
strong argument to support its thesis. The law itself has provided strict
safeguards on foreign participation in that business. Thus –
Republic of the Philippines Park".2 On 12 July 1990, the group, led by their President Roque Fermo,
SUPREME COURT filed a letter-complaint (Pinag-samang Sinumpaang Salaysay) with the
Manila CHR against the petitioners, asking the late CHR Chairman Mary
Concepcion Bautista for a letter to be addressed to then Mayor Brigido
EN BANC Simon, Jr., of Quezon City to stop the demolition of the private
respondents' stalls, sari-sari stores, and carinderia along North EDSA. The
complaint was docketed as CHR Case No. 90-1580.3 On 23 July 1990, the
CHR issued an Order, directing the petitioners "to desist from demolishing
the stalls and shanties at North EDSA pending resolution of the
G.R. No. 100150 January 5, 1994 vendors/squatters' complaint before the Commission" and ordering said
petitioners to appear before the CHR.4
BRIGIDO R. SIMON, JR., CARLOS QUIMPO, CARLITO ABELARDO,
AND GENEROSO OCAMPO, petitioners, On the basis of the sworn statements submitted by the private
vs. respondents on 31 July 1990, as well as CHR's own ocular inspection, and
COMMISSION ON HUMAN RIGHTS, ROQUE FERMO, AND OTHERS AS convinced that on 28 July 1990 the petitioners carried out the demolition
JOHN DOES, respondents. of private respondents' stalls, sari-sari stores and carinderia,5 the CHR, in
its resolution of 1 August 1990, ordered the disbursement of financial
The City Attorney for petitioners. assistance of not more than P200,000.00 in favor of the private
respondents to purchase light housing materials and food under the
The Solicitor General for public respondent. Commission's supervision and again directed the petitioners to "desist
from further demolition, with the warning that violation of said order would
lead to a citation for contempt and arrest." 6

A motion to dismiss,7 dated 10 September 1990, questioned CHR's


VITUG, J.: jurisdiction. The motion also averred, among other things, that:

The extent of the authority and power of the Commission on Human Rights 1. this case came about due to the alleged violation by the
("CHR") is again placed into focus in this petition for prohibition, with (petitioners) of the Inter-Agency Memorandum of
prayer for a restraining order and preliminary injunction. The petitioners Agreement whereby Metro-Manila Mayors agreed on a
ask us to prohibit public respondent CHR from further hearing and moratorium in the demolition of the dwellings of poor
investigating CHR Case No. 90-1580, entitled "Fermo, et al. vs. Quimpo, et dwellers in Metro-Manila;
al."
xxx xxx xxx
The case all started when a "Demolition Notice," dated 9 July 1990, signed
by Carlos Quimpo (one of the petitioners) in his capacity as an Executive
Officer of the Quezon City Integrated Hawkers Management Council under 3. . . . , a perusal of the said Agreement (revealed) that
the Office of the City Mayor, was sent to, and received by, the private the moratorium referred to therein refers to moratorium in
respondents (being the officers and members of the North EDSA Vendors the demolition of the structures of poor dwellers;
Association, Incorporated). In said notice, the respondents were given a
grace-period of three (3) days (up to 12 July 1990) within which to vacate 4. that the complainants in this case (were) not poor
the questioned premises of North EDSA.1Prior to their receipt of the dwellers but independent business entrepreneurs even this
demolition notice, the private respondents were informed by petitioner Honorable Office admitted in its resolution of 1 August
Quimpo that their stalls should be removed to give way to the "People's 1990 that the complainants are indeed, vendors;
5. that the complainants (were) occupying government The CHR opined that "it was not the intention of the (Constitutional)
land, particularly the sidewalk of EDSA corner North Commission to create only a paper tiger limited only to investigating civil
Avenue, Quezon City; . . . and and political rights, but it (should) be (considered) a quasi-judicial body
with the power to provide appropriate legal measures for the protection of
6. that the City Mayor of Quezon City (had) the sole and human rights of all persons within the Philippines . . . ." It added:
exclusive discretion and authority whether or not a certain
business establishment (should) be allowed to operate The right to earn a living is a right essential to one's right
within the jurisdiction of Quezon City, to revoke or cancel a to development, to life and to dignity. All these brazenly
permit, if already issued, upon grounds clearly specified by and violently ignored and trampled upon by respondents
law and ordinance.8 with little regard at the same time for the basic rights of
women and children, and their health, safety and welfare.
During the 12 September 1990 hearing, the petitioners moved for Their actions have psychologically scarred and traumatized
postponement, arguing that the motion to dismiss set for 21 September the children, who were witness and exposed to such a
1990 had yet to be resolved. The petitioners likewise manifested that they violent demonstration of Man's inhumanity to man.
would bring the case to the courts.
In an Order,14 dated 25 April 1991, petitioners' motion for reconsideration
On 18 September 1990 a supplemental motion to dismiss was filed by the was denied.
petitioners, stating that the Commission's authority should be understood
as being confined only to the investigation of violations of civil and political Hence, this recourse.
rights, and that "the rights allegedly violated in this case (were) not civil
and political rights, (but) their privilege to engage in business." 9 The petition was initially dismissed in our resolution 15 of 25 June 1991; it
was subsequently reinstated, however, in our resolution 16 of 18 June 1991,
On 21 September 1990, the motion to dismiss was heard and submitted in which we also issued a temporary restraining order, directing the CHR
for resolution, along with the contempt charge that had meantime been to "CEASE and DESIST from further hearing CHR No. 90-1580." 17
filed by the private respondents, albeit vigorously objected to by
petitioners (on the ground that the motion to dismiss was still then The petitioners pose the following:
unresolved).10
Whether or not the public respondent has jurisdiction:
In an Order,11 dated 25 September 1990, the CHR cited the petitioners in
contempt for carrying out the demolition of the stalls, sari-sari stores
a) to investigate the alleged violations of the "business rights" of the
and carinderia despite the "order to desist", and it imposed a fine of
private respondents whose stalls were demolished by the petitioners at the
P500.00 on each of them.
instance and authority given by the Mayor of Quezon City;

On 1 March 1991,12 the CHR issued an Order, denying petitioners' motion


b) to impose the fine of P500.00 each on the petitioners; and
to dismiss and supplemental motion to dismiss, in this wise:

c) to disburse the amount of P200,000.00 as financial aid to the vendors


Clearly, the Commission on Human Rights under its
affected by the demolition.
constitutional mandate had jurisdiction over the complaint
filed by the squatters-vendors who complained of the gross
violations of their human and constitutional rights. The In the Court's resolution of 10 October 1991, the Solicitor-General was
motion to dismiss should be and is hereby DENIED for lack excused from filing his comment for public respondent CHR. The latter
of merit.13 thus filed its own comment,18 through Hon. Samuel Soriano, one of its
Commissioners. The Court also resolved to dispense with the comment of
private respondent Roque Fermo, who had since failed to comply with the (8) Grant immunity from prosecution to any person whose
resolution, dated 18 July 1991, requiring such comment. testimony or whose possession of documents or other
evidence is necessary or convenient to determine the truth
The petition has merit. in any investigation conducted by it or under its authority;

The Commission on Human Rights was created by the 1987 (9) Request the assistance of any department, bureau,
Constitution.19 It was formally constituted by then President Corazon office, or agency in the performance of its functions;
Aquino via Executive Order No. 163,20 issued on 5 May 1987, in the
exercise of her legislative power at the time. It succeeded, but so (10) Appoint its officers and employees in accordance with
superseded as well, the Presidential Committee on Human Rights. 21 law; and

The powers and functions22 of the Commission are defined by the 1987 (11) Perform such other duties and functions as may be
Constitution, thus: to — provided by law.

(1) Investigate, on its own or on complaint by any party, In its Order of 1 March 1991, denying petitioners' motion to dismiss, the
all forms of human rights violations involving civil and CHR theorizes that the intention of the members of the Constitutional
political rights; Commission is to make CHR a quasi-judicial body. 23 This view, however,
has not heretofore been shared by this Court. In Cariño v. Commission on
(2) Adopt its operational guidelines and rules of procedure, Human Rights,24 the Court, through then Associate Justice, now Chief
and cite for contempt for violations thereof in accordance Justice Andres Narvasa, has observed that it is "only the first of the
with the Rules of Court; enumerated powers and functions that bears any resemblance to
adjudication or adjudgment," but that resemblance can in no way be
synonymous to the adjudicatory power itself. The Court explained:
(3) Provide appropriate legal measures for the protection
of human rights of all persons within the Philippines, as
well as Filipinos residing abroad, and provide for . . . (T)he Commission on Human Rights . . . was not
preventive measures and legal aid services to the meant by the fundamental law to be another court or
underprivileged whose human rights have been violated or quasi-judicial agency in this country, or duplicate much
need protection; less take over the functions of the latter.

(4) Exercise visitorial powers over jails, prisons, or The most that may be conceded to the Commission in the
detention facilities; way of adjudicative power is that it may investigate, i.e.,
receive evidence and make findings of fact as regards
claimed human rights violations involving civil and political
(5) Establish a continuing program of research, education,
rights. But fact finding is not adjudication, and cannot be
and information to enhance respect for the primacy of
likened to the judicial function of a court of justice, or even
human rights;
a quasi-judicial agency or official. The function of receiving
evidence and ascertaining therefrom the facts of a
(6) Recommend to the Congress effective measures to controversy is not a judicial function, properly speaking. To
promote human rights and to provide for compensation to be considered such, the faculty of receiving evidence and
victims of violations of human rights, or their families; making factual conclusions in a controversy must be
accompanied by the authority of applying the law to those
(7) Monitor the Philippine Government's compliance with factual conclusions to the end that the controversy may be
international treaty obligations on human rights; decided or determined authoritatively, finally and
definitively, subject to such appeals or modes of review as
may be provided by law. This function, to repeat, the The Universal Declaration of Human Rights, as well as, or more
Commission does not have. specifically, the International Covenant on Economic, Social and Cultural
Rights and International Covenant on Civil and Political Rights, suggests
After thus laying down at the outset the above rule, we now proceed to that the scope of human rights can be understood to include those that
the other kernel of this controversy and, its is, to determine the extent of relate to an individual's social, economic, cultural, political and civil
CHR's investigative power. relations. It thus seems to closely identify the term to the universally
accepted traits and attributes of an individual, along with what is generally
considered to be his inherent and inalienable rights, encompassing almost
It can hardly be disputed that the phrase "human rights" is so generic a
all aspects of life.
term that any attempt to define it, albeit not a few have tried, could at
best be described as inconclusive. Let us observe. In a symposium on
human rights in the Philippines, sponsored by the University of the Have these broad concepts been equally contemplated by the framers of
Philippines in 1977, one of the questions that has been propounded is our 1986 Constitutional Commission in adopting the specific provisions on
"(w)hat do you understand by "human rights?" The participants, human rights and in creating an independent commission to safeguard
representing different sectors of the society, have given the following these rights? It may of value to look back at the country's experience
varied answers: under the martial law regime which may have, in fact, impelled the
inclusions of those provisions in our fundamental law. Many voices have
been heard. Among those voices, aptly represented perhaps of the
Human rights are the basic rights which inhere in man by
sentiments expressed by others, comes from Mr. Justice J.B.L. Reyes, a
virtue of his humanity. They are the same in all parts of
respected jurist and an advocate of civil liberties, who, in his paper,
the world, whether the Philippines or England, Kenya or
entitled "Present State of Human Rights in the Philippines," 29 observes:
the Soviet Union, the United States or Japan, Kenya or
Indonesia . . . .
But while the Constitution of 1935 and that of 1973
enshrined in their Bill of Rights most of the human rights
Human rights include civil rights, such as the right to life,
expressed in the International Covenant, these rights
liberty, and property; freedom of speech, of the press, of
became unavailable upon the proclamation of Martial Law
religion, academic freedom, and the rights of the accused
on 21 September 1972. Arbitrary action then became the
to due process of law; political rights, such as the right to
rule. Individuals by the thousands became subject to
elect public officials, to be elected to public office, and to
arrest upon suspicion, and were detained and held for
form political associations and engage in politics; and
indefinite periods, sometimes for years, without charges,
social rights, such as the right to an education,
until ordered released by the Commander-in-Chief or this
employment, and social services.25
representative. The right to petition for the redress of
grievances became useless, since group actions were
Human rights are the entitlement that inhere in the forbidden. So were strikes. Press and other mass media
individual person from the sheer fact of his humanity. . . . were subjected to censorship and short term licensing.
Because they are inherent, human rights are not granted Martial law brought with it the suspension of the writ of
by the State but can only be recognized and protected by habeas corpus, and judges lost independence and security
it.26 of tenure, except members of the Supreme Court. They
were required to submit letters of resignation and were
(Human rights include all) the civil, political, economic, dismissed upon the acceptance thereof. Torture to extort
social, and cultural rights defined in the Universal confessions were practiced as declared by international
Declaration of Human Rights.27 bodies like Amnesty International and the International
Commission of Jurists.
Human rights are rights that pertain to man simply
because he is human. They are part of his natural birth,
right, innate and inalienable.28
Converging our attention to the records of the Constitutional Commission, MR. GARCIA. Yes, because the other rights will encompass
we can see the following discussions during its 26 August 1986 social and economic rights, and there are other violations
deliberations: of rights of citizens which can be addressed to the proper
courts and authorities.
MR. GARCIA . . . , the primacy of its (CHR) task must be
made clear in view of the importance of human rights and xxx xxx xxx
also because civil and political rights have been
determined by many international covenants and human MR. BENGZON. So, we will authorize the commission to
rights legislations in the Philippines, as well as the define its functions, and, therefore, in doing that the
Constitution, specifically the Bill of Rights and subsequent commission will be authorized to take under its wings
legislation. Otherwise, if we cover such a wide territory in cases which perhaps heretofore or at this moment are
area, we might diffuse its impact and the precise nature of under the jurisdiction of the ordinary investigative and
its task, hence, its effectivity would also be curtailed. prosecutorial agencies of the government. Am I correct?

So, it is important to delienate the parameters of its tasks MR. GARCIA. No. We have already mentioned earlier that
so that the commission can be most effective. we would like to define the specific parameters which
cover civil and political rights as covered by the
MR. BENGZON. That is precisely my difficulty because civil international standards governing the behavior of
and political rights are very broad. The Article on the Bill of governments regarding the particular political and civil
Rights covers civil and political rights. Every single right of rights of citizens, especially of political detainees or
an individual involves his civil right or his political right. prisoners. This particular aspect we have experienced
So, where do we draw the line? during martial law which we would now like to safeguard.

MR. GARCIA. Actually, these civil and political rights have MR. BENGZON. Then, I go back to that question that I had.
been made clear in the language of human rights Therefore, what we are really trying to say is, perhaps, at
advocates, as well as in the Universal Declaration of the proper time we could specify all those rights stated in
Human Rights which addresses a number of articles on the the Universal Declaration of Human Rights and defined as
right to life, the right against torture, the right to fair and human rights. Those are the rights that we envision here?
public hearing, and so on. These are very specific rights
that are considered enshrined in many international MR. GARCIA. Yes. In fact, they are also enshrined in the
documents and legal instruments as constituting civil and Bill of Rights of our Constitution. They are integral parts of
political rights, and these are precisely what we want to that.
defend here.
MR. BENGZON. Therefore, is the Gentleman saying that all
MR. BENGZON. So, would the commissioner say civil and the rights under the Bill of Rights covered by human
political rights as defined in the Universal Declaration of rights?
Human Rights?
MR. GARCIA. No, only those that pertain to civil and
MR. GARCIA. Yes, and as I have mentioned, the political rights.
International Covenant of Civil and Political Rights
distinguished this right against torture.
xxx xxx xxx

MR. BENGZON. So as to distinguish this from the other


rights that we have?
MR. RAMA. In connection with the discussion on the scope Human Rights of 1948, mentioned or linked the concept of
of human rights, I would like to state that in the past human right with other human rights specified in other
regime, everytime we invoke the violation of human convention which I do not remember. Am I correct?
rights, the Marcos regime came out with the defense that,
as a matter of fact, they had defended the rights of people MR. GARCIA. Is Commissioner Guingona referring to the
to decent living, food, decent housing and a life consistent Declaration of Torture of 1985?
with human dignity.
MR. GUINGONA. I do not know, but the commissioner
So, I think we should really limit the definition of human mentioned another.
rights to political rights. Is that the sense of the
committee, so as not to confuse the issue?
MR. GARCIA. Madam President, the other one is the
International Convention on Civil and Political Rights of
MR. SARMIENTO. Yes, Madam President. which we are signatory.

MR. GARCIA. I would like to continue and respond also to MR. GUINGONA. I see. The only problem is that, although
repeated points raised by the previous speaker. I have a copy of the Universal Declaration of Human Rights
here, I do not have a copy of the other covenant
There are actually six areas where this Commission on mentioned. It is quite possible that there are rights
Human Rights could act effectively: 1) protection of rights specified in that other convention which may not be
of political detainees; 2) treatment of prisoners and the specified here. I was wondering whether it would be wise
prevention of tortures; 3) fair and public trials; 4) cases of to link our concept of human rights to general terms like
disappearances; 5) salvagings and hamletting; and 6) "convention," rather than specify the rights contained in
other crimes committed against the religious. the convention.

xxx xxx xxx As far as the Universal Declaration of Human Rights is


concerned, the Committee, before the period of
The PRESIDENT. Commissioner Guingona is recognized. amendments, could specify to us which of these articles in
the Declaration will fall within the concept of civil and
political rights, not for the purpose of including these in the
MR. GUINGONA. Thank You Madam President.
proposed constitutional article, but to give the sense of the
Commission as to what human rights would be included,
I would like to start by saying that I agree with without prejudice to expansion later on, if the need arises.
Commissioner Garcia that we should, in order to make the For example, there was no definite reply to the question of
proposed Commission more effective, delimit as much as Commissioner Regalado as to whether the right to marry
possible, without prejudice to future expansion. The would be considered a civil or a social right. It is not a civil
coverage of the concept and jurisdictional area of the right?
term "human rights". I was actually disturbed this morning
when the reference was made without qualification to the
MR. GARCIA. Madam President, I have to repeat the
rights embodied in the universal Declaration of Human
various specific civil and political rights that we felt must
Rights, although later on, this was qualified to refer to civil
be envisioned initially by this provision — freedom from
and political rights contained therein.
political detention and arrest prevention of torture, right to
fair and public trials, as well as crimes involving
If I remember correctly, Madam President, Commissioner disappearance, salvagings, hamlettings and collective
Garcia, after mentioning the Universal Declaration of violations. So, it is limited to politically related crimes
precisely to protect the civil and political rights of a specific SR. TAN. Madam President, from the standpoint of the
group of individuals, and therefore, we are not opening it victims of human rights, I cannot stress more on how
up to all of the definite areas. much we need a Commission on Human Rights. . . .

MR. GUINGONA. Correct. Therefore, just for the record, . . . human rights victims are usually penniless. They
the Gentlemen is no longer linking his concept or the cannot pay and very few lawyers will accept clients who do
concept of the Committee on Human Rights with the so- not pay. And so, they are the ones more abused and
called civil or political rights as contained in the Universal oppressed. Another reason is, the cases involved are very
Declaration of Human Rights. delicate — torture, salvaging, picking up without any
warrant of arrest, massacre — and the persons who are
MR. GARCIA. When I mentioned earlier the Universal allegedly guilty are people in power like politicians, men in
Declaration of Human Rights, I was referring to an the military and big shots. Therefore, this Human Rights
international instrument. Commission must be independent.

MR. GUINGONA. I know. I would like very much to emphasize how much we need
this commission, especially for the little Filipino, the little
individual who needs this kind of help and cannot get
MR. GARCIA. But it does not mean that we will refer to
it. And I think we should concentrate only on civil and
each and every specific article therein, but only to those
political violations because if we open this to land, housing
that pertain to the civil and politically related, as we
and health, we will have no place to go again and we will
understand it in this Commission on Human Rights.
not receive any response. . . .30 (emphasis supplied)

MR. GUINGONA. Madam President, I am not even clear as


The final outcome, now written as Section 18, Article XIII, of the 1987
to the distinction between civil and social rights.
Constitution, is a provision empowering the Commission on Human Rights
to "investigate, on its own or on complaint by any party, all forms of
MR. GARCIA. There are two international covenants: the human rights violations involving civil and political rights" (Sec. 1).
International Covenant and Civil and Political Rights and
the International Covenant on Economic, Social and
The term "civil rights,"31 has been defined as referring —
Cultural Rights. The second covenant contains all the
different rights-the rights of labor to organize, the right to
education, housing, shelter, et cetera. (t)o those (rights) that belong to every citizen of the state
or country, or, in wider sense, to all its inhabitants, and
are not connected with the organization or administration
MR. GUINGONA. So we are just limiting at the moment the
of the government. They include the rights of property,
sense of the committee to those that the Gentlemen has
marriage, equal protection of the laws, freedom of
specified.
contract, etc. Or, as otherwise defined civil rights are
rights appertaining to a person by virtue of his citizenship
MR. GARCIA. Yes, to civil and political rights. in a state or community. Such term may also refer, in its
general sense, to rights capable of being enforced or
MR. GUINGONA. Thank you. redressed in a civil action.

xxx xxx xxx Also quite often mentioned are the guarantees against involuntary
servitude, religious persecution, unreasonable searches and seizures, and
imprisonment for debt.32
Political rights,33 on the other hand, are said to refer to the right to provided for in the Rules of Court." That power to cite for contempt,
participate, directly or indirectly, in the establishment or administration of however, should be understood to apply only to violations of its adopted
government, the right of suffrage, the right to hold public office, the right operational guidelines and rules of procedure essential to carry out its
of petition and, in general, the rights appurtenant to citizenship vis-a- investigatorial powers. To exemplify, the power to cite for contempt could
vis the management of government.34 be exercised against persons who refuse to cooperate with the said body,
or who unduly withhold relevant information, or who decline to honor
Recalling the deliberations of the Constitutional Commission, aforequoted, summons, and the like, in pursuing its investigative work. The "order to
it is readily apparent that the delegates envisioned a Commission on desist" (a semantic interplay for a restraining order) in the instance before
Human Rights that would focus its attention to the more severe cases of us, however, is not investigatorial in character but prescinds from an
human rights violations. Delegate Garcia, for instance, mentioned such adjudicative power that it does not possess. In Export Processing Zone
areas as the "(1) protection of rights of political detainees, (2) treatment Authority vs. Commission on Human Rights,36 the Court, speaking through
of prisoners and the prevention of tortures, (3) fair and public trials, (4) Madame Justice Carolina Griño-Aquino, explained:
cases of disappearances, (5) salvagings and hamletting, and (6) other
crimes committed against the religious." While the enumeration has not The constitutional provision directing the CHR to "provide
likely been meant to have any preclusive effect, more than just expressing for preventive measures and legal aid services to the
a statement of priority, it is, nonetheless, significant for the tone it has underprivileged whose human rights have been violated or
set. In any event, the delegates did not apparently take comfort in need protection" may not be construed to confer
peremptorily making a conclusive delineation of the CHR's scope of jurisdiction on the Commission to issue a restraining order
investigatorial jurisdiction. They have thus seen it fit to resolve, instead, or writ of injunction for, it that were the intention, the
that "Congress may provide for other cases of violations of human rights Constitution would have expressly said so. "Jurisdiction is
that should fall within the authority of the Commission, taking into account conferred only by the Constitution or by law". It is never
its recommendation."35 derived by implication.

In the particular case at hand, there is no cavil that what are sought to be Evidently, the "preventive measures and legal aid services"
demolished are the stalls, sari-sari stores and carinderia, as well as mentioned in the Constitution refer to extrajudicial and
temporary shanties, erected by private respondents on a land which is judicial remedies (including a writ of preliminary
planned to be developed into a "People's Park". More than that, the land injunction) which the CHR may seek from proper courts on
adjoins the North EDSA of Quezon City which, this Court can take judicial behalf of the victims of human rights violations. Not being
notice of, is a busy national highway. The consequent danger to life and a court of justice, the CHR itself has no jurisdiction to issue
limb is not thus to be likewise simply ignored. It is indeed paradoxical that the writ, for a writ of preliminary injunction may only be
a right which is claimed to have been violated is one that cannot, in the issued "by the judge of any court in which the action is
first place, even be invoked, if it is, in fact, extant. Be that as it may, pending [within his district], or by a Justice of the Court of
looking at the standards hereinabove discoursed vis-a-vis the Appeals, or of the Supreme Court. . . . A writ of
circumstances obtaining in this instance, we are not prepared to conclude preliminary injunction is an ancillary remedy. It is available
that the order for the demolition of the stalls, sari-sari stores only in a pending principal action, for the preservation or
and carinderia of the private respondents can fall within the compartment protection of the rights and interests of a party thereto,
of "human rights violations involving civil and political rights" intended by and for no other purpose." (footnotes omitted).
the Constitution.
The Commission does have legal standing to indorse, for appropriate
On its contempt powers, the CHR is constitutionally authorized to "adopt action, its findings and recommendations to any appropriate agency of
its operational guidelines and rules of procedure, and cite for contempt for government.37
violations thereof in accordance with the Rules of Court." Accordingly, the
CHR acted within its authority in providing in its revised rules, its power The challenge on the CHR's disbursement of the amount of P200,000.00
"to cite or hold any person in direct or indirect contempt, and to impose by way of financial aid to the vendors affected by the demolition is not an
the appropriate penalties in accordance with the procedure and sanctions
appropriate issue in the instant petition. Not only is there lack of locus
standi on the part of the petitioners to question the disbursement but,
more importantly, the matter lies with the appropriate administrative
agencies concerned to initially consider.

The public respondent explains that this petition for prohibition filed by the
petitioners has become moot and academic since the case before it (CHR
Case No. 90-1580) has already been fully heard, and that the matter is
merely awaiting final resolution. It is true that prohibition is a preventive
remedy to restrain the doing of an act about to be done, and not intended
to provide a remedy for an act already accomplished. 38 Here, however,
said Commission admittedly has yet to promulgate its resolution in CHR
Case No. 90-1580. The instant petition has been intended, among other
things, to also prevent CHR from precisely doing that. 39

WHEREFORE, the writ prayed for in this petition is GRANTED. The


Commission on Human Rights is hereby prohibited from further proceeding
with CHR Case No. 90-1580 and from implementing the P500.00 fine for
contempt. The temporary restraining order heretofore issued by this Court
is made permanent. No costs.

SO ORDERED.

Narvasa, C.J., Cruz, Feliciano, Bidin, Regalado, Davide, Jr., Romero,


Nocon, Bellosillo, Melo, Quiason and Puno, JJ., concur.
SECOND DIVISION It now appears that private respondent had made the same
representation in the two successive reliever agreements which she signed
on June 10, 1991 and July 8, 1991. When petitioner supposedly learned
about the same later, its branch supervisor in Baguio City, Delia M. Oficial,
[G.R. No. 118978. May 23, 1997] sent to private respondent a memorandum dated January 15, 1992
requiring her to explain the discrepancy. In that memorandum, she was
reminded about the companys policy of not accepting married women for
employment.[4]
PHILIPPINE TELEGRAPH AND TELEPHONE COMPANY, * petitioner, In her reply letter dated January 17, 1992, private respondent stated
vs. NATIONAL LABOR RELATIONS COMMISSION and GRACE that she was not aware of PT&Ts policy regarding married women at the
DE GUZMAN, respondents. time, and that all along she had not deliberately hidden her true civil
status.[5] Petitioner nonetheless remained unconvinced by her
DECISION explanations. Private respondent was dismissed from the company
effective January 29, 1992,[6] which she readily contested by initiating a
REGALADO, J.: complaint for illegal dismissal, coupled with a claim for non-payment of
cost of living allowances (COLA), before the Regional Arbitration Branch of
Seeking relief through the extraordinary writ of certiorari, petitioner the National Labor Relations Commission in Baguio City.
Philippine Telegraph and Telephone Company (hereafter, PT&T) invokes
At the preliminary conference conducted in connection therewith,
the alleged concealment of civil status and defalcation of company funds
private respondent volunteered the information, and this was incorporated
as grounds to terminate the services of an employee. That employee,
in the stipulation of facts between the parties, that she had failed to remit
herein private respondent Grace de Guzman, contrarily argues that what
the amount of P2,380.75 of her collections. She then executed a
really motivated PT&T to terminate her services was her having contracted
promissory note for that amount in favor of petitioner. [7] All of these took
marriage during her employment, which is prohibited by petitioner in its
place in a formal proceeding and with the agreement of the parties and/or
company policies. She thus claims that she was discriminated against in
their counsel.
gross violation of law, such a proscription by an employer being outlawed
by Article 136 of the Labor Code. On November 23, 1993, Labor Arbiter Irenarco R. Rimando handed
down a decision declaring that private respondent, who had already gained
Grace de Guzman was initially hired by petitioner as a reliever,
the status of a regular employee, was illegally dismissed by petitioner. Her
specifically as a Supernumerary Project Worker, for a fixed period from
reinstatement, plus payment of the corresponding back wages and COLA,
November 21, 1990 until April 20, 1991 vice one C.F. Tenorio who went on
was correspondingly ordered, the labor arbiter being of the firmly
maternity leave.[1] Under the Reliever Agreement which she signed with
expressed view that the ground relied upon by petitioner in dismissing
petitioner company, her employment was to be immediately terminated
private respondent was clearly insufficient, and that it was apparent that
upon expiration of the agreed period. Thereafter, from June 10, 1991 to
she had been discriminated against on account of her having contracted
July 1, 1991, and from July 19, 1991 to August 8, 1991, private
marriage in violation of company rules.
respondents services as reliever were again engaged by petitioner, this
time in replacement of one Erlinda F. Dizon who went on leave during both On appeal to the National Labor Relations Commission (NLRC), said
periods.[2] After August 8, 1991, and pursuant to their Reliever Agreement, public respondent upheld the labor arbiter and, in its decision dated April
her services were terminated. 29, 1994, it ruled that private respondent had indeed been the subject of
an unjust and unlawful discrimination by her employer, PT&T. However,
On September 2, 1991, private respondent was once more asked to
the decision of the labor arbiter was modified with the qualification that
join petitioner company as a probationary employee, the probationary
Grace de Guzman deserved to be suspended for three months in view of
period to cover 150 days. In the job application form that was furnished
the dishonest nature of her acts which should not be condoned. In all
her to be filled up for the purpose, she indicated in the portion for civil
other respects, the NLRC affirmed the decision of the labor arbiter,
status therein that she was single although she had contracted marriage a
few months earlier, that is, on May 26, 1991.[3]
including the order for the reinstatement of private respondent in her equal opportunities with men to act and to enter into contracts, and for
employment with PT&T. appointment, admission, training, graduation, and commissioning in all
military or similar schools of the Armed Forces of the Philippines and the
The subsequent motion for reconsideration filed by petitioner was Philippine National Police; Republic Act No. 7322 [15]increasing the
rebuffed by respondent NLRC in its resolution of November 9, 1994, hence maternity benefits granted to women in the private sector; Republic Act
this special civil action assailing the aforestated decisions of the labor No. 7877[16] which outlaws and punishes sexual harassment in the
arbiter and respondent NLRC, as well as the denial resolution of the latter. workplace and in the education and training environment; and Republic
1. Decreed in the Bible itself is the universal norm that women should Act No. 8042,[17] or the Migrant Workers and Overseas Filipinos Act of
be regarded with love and respect but, through the ages, men have 1995, which prescribes as a matter of policy, inter alia, the deployment of
responded to that injunction with indifference, on the hubristic conceit that migrant workers, with emphasis on women, only in countries where their
women constitute the inferior sex. Nowhere has that prejudice against rights are secure. Likewise, it would not be amiss to point out that in the
womankind been so pervasive as in the field of labor, especially on the Family Code,[18] womens rights in the field of civil law have been greatly
matter of equal employment opportunities and standards. In the Philippine enhanced and expanded.
setting, women have traditionally been considered as falling within the In the Labor Code, provisions governing the rights of women workers
vulnerable groups or types of workers who must be safeguarded with are found in Articles 130 to 138 thereof. Article 130 involves the right
preventive and remedial social legislation against discriminatory and against particular kinds of night work while Article 132 ensures the right of
exploitative practices in hiring, training, benefits, promotion and retention. women to be provided with facilities and standards which the Secretary of
The Constitution, cognizant of the disparity in rights between men and Labor may establish to ensure their health and safety. For purposes of
women in almost all phases of social and political life, provides a gamut of labor and social legislation, a woman working in a nightclub, cocktail
protective provisions. To cite a few of the primordial ones, Section 14, lounge, massage clinic, bar or other similar establishments shall be
Article II[8] on the Declaration of Principles and State Policies, expressly considered as an employee under Article 138. Article 135, on the other
recognizes the role of women in nation-building and commands the State hand, recognizes a womans right against discrimination with respect to
to ensure, at all times, the fundamental equality before the law of women terms and conditions of employment on account simply of sex. Finally, and
and men. Corollary thereto, Section 3 of Article XIII [9] (the progenitor this brings us to the issue at hand, Article 136 explicitly prohibits
whereof dates back to both the 1935 and 1973 Constitution) pointedly discrimination merely by reason of the marriage of a female employee.
requires the State to afford full protection to labor and to promote full 3. Acknowledged as paramount in the due process scheme is the
employment and equality of employment opportunities for all, including an constitutional guarantee of protection to labor and security of
assurance of entitlement to tenurial security of all workers.Similarly, tenure. Thus, an employer is required, as a condition sine qua nonprior to
Section 14 of Article XIII[10] mandates that the State shall protect working severance of the employment ties of an individual under his employ, to
women through provisions for opportunities that would enable them to convincingly establish, through substantial evidence, the existence of a
reach their full potential. valid and just cause in dispensing with the services of such employee,
2. Corrective labor and social laws on gender inequality have emerged ones labor being regarded as constitutionally protected property.
with more frequency in the years since the Labor Code was enacted on On the other hand, it is recognized that regulation of manpower by
May 1, 1974 as Presidential Decree No. 442, largely due to our countrys the company falls within the so-called management prerogatives, which
commitment as a signatory to the United Nations Convention on the prescriptions encompass the matter of hiring, supervision of workers, work
Elimination of All Forms of Discrimination Against Women (CEDAW). [11] assignments, working methods and assignments, as well as regulations on
Principal among these laws are Republic Act No. 6727 [12] which the transfer of employees, lay-off of workers, and the discipline, dismissal,
explicitly prohibits discrimination against women with respect to terms and and recall of employees.[19] As put in a case, an employer is free to
conditions of employment, promotion, and training opportunities; Republic regulate, according to his discretion and best business judgment, all
Act No. 6955[13] which bans the mail-order-bride practice for a fee and the aspects of employment, from hiring to firing, except in cases of unlawful
export of female labor to countries that cannot guarantee protection to the discrimination or those which may be provided by law. [20]
rights of women workers; Republic Act No. 7192, [14] also known as the In the case at bar, petitioners policy of not accepting or considering
Women in Development and Nation Building Act, which affords women as disqualified from work any woman worker who contracts marriage runs
afoul of the test of, and the right against, discrimination, afforded all Petitioner would have the Court believe that although private
women workers by our labor laws and by no less than the respondent defied its policy against its female employees contracting
Constitution. Contrary to petitioners assertion that it dismissed private marriage, what could be an act of insubordination was
respondent from employment on account of her dishonesty, the record inconsequential. What it submits as unforgivable is her concealment of
discloses clearly that her ties with the company were dissolved principally that marriage yet, at the same time, declaring that marriage as a trivial
because of the companys policy that married women are not qualified for matter to which it supposedly has no objection. In other words, PT&T says
employment in PT&T, and not merely because of her supposed acts of it gives its blessings to its female employees contracting marriage, despite
dishonesty. the maternity leaves and other benefits it would consequently respond for
and which obviously it would have wanted to avoid. If that employee
That it was so can easily be seen from the memorandum sent to confesses such fact of marriage, there will be no sanction; but if such
private respondent by Delia M. Oficial, the branch supervisor of the employee conceals the same instead of proceeding to the confessional, she
company, with the reminder, in the words of the latter, that youre fully will be dismissed. This line of reasoning does not impress us as reflecting
aware that the company is not accepting married women employee (sic), its true management policy or that we are being regaled with responsible
as it was verbally instructed to you. [21] Again, in the termination notice advocacy.
sent to her by the same branch supervisor, private respondent was made
to understand that her severance from the service was not only by reason This Court should be spared the ennui of strained reasoning and
of her concealment of her married status but, over and on top of that, was the tedium of propositions which confuse through less than candid
her violation of the companys policy against marriage (and even told you arguments. Indeed, petitioner glosses over the fact that it was its unlawful
that married women employees are not applicable [sic] or accepted in our policy against married women, both on the aspects of qualification and
company.)[22] Parenthetically, this seems to be the curious reason why it retention, which compelled private respondent to conceal her supervenient
was made to appear in the initiatory pleadings that petitioner was marriage. It was, however, that very policy alone which was the cause of
represented in this case only by its said supervisor and not by its highest private respondents secretive conduct now complained of. It is
ranking officers who would otherwise be solidarily liable with the then apropos to recall the familiar saying that he who is the cause of the
corporation.[23] cause is the cause of the evil caused.

Verily, private respondents act of concealing the true nature of her Finally, petitioners collateral insistence on the admission of private
status from PT&T could not be properly characterized as willful or in bad respondent that she supposedly misappropriated company funds, as an
faith as she was moved to act the way she did mainly because she wanted additional ground to dismiss her from employment, is somewhat insincere
to retain a permanent job in a stable company. In other words, she was and self-serving. Concededly, private respondent admitted in the course of
practically forced by that very same illegal company policy into the proceedings that she failed to remit some of her collections, but that is
misrepresenting her civil status for fear of being disqualified from an altogether different story.The fact is that she was dismissed solely
work. While loss of confidence is a just cause for termination of because of her concealment of her marital status, and not on the basis of
employment, it should not be simulated. [24] It must rest on an actual that supposed defalcation of company funds. That the labor arbiter would
breach of duty committed by the employee and not on the employers thus consider petitioners submissions on this supposed dishonesty as a
caprices.[25] Furthermore, it should never be used as a subterfuge for mere afterthought, just to bolster its case for dismissal, is a perceptive
causes which are improper, illegal, or unjustified. [26] conclusion born of experience in labor cases. For, there was no showing
that private respondent deliberately misappropriated the amount or
In the present controversy, petitioners expostulations that it whether her failure to remit the same was through negligence and, if so,
dismissed private respondent, not because the latter got married but whether the negligence was in nature simple or grave. In fact, it was
because she concealed that fact, does have a hollow ring. Her merely agreed that private respondent execute a promissory note to
concealment, so it is claimed, bespeaks dishonesty hence the consequent refund the same, which she did, and the matter was deemed settled as a
loss of confidence in her which justified her dismissal. Petitioner would peripheral issue in the labor case.
asseverate, therefore, that while it has nothing against marriage, it
nonetheless takes umbrage over the concealment of that fact. This Private respondent, it must be observed, had gained regular status at
improbable reasoning, with interstitial distinctions, perturbs the Court the time of her dismissal. When she was served her walking papers on
since private respondent may well be minded to claim that the imputation January 29, 1992, she was about to complete the probationary period of
of dishonesty should be the other way around. 150 days as she was contracted as a probationary employee on September
2, 1991. That her dismissal would be effected just when her probationary industrial, agricultural, and mercantile establishments and other places of
period was winding down clearly raises the plausible conclusion that it was labor in the then Philippine Islands.
done in order to prevent her from earning security of tenure. [27] On the
other hand, her earlier stints with the company as reliever were It would be worthwhile to reflect upon and adopt here the
undoubtedly those of a regular employee, even if the same were for fixed rationalization in Zialcita, et al. vs. Philippine Air Lines,[33] a decision that
periods, as she performed activities which were essential or necessary in emanated from the Office of the President. There, a policy of Philippine Air
the usual trade and business of PT&T. [28] The primary standard of Lines requiring that prospective flight attendants must be single and that
determining regular employment is the reasonable connection between the they will be automatically separated from the service once they marry was
activity performed by the employee in relation to the business or trade of declared void, it being violative of the clear mandate in Article 136 of the
the employer.[29] Labor Code with regard to discrimination against married women. Thus:

As an employee who had therefore gained regular status, and as she Of first impression is the incompatibility of the respondents policy or
had been dismissed without just cause, she is entitled to reinstatement regulation with the codal provision of law. Respondent is resolute in its
without loss of seniority rights and other privileges and to full back wages, contention that Article 136 of the Labor Code applies only to women
inclusive of allowances and other benefits or their monetary equivalent. employed in ordinary occupations and that the prohibition against
[30]
However, as she had undeniably committed an act of dishonesty in marriage of women engaged in extraordinary occupations, like flight
concealing her status, albeit under the compulsion of an unlawful attendants, is fair and reasonable, considering the pecularities of their
imposition of petitioner, the three-month suspension imposed by chosen profession.
respondent NLRC must be upheld to obviate the impression or inference
that such act should be condoned. It would be unfair to the employer if
We cannot subscribe to the line of reasoning pursued by respondent. All
she were to return to its fold without any sanction whatsoever for her act
along, it knew that the controverted policy has already met its doom as
which was not totally justified. Thus, her entitlement to back wages, which
early as March 13, 1973 when Presidential Decree No. 148, otherwise
shall be computed from the time her compensation was withheld up to the
known as the Women and Child Labor Law, was promulgated. But for the
time of her actual reinstatement, shall be reduced by deducting therefrom
timidity of those affected or their labor unions in challenging the validity of
the amount corresponding to her three months suspension.
the policy, the same was able to obtain a momentary reprieve. A close
4. The government, to repeat, abhors any stipulation or policy in the look at Section 8 of said decree, which amended paragraph (c) of Section
nature of that adopted by petitioner PT&T. The Labor Code states, in no 12 of Republic Act No. 679, reveals that it is exactly the same provision
uncertain terms, as follows: reproduced verbatim in Article 136 of the Labor Code, which was
promulgated on May 1, 1974 to take effect six (6) months later, or on
ART. 136. Stipulation against marriage. - It shall be unlawful for an November 1, 1974.
employer to require as a condition of employment or continuation of
employment that a woman shall not get married, or to stipulate expressly It cannot be gainsaid that, with the reiteration of the same provision in the
or tacitly that upon getting married, a woman employee shall be deemed new Labor Code, all policies and acts against it are deemed illegal and
resigned or separated, or to actually dismiss, discharge, discriminate or therefore abrogated. True, Article 132 enjoins the Secretary of Labor to
otherwise prejudice a woman employee merely by reason of marriage. establish standards that will ensure the safety and health of women
employees and in appropriate cases shall by regulation require employers
This provision had a studied history for its origin can be traced to to determine appropriate minimum standards for termination in special
Section 8 of Presidential Decree No. 148, [31] better known as the Women occupations, such as those of flight attendants, but that is precisely the
and Child Labor Law, which amended paragraph (c), Section 12 of factor that militates against the policy of respondent. The standards have
Republic Act No. 679,[32] entitled An Act to Regulate the Employment of not yet been established as set forth in the first paragraph, nor has the
Women and Children, to Provide Penalties for Violations Thereof, and for Secretary of Labor issued any regulation affecting flight attendants.
Other Purposes. The forerunner to Republic Act No. 679, on the other
hand, was Act No. 3071 which became law on March 16, 1923 and which It is logical to presume that, in the absence of said standards or
regulated the employment of women and children in shops, factories, regulations which are as yet to be established, the policy of respondent
against marriage is patently illegal. This finds support in Section 9 of the of the same nature. In said case, respondent, in dismissing from the
New Constitution, which provides: service the complainant, invoked a policy of the firm to consider female
employees in the project it was undertaking as separated the moment
Sec. 9. The State shall afford protection to labor, promote full employment they get married due to lack of facilities for married women. Respondent
and equality in employment, ensure equal work opportunities regardless of further claimed that complainant was employed in the project with an oral
sex, race, or creed, and regulate the relations between workers and understanding that her services would be terminated when she gets
employees. The State shall assure the rights of workers to self- married. Branding the policy of the employer as an example of
organization, collective bargaining, security of tenure, and just and discriminatory chauvinism tantamount to denying equal employment
humane conditions of work x x x. opportunities to women simply on account of their sex, the appellate court
struck down said employer policy as unlawful in view of its repugnance to
the Civil Code, Presidential Decree No. 148 and the Constitution.
Moreover, we cannot agree to the respondents proposition that
termination from employment of flight attendants on account of marriage Under American jurisprudence, job requirements which establish
is a fair and reasonable standard designed for their own health, safety, employer preference or conditions relating to the marital status of an
protection and welfare, as no basis has been laid therefor. Actually, employee are categorized as a sex-plus discrimination where it is imposed
respondent claims that its concern is not so much against the continued on one sex and not on the other. Further, the same should be evenly
employment of the flight attendant merely by reason of marriage as applied and must not inflict adverse effects on a racial or sexual group
observed by theSecretary of Labor, but rather on the consequence of which is protected by federal job discrimination laws. Employment rules
marriage-pregnancy. Respondent discussed at length in the instant appeal that forbid or restrict the employment of married women, but do not apply
the supposed ill effects of pregnancy on flight attendants in the course of to married men, have been held to violate Title VII of the United States
their employment.We feel that this needs no further discussion as it had Civil Rights Act of 1964, the main federal statute prohibiting job
been adequately explained by the Secretary of Labor in his decision of May discrimination against employees and applicants on the basis of, among
2, 1976. other things, sex.[35]

Further, it is not relevant that the rule is not directed against all
In a vain attempt to give meaning to its position, respondent went as far
women but just against married women. And, where the employer
as invoking the provisions of Articles 52 and 216 of the New Civil Code on
discriminates against married women, but not against married men, the
the preservation of marriage as an inviolable social institution and the
variable is sex and the discrimination is unlawful. [36] Upon the other hand,
family as a basic social institution, respectively, as bases for its policy of
a requirement that a woman employee must remain unmarried could be
non-marriage. In both instances, respondent predicates absence of a flight
justified as a bona fide occupational qualification, or BFOQ, where the
attendant from her home for long periods of time as contributory to an
particular requirements of the job would justify the same, but not on the
unhappy married life. This is pure conjecture not based on actual
ground of a general principle, such as the desirability of spreading work in
conditions, considering that, in this modern world, sophisticated
the workplace. A requirement of that nature would be valid provided it
technology has narrowed the distance from one place to
reflects an inherent quality reasonably necessary for satisfactory job
another. Moreover, respondent overlooked the fact that married flight
performance. Thus, in one case, a no-marriage rule applicable to both
attendants can program their lives to adapt to prevailing circumstances
male and female flight attendants, was regarded as unlawful since the
and events.
restriction was not related to the job performance of the flight attendants.
[37]

Article 136 is not intended to apply only to women employed in ordinary


occupations, or it should have categorically expressed so. The sweeping 5. Petitioners policy is not only in derogation of the provisions of
intendment of the law, be it on special or ordinary occupations, is reflected Article 136 of the Labor Code on the right of a woman to be free from any
in the whole text and supported by Article 135 that speaks of non- kind of stipulation against marriage in connection with her employment,
discrimination on the employment of women. but it likewise assaults good morals and public policy, tending as it does to
deprive a woman of the freedom to choose her status, a privilege that by
all accounts inheres in the individual as an intangible and inalienable right.
The judgment of the Court of Appeals in Gualberto, et al. vs. [38]
Hence, while it is true that the parties to a contract may establish any
Marinduque Mining & Industrial Corporation [34] considered as void a policy
agreements, terms, and conditions that they may deem convenient, the
same should not be contrary to law, morals, good customs, public order,
or public policy.[39] Carried to its logical consequences, it may even be said
that petitioners policy against legitimate marital bonds would encourage
illicit or common-law relations and subvert the sacrament of marriage.

Parenthetically, the Civil Code provisions on the contract of labor state


that the relations between the parties, that is, of capital and labor, are not
merely contractual, impressed as they are with so much public interest
that the same should yield to the common good. [40] It goes on to intone
that neither capital nor labor should visit acts of oppression against the
other, nor impair the interest or convenience of the public. [41] In the final
reckoning, the danger of just such a policy against marriage followed by
petitioner PT&T is that it strikes at the very essence, ideals and purpose of
marriage as an inviolable social institution and, ultimately, of the family as
the foundation of the nation. [42] That it must be effectively interdicted here
in all its indirect, disguised or dissembled forms as discriminatory conduct
derogatory of the laws of the land is not only in order but imperatively
required.

ON THE FOREGOING PREMISES, the petition of Philippine


Telegraph and Telephone Company is hereby DISMISSED for lack of merit,
with double costs against petitioner.

SO ORDERED.

Romero, Puno, Mendoza, and Torres, Jr., JJ., concur.


Republic of the Philippines
SUPREME COURT VELASCO, JR., J.:
Manila
The need to address environmental pollution, as a cause of climate
EN BANC change, has of late gained the attention of the international
community. Media have finally trained their sights on the ill effects of
pollution, the destruction of forests and other critical habitats, oil spills,
METROPOLITAN MANILA G.R. Nos. 171947-48 and the unabated improper disposal of garbage. And rightly so, for the
DEVELOPMENT AUTHORITY, magnitude of environmental destruction is now on a scale few ever
DEPARTMENT OF ENVIRONMENT foresaw and the wound no longer simply heals by itself. [2] But amidst hard
AND NATURAL RESOURCES, Present: evidence and clear signs of a climate crisis that need bold action, the voice
DEPARTMENT OF EDUCATION, of cynicism, naysayers, and procrastinators can still be heard.
CULTURE AND SPORTS,[1] PUNO, C.J.,
DEPARTMENT OF HEALTH, QUISUMBING, This case turns on government agencies and their officers who, by
DEPARTMENT OF AGRICULTURE, YNARES-SANTIAGO, the nature of their respective offices or by direct statutory command, are
DEPARTMENT OF PUBLIC CARPIO, tasked to protect and preserve, at the first instance, our internal waters,
WORKS AND HIGHWAYS, AUSTRIA-MARTINEZ, rivers, shores, and seas polluted by human activities. To most of these
DEPARTMENT OF BUDGET AND CORONA, agencies and their official complement, the pollution menace does not
MANAGEMENT, PHILIPPINE CARPIO MORALES, seem to carry the high national priority it deserves, if their track records
COAST GUARD, PHILIPPINE AZCUNA, are to be the norm. Their cavalier attitude towards solving, if not
NATIONAL POLICE MARITIME TINGA, mitigating, the environmental pollution problem, is a sad commentary on
GROUP, and DEPARTMENT OF CHICO-NAZARIO, bureaucratic efficiency and commitment.
THE INTERIOR AND LOCAL VELASCO, JR.,
GOVERNMENT, NACHURA, At the core of the case is the Manila Bay, a place with a proud
Petitioners, REYES, historic past, once brimming with marine life and, for so many decades in
LEONARDO-DE CASTRO, and the past, a spot for different contact recreation activities, but now a dirty
- versus - BRION, JJ. and slowly dying expanse mainly because of the abject official indifference
CONCERNED RESIDENTS OF of people and institutions that could have otherwise made a difference.
MANILA BAY, represented and
joined by DIVINA V. ILAS,
SABINIANO ALBARRACIN, This case started when, on January 29, 1999, respondents
MANUEL SANTOS, JR., DINAH Concerned Residents of Manila Bay filed a complaint before the Regional
DELA PEA, PAUL DENNIS Trial Court (RTC) in Imus, Cavite against several government agencies,
QUINTERO, MA. VICTORIA among them the petitioners, for the cleanup, rehabilitation, and protection
LLENOS, DONNA CALOZA, of the Manila Bay. Raffled to Branch 20 and docketed as Civil Case No.
FATIMA QUITAIN, VENICE 1851-99 of the RTC, the complaint alleged that the water quality of
SEGARRA, FRITZIE TANGKIA, the Manila Bay had fallen way below the allowable standards set by law,
SARAH JOELLE LINTAG, specifically Presidential Decree No. (PD) 1152 or the Philippine
HANNIBAL AUGUSTUS BOBIS, Environment Code. This environmental aberration, the complaint stated,
FELIMON SANTIAGUEL, and Promulgated: stemmed from:
JAIME AGUSTIN R. OPOSA,
Respondents. December 18, 2008 x x x [The] reckless, wholesale, accumulated and ongoing
x---------------------------------------------------------------------------------- acts of omission or commission [of the defendants]
-------x resulting in the clear and present danger to public health
and in the depletion and contamination of the marine life
DECISION of Manila Bay, [for which reason] ALL defendants must be
held jointly and/or solidarily liable and be collectively being conducted on ship-generated waste treatment and disposal, and
ordered to clean up Manila Bay and to restore its water its Linis Dagat (Clean the Ocean) project for the cleaning of wastes
quality to class B waters fit for swimming, skin-diving, and accumulated or washed to shore.
other forms of contact recreation.[3]
The RTC Ordered Petitioners to Clean Up
and Rehabilitate Manila Bay
In their individual causes of action, respondents alleged that the
continued neglect of petitioners in abating the pollution of On September 13, 2002, the RTC rendered a Decision [5] in favor of
the Manila Bay constitutes a violation of, among others: respondents. The dispositive portion reads:

(1) Respondents constitutional right to life, health, and a WHEREFORE, finding merit in the complaint, judgment is
balanced ecology; hereby rendered ordering the abovenamed defendant-
(2) The Environment Code (PD 1152); government agencies, jointly and solidarily, to clean up
(3) The Pollution Control Law (PD 984); and rehabilitate Manila Bay and restore its waters to SB
(4) The Water Code (PD 1067); classification to make it fit for swimming, skin-diving and
(5) The Sanitation Code (PD 856); other forms of contact recreation. To attain this,
(6) The Illegal Disposal of Wastes Decree (PD 825); defendant-agencies, with defendant DENR as the lead
(7) The Marine Pollution Law (PD 979); agency, are directed, within six (6) months from receipt
(8) Executive Order No. 192; hereof, to act and perform their respective duties by
(9) The Toxic and Hazardous Wastes Law (Republic Act devising a consolidated, coordinated and concerted scheme
No. 6969); of action for the rehabilitation and restoration of the bay.
(10) Civil Code provisions on nuisance and human relations; In particular:
(11) The Trust Doctrine and the Principle of Guardianship;
and Defendant MWSS is directed to install, operate and
(12) International Law maintain adequate [sewerage] treatment facilities in
strategic places under its jurisdiction and increase their
Inter alia, respondents, as plaintiffs a quo, prayed that petitioners capacities.
be ordered to clean the Manila Bay and submit to the RTC a concerted
concrete plan of action for the purpose. Defendant LWUA, to see to it that the water districts under
its wings, provide, construct and operate sewage facilities
The trial of the case started off with a hearing at the Manila Yacht for the proper disposal of waste.
Club followed by an ocular inspection of the Manila Bay. Renato T. Cruz,
the Chief of the Water Quality Management Section, Environmental Defendant DENR, which is the lead agency in cleaning up
Management Bureau, Department of Environment and Natural Resources Manila Bay, to install, operate and maintain waste facilities
(DENR), testifying for petitioners, stated that water samples collected from to rid the bay of toxic and hazardous substances.
different beaches around the Manila Bay showed that the amount of fecal
coliform content ranged from 50,000 to 80,000 most probable number Defendant PPA, to prevent and also to treat the discharge
(MPN)/ml when what DENR Administrative Order No. 34-90 prescribed as not only of ship-generated wastes but also of other solid
a safe level for bathing and other forms of contact recreational activities, and liquid wastes from docking vessels that contribute to
or the SB level, is one not exceeding 200 MPN/100 ml. [4] the pollution of the bay.

Rebecca de Vera, for Metropolitan Waterworks and Sewerage Defendant MMDA, to establish, operate and maintain an
System (MWSS) and in behalf of other petitioners, testified about the adequate and appropriate sanitary landfill and/or adequate
MWSS efforts to reduce pollution along the Manila Bay through the Manila solid waste and liquid disposal as well as other alternative
Second Sewerage Project. For its part, the Philippine Ports Authority (PPA) garbage disposal system such as re-use or recycling of
presented, as part of its evidence, its memorandum circulars on the study wastes.
consolidation with the consolidated appeals of MWSS, LWUA, and PPA,
Defendant DA, through the Bureau of Fisheries and Aquatic docketed as CA-G.R. SP No. 74944.
Resources, to revitalize the marine life in Manila Bay and
restock its waters with indigenous fish and other aquatic Petitioners, before the CA, were one in arguing in the main that
animals. the pertinent provisions of the Environment Code (PD 1152) relate only to
the cleaning of specific pollution incidents and do not cover cleaning in
Defendant DBM, to provide and set aside an adequate general. And apart from raising concerns about the lack of funds
budget solely for the purpose of cleaning up and appropriated for cleaning purposes, petitioners also asserted that the
rehabilitation of Manila Bay. cleaning of the ManilaBay is not a ministerial act which can be compelled
by mandamus.
Defendant DPWH, to remove and demolish structures and
other nuisances that obstruct the free flow of waters to the
bay. These nuisances discharge solid and liquid wastes The CA Sustained the RTC
which eventually end up in Manila Bay. As the construction By a Decision[6] of September 28, 2005, the CA denied petitioners
and engineering arm of the government, DPWH is ordered appeal and affirmed the Decision of the RTC in toto, stressing that the trial
to actively participate in removing debris, such as carcass courts decision did not require petitioners to do tasks outside of their usual
of sunken vessels, and other non-biodegradable garbage in basic functions under existing laws.[7]
the bay.
Petitioners are now before this Court praying for the allowance of
Defendant DOH, to closely supervise and monitor the their Rule 45 petition on the following ground and supporting arguments:
operations of septic and sludge companies and require THE [CA] DECIDED A QUESTION OF SUBSTANCE
them to have proper facilities for the treatment and NOT HERETOFORE PASSED UPON BY THE HONORABLE
disposal of fecal sludge and sewage coming from septic COURT, I.E., IT AFFIRMED THE TRIAL COURTS DECISION
tanks. DECLARING THAT SECTION 20 OF [PD] 1152 REQUIRES
CONCERNED GOVERNMENT AGENCIES TO REMOVE ALL
Defendant DECS, to inculcate in the minds and hearts of POLLUTANTS SPILLED AND DISCHARGED IN THE WATER
the people through education the importance of preserving SUCH AS FECAL COLIFORMS.
and protecting the environment.
ARGUMENTS
Defendant Philippine Coast Guard and the PNP Maritime
Group, to protect at all costs the Manila Bay from all forms I
of illegal fishing. [SECTIONS] 17 AND 20 OF [PD] 1152 RELATE
ONLY TO THE CLEANING OF SPECIFIC POLLUTION
No pronouncement as to damages and costs. INCIDENTS AND [DO] NOT COVER CLEANING IN GENERAL

SO ORDERED. II
The MWSS, Local Water Utilities Administration (LWUA), and PPA filed THE CLEANING OR REHABILITATION OF
before the Court of Appeals (CA) individual Notices of Appeal which were THE MANILA BAY IS NOT A MINISTERIAL ACT OF
eventually consolidated and docketed as CA-G.R. CV No. 76528. PETITIONERS THAT CAN BE COMPELLED BY MANDAMUS.

On the other hand, the DENR, Department of Public Works and


Highways (DPWH), Metropolitan Manila Development Authority (MMDA), The issues before us are two-fold. First, do Sections 17 and 20 of
Philippine Coast Guard (PCG), Philippine National Police (PNP) Maritime PD 1152 under the headings, Upgrading of Water Quality and Clean-up
Group, and five other executive departments and agencies filed directly Operations, envisage a cleanup in general or are they limited only to the
with this Court a petition for review under Rule 45. The Court, in a cleanup of specific pollution incidents? And second, can petitioners be
Resolution of December 9, 2002, sent the said petition to the CA for compelled by mandamus to clean up and rehabilitate the Manila Bay?
implementation of the MMDAs mandated tasks may entail a decision-
On August 12, 2008, the Court conducted and heard the parties on making process, the enforcement of the law or the very act of doing what
oral arguments. the law exacts to be done is ministerial in nature and may be compelled by
mandamus. We said so in Social Justice Society v. Atienza [11] in which the
Our Ruling Court directed the City of Manila to enforce, as a matter of ministerial
duty, its Ordinance No. 8027 directing the three big local oil players to
We shall first dwell on the propriety of the issuance of mandamus cease and desist from operating their business in the so-called Pandacan
under the premises. Terminals within six months from the effectivity of the ordinance. But to
illustrate with respect to the instant case, the MMDAs duty to put up an
adequate and appropriate sanitary landfill and solid waste and liquid
The Cleaning or Rehabilitation of Manila Bay disposal as well as other alternative garbage disposal systems is
Can be Compelled by Mandamus ministerial, its duty being a statutory imposition. The MMDAs duty in this
regard is spelled out in Sec. 3(c) of Republic Act No. (RA) 7924 creating
Generally, the writ of mandamus lies to require the execution of a the MMDA. This section defines and delineates the scope of the MMDAs
ministerial duty.[8] A ministerial duty is one that requires neither the waste disposal services to include:
exercise of official discretion nor judgment. [9] It connotes an act in which
nothing is left to the discretion of the person executing it. It is a simple, Solid waste disposal and management which
definite duty arising under conditions admitted or proved to exist and include formulation and implementation of policies,
imposed by law.[10]Mandamus is available to compel action, when refused, standards, programs and projects for proper and sanitary
on matters involving discretion, but not to direct the exercise of judgment waste disposal. It shall likewise include
or discretion one way or the other. the establishment and operation of sanitary land fill
and related facilities and the implementation of other
Petitioners maintain that the MMDAs duty to take measures and alternative programs intended to reduce, reuse and recycle
maintain adequate solid waste and liquid disposal systems necessarily solid waste. (Emphasis added.)
involves policy evaluation and the exercise of judgment on the part of the
agency concerned. They argue that the MMDA, in carrying out its
mandate, has to make decisions, including choosing where a landfill should The MMDA is duty-bound to comply with Sec. 41 of the Ecological
be located by undertaking feasibility studies and cost estimates, all of Solid Waste Management Act (RA 9003) which prescribes the minimum
which entail the exercise of discretion. criteria for the establishment of sanitary landfills and Sec. 42 which
provides the minimum operating requirements that each site operator shall
Respondents, on the other hand, counter that the statutory maintain in the operation of a sanitary landfill. Complementing Sec. 41 are
command is clear and that petitioners duty to comply with and act Secs. 36 and 37 of RA 9003,[12] enjoining the MMDA and local government
according to the clear mandate of the law does not require the exercise of units, among others, after the effectivity of the law on February 15, 2001,
discretion. According to respondents, petitioners, the MMDA in particular, from using and operating open dumps for solid waste and disallowing, five
are without discretion, for example, to choose which bodies of water they years after such effectivity, the use of controlled dumps.
are to clean up, or which discharge or spill they are to contain. By the
same token, respondents maintain that petitioners are bereft of discretion The MMDAs duty in the area of solid waste disposal, as may be
on whether or not to alleviate the problem of solid and liquid waste noted, is set forth not only in the Environment Code (PD 1152) and RA
disposal; in other words, it is the MMDAs ministerial duty to attend to such 9003, but in its charter as well. This duty of putting up a proper waste
services. disposal system cannot be characterized as discretionary, for, as earlier
stated, discretion presupposes the power or right given by law to public
We agree with respondents. functionaries to act officially according to their judgment or conscience.
[13]
A discretionary duty is one that allows a person to exercise judgment
First off, we wish to state that petitioners obligation to perform and choose to perform or not to perform. [14] Any suggestion that the MMDA
their duties as defined by law, on one hand, and how they are to carry out has the option whether or not to perform its solid waste disposal-related
such duties, on the other, are two different concepts. While the duties ought to be dismissed for want of legal basis.
A perusal of other petitioners respective charters or like enabling statutes
and pertinent laws would yield this conclusion: these government agencies The DENR has prepared the status report for the period 2001 to 2005 and
are enjoined, as a matter of statutory obligation, to perform certain is in the process of completing the preparation of the Integrated Water
functions relating directly or indirectly to the cleanup, rehabilitation, Quality Management Framework. [16]Within twelve (12) months thereafter,
protection, and preservation of the Manila Bay. They are precluded from it has to submit a final Water Quality Management Area Action Plan.
choosing not to perform these duties. Consider: [17]
Again, like the MMDA, the DENR should be made to accomplish the
tasks assigned to it under RA 9275.
(1) The DENR, under Executive Order No. (EO) 192, [15] is the primary Parenthetically, during the oral arguments, the DENR Secretary manifested
agency responsible for the conservation, management, development, and that the DENR, with the assistance of and in partnership with various
proper use of the countrys environment and natural resources. Sec. 19 of government agencies and non-government organizations, has completed,
the Philippine Clean Water Act of 2004 (RA 9275), on the other hand, as of December 2005, the final draft of a comprehensive action plan with
designates the DENR as the primary government agency responsible for its estimated budget and time frame, denominated as Operation Plan for the
enforcement and implementation, more particularly over all aspects of Manila Bay Coastal Strategy, for the rehabilitation, restoration, and
water quality management. On water pollution, the DENR, under the Acts rehabilitation of the Manila Bay.
Sec. 19(k), exercises jurisdiction over all aspects of water pollution,
determine[s] its location, magnitude, extent, severity, causes and effects The completion of the said action plan and even the implementation of
and other pertinent information on pollution, and [takes] measures, using some of its phases should more than ever prod the concerned agencies to
available methods and technologies, to prevent and abate such pollution. fast track what are assigned them under existing laws.
The DENR, under RA 9275, is also tasked to prepare a National Water
Quality Status Report, an Integrated Water Quality Management (2) The MWSS, under Sec. 3 of RA 6234, [18] is vested with
Framework, and a 10-year Water Quality Management Area Action Plan jurisdiction, supervision, and control over all waterworks and sewerage
which is nationwide in scope covering the Manila Bay and adjoining areas. systems in the territory comprising what is now the cities of Metro Manila
Sec. 19 of RA 9275 provides: and several towns of the provinces of Rizal and Cavite, and charged with
the duty:
Sec. 19 Lead Agency.The [DENR] shall be the primary (g) To construct, maintain, and operate such sanitary
government agency responsible for the implementation sewerages as may be necessary for the proper sanitation
and enforcement of this Act x x x unless otherwise and other uses of the cities and towns comprising the
provided herein. As such, it shall have the following System; x x x
functions, powers and responsibilities:
a) Prepare a National Water Quality Status report
within twenty-four (24) months from the effectivity of (3) The LWUA under PD 198 has the power of supervision and
this Act: Provided, That the Department shall control over local water districts. It can prescribe the minimum standards
thereafter review or revise and publish annually, or as and regulations for the operations of these districts and shall monitor and
the need arises, said report; evaluate local water standards. The LWUA can direct these districts to
construct, operate, and furnish facilities and services for the collection,
b) Prepare an Integrated Water Quality Management treatment, and disposal of sewerage, waste, and storm water.
Framework within twelve (12) months following the Additionally, under RA 9275, the LWUA, as attached agency of the DPWH,
completion of the status report; is tasked with providing sewerage and sanitation facilities, inclusive of the
setting up of efficient and safe collection, treatment, and sewage disposal
c) Prepare a ten (10) year Water Quality Management system in the different parts of the country. [19] In relation to the instant
Area Action Plan within 12 months following the petition, the LWUA is mandated to provide sewerage and sanitation
completion of the framework for each designated water facilities in Laguna, Cavite, Bulacan, Pampanga, and Bataan to prevent
management area. Such action plan shall be reviewed pollution in the Manila Bay.
by the water quality management area governing (4) The Department of Agriculture (DA), pursuant to the Administrative
board every five (5) years or as need arises. Code of 1987 (EO 292),[20] is designated as the agency tasked to
promulgate and enforce all laws and issuances respecting the conservation a. discharge, dump x x x harmful substances from or out
and proper utilization of agricultural and fishery resources. Furthermore, of any ship, vessel, barge, or any other floating craft, or
the DA, under the Philippine Fisheries Code of 1998 (RA 8550), is, in other man-made structures at sea, by any method, means
coordination with local government units (LGUs) and other concerned or manner, into or upon the territorial and inland navigable
sectors, in charge of establishing a monitoring, control, and surveillance waters of the Philippines;
system to ensure that fisheries and aquatic resources in Philippine waters
are judiciously utilized and managed on a sustainable basis. [21] Likewise b. throw, discharge or deposit, dump, or cause, suffer or
under RA 9275, the DA is charged with coordinating with the PCG and procure to be thrown, discharged, or deposited either from
DENR for the enforcement of water quality standards in marine waters. or out of any ship, barge, or other floating craft or vessel
[22]
More specifically, its Bureau of Fisheries and Aquatic Resources (BFAR) of any kind, or from the shore, wharf, manufacturing
under Sec. 22(c) of RA 9275 shall primarily be responsible for the establishment, or mill of any kind, any refuse matter of
prevention and control of water pollution for the development, any kind or description whatever other than that flowing
management, and conservation of the fisheries and aquatic resources. from streets and sewers and passing therefrom in a liquid
state into tributary of any navigable water from which the
(5) The DPWH, as the engineering and construction arm of the national same shall float or be washed into such navigable water;
government, is tasked under EO 292[23] to provide integrated planning, and
design, and construction services for, among others, flood control and
water resource development systems in accordance with national c. deposit x x x material of any kind in any place on the
development objectives and approved government plans and bank of any navigable water or on the bank of any
specifications. tributary of any navigable water, where the same shall be
liable to be washed into such navigable water, either by
In Metro Manila, however, the MMDA is authorized by Sec. 3(d), RA 7924 ordinary or high tides, or by storms or floods, or
to perform metro-wide services relating to flood control and sewerage otherwise, whereby navigation shall or may be impeded or
management which include the formulation and implementation of policies, obstructed or increase the level of pollution of such water.
standards, programs and projects for an integrated flood control, drainage
and sewerage system.
(7) When RA 6975 or the Department of the Interior and Local
On July 9, 2002, a Memorandum of Agreement was entered into between Government (DILG) Act of 1990 was signed into law on December 13,
the DPWH and MMDA, whereby MMDA was made the agency primarily 1990, the PNP Maritime Group was tasked to perform all police functions
responsible for flood control in Metro Manila. For the rest of the country, over the Philippine territorial waters and rivers. Under Sec. 86, RA 6975,
DPWH shall remain as the implementing agency for flood control the police functions of the PCG shall be taken over by the PNP when the
services. The mandate of the MMDA and DPWH on flood control and latter acquires the capability to perform such functions. Since the PNP
drainage services shall include the removal of structures, constructions, Maritime Group has not yet attained the capability to assume and perform
and encroachments built along rivers, waterways, and esteros (drainages) the police functions of PCG over marine pollution, the PCG and PNP
in violation of RA 7279, PD 1067, and other pertinent laws. Maritime Group shall coordinate with regard to the enforcement of laws,
rules, and regulations governing marine pollution within the territorial
(6) The PCG, in accordance with Sec. 5(p) of PD 601, or the waters of the Philippines. This was made clear in Sec. 124, RA 8550 or the
Revised Coast Guard Law of 1974, and Sec. 6 of PD 979, [24] or the Marine Philippine Fisheries Code of 1998, in which both the PCG and PNP Maritime
Pollution Decree of 1976, shall have the primary responsibility of enforcing Group were authorized to enforce said law and other fishery laws, rules,
laws, rules, and regulations governing marine pollution within the and regulations.[25]
territorial waters of the Philippines. It shall promulgate its own rules and
regulations in accordance with the national rules and policies set by the (8) In accordance with Sec. 2 of EO 513, the PPA is mandated to
National Pollution Control Commission upon consultation with the latter for establish, develop, regulate, manage and operate a rationalized national
the effective implementation and enforcement of PD 979. It shall, under port system in support of trade and national development. [26] Moreover,
Sec. 4 of the law, apprehend violators who: Sec. 6-c of EO 513 states that the PPA has police authority within the
ports administered by it as may be necessary to carry out and remove all structures, constructions, and other encroachments built in
its powers and functions and attain its purposes and breach of RA 7279 and other pertinent laws along the rivers, waterways,
objectives, without prejudice to the exercise of the and esteros in Metro Manila. With respect to rivers, waterways,
functions of the Bureau of Customs and other law and esteros in Bulacan, Bataan, Pampanga, Cavite, and Laguna that
enforcement bodies within the area. Such police authority discharge wastewater directly or eventually into the Manila Bay, the DILG
shall include the following: shall direct the concerned LGUs to implement the demolition and removal
xxxx of such structures, constructions, and other encroachments built in
violation of RA 7279 and other applicable laws in coordination with the
b) To regulate the entry to, exit from, and movement DPWH and concerned agencies.
within the port, of persons and vehicles, as well as
movement within the port of watercraft.[27] (10) The Department of Health (DOH), under Article 76 of PD 1067
(the Water Code), is tasked to promulgate rules and regulations for the
establishment of waste disposal areas that affect the source of a water
Lastly, as a member of the International Marine Organization and supply or a reservoir for domestic or municipal use. And under Sec. 8 of
a signatory to the International Convention for the Prevention of Pollution RA 9275, the DOH, in coordination with the DENR, DPWH, and other
from Ships, as amended by MARPOL 73/78, [28] the Philippines, through the concerned agencies, shall formulate guidelines and standards for the
PPA, must ensure the provision of adequate reception facilities at ports collection, treatment, and disposal of sewage and the establishment and
and terminals for the reception of sewage from the ships docking in operation of a centralized sewage treatment system. In areas not
Philippine ports. Thus, the PPA is tasked to adopt such measures as are considered as highly urbanized cities, septage or a mix sewerage-septage
necessary to prevent the discharge and dumping of solid and liquid wastes management system shall be employed.
and other ship-generated wastes into the Manila Bay waters from vessels
docked at ports and apprehend the violators. When the vessels are not In accordance with Sec. 72[30] of PD 856, the Code of Sanitation of
docked at ports but within Philippine territorial waters, it is the PCG and the Philippines, and Sec. 5.1.1 [31] of Chapter XVII of its implementing
PNP Maritime Group that have jurisdiction over said vessels. rules, the DOH is also ordered to ensure the regulation and monitoring of
the proper disposal of wastes by private sludge companies through the
(9) The MMDA, as earlier indicated, is duty-bound to put up and maintain strict enforcement of the requirement to obtain an environmental
adequate sanitary landfill and solid waste and liquid disposal system as sanitation clearance of sludge collection treatment and disposal before
well as other alternative garbage disposal systems. It is primarily these companies are issued their environmental sanitation permit.
responsible for the implementation and enforcement of the provisions of
RA 9003, which would necessary include its penal provisions, within its (11) The Department of Education (DepEd), under the Philippine
area of jurisdiction.[29] Environment Code (PD 1152), is mandated to integrate subjects on
environmental education in its school curricula at all levels. [32] Under Sec.
Among the prohibited acts under Sec. 48, Chapter VI of RA 9003 that are 118 of RA 8550, the DepEd, in collaboration with the DA, Commission on
frequently violated are dumping of waste matters in public places, such as Higher Education, and Philippine Information Agency, shall launch and
roads, canals or esteros, open burning of solid waste, squatting in open pursue a nationwide educational campaign to promote the development,
dumps and landfills, open dumping, burying of biodegradable or non- management, conservation, and proper use of the environment. Under the
biodegradable materials in flood-prone areas, establishment or operation Ecological Solid Waste Management Act (RA 9003), on the other hand, it is
of open dumps as enjoined in RA 9003, and operation of waste directed to strengthen the integration of environmental concerns in school
management facilities without an environmental compliance certificate. curricula at all levels, with an emphasis on waste management principles.
[33]

Under Sec. 28 of the Urban Development and Housing Act of 1992 (RA (12) The Department of Budget and Management (DBM) is tasked under
7279), eviction or demolition may be allowed when persons or entities Sec. 2, Title XVII of the Administrative Code of 1987 to ensure the efficient
occupy danger areas such as esteros, railroad tracks, garbage dumps, and sound utilization of government funds and revenues so as to
riverbanks, shorelines, waterways, and other public places such as effectively achieve the countrys development objectives. [34]
sidewalks, roads, parks and playgrounds. The MMDA, as lead agency, in
coordination with the DPWH, LGUs, and concerned agencies, can dismantle
One of the countrys development objectives is enshrined in RA 9275 or the operations and expenses incurred in said operations shall
Philippine Clean Water Act of 2004. This law stresses that the State shall be charged against the persons and/or entities responsible
pursue a policy of economic growth in a manner consistent with the for such pollution.
protection, preservation, and revival of the quality of our fresh, brackish,
and marine waters. It also provides that it is the policy of the government,
among others, to streamline processes and procedures in the prevention, When the Clean Water Act (RA 9275) took effect, its Sec. 16 on
control, and abatement of pollution mechanisms for the protection of the subject, Cleanup Operations, amended the counterpart provision (Sec.
water resources; to promote environmental strategies and use of 20) of the Environment Code (PD 1152). Sec. 17 of PD 1152 continues,
appropriate economic instruments and of control mechanisms for the however, to be operational.
protection of water resources; to formulate a holistic national program of
water quality management that recognizes that issues related to this
management cannot be separated from concerns about water sources and The amendatory Sec. 16 of RA 9275 reads:
ecological protection, water supply, public health, and quality of life; and
to provide a comprehensive management program for water pollution SEC. 16. Cleanup Operations.Notwithstanding the
focusing on pollution prevention. provisions of Sections 15 and 26 hereof, any person who
causes pollution in or pollutes water bodies in excess of
Thus, the DBM shall then endeavor to provide an adequate budget to the applicable and prevailing standards shall be
attain the noble objectives of RA 9275 in line with the countrys responsible to contain, remove and clean up any pollution
development objectives. incident at his own expense to the extent that the same
water bodies have been rendered unfit for utilization and
All told, the aforementioned enabling laws and issuances are in beneficial use: Provided, That in the event emergency
themselves clear, categorical, and complete as to what are the obligations cleanup operations are necessary and the polluter fails to
and mandate of each agency/petitioner under the law. We need not immediately undertake the same, the [DENR] in
belabor the issue that their tasks include the cleanup of the Manila Bay. coordination with other government agencies concerned,
shall undertake containment, removal and cleanup
Now, as to the crux of the petition. Do Secs. 17 and 20 of the operations. Expenses incurred in said operations shall be
Environment Code encompass the cleanup of water pollution in general, reimbursed by the persons found to have caused such
not just specific pollution incidents? pollution under proper administrative determination x x x.
Reimbursements of the cost incurred shall be made to the
Secs. 17 and 20 of the Environment Code Water Quality Management Fund or to such other funds
Include Cleaning in General where said disbursements were sourced.

The disputed sections are quoted as follows:


As may be noted, the amendment to Sec. 20 of the Environment Code is
Section 17. Upgrading of Water Quality.Where the quality more apparent than real since the amendment, insofar as it is relevant to
of water has deteriorated to a degree where its state will this case, merely consists in the designation of the DENR as lead agency in
adversely affect its best usage, the government agencies the cleanup operations.
concerned shall take such measures as may be necessary
to upgrade the quality of such water to meet the Petitioners contend at every turn that Secs. 17 and 20 of the
prescribed water quality standards. Environment Code concern themselves only with the matter of cleaning up
in specific pollution incidents, as opposed to cleanup in general. They aver
Section 20. Clean-up Operations.It shall be the that the twin provisions would have to be read alongside the succeeding
responsibility of the polluter to contain, remove and clean- Sec. 62(g) and (h), which defines the terms cleanup operations and
up water pollution incidents at his own expense. In case of accidental spills, as follows:
his failure to do so, the government agencies concerned
shall undertake containment, removal and clean-up
g. Clean-up Operations [refer] to activities conducted in Respondents are correct. For one thing, said Sec. 17 does not in
removing the pollutants discharged or spilled in water to any way state that the government agencies concerned ought to confine
restore it to pre-spill condition. themselves to the containment, removal, and cleaning operations when a
specific pollution incident occurs. On the contrary, Sec. 17 requires them
h. Accidental Spills [refer] to spills of oil or to act even in the absence of a specific pollution incident, as long as water
other hazardous substances in water that result quality has deteriorated to a degree where its state will adversely affect its
from accidents such as collisions and groundings. best usage. This section, to stress, commands concerned government
agencies, when appropriate, to take such measures as may be necessary
to meet the prescribed water quality standards. In fine, the underlying
Petitioners proffer the argument that Secs. 17 and 20 of PD 1152 duty to upgrade the quality of water is not conditional on the occurrence of
merely direct the government agencies concerned to undertake any pollution incident.
containment, removal, and cleaning operations of a specific polluted
portion or portions of the body of water concerned. They maintain that the For another, a perusal of Sec. 20 of the Environment Code, as
application of said Sec. 20 is limited only to water pollution incidents, couched, indicates that it is properly applicable to a specific situation in
which are situations that presuppose the occurrence of specific, isolated which the pollution is caused by polluters who fail to clean up the mess
pollution events requiring the corresponding containment, removal, and they left behind. In such instance, the concerned government agencies
cleaning operations. Pushing the point further, they argue that the shall undertake the cleanup work for the polluters account. Petitioners
aforequoted Sec. 62(g) requires cleanup operations to restore the body of assertion, that they have to perform cleanup operations in
water to pre-spill condition, which means that there must have been a the Manila Bay only when there is a water pollution incident and the erring
specific incident of either intentional or accidental spillage of oil or other polluters do not undertake the containment, removal, and cleanup
hazardous substances, as mentioned in Sec. 62(h). operations, is quite off mark. As earlier discussed, the complementary Sec.
17 of the Environment Code comes into play and the specific duties of the
As a counterpoint, respondents argue that petitioners erroneously read agencies to clean up come in even if there are no pollution incidents
Sec. 62(g) as delimiting the application of Sec. 20 to the containment, staring at them. Petitioners, thus, cannot plausibly invoke and hide behind
removal, and cleanup operations for accidental spills only. Contrary to Sec. 20 of PD 1152 or Sec. 16 of RA 9275 on the pretext that their cleanup
petitioners posture, respondents assert that Sec. 62(g), in fact, even mandate depends on the happening of a specific pollution incident. In this
expanded the coverage of Sec. 20. Respondents explain that without its regard, what the CA said with respect to the impasse over Secs. 17 and 20
Sec. 62(g), PD 1152 may have indeed covered only pollution accumulating of PD 1152 is at once valid as it is practical. The appellate court wrote: PD
from the day-to-day operations of businesses around the Manila Bay and 1152 aims to introduce a comprehensive program of environmental
other sources of pollution that slowly accumulated in the bay. protection and management. This is better served by making Secs. 17 &
Respondents, however, emphasize that Sec. 62(g), far from being a 20 of general application rather than limiting them to specific pollution
delimiting provision, in fact even enlarged the operational scope of Sec. incidents.[35]
20, by including accidental spills as among the water pollution incidents
contemplated in Sec. 17 in relation to Sec. 20 of PD 1152. Granting arguendo that petitioners position thus described vis--vis
the implementation of Sec. 20 is correct, they seem to have overlooked
To respondents, petitioners parochial view on environmental the fact that the pollution of the ManilaBay is of such magnitude and scope
issues, coupled with their narrow reading of their respective mandated that it is well-nigh impossible to draw the line between a specific and a
roles, has contributed to the worsening water quality of the Manila Bay. general pollution incident. And such impossibility extends to pinpointing
Assuming, respondents assert, that petitioners are correct in saying that with reasonable certainty who the polluters are. We note that Sec. 20 of
the cleanup coverage of Sec. 20 of PD 1152 is constricted by the definition PD 1152 mentions water pollution incidents which may be caused by
of the phrase cleanup operations embodied in Sec. 62(g), Sec. 17 is not polluters in the waters of the Manila Bay itself or by polluters in adjoining
hobbled by such limiting definition. As pointed out, the phrases cleanup lands and in water bodies or waterways that empty into the bay. Sec. 16
operations and accidental spills do not appear in said Sec. 17, not even in of RA 9275, on the other hand, specifically adverts to any person who
the chapter where said section is found. causes pollution in or pollutes water bodies, which may refer to an
individual or an establishment that pollutes the land mass near
the Manila Bay or the waterways, such that the contaminants eventually
end up in the bay. In this situation, the water pollution incidents are so resolve, then practically all efforts to cleanse these important bodies of
numerous and involve nameless and faceless polluters that they can water would be for naught. The DENR Secretary said as much.[38]
validly be categorized as beyond the specific pollution incident level.
Not to be ignored of course is the reality that the government Giving urgent dimension to the necessity of removing these illegal
agencies concerned are so undermanned that it would be almost structures is Art. 51 of PD 1067 or the Water Code, [39] which prohibits the
impossible to apprehend the numerous polluters of the Manila Bay. It may building of structures within a given length along banks of rivers and other
perhaps not be amiss to say that the apprehension, if any, of waterways. Art. 51 reads:
the Manila Bay polluters has been few and far between. Hence, practically
nobody has been required to contain, remove, or clean up a given water The banks of rivers and streams and the
pollution incident. In this kind of setting, it behooves the Government to shores of the seas and lakes throughout their entire
step in and undertake cleanup operations. Thus, Sec. 16 of RA 9275, length and within a zone of three (3) meters in
previously Sec. 20 of PD 1152, covers for all intents and purposes a urban areas, twenty (20) meters in agricultural areas and
general cleanup situation. forty (40) meters in forest areas, along their margins,
are subject to the easement of public use in the
The cleanup and/or restoration of the Manila Bay is only an aspect and the interest of recreation, navigation, floatage, fishing
initial stage of the long-term solution. The preservation of the water and salvage. No person shall be allowed to stay in
quality of the bay after the rehabilitation process is as important as the this zone longer than what is necessary for recreation,
cleaning phase. It is imperative then that the wastes and contaminants navigation, floatage, fishing or salvage or to build
found in the rivers, inland bays, and other bodies of water be stopped from structures of any kind. (Emphasis added.)
reaching the Manila Bay. Otherwise, any cleanup effort would just be a
futile, cosmetic exercise, for, in no time at all, the Manila Bay water quality
would again deteriorate below the ideal minimum standards set by PD Judicial notice may likewise be taken of factories and other industrial
1152, RA 9275, and other relevant laws. It thus behooves the Court to put establishments standing along or near the banks of the Pasig River, other
the heads of the petitioner-department-agencies and the bureaus and major rivers, and connecting waterways. But while they may not be
offices under them on continuing notice about, and to enjoin them to treated as unauthorized constructions, some of these establishments
perform, their mandates and duties towards cleaning up undoubtedly contribute to the pollution of the Pasig River and
the Manila Bay and preserving the quality of its water to the ideal level. waterways. The DILG and the concerned LGUs, have, accordingly, the duty
Under what other judicial discipline describes as continuing mandamus, to see to it that non-complying industrial establishments set up, within a
[36]
the Court may, under extraordinary circumstances, issue directives reasonable period, the necessary waste water treatment facilities and
with the end in view of ensuring that its decision would not be set to infrastructure to prevent their industrial discharge, including their sewage
naught by administrative inaction or indifference. In India, the doctrine of waters, from flowing into the Pasig River, other major rivers, and
continuing mandamus was used to enforce directives of the court to clean connecting waterways. After such period, non-complying establishments
up the length of the Ganges River from industrial and municipal pollution. shall be shut down or asked to transfer their operations.
[37]

The Court can take judicial notice of the presence of shanties and other At this juncture, and if only to dramatize the urgency of the need for
unauthorized structures which do not have septic tanks along the Pasig- petitioners-agencies to comply with their statutory tasks, we cite the Asian
Marikina-San Juan Rivers, the National Capital Region (NCR) (Paraaque- Development Bank-commissioned study on the garbage problem in Metro
Zapote, Las Pias) Rivers, the Navotas-Malabon-Tullahan-Tenejeros Rivers, Manila, the results of which are embodied in the The Garbage Book. As
the Meycuayan-Marilao-Obando (Bulacan) Rivers, the Talisay (Bataan) there reported, the garbage crisis in the metropolitan area is as alarming
River, the Imus (Cavite) River, the Laguna De Bay, and other minor rivers as it is shocking. Some highlights of the report:
and connecting waterways, river banks, and esteros which discharge their
waters, with all the accompanying filth, dirt, and garbage, into the major 1. As early as 2003, three land-filled dumpsites in
rivers and eventually the Manila Bay. If there is one factor responsible for Metro Manila - the Payatas, Catmon and Rodriquez
the pollution of the major river systems and the Manila Bay, these dumpsites - generate an alarming quantity of lead and
unauthorized structures would be on top of the list. And if the issue of leachate or liquid run-off. Leachate are toxic liquids that
illegal or unauthorized structures is not seriously addressed with sustained flow along the surface and seep into the earth and poison
the surface and groundwater that are used for drinking, dumping/disposal of waste and other marine litters, discharge of
aquatic life, and the environment. petroleum or residual products of petroleum of carbonaceous
materials/substances [and other] radioactive, noxious or harmful liquid,
2. The high level of fecal coliform confirms the gaseous or solid substances, from any water, land or air transport or other
presence of a large amount of human waste in the dump human-made structure.
sites and surrounding areas, which is presumably
generated by households that lack alternatives to In the light of the ongoing environmental degradation, the Court
sanitation. To say that Manila Bay needs rehabilitation is wishes to emphasize the extreme necessity for all concerned executive
an understatement. departments and agencies to immediately act and discharge their
respective official duties and obligations. Indeed, time is of the essence;
3. Most of the deadly leachate, lead and other hence, there is a need to set timetables for the performance and
dangerous contaminants and possibly strains of pathogens completion of the tasks, some of them as defined for them by law and the
seeps untreated into ground water and runs into nature of their respective offices and mandates.
the Marikina and Pasig River systems and Manila Bay.[40]
The importance of the Manila Bay as a sea resource, playground,
Given the above perspective, sufficient sanitary landfills should and as a historical landmark cannot be over-emphasized. It is not yet too
now more than ever be established as prescribed by the Ecological Solid late in the day to restore the Manila Bay to its former splendor and bring
Waste Management Act (RA 9003). Particular note should be taken of the back the plants and sea life that once thrived in its blue waters. But the
blatant violations by some LGUs and possibly the MMDA of Sec. 37, tasks ahead, daunting as they may be, could only be accomplished if those
reproduced below: mandated, with the help and cooperation of all civic-minded individuals,
Sec. 37. Prohibition against the Use of Open would put their minds to these tasks and take responsibility. This means
Dumps for Solid Waste.No open dumps shall be that the State, through petitioners, has to take the lead in the
established and operated, nor any practice or disposal of preservation and protection of the Manila Bay.
solid waste by any person, including LGUs which
[constitute] the use of open dumps for solid waste, be The era of delays, procrastination, and ad hoc measures is over.
allowed after the effectivity of this Act: Provided, further Petitioners must transcend their limitations, real or imaginary, and buckle
that no controlled dumps shall be allowed (5) years down to work before the problem at hand becomes unmanageable. Thus,
following the effectivity of this Act. (Emphasis added.) we must reiterate that different government agencies and
instrumentalities cannot shirk from their mandates; they must perform
their basic functions in cleaning up and rehabilitating the Manila Bay. We
RA 9003 took effect on February 15, 2001 and the adverted grace are disturbed by petitioners hiding behind two untenable claims: (1) that
period of five (5) years which ended on February 21, 2006 has come and there ought to be a specific pollution incident before they are required to
gone, but no single sanitary landfill which strictly complies with the act; and (2) that the cleanup of the bay is a discretionary duty.
prescribed standards under RA 9003 has yet been set up.
RA 9003 is a sweeping piece of legislation enacted to radically
In addition, there are rampant and repeated violations of Sec. 48 transform and improve waste management. It implements Sec. 16, Art. II
of RA 9003, like littering, dumping of waste matters in roads, of the 1987 Constitution, which explicitly provides that the State shall
canals, esteros, and other public places, operation of open dumps, open protect and advance the right of the people to a balanced and healthful
burning of solid waste, and the like. Some sludge companies which do not ecology in accord with the rhythm and harmony of nature.
have proper disposal facilities simply discharge sludge into the Metro
Manila sewerage system that ends up in the Manila Bay. Equally unabated So it was that in Oposa v. Factoran, Jr. the Court stated that the
are violations of Sec. 27 of RA 9275, which enjoins the pollution of water right to a balanced and healthful ecology need not even be written in the
bodies, groundwater pollution, disposal of infectious wastes from vessels, Constitution for it is assumed, like other civil and political rights
and unauthorized transport or dumping into sea waters of sewage or solid guaranteed in the Bill of Rights, to exist from the inception of mankind and
waste and of Secs. 4 and 102 of RA 8550 which proscribes the introduction it is an issue of transcendental importance with intergenerational
by human or machine of substances to the aquatic environment including implications.[41]Even assuming the absence of a categorical legal provision
specifically prodding petitioners to clean up the bay, they and the men and Bay, and other minor rivers and waterways that eventually discharge
women representing them cannot escape their obligation to future water into the Manila Bay; and the lands abutting the bay, to determine
generations of Filipinos to keep the waters of the Manila Bay clean and whether they have wastewater treatment facilities or hygienic septic tanks
clear as humanly as possible. Anything less would be a betrayal of the as prescribed by existing laws, ordinances, and rules and regulations. If
trust reposed in them. none be found, these LGUs shall be ordered to require non-complying
establishments and homes to set up said facilities or septic tanks within a
WHEREFORE, the petition is DENIED. The September 28, 2005 Decision reasonable time to prevent industrial wastes, sewage water, and human
of the CA in CA-G.R. CV No. 76528 and SP No. 74944 and the September wastes from flowing into these rivers, waterways, esteros, and the Manila
13, 2002 Decision of the RTC in Civil Case No. 1851-99 Bay, under pain of closure or imposition of fines and other sanctions.
are AFFIRMED but with MODIFICATIONS in view of subsequent
developments or supervening events in the case. The fallo of the RTC (3) As mandated by Sec. 8 of RA 9275, [43] the MWSS is directed to
Decision shall now read: provide, install, operate, and maintain the necessary adequate waste
WHEREFORE, judgment is hereby rendered ordering the water treatment facilities in Metro Manila, Rizal, and Cavite where needed
abovenamed defendant-government agencies to clean up, at the earliest possible time.
rehabilitate, and preserve Manila Bay, and restore and
maintain its waters to SB level (Class B sea waters per (4) Pursuant to RA 9275, [44] the LWUA, through the local water
Water Classification Tables under DENR Administrative districts and in coordination with the DENR, is ordered to provide, install,
Order No. 34 [1990]) to make them fit for swimming, skin- operate, and maintain sewerage and sanitation facilities and the efficient
diving, and other forms of contact recreation. and safe collection, treatment, and disposal of sewage in the provinces of
Laguna, Cavite, Bulacan, Pampanga, and Bataan where needed at the
In particular: earliest possible time.
(5) Pursuant to Sec. 65 of RA 8550, [45] the DA, through the BFAR, is
(1) Pursuant to Sec. 4 of EO 192, assigning the DENR as the primary ordered to improve and restore the marine life of the Manila Bay. It is also
agency responsible for the conservation, management, development, and directed to assist the LGUs in Metro Manila, Rizal, Cavite, Laguna, Bulacan,
proper use of the countrys environment and natural resources, and Sec. Pampanga, and Bataan in developing, using recognized methods, the
19 of RA 9275, designating the DENR as the primary government agency fisheries and aquatic resources in the Manila Bay.
responsible for its enforcement and implementation, the DENR is directed
to fully implement its Operational Plan for the Manila Bay Coastal (6) The PCG, pursuant to Secs. 4 and 6 of PD 979, and the PNP Maritime
Strategy for the rehabilitation, restoration, and conservation of the Manila Group, in accordance with Sec. 124 of RA 8550, in coordination with each
Bay at the earliest possible time. It is ordered to call regular coordination other, shall apprehend violators of PD 979, RA 8550, and other existing
meetings with concerned government departments and agencies to ensure laws and regulations designed to prevent marine pollution in
the successful implementation of the aforesaid plan of action in accordance the Manila Bay.
with its indicated completion schedules.
(7) Pursuant to Secs. 2 and 6-c of EO 513 [46] and the International
(2) Pursuant to Title XII (Local Government) of the Administrative Code of Convention for the Prevention of Pollution from Ships, the PPA is ordered
1987 and Sec. 25 of the Local Government Code of 1991, [42] the DILG, in to immediately adopt such measures to prevent the discharge and
exercising the Presidents power of general supervision and its duty to dumping of solid and liquid wastes and other ship-generated wastes into
promulgate guidelines in establishing waste management programs under the Manila Bay waters from vessels docked at ports and apprehend the
Sec. 43 of the Philippine Environment Code (PD 1152), shall direct all violators.
LGUs in Metro Manila, Rizal, Laguna, Cavite, Bulacan, Pampanga, and
Bataan to inspect all factories, commercial establishments, and private (8) The MMDA, as the lead agency and implementor of programs
homes along the banks of the major river systems in their respective areas and projects for flood control projects and drainage services in Metro
of jurisdiction, such as but not limited to the Pasig-Marikina-San Juan Manila, in coordination with the DPWH, DILG, affected LGUs, PNP Maritime
Rivers, the NCR (Paraaque-Zapote, Las Pias) Rivers, the Navotas-Malabon- Group, Housing and Urban Development Coordinating Council (HUDCC),
Tullahan-Tenejeros Rivers, the Meycauayan-Marilao-Obando (Bulacan) and other agencies, shall dismantle and remove all structures,
Rivers, the Talisay (Bataan) River, the Imus (Cavite) River, the Laguna De constructions, and other encroachments established or built in violation of
RA 7279, and other applicable laws along the Pasig-Marikina-San Juan and PPA, in line with the principle of continuing mandamus, shall, from
Rivers, the NCR (Paraaque-Zapote, Las Pias) Rivers, the Navotas-Malabon- finality of this Decision, each submit to the Court a quarterly progressive
Tullahan-Tenejeros Rivers, and connecting waterways and esteros in Metro report of the activities undertaken in accordance with this Decision.
Manila. The DPWH, as the principal implementor of programs and projects
for flood control services in the rest of the country more particularly in No costs.
Bulacan, Bataan, Pampanga, Cavite, and Laguna, in coordination with the
DILG, affected LGUs, PNP Maritime Group, HUDCC, and other concerned SO ORDERED.
government agencies, shall remove and demolish all structures,
constructions, and other encroachments built in breach of RA 7279 and
other applicable laws along the Meycauayan-Marilao-Obando (Bulacan)
Rivers, the Talisay (Bataan) River, the Imus (Cavite) River, the Laguna De
Bay, and other rivers, connecting waterways, and esteros that discharge
wastewater into the Manila Bay.

In addition, the MMDA is ordered to establish, operate, and maintain a


sanitary landfill, as prescribed by RA 9003, within a period of one (1) year
from finality of this Decision. On matters within its territorial jurisdiction
and in connection with the discharge of its duties on the maintenance of
sanitary landfills and like undertakings, it is also ordered to cause the
apprehension and filing of the appropriate criminal cases against violators
of the respective penal provisions of RA 9003, [47] Sec. 27 of RA 9275 (the
Clean Water Act), and other existing laws on pollution.
(9) The DOH shall, as directed by Art. 76 of PD 1067 and Sec. 8 of RA
9275, within one (1) year from finality of this Decision, determine if all
licensed septic and sludge companies have the proper facilities for the
treatment and disposal of fecal sludge and sewage coming from septic
tanks. The DOH shall give the companies, if found to be non-complying, a
reasonable time within which to set up the necessary facilities under pain
of cancellation of its environmental sanitation clearance.

(10) Pursuant to Sec. 53 of PD 1152, [48] Sec. 118 of RA 8550, and Sec. 56
of RA 9003,[49] the DepEd shall integrate lessons on pollution prevention,
waste management, environmental protection, and like subjects in the
school curricula of all levels to inculcate in the minds and hearts of
students and, through them, their parents and friends, the importance of
their duty toward achieving and maintaining a balanced and healthful
ecosystem in the Manila Bay and the entire Philippine archipelago.

(11) The DBM shall consider incorporating an adequate budget in the


General Appropriations Act of 2010 and succeeding years to cover the
expenses relating to the cleanup, restoration, and preservation of the
water quality of the Manila Bay, in line with the countrys development
objective to attain economic growth in a manner consistent with the
protection, preservation, and revival of our marine waters.
(12) The heads of petitioners-agencies MMDA, DENR, DepEd, DOH, DA,
DPWH, DBM, PCG, PNP Maritime Group, DILG, and also of MWSS, LWUA,
FIRST DIVISION on the training, testing certification and deployment of performing artists
abroad.

Pursuant to the EIAC's recommendations, [1] the Secretary of Labor, on


January 6, 1994, issued Department Order No. 3 establishing various
[G.R. No. 120095. August 5, 1996] procedures and requirements for screening performing artists under a new
system of training, testing, certification and deployment of the
former. Performing artists successfully hurdling the test, training and
certification requirement were to be issued an Artist's Record Book (ARB),
JMM PROMOTION AND MANAGEMENT, INC., and KARY a necessary prerequisite to processing of any contract of employment by
INTERNATIONAL, INC., petitioner, vs. HON. COURT OF the POEA. Upon request of the industry, implementation of the process,
APPEALS, HON. MA. NIEVES CONFESSOR, then Secretary of originally scheduled for April 1, 1994, was moved to October 1, 1994.
the Department of the Labor and Employment, HON. JOSE
BRILLANTES, in his capacity as acting Secretary of the Thereafter, the Department of Labor, following the EIAC's
Department of Labor and Employment and recommendation, issued a series of orders fine-tuning and implementing
HON. FELICISIMO JOSON, in his capacity as Administrator the new system. Prominent among these orders were the following
of the Philippine Overseas Employment issuances:
Administration, respondents.
1. Department Order No. 3-A, providing for additional guidelines on the
DECISION training, testing, certification and deployment of performing artists.

KAPUNAN, J.: 2. Department Order No. 3-B, pertaining to the Artist Record Book (ARB)
requirement, which could be processed only after the artist could show
The limits of government regulation under the State's Police Power proof of academic and skills training and has passed the required tests.
are once again at the vortex of the instant controversy. Assailed is the
government's power to control deployment of female entertainers to Japan 3. Department Order No. 3-E, providing the minimum salary a performing
by requiring an Artist Record Book (ARB) as a precondition to the artist ought to receive (not less than US$600.00 for those bound for
processing by the POEA of any contract for overseas employment. By Japan) and the authorized deductions therefrom.
contending that the right to overseas employment, is a property right
within the meaning of the Constitution, petitioners vigorously aver that
4. Department Order No. 3-F, providing for the guidelines on the issuance
deprivation thereof allegedly through the onerous requirement of an ARB
and use of the ARB by returning performing artists who, unlike new artists,
violates the due process clause and constitutes an invalid exercise of the
shall only undergo a Special Orientation Program (shorter than the basic
police power.
program) although they must pass the academic test.
The factual antecedents are undisputed.
In Civil Case No. 95-72750, the Federation of Entertainment Talent
Following the much-publicized death of Maricris Sioson in 1991,
Managers of the Philippines (FETMOP), on January 27, 1995 filed a class
former President Corazon C. Aquino ordered a total ban against the
suit assailing these department orders, principally contending that said
deployment of performing artists to Japan and other foreign
orders 1) violated the constitutional right to travel; 2) abridged existing
destinations. The ban was, however, rescinded after leaders of the
contracts for employment; and 3) deprived individual artists of their
overseas employment industry promised to extend full support for a
licenses without due process of law.FETMOP, likewise, averred that the
program aimed at removing kinks in the system of deployment.In its
issuance of the Artist Record Book (ARB) was discriminatory and illegal
place, the government, through the Secretary of Labor and Employment,
and "in gross violation of the constitutional right... to life liberty and
subsequently issued Department Order No. 28, creating the Entertainment
property." Said Federation consequently prayed for the issuance of a writ
Industry Advisory Council (EIAC), which was tasked with issuing guidelines
of preliminary injunction against the aforestated orders.
On February 2, 1992, JMM Promotion and Management, Inc. and Kary A thorough review of the facts and circumstances leading to the
International, Inc., herein petitioners, filed a Motion for Intervention in issuance of the assailed orders compels us to rule that the Artist Record
said civil case, which was granted by the trial court in an Order dated 15 Book requirement and the questioned Department Order related to its
February, 1995. issuance were issued by the Secretary of Labor pursuant to a valid
exercise of the police power.
However, on February 21, 1995, the trial court issued an Order
denying petitioners' prayer for a writ of preliminary injunction and In 1984, the Philippines emerged as the largest labor sending country
dismissed the complaint. in Asia dwarfing the labor export of countries with mammoth populations
such as India and China. According to the National Statistics Office,
On appeal from the trial court's Order, respondent court, in CA G.R. this diaspora was augmented annually by over 450,000 documented and
SP No. 36713 dismissed the same. Tracing the circumstances which led to clandestine or illegal (undocumented) workers who left the country for
the issuance of the ARB requirement and the assailed Department Order, various destinations abroad, lured by higher salaries, better work
respondent court concluded that the issuances constituted a valid exercise opportunities and sometimes better living conditions.
by the state of the police power.
Of the hundreds of thousands of workers who left the country for
We agree. greener pastures in the last few years, women composed slightly close to
The latin maxim salus populi est suprema lex embodies the character half of those deployed, constituting 47% between 1987-1991, exceeding
of the entire spectrum of public laws aimed at promoting the general this proportion (58%) by the end of 1991, [6] the year former President
welfare of the people under the State's police power. As an inherent Aquino instituted the ban on deployment of performing artists to Japan
attribute of sovereignty which virtually "extends to all public needs," [2] this and other countries as a result of the gruesome death of Filipino
"least limitable"[3] of governmental powers grants a wide panoply of entertainer Maricris Sioson.
instruments through which the state, as parens patriae gives effect to a It was during the same period that this Court took judicial notice not
host of its regulatory powers. only of the trend, but also of the fact that most of our women, a large
Describing the nature and scope of the police power, Justice Malcolm, number employed as domestic helpers and entertainers, worked under
in the early case of Rubi v. Provincial Board of Mindoro[4] wrote: exploitative conditions "marked by physical and personal abuse." [7] Even
then, we noted that "[t]he sordid tales of maltreatment suffered by
migrant Filipina workers, even rape and various forms of torture,
"The police power of the State," one court has said...'is a power confirmed by testimonies of returning workers" compelled "urgent
coextensive with self-protection, and is not inaptly termed 'the law of government action."[8]
overruling necessity.' It may be said to be that inherent and plenary power
in the state which enables it to prohibit all things hurtful to the comfort, Pursuant to the alarming number of reports that a significant number
safety and welfare of society.' Carried onward by the current of legislature, of Filipina performing artists ended up as prostitutes abroad (many of
the judiciary rarely attempts to dam the onrushing power of legislative whom were beaten, drugged and forced into prostitution), and following
discretion, provided the purposes of the law do not go beyond the great the deaths of a number of these women, the government began instituting
principles that mean security for the public welfare or do not arbitrarily measures aimed at deploying only those individuals who met set standards
interfere with the right of the individual."[5] which would qualify them as legitimate performing artists. In spite of these
measures, however, a number of our countrymen have nonetheless fallen
Thus, police power concerns government enactments which precisely victim to unscrupulous recruiters, ending up as virtual slaves controlled by
interfere with personal liberty or property in order to promote the general foreign crime syndicates and forced into jobs other than those indicated in
welfare or the common good. As the assailed Department Order enjoys a their employment contracts. Worse, some of our women have been forced
presumed validity, it follows that the burden rests upon petitioners to into prostitution.
demonstrate that the said order, particularly, its ARB requirement, does
Thus, after a number of inadequate and failed accreditation schemes,
not enhance the public welfare or was exercised arbitrarily or
the Secretary of Labor issued on August 16, 1993, D.O. No. 28,
unreasonably.
establishing the Entertainment Industry Advisory Council (EIAC), the
policy advisory body of DOLE on entertainment industry matters. [9] Acting
on the recommendations of the said body, the Secretary of Labor, on they are aimed at enhancing the safety and security of entertainers and
January 6, 1994, issued the assailed orders. These orders embodied artists bound for Japan and other destinations, without stifling the
EIAC's Resolution No. 1, which called for guidelines on screening, testing industry's concerns for expansion and growth.
and accrediting performing overseas Filipino artists. Significantly, as the
respondent court noted, petitioners were duly represented in the EIAC, In any event, apart from the State's police power, the Constitution
[10]
which gave the recommendations on which the ARB and other itself mandates government to extend the fullest protection to our
requirements were based. overseas workers. The basic constitutional statement on labor, embodied
in Section 18 of Article II of the Constitution provides:
Clearly, the welfare of Filipino performing artists, particularly the
women was paramount in the issuance of Department Order No. 3. Short Sec. 18. The State affirms labor as a primary social economic force. It
of a total and absolute ban against the deployment of performing artists to shall protect the rights of workers and promote their welfare.
"high risk" destinations, a measure which would only drive recruitment
further underground, the new scheme at the very least rationalizes the
More emphatically, the social justice provision on labor of the 1987
method of screening performing artists by requiring reasonable
Constitution in its first paragraph states:
educational and artistic skills from them and limits deployment to only
those individuals adequately prepared for the unpredictable demands of The State shall afford full protection to labor, local and overseas,
employment as artists abroad. It cannot be gainsaid that this scheme at organized and unorganized and promote full employment and equality of
least lessens the room for exploitation by unscrupulous individuals and employment opportunities for all.
agencies.
Obviously, protection to labor does not indicate promotion of
Moreover, here or abroad, selection of performing artists is usually employment alone. Under the welfare and social justice provisions of the
accomplished by auditions, where those deemed unfit are usually weeded Constitution, the promotion of full employment, while desirable, cannot
out through a process which is inherently subjective and vulnerable to bias take a backseat to the government's constitutional duty to provide
and differences in taste. The ARB requirement goes one step further, mechanisms for the protection of our workforce, local or overseas. As this
however, attempting to minimize the subjectivity of the process by Court explained in Philippine Association of Service Exporters (PASEI) v.
defining the minimum skills required from entertainers and performing Drilon,[11] in reference to the recurring problems faced by our overseas
artists. As the Solicitor General observed, this should be easily met by workers:
experienced artists possessing merely basic skills. The tests are aimed at
segregating real artists or performers from those passing themselves off
What concerns the Constitution more paramountly is that such an
as such, eager to accept any available job and therefore exposing
employment be above all, decent, just, and humane. It is bad enough that
themselves to possible exploitation.
the country has to send its sons and daughters to strange lands because it
As to the other provisions of Department Order No. 3 questioned by cannot satisfy their employment needs at home. Under these
petitioners, we see nothing wrong with the requirement for document and circumstances, the Government is duty-bound to insure that our toiling
booking confirmation (D.O. 3-C), a minimum salary scale (D.O. 3-E), or expatriates have adequate protection, personally and economically, while
the requirement for registration of returning performers. The requirement away from home.
for a venue certificate or other documents evidencing the place and nature
of work allows the government closer monitoring of foreign employers and We now go to petitioners' assertion that the police power cannot,
helps keep our entertainers away from prostitution fronts and other nevertheless, abridge the right of our performing workers to return to
worksites associated with unsavory, immoral, illegal or exploitative work abroad after having earlier qualified under the old process, because,
practices.Parenthetically, none of these issuances appear to us, by any having previously been accredited, their accreditation became a property
stretch of the imagination, even remotely unreasonable or arbitrary. They right," protected by the due process clause. We find this contention
address a felt need of according greater protection for an oft-exploited untenable.
segment of our OCW's. They respond to the industry's demand for clearer
and more practicable rules and guidelines. Many of these provisions were A profession, trade or calling is a property right within the meaning of
fleshed out following recommendations by, and after consultations with, our constitutional guarantees. One cannot be deprived of the right to work
the affected sectors and non-government organizations. On the whole, and the right to make a living because these rights are property rights, the
arbitrary and unwarranted deprivation of which normally constitutes an which is limited to the object to which it is directed or by the territory in
actionable wrong.[12] which it is to operate. It does not require absolute equality, but merely
that all persons be treated alike under like conditions both as to privileges
Nevertheless, no right is absolute, and the proper regulation of a conferred and liabilities imposed. [16]We have held, time and again, that the
profession, calling, business or trade has always been upheld as a equal protection clause of the Constitution does not forbid classification for
legitimate subject of a valid exercise of the police power by the state so long as such classification is based on real and substantial differences
particularly when their conduct affects either the execution of legitimate having a reasonable relation to the subject of the particular legislation.
governmental functions, the preservation of the State, the public health [17]
If classification is germane to the purpose of the law, concerns all
and welfare and public morals. According to the maxim, sic utere tuo ut members of the class, and applies equally to present and future
alienum non laedas, it must of course be within the legitimate range of conditions, the classification does not violate the equal protection
legislative action to define the mode and manner in which every one may guarantee.
so use his own property so as not to pose injury to himself or others. [13]
In the case at bar, the challenged Department Order clearly applies to
In any case, where the liberty curtailed affects at most the rights of all performing artists and entertainers destined for jobs abroad. These
property, the permissible scope of regulatory measures is certainly much orders, we stressed hereinbefore, further the Constitutional mandate
wider.[14] To pretend that licensing or accreditation requirements violates requiring Government to protect our workforce, particularly those who
the due process clause is to ignore the settled practice, under the mantle may be prone to abuse and exploitation as they are beyond the physical
of the police power, of regulating entry to the practice of various trades or reach of government regulatory agencies. The tragic incidents must
professions. Professionals leaving for abroad are required to pass rigid somehow stop, but short of absolutely curtailing the right of these
written and practical exams before they are deemed fit to practice their performers and entertainers to work abroad, the assailed measures enable
trade. Seamen are required to take tests determining their our government to assume a measure of control.
seamanship. Locally, the Professional Regulation Commission has began to
require previously licensed doctors and other professionals to furnish WHEREFORE, finding no reversible error in the decision sought to be
documentary proof that they had either re-trained or had undertaken reviewed, petition is hereby DENIED.
continuing education courses as a requirement for renewal of their
licenses. It is not claimed that these requirements pose an unwarranted SO ORDERED.
deprivation of a property right under the due process clause. So long as Padilla (Chairman), Bellosillo, Vitug, and Hermosisima, Jr.,
Professionals and other workers meet reasonable regulatory standards no JJ., concur.
such deprivation exists.

Finally, it is a futile gesture on the part of petitioners to invoke the


non-impairment clause of the Constitution to support their argument that
the government cannot enact the assailed regulatory measures because
they abridge the freedom to contract. In Philippine Association of Service
Exporters, Inc. vs. Drilon, we held that "[t]he non-impairment clause of
the Constitution... must yield to the loftier purposes targeted by the
government."[15] Equally important, into every contract is read provisions
of existing law, and always, a reservation of the police power for so long
as the agreement deals with a subject impressed with the public welfare.

A last point. Petitioners suggest that the singling out of entertainers


and performing artists under the assailed department orders constitutes
class legislation which violates the equal protection clause of the
Constitution. We do not agree.

The equal protection clause is directed principally against undue favor


and individual or class privilege. It is not intended to prohibit legislation
Republic of the Philippines to impose local taxes and license fees. This, in contravention of the
SUPREME COURT constitutionally enshrined principle of local autonomy;
Manila
C. It violates the equal protection clause of the constitution in that
EN BANC it legalizes PAGCOR — conducted gambling, while most other
forms of gambling are outlawed, together with prostitution, drug
trafficking and other vices;

G.R. No. 91649 May 14, 1991 D. It violates the avowed trend of the Cory government away from
monopolistic and crony economy, and toward free enterprise and
privatization. (p. 2, Amended Petition; p. 7, Rollo)
ATTORNEYS HUMBERTO BASCO, EDILBERTO BALCE, SOCRATES
MARANAN AND LORENZO SANCHEZ,petitioners,
vs. In their Second Amended Petition, petitioners also claim that PD 1869 is
PHILIPPINE AMUSEMENTS AND GAMING CORPORATION contrary to the declared national policy of the "new restored democracy"
(PAGCOR), respondent. and the people's will as expressed in the 1987 Constitution. The decree is
said to have a "gambling objective" and therefore is contrary to Sections
11, 12 and 13 of Article II, Sec. 1 of Article VIII and Section 3 (2) of
H.B. Basco & Associates for petitioners.
Article XIV, of the present Constitution (p. 3, Second Amended Petition; p.
Valmonte Law Offices collaborating counsel for petitioners.
21, Rollo).
Aguirre, Laborte and Capule for respondent PAGCOR.

The procedural issue is whether petitioners, as taxpayers and practicing


lawyers (petitioner Basco being also the Chairman of the Committee on
Laws of the City Council of Manila), can question and seek the annulment
of PD 1869 on the alleged grounds mentioned above.
PARAS, J.:
The Philippine Amusements and Gaming Corporation (PAGCOR) was
A TV ad proudly announces: created by virtue of P.D. 1067-A dated January 1, 1977 and was granted a
franchise under P.D. 1067-B also dated January 1, 1977 "to establish,
"The new PAGCOR — responding through responsible gaming." operate and maintain gambling casinos on land or water within the
territorial jurisdiction of the Philippines." Its operation was originally
conducted in the well known floating casino "Philippine Tourist." The
But the petitioners think otherwise, that is why, they filed the instant operation was considered a success for it proved to be a potential source
petition seeking to annul the Philippine Amusement and Gaming of revenue to fund infrastructure and socio-economic projects, thus, P.D.
Corporation (PAGCOR) Charter — PD 1869, because it is allegedly contrary 1399 was passed on June 2, 1978 for PAGCOR to fully attain this
to morals, public policy and order, and because — objective.

A. It constitutes a waiver of a right prejudicial to a third person Subsequently, on July 11, 1983, PAGCOR was created under P.D. 1869 to
with a right recognized by law. It waived the Manila City enable the Government to regulate and centralize all games of chance
government's right to impose taxes and license fees, which is authorized by existing franchise or permitted by law, under the following
recognized by law; declared policy —

B. For the same reason stated in the immediately preceding


paragraph, the law has intruded into the local government's right
Sec. 1. Declaration of Policy. — It is hereby declared to be the But the petitioners, are questioning the validity of P.D. No. 1869. They
policy of the State to centralize and integrate all games of chance allege that the same is "null and void" for being "contrary to morals, public
not heretofore authorized by existing franchises or permitted by policy and public order," monopolistic and tends toward "crony economy",
law in order to attain the following objectives: and is violative of the equal protection clause and local autonomy as well
as for running counter to the state policies enunciated in Sections 11
(a) To centralize and integrate the right and authority to operate (Personal Dignity and Human Rights), 12 (Family) and 13 (Role of Youth)
and conduct games of chance into one corporate entity to be of Article II, Section 1 (Social Justice) of Article XIII and Section 2
controlled, administered and supervised by the Government. (Educational Values) of Article XIV of the 1987 Constitution.

(b) To establish and operate clubs and casinos, for amusement This challenge to P.D. No. 1869 deserves a searching and thorough
and recreation, including sports gaming pools, (basketball, scrutiny and the most deliberate consideration by the Court, involving as it
football, lotteries, etc.) and such other forms of amusement and does the exercise of what has been described as "the highest and most
recreation including games of chance, which may be allowed by delicate function which belongs to the judicial department of the
law within the territorial jurisdiction of the Philippines and which government." (State v. Manuel, 20 N.C. 144; Lozano v. Martinez, 146
will: (1) generate sources of additional revenue to fund SCRA 323).
infrastructure and socio-civic projects, such as flood control
programs, beautification, sewerage and sewage projects, Tulungan As We enter upon the task of passing on the validity of an act of a co-
ng Bayan Centers, Nutritional Programs, Population Control and equal and coordinate branch of the government We need not be reminded
such other essential public services; (2) create recreation and of the time-honored principle, deeply ingrained in our jurisprudence, that a
integrated facilities which will expand and improve the country's statute is presumed to be valid. Every presumption must be indulged in
existing tourist attractions; and (3) minimize, if not totally favor of its constitutionality. This is not to say that We approach Our task
eradicate, all the evils, malpractices and corruptions that are with diffidence or timidity. Where it is clear that the legislature or the
normally prevalent on the conduct and operation of gambling clubs executive for that matter, has over-stepped the limits of its authority
and casinos without direct government involvement. (Section 1, under the constitution, We should not hesitate to wield the axe and let it
P.D. 1869) fall heavily, as fall it must, on the offending statute (Lozano v.
Martinez, supra).
To attain these objectives PAGCOR is given territorial jurisdiction all over
the Philippines. Under its Charter's repealing clause, all laws, decrees, In Victoriano v. Elizalde Rope Workers' Union, et al, 59 SCRA 54, the Court
executive orders, rules and regulations, inconsistent therewith, are thru Mr. Justice Zaldivar underscored the —
accordingly repealed, amended or modified.
. . . thoroughly established principle which must be followed in all
It is reported that PAGCOR is the third largest source of government cases where questions of constitutionality as obtain in the instant
revenue, next to the Bureau of Internal Revenue and the Bureau of cases are involved. All presumptions are indulged in favor of
Customs. In 1989 alone, PAGCOR earned P3.43 Billion, and directly constitutionality; one who attacks a statute alleging
remitted to the National Government a total of P2.5 Billion in form of unconstitutionality must prove its invalidity beyond a reasonable
franchise tax, government's income share, the President's Social Fund and doubt; that a law may work hardship does not render it
Host Cities' share. In addition, PAGCOR sponsored other socio-cultural and unconstitutional; that if any reasonable basis may be conceived
charitable projects on its own or in cooperation with various governmental which supports the statute, it will be upheld and the challenger
agencies, and other private associations and organizations. In its 3 1/2 must negate all possible basis; that the courts are not concerned
years of operation under the present administration, PAGCOR remitted to with the wisdom, justice, policy or expediency of a statute and that
the government a total of P6.2 Billion. As of December 31, 1989, PAGCOR a liberal interpretation of the constitution in favor of the
was employing 4,494 employees in its nine (9) casinos nationwide, directly constitutionality of legislation should be adopted. (Danner v. Hass,
supporting the livelihood of Four Thousand Four Hundred Ninety-Four 194 N.W. 2nd534, 539; Spurbeck v. Statton, 106 N.W. 2nd 660,
(4,494) families. 663; 59 SCRA 66; see also e.g. Salas v. Jarencio, 46 SCRA 734,
739 [1970]; Peralta v. Commission on Elections, 82 SCRA 30, 55 Gambling in all its forms, unless allowed by law, is generally prohibited.
[1978]; and Heirs of Ordona v. Reyes, 125 SCRA 220, 241-242 But the prohibition of gambling does not mean that the Government
[1983] cited in Citizens Alliance for Consumer Protection v. Energy cannot regulate it in the exercise of its police power.
Regulatory Board, 162 SCRA 521, 540)
The concept of police power is well-established in this jurisdiction. It has
Of course, there is first, the procedural issue. The respondents are been defined as the "state authority to enact legislation that may interfere
questioning the legal personality of petitioners to file the instant petition. with personal liberty or property in order to promote the general welfare."
(Edu v. Ericta, 35 SCRA 481, 487) As defined, it consists of (1) an
Considering however the importance to the public of the case at bar, and imposition or restraint upon liberty or property, (2) in order to foster the
in keeping with the Court's duty, under the 1987 Constitution, to common good. It is not capable of an exact definition but has been,
determine whether or not the other branches of government have kept purposely, veiled in general terms to underscore its all-comprehensive
themselves within the limits of the Constitution and the laws and that they embrace. (Philippine Association of Service Exporters, Inc. v. Drilon, 163
have not abused the discretion given to them, the Court has brushed aside SCRA 386).
technicalities of procedure and has taken cognizance of this petition.
(Kapatiran ng mga Naglilingkod sa Pamahalaan ng Pilipinas Inc. v. Tan, Its scope, ever-expanding to meet the exigencies of the times, even to
163 SCRA 371) anticipate the future where it could be done, provides enough room for an
efficient and flexible response to conditions and circumstances thus
With particular regard to the requirement of proper party as assuming the greatest benefits. (Edu v. Ericta, supra)
applied in the cases before us, We hold that the same is satisfied
by the petitioners and intervenors because each of them has It finds no specific Constitutional grant for the plain reason that it does not
sustained or is in danger of sustaining an immediate injury as a owe its origin to the charter. Along with the taxing power and eminent
result of the acts or measures complained of. And even if, strictly domain, it is inborn in the very fact of statehood and sovereignty. It is a
speaking they are not covered by the definition, it is still within the fundamental attribute of government that has enabled it to perform the
wide discretion of the Court to waive the requirement and so most vital functions of governance. Marshall, to whom the expression has
remove the impediment to its addressing and resolving the serious been credited, refers to it succinctly as the plenary power of the state "to
constitutional questions raised. govern its citizens". (Tribe, American Constitutional Law, 323, 1978). The
police power of the State is a power co-extensive with self-protection and
In the first Emergency Powers Cases, ordinary citizens and is most aptly termed the "law of overwhelming necessity." (Rubi v.
taxpayers were allowed to question the constitutionality of several Provincial Board of Mindoro, 39 Phil. 660, 708) It is "the most essential,
executive orders issued by President Quirino although they were insistent, and illimitable of powers." (Smith Bell & Co. v. National, 40 Phil.
involving only an indirect and general interest shared in common 136) It is a dynamic force that enables the state to meet the agencies of
with the public. The Court dismissed the objection that they were the winds of change.
not proper parties and ruled that "the transcendental importance
to the public of these cases demands that they be settled promptly What was the reason behind the enactment of P.D. 1869?
and definitely, brushing aside, if we must technicalities of
procedure." We have since then applied the exception in many P.D. 1869 was enacted pursuant to the policy of the government to
other cases. (Association of Small Landowners in the Philippines, "regulate and centralize thru an appropriate institution all games of chance
Inc. v. Sec. of Agrarian Reform, 175 SCRA 343). authorized by existing franchise or permitted by law" (1st whereas clause,
PD 1869). As was subsequently proved, regulating and centralizing
Having disposed of the procedural issue, We will now discuss the gambling operations in one corporate entity — the PAGCOR, was beneficial
substantive issues raised. not just to the Government but to society in general. It is a reliable source
of much needed revenue for the cash strapped Government. It provided
funds for social impact projects and subjected gambling to "close scrutiny,
regulation, supervision and control of the Government" (4th Whereas
Clause, PD 1869). With the creation of PAGCOR and the direct intervention "general legislative powers" (Asuncion v. Yriantes, 28 Phil. 67; Merdanillo
of the Government, the evil practices and corruptions that go with v. Orandia, 5 SCRA 541). Congress, therefore, has the power of control
gambling will be minimized if not totally eradicated. Public welfare, then, over Local governments (Hebron v. Reyes, G.R. No. 9124, July 2, 1950).
lies at the bottom of the enactment of PD 1896. And if Congress can grant the City of Manila the power to tax certain
matters, it can also provide for exemptions or even take back the power.
Petitioners contend that P.D. 1869 constitutes a waiver of the right of the
City of Manila to impose taxes and legal fees; that the exemption clause in (c) The City of Manila's power to impose license fees on gambling, has
P.D. 1869 is violative of the principle of local autonomy. They must be long been revoked. As early as 1975, the power of local governments to
referring to Section 13 par. (2) of P.D. 1869 which exempts PAGCOR, as regulate gambling thru the grant of "franchise, licenses or permits" was
the franchise holder from paying any "tax of any kind or form, income or withdrawn by P.D. No. 771 and was vested exclusively on the National
otherwise, as well as fees, charges or levies of whatever nature, whether Government, thus:
National or Local."
Sec. 1. Any provision of law to the contrary notwithstanding, the
(2) Income and other taxes. — a) Franchise Holder: No tax of any authority of chartered cities and other local governments to issue
kind or form, income or otherwise as well as fees, charges or license, permit or other form of franchise to operate, maintain and
levies of whatever nature, whether National or Local, shall be establish horse and dog race tracks, jai-alai and other forms of
assessed and collected under this franchise from the Corporation; gambling is hereby revoked.
nor shall any form or tax or charge attach in any way to the
earnings of the Corporation, except a franchise tax of five (5%) Sec. 2. Hereafter, all permits or franchises to operate, maintain
percent of the gross revenues or earnings derived by the and establish, horse and dog race tracks, jai-alai and other forms
Corporation from its operations under this franchise. Such tax shall of gambling shall be issued by the national government upon
be due and payable quarterly to the National Government and proper application and verification of the qualification of the
shall be in lieu of all kinds of taxes, levies, fees or assessments of applicant . . .
any kind, nature or description, levied, established or collected by
any municipal, provincial or national government authority
Therefore, only the National Government has the power to issue "licenses
(Section 13 [2]).
or permits" for the operation of gambling. Necessarily, the power to
demand or collect license fees which is a consequence of the issuance of
Their contention stated hereinabove is without merit for the following "licenses or permits" is no longer vested in the City of Manila.
reasons:
(d) Local governments have no power to tax instrumentalities of the
(a) The City of Manila, being a mere Municipal corporation has no inherent National Government. PAGCOR is a government owned or controlled
right to impose taxes (Icard v. City of Baguio, 83 Phil. 870; City of Iloilo v. corporation with an original charter, PD 1869. All of its shares of stocks
Villanueva, 105 Phil. 337; Santos v. Municipality of Caloocan, 7 SCRA are owned by the National Government. In addition to its corporate powers
643). Thus, "the Charter or statute must plainly show an intent to confer (Sec. 3, Title II, PD 1869) it also exercises regulatory powers thus:
that power or the municipality cannot assume it" (Medina v. City of
Baguio, 12 SCRA 62). Its "power to tax" therefore must always yield to a
Sec. 9. Regulatory Power. — The Corporation shall maintain a
legislative act which is superior having been passed upon by the state
Registry of the affiliated entities, and shall exercise all the powers,
itself which has the "inherent power to tax" (Bernas, the Revised [1973]
authority and the responsibilities vested in the Securities and
Philippine Constitution, Vol. 1, 1983 ed. p. 445).
Exchange Commission over such affiliating entities mentioned
under the preceding section, including, but not limited to
(b) The Charter of the City of Manila is subject to control by Congress. It amendments of Articles of Incorporation and By-Laws, changes in
should be stressed that "municipal corporations are mere creatures of corporate term, structure, capitalization and other matters
Congress" (Unson v. Lacson, G.R. No. 7909, January 18, 1957) which has concerning the operation of the affiliated entities, the provisions of
the power to "create and abolish municipal corporations" due to its
the Corporation Code of the Philippines to the contrary Sec. 5. Each local government unit shall have the power to create
notwithstanding, except only with respect to original incorporation. its own source of revenue and to levy taxes, fees, and other
charges subject to such guidelines and limitation as the congress
PAGCOR has a dual role, to operate and to regulate gambling casinos. The may provide, consistent with the basic policy on local autonomy.
latter role is governmental, which places it in the category of an agency or Such taxes, fees and charges shall accrue exclusively to the local
instrumentality of the Government. Being an instrumentality of the government. (emphasis supplied)
Government, PAGCOR should be and actually is exempt from local taxes.
Otherwise, its operation might be burdened, impeded or subjected to The power of local government to "impose taxes and fees" is always
control by a mere Local government. subject to "limitations" which Congress may provide by law. Since PD 1869
remains an "operative" law until "amended, repealed or revoked" (Sec. 3,
The states have no power by taxation or otherwise, to retard, Art. XVIII, 1987 Constitution), its "exemption clause" remains as an
impede, burden or in any manner control the operation of exception to the exercise of the power of local governments to impose
constitutional laws enacted by Congress to carry into execution the taxes and fees. It cannot therefore be violative but rather is consistent
powers vested in the federal government. (MC Culloch v. Marland, with the principle of local autonomy.
4 Wheat 316, 4 L Ed. 579)
Besides, the principle of local autonomy under the 1987 Constitution
This doctrine emanates from the "supremacy" of the National Government simply means "decentralization" (III Records of the 1987 Constitutional
over local governments. Commission, pp. 435-436, as cited in Bernas, The Constitution of the
Republic of the Philippines, Vol. II, First Ed., 1988, p. 374). It does not
make local governments sovereign within the state or an "imperium in
Justice Holmes, speaking for the Supreme Court, made reference
imperio."
to the entire absence of power on the part of the States to touch,
in that way (taxation) at least, the instrumentalities of the United
States (Johnson v. Maryland, 254 US 51) and it can be agreed Local Government has been described as a political subdivision of
that no state or political subdivision can regulate a federal a nation or state which is constituted by law and has substantial
instrumentality in such a way as to prevent it from consummating control of local affairs. In a unitary system of government, such as
its federal responsibilities, or even to seriously burden it in the the government under the Philippine Constitution, local
accomplishment of them. (Antieau, Modern Constitutional Law, governments can only be an intra sovereign subdivision of one
Vol. 2, p. 140, emphasis supplied) sovereign nation, it cannot be an imperium in imperio. Local
government in such a system can only mean a measure of
decentralization of the function of government. (emphasis
Otherwise, mere creatures of the State can defeat National policies thru
supplied)
extermination of what local authorities may perceive to be undesirable
activities or enterprise using the power to tax as "a tool for regulation"
(U.S. v. Sanchez, 340 US 42). As to what state powers should be "decentralized" and what may be
delegated to local government units remains a matter of policy, which
concerns wisdom. It is therefore a political question. (Citizens Alliance for
The power to tax which was called by Justice Marshall as the "power to
Consumer Protection v. Energy Regulatory Board, 162 SCRA 539).
destroy" (Mc Culloch v. Maryland, supra) cannot be allowed to defeat an
instrumentality or creation of the very entity which has the inherent power
to wield it. What is settled is that the matter of regulating, taxing or otherwise dealing
with gambling is a State concern and hence, it is the sole prerogative of
the State to retain it or delegate it to local governments.
(e) Petitioners also argue that the Local Autonomy Clause of the
Constitution will be violated by P.D. 1869. This is a pointless argument.
Article X of the 1987 Constitution (on Local Autonomy) provides: As gambling is usually an offense against the State, legislative
grant or express charter power is generally necessary to empower
the local corporation to deal with the subject. . . . In the absence
of express grant of power to enact, ordinance provisions on this that would insure if the rule laid down were made mathematically
subject which are inconsistent with the state laws are void. (Ligan exact. (Dominican Hotel v. Arizona, 249 US 2651).
v. Gadsden, Ala App. 107 So. 733 Ex-Parte Solomon, 9, Cals. 440,
27 PAC 757 following in re Ah You, 88 Cal. 99, 25 PAC 974, 22 Am Anent petitioners' claim that PD 1869 is contrary to the "avowed trend of
St. Rep. 280, 11 LRA 480, as cited in Mc Quinllan Vol. 3 Ibid, p. the Cory Government away from monopolies and crony economy and
548, emphasis supplied) toward free enterprise and privatization" suffice it to state that this is not a
ground for this Court to nullify P.D. 1869. If, indeed, PD 1869 runs counter
Petitioners next contend that P.D. 1869 violates the equal protection to the government's policies then it is for the Executive Department to
clause of the Constitution, because "it legalized PAGCOR — conducted recommend to Congress its repeal or amendment.
gambling, while most gambling are outlawed together with prostitution,
drug trafficking and other vices" (p. 82, Rollo). The judiciary does not settle policy issues. The Court can only
declare what the law is and not what the law should
We, likewise, find no valid ground to sustain this contention. The be.1âwphi1 Under our system of government, policy issues are
petitioners' posture ignores the well-accepted meaning of the clause within the domain of the political branches of government and of
"equal protection of the laws." The clause does not preclude classification the people themselves as the repository of all state power.
of individuals who may be accorded different treatment under the law as (Valmonte v. Belmonte, Jr., 170 SCRA 256).
long as the classification is not unreasonable or arbitrary (Itchong v.
Hernandez, 101 Phil. 1155). A law does not have to operate in equal force On the issue of "monopoly," however, the Constitution provides that:
on all persons or things to be conformable to Article III, Section 1 of the
Constitution (DECS v. San Diego, G.R. No. 89572, December 21, 1989).
Sec. 19. The State shall regulate or prohibit monopolies when
public interest so requires. No combinations in restraint of trade or
The "equal protection clause" does not prohibit the Legislature from unfair competition shall be allowed. (Art. XII, National Economy
establishing classes of individuals or objects upon which different rules and Patrimony)
shall operate (Laurel v. Misa, 43 O.G. 2847). The Constitution does not
require situations which are different in fact or opinion to be treated in law
It should be noted that, as the provision is worded, monopolies are not
as though they were the same (Gomez v. Palomar, 25 SCRA 827).
necessarily prohibited by the Constitution. The state must still decide
whether public interest demands that monopolies be regulated or
Just how P.D. 1869 in legalizing gambling conducted by PAGCOR is prohibited. Again, this is a matter of policy for the Legislature to decide.
violative of the equal protection is not clearly explained in the petition. The
mere fact that some gambling activities like cockfighting (P.D 449) horse
On petitioners' allegation that P.D. 1869 violates Sections 11 (Personality
racing (R.A. 306 as amended by RA 983), sweepstakes, lotteries and races
Dignity) 12 (Family) and 13 (Role of Youth) of Article II; Section 13 (Social
(RA 1169 as amended by B.P. 42) are legalized under certain conditions,
Justice) of Article XIII and Section 2 (Educational Values) of Article XIV of
while others are prohibited, does not render the applicable laws, P.D. 1869
the 1987 Constitution, suffice it to state also that these are merely
for one, unconstitutional.
statements of principles and, policies. As such, they are basically not self-
executing, meaning a law should be passed by Congress to clearly define
If the law presumably hits the evil where it is most felt, it is not to and effectuate such principles.
be overthrown because there are other instances to which it might
have been applied. (Gomez v. Palomar, 25 SCRA 827)
In general, therefore, the 1935 provisions were not intended to be
self-executing principles ready for enforcement through the courts.
The equal protection clause of the 14th Amendment does not They were rather directives addressed to the executive and the
mean that all occupations called by the same name must be legislature. If the executive and the legislature failed to heed the
treated the same way; the state may do what it can to prevent directives of the articles the available remedy was not judicial or
which is deemed as evil and stop short of those cases in which political. The electorate could express their displeasure with the
harm to the few concerned is not less than the harm to the public
failure of the executive and the legislature through the language of does not necessarily mean that the same are directly attributable to
the ballot. (Bernas, Vol. II, p. 2) gambling. Gambling may have been the antecedent, but certainly not
necessarily the cause. For the same consequences could have been
Every law has in its favor the presumption of constitutionality (Yu Cong preceded by an overdose of food, drink, exercise, work, and even sex.
Eng v. Trinidad, 47 Phil. 387; Salas v. Jarencio, 48 SCRA 734; Peralta v.
Comelec, 82 SCRA 30; Abbas v. Comelec, 179 SCRA 287). Therefore, for WHEREFORE, the petition is DISMISSED for lack of merit.
PD 1869 to be nullified, it must be shown that there is a clear and
unequivocal breach of the Constitution, not merely a doubtful and SO ORDERED.
equivocal one. In other words, the grounds for nullity must be clear and
beyond reasonable doubt. (Peralta v. Comelec, supra) Those who petition
Fernan, C.J., Narvasa, Gutierrez, Jr., Cruz, Feliciano, Gancayco, Bidin,
this Court to declare a law, or parts thereof, unconstitutional must clearly
Sarmiento, Griño-Aquino, Medialdea, Regalado and Davide, Jr., JJ.,
establish the basis for such a declaration. Otherwise, their petition must
concur.
fail. Based on the grounds raised by petitioners to challenge the
constitutionality of P.D. 1869, the Court finds that petitioners have failed
to overcome the presumption. The dismissal of this petition is therefore,
inevitable. But as to whether P.D. 1869 remains a wise legislation
considering the issues of "morality, monopoly, trend to free enterprise,
privatization as well as the state principles on social justice, role of youth
and educational values" being raised, is up for Congress to determine.

As this Court held in Citizens' Alliance for Consumer Protection v. Energy


Regulatory Board, 162 SCRA 521 —

Presidential Decree No. 1956, as amended by Executive Order No.


137 has, in any case, in its favor the presumption of validity and
constitutionality which petitioners Valmonte and the KMU have not
overturned. Petitioners have not undertaken to identify the
provisions in the Constitution which they claim to have been
violated by that statute. This Court, however, is not compelled to
speculate and to imagine how the assailed legislation may possibly
offend some provision of the Constitution. The Court notes,
further, in this respect that petitioners have in the main put in
question the wisdom, justice and expediency of the establishment
of the OPSF, issues which are not properly addressed to this Court
and which this Court may not constitutionally pass upon. Those
issues should be addressed rather to the political departments of
government: the President and the Congress.

Parenthetically, We wish to state that gambling is generally immoral, and


this is precisely so when the gambling resorted to is excessive. This
excessiveness necessarily depends not only on the financial resources of
the gambler and his family but also on his mental, social, and spiritual
outlook on life. However, the mere fact that some persons may have lost
their material fortunes, mental control, physical health, or even their lives
SECOND DIVISION Larano at buong pagkakaisang sinangayunan ng lahat ng dumalo sa
pulong;

IPINASIYA, na tutulan gaya ng dito ay mahigpit na TINUTUTULAN ang ano


[G.R. No. 129093. August 30, 2001] mang uri ng sugal dito sa lalawigan ng Laguna lalot higit ang Lotto;

IPINASIYA PA RIN na hilingin tulad ng dito ay hinihiling sa Panlalawigang


pinuno ng Philippine National Police (PNP) Col. [illegible] na mahigpit na
HON. JOSE D. LINA, JR., SANGGUNIANG PANLALAWIGAN OF pag-ibayuhin ang pagsugpo sa lahat ng uri ng illegal na sugal sa buong
LAGUNA, and HON. CALIXTO CATAQUIZ, petitioners, vs. lalawigan ng Laguna lalo na ang Jueteng. [3]
HON. FRANCISCO DIZON PAO and TONY
CALVENTO, respondents. As a result of this resolution of denial, respondent Calvento filed a
complaint for declaratory relief with prayer for preliminary injunction and
DECISION temporary restraining order. In the said complaint, respondent Calvento
asked the Regional Trial Court of San Pedro Laguna, Branch 93, for the
QUISUMBING, J.: following reliefs: (1) a preliminary injunction or temporary restraining
order, ordering the defendants to refrain from implementing or
For our resolution is a petition for review on certiorari seeking the enforcing Kapasiyahan Blg. 508, T. 1995; (2) an order requiring Hon.
reversal of the decision[1] dated February 10, 1997 of the Regional Trial Municipal Mayor Calixto R. Cataquiz to issue a business permit for the
Court of San Pedro, Laguna, Branch 93, enjoining petitioners from operation of a lotto outlet; and (3) an order annulling or declaring as
implementing or enforcing Kapasiyahan Bilang 508, Taon 1995, of invalid Kapasiyahan Blg. 508, T. 1995.
the Sangguniang Panlalawigan of Laguna and its subsequent Order [2] dated
April 21, 1997 denying petitioners motion for reconsideration. On February 10, 1997, the respondent judge, Francisco Dizon Pao,
promulgated his decision enjoining the petitioners from implementing or
On December 29, 1995, respondent Tony Calvento was appointed enforcing resolution or Kapasiyahan Blg. 508, T. 1995. The dispositive
agent by the Philippine Charity Sweepstakes Office (PCSO) to install portion of said decision reads:
Terminal OM 20 for the operation of lotto. He asked Mayor Calixto
Cataquiz, Mayor of San Pedro, Laguna, for a mayors permit to open the WHEREFORE, premises considered, defendants, their agents and
lotto outlet. This was denied by Mayor Cataquiz in a letter dated February representatives are hereby enjoined from implementing or enforcing
19, 1996. The ground for said denial was an ordinance passed by resolution or kapasiyahan blg. 508, T. 1995 of the Sangguniang
the Sangguniang Panlalawigan of Laguna entitled Kapasiyahan Blg. 508, T. Panlalawigan ng Laguna prohibiting the operation of the lotto in the
1995 which was issued on September 18, 1995. The ordinance reads: province of Laguna.

ISANG KAPASIYAHAN TINUTUTULAN ANG MGA ILLEGAL GAMBLING LALO SO ORDERED.[4]


NA ANG LOTTO SA LALAWIGAN NG LAGUNA
Petitioners filed a motion for reconsideration which was subsequently
SAPAGKAT, ang sugal dito sa lalawigan ng Laguna ay talamak na; denied in an Order dated April 21, 1997, which reads:

SAPAGKAT, ang sugal ay nagdudulot ng masasamang impluwensiya lalot Acting on the Motion for Reconsideration filed by defendants Jose D. Lina,
higit sa mga kabataan; Jr. and the Sangguniang Panlalawigan of Laguna, thru counsel, with the
opposition filed by plaintiffs counsel and the comment thereto filed by
KUNG KAYAT DAHIL DITO, at sa mungkahi nina Kgg. Kgd. Juan M. Unico counsel for the defendants which were duly noted, the Court hereby denies
at Kgg. Kgd. Gat-Ala A. Alatiit, pinangalawahan ni Kgg. Kgd. Meliton C. the motion for lack of merit.
SO ORDERED.[5] gambling which has been authorized by the national government. [11]He
argues that this is based on the principle that ordinances should not
On May 23, 1997, petitioners filed this petition alleging that the contravene statutes as municipal governments are merely agents of the
following errors were committed by the respondent trial court: national government. The local councils exercise only delegated legislative
powers which have been conferred on them by Congress. This being the
I case, these councils, as delegates, cannot be superior to the principal or
exercise powers higher than those of the latter. The OSG also adds that
THE TRIAL COURT ERRED IN ENJOINING THE PETITIONERS FROM the question of whether gambling should be permitted is for Congress to
IMPLEMENTING KAPASIYAHAN BLG. 508, T. 1995 OF THE SANGGUNIANG determine, taking into account national and local interests. Since Congress
PANLALAWIGAN OF LAGUNA PROHIBITING THE OPERATION OF THE has allowed the PCSO to operate lotteries which PCSO seeks to conduct in
LOTTO IN THE PROVINCE OF LAGUNA. Laguna, pursuant to its legislative grant of authority, the
provinces Sangguniang Panlalawigan cannot nullify the exercise of said
authority by preventing something already allowed by Congress.
II
The issues to be resolved now are the following: (1)
THE TRIAL COURT FAILED TO APPRECIATE THE ARGUMENT POSITED BY whether Kapasiyahan Blg. 508, T. 1995 of the Sangguniang
THE PETITIONERS THAT BEFORE ANY GOVERNMENT PROJECT OR Panlalawigan of Laguna and the denial of a mayors permit based thereon
PROGRAM MAY BE IMPLEMENTED BY THE NATIONAL AGENCIES OR are valid; and (2) whether prior consultations and approval by the
OFFICES, PRIOR CONSULTATION AND APPROVAL BY THE LOCAL concerned Sanggunian are needed before a lotto system can be operated
GOVERNMENT UNITS CONCERNED AND OTHER CONCERNED SECTORS IS in a given local government unit.
REQUIRED.
The entire controversy stemmed from the refusal of Mayor Cataquiz
to issue a mayors permit for the operation of a lotto outlet in favor of
Petitioners contend that the assailed resolution is a valid policy private respondent. According to the mayor, he based his decision on an
declaration of the Provincial Government of Laguna of its vehement existing ordinance prohibiting the operation of lotto in the province of
objection to the operation of lotto and all forms of gambling. It is likewise Laguna. The ordinance, however, merely states the objection of the
a valid exercise of the provincial governments police power under the council to the said game. It is but a mere policy statement on the part of
General Welfare Clause of Republic Act 7160, otherwise known as the the local council, which is not self-executing. Nor could it serve as a valid
Local Government Code of 1991. [6] They also maintain that respondents ground to prohibit the operation of the lotto system in the province of
lotto operation is illegal because no prior consultations and approval by the Laguna. Even petitioners admit as much when they stated in their petition
local government were sought before it was implemented contrary to the that:
express provisions of Sections 2 (c) and 27 of R.A. 7160. [7]

For his part, respondent Calvento argues that the questioned 5.7. The terms of the Resolution and the validity thereof are express and
resolution is, in effect, a curtailment of the power of the state since in this clear. The Resolution is a policy declaration of the Provincial Government
case the national legislature itself had already declared lotto as legal and of Laguna of its vehement opposition and/or objection to the operation of
permitted its operations around the country. [8] As for the allegation that no and/or all forms of gambling including the Lotto operation in the Province
prior consultations and approval were sought from the sangguniang of Laguna.[12]
panlalawigan of Laguna, respondent Calvento contends this is not
mandatory since such a requirement is merely stated as a declaration of As a policy statement expressing the local governments objection to
policy and not a self-executing provision of the Local Government Code of the lotto, such resolution is valid. This is part of the local governments
1991.[9] He also states that his operation of the lotto system is legal autonomy to air its views which may be contrary to that of the national
because of the authority given to him by the PCSO, which in turn had been governments. However, this freedom to exercise contrary views does not
granted a franchise to operate the lotto by Congress. [10] mean that local governments may actually enact ordinances that go
against laws duly enacted by Congress. Given this premise, the assailed
The Office of the Solicitor General (OSG), for the State, contends that
resolution in this case could not and should not be interpreted as a
the Provincial Government of Laguna has no power to prohibit a form of
measure or ordinance prohibiting the operation of lotto.
The game of lotto is a game of chance duly authorized by the national limitation on the right, the legislature might, by a single act, and if we can
government through an Act of Congress. Republic Act 1169, as amended suppose it capable of so great a folly and so great a wrong, sweep from
by Batas Pambansa Blg. 42, is the law which grants a franchise to the existence all of the municipal corporations in the state, and the corporation
PCSO and allows it to operate the lotteries. The pertinent provision reads: could not prevent it. We know of no limitation on the right so far as the
corporation themselves are concerned. They are, so to phrase it, the mere
Section 1. The Philippine Charity Sweepstakes Office.- The Philippine tenants at will of the legislature (citing Clinton vs. Ceder Rapids, etc.
Charity Sweepstakes Office, hereinafter designated the Office, shall be the Railroad Co., 24 Iowa 455).
principal government agency for raising and providing for funds for health
programs, medical assistance and services and charities of national Nothing in the present constitutional provision enhancing local
character, and as such shall have the general powers conferred in section autonomy dictates a different conclusion.
thirteen of Act Numbered One thousand four hundred fifty-nine, as
amended, and shall have the authority: The basic relationship between the national legislature and the local
government units has not been enfeebled by the new provisions in the
A. To hold and conduct charity sweepstakes races, lotteries, and other Constitution strengthening the policy of local autonomy. Without meaning
similar activities, in such frequency and manner, as shall be determined, to detract from that policy, we here confirm that Congress retains control
and subject to such rules and regulations as shall be promulgated by the of the local government units although in significantly reduced degree now
Board of Directors. than under our previous Constitutions. The power to create still includes
the power to destroy. The power to grant still includes the power to
This statute remains valid today. While lotto is clearly a game of withhold or recall. True, there are certain notable innovations in the
chance, the national government deems it wise and proper to permit Constitution, like the direct conferment on the local government units of
it. Hence, the Sangguniang Panlalawigan of Laguna, a local government the power to tax (citing Art. X, Sec. 5, Constitution), which cannot now be
unit, cannot issue a resolution or an ordinance that would seek to prohibit withdrawn by mere statute. By and large, however, the national legislature
permits. Stated otherwise, what the national legislature expressly allows is still the principal of the local government units, which cannot defy its will
by law, such as lotto, a provincial board may not disallow by ordinance or or modify or violate it.[15]
resolution.
Ours is still a unitary form of government, not a federal state. Being
In our system of government, the power of local government units to so, any form of autonomy granted to local governments will necessarily be
legislate and enact ordinances and resolutions is merely a delegated power limited and confined within the extent allowed by the central
coming from Congress. As held in Tatel vs. Virac,[13] ordinances should not authority.Besides, the principle of local autonomy under the 1987
contravene an existing statute enacted by Congress. The reasons for this Constitution simply means decentralization. It does not make local
is obvious, as elucidated in Magtajas v. Pryce Properties Corp.[14] governments sovereign within the state or an imperium in imperio.[16]

Municipal governments are only agents of the national government. Local To conclude our resolution of the first issue, respondent mayor of San
councils exercise only delegated legislative powers conferred upon them by Pedro, cannot avail of Kapasiyahan Bilang 508, Taon 1995, of the
Congress as the national lawmaking body. The delegate cannot be superior Provincial Board of Laguna as justification to prohibit lotto in his
to the principal or exercise powers higher than those of the latter. It is a municipality. For said resolution is nothing but an expression of the local
heresy to suggest that the local government units can undo the acts of legislative unit concerned. The Boards enactment, like spring water, could
Congress, from which they have derived their power in the first place, and not rise above its source of power, the national legislature.
negate by mere ordinance the mandate of the statute. As for the second issue, we hold that petitioners erred in declaring
that Sections 2 (c) and 27 of Republic Act 7160, otherwise known as the
Municipal corporations owe their origin to, and derive their powers and Local Government Code of 1991, apply mandatorily in the setting up of
rights wholly from the legislature. It breathes into them the breath of life, lotto outlets around the country. These provisions state:
without which they cannot exist. As it creates, so it may destroy. As it may
destroy, it may abridge and control. Unless there is some constitutional Section 2. Declaration of Policy. x x x
(c) It is likewise the policy of the State to require all national agencies and that may call for the eviction of a particular group of people residing in the
offices to conduct periodic consultations with appropriate local government locality where these will be implemented. Obviously, none of these effects
units, non-governmental and peoples organizations, and other concerned will be produced by the introduction of lotto in the province of Laguna.
sectors of the community before any project or program is implemented in
their respective jurisdictions. Moreover, the argument regarding lack of consultation raised by
petitioners is clearly an afterthought on their part. There is no indication in
the letter of Mayor Cataquiz that this was one of the reasons for his refusal
Section 27. Prior Consultations Required. No project or program shall be to issue a permit. That refusal was predicated solely but erroneously on
implemented by government authorities unless the consultations the provisions of Kapasiyahan Blg. 508, Taon 1995, of the Sangguniang
mentioned in Section 2 (c) and 26 hereof are complied with, and prior Panlalawigan of Laguna.
approval of the sanggunian concerned is obtained; Provided, that
occupants in areas where such projects are to be implemented shall not be In sum, we find no reversible error in the RTC decision enjoining
evicted unless appropriate relocation sites have been provided, in Mayor Cataquiz from enforcing or implementing the Kapasiyahan Blg. 508,
accordance with the provisions of the Constitution. T. 1995, of the Sangguniang Panlalawigan of Laguna. That resolution
expresses merely a policy statement of the Laguna provincial board. It
From a careful reading of said provisions, we find that these apply possesses no binding legal force nor requires any act of implementation. It
only to national programs and/or projects which are to be implemented in provides no sufficient legal basis for respondent mayors refusal to issue
a particular local community. Lotto is neither a program nor a project of the permit sought by private respondent in connection with a legitimate
the national government, but of a charitable institution, the PCSO. Though business activity authorized by a law passed by Congress.
sanctioned by the national government, it is far fetched to say that lotto WHEREFORE, the petition is DENIED for lack of merit. The Order of
falls within the contemplation of Sections 2 (c) and 27 of the Local
the Regional Trial Court of San Pedro, Laguna enjoining the petitioners
Government Code. from implementing or enforcing Resolution or Kapasiyahan Blg. 508, T.
Section 27 of the Code should be read in conjunction with Section 26 1995, of the Provincial Board of Laguna is hereby AFFIRMED. No costs.
thereof.[17] Section 26 reads: SO ORDERED.

Section 26. Duty of National Government Agencies in the Maintenance of Bellosillo, (Chairman), Mendoza, Buena, and De Leon, Jr., JJ., concur.
Ecological Balance. It shall be the duty of every national agency or
government-owned or controlled corporation authorizing or involved in the
planning and implementation of any project or program that may cause
pollution, climatic change, depletion of non-renewable resources, loss of
crop land, range-land, or forest cover, and extinction of animal or plant
species, to consult with the local government units, nongovernmental
organizations, and other sectors concerned and explain the goals and
objectives of the project or program, its impact upon the people and the
community in terms of environmental or ecological balance, and the
measures that will be undertaken to prevent or minimize the adverse
effects thereof.

Thus, the projects and programs mentioned in Section 27 should be


interpreted to mean projects and programs whose effects are among those
enumerated in Section 26 and 27, to wit, those that: (1) may cause
pollution; (2) may bring about climatic change; (3) may cause the
depletion of non-renewable resources; (4) may result in loss of crop land,
range-land, or forest cover; (5) may eradicate certain animal or plant
species from the face of the planet; and (6) other projects or programs
Republic of the Philippines passed. Unfortunately, petitioner did not make it. When the results were
SUPREME COURT released, she received failing grades in four out of the seven subjects. 2
Manila
Subject Petitioner’s Grade
FIRST DIVISION
Theory of Accounts 65 %
G.R. No. 165036 July 5, 2010
Business Law 66 %

HAZEL MA. C. ANTOLIN, Petitioner, Management Services 69 %


vs.
Auditing Theory 82 %
ABELARDO T. DOMONDON, JOSE A. GANGAN, and VIOLETA J.
JOSEF, Respondents. Auditing Problems 70 %

x - - - - - - - - - - - - - - - - - - - - - - -x Practical Accounting I 68 %

Practical Accounting II 77 %
G.R. No. 175705

Convinced that she deserved to pass the examinations, she wrote to


HAZEL MA. C. ANTOLIN Petitioner, respondent Abelardo T. Domondon (Domondon), Acting Chairman of the
vs. Board of Accountancy, and requested that her answer sheets be re-
ANTONIETA FORTUNA-IBE, Respondent. corrected.3 On November 3, 1997, petitioner was shown her answer
sheets, but these consisted merely of shaded marks, so she was unable to
DECISION determine why she failed the exam.4 Thus, on November 10, 1997, she
again wrote to the Board to request for copies of (a) the questionnaire in
DEL CASTILLO, J.: each of the seven subjects (b) her answer sheets; (c) the answer keys to
the questionnaires, and (d) an explanation of the grading system used in
each subject (collectively, the Examination Papers). 5
Examinations have a two-fold purpose. First, they are summative;
examinations are intended to assess and record what and how much the
students have learned. Second, and perhaps more importantly, they are Acting Chairman Domondon denied petitioner’s request on two grounds:
formative; examinations are intended to be part and parcel of the learning first, that Section 36, Article III of the Rules and Regulations Governing
process. In a perfect system, they are tools for learning. In view of the the Regulation and Practice of Professionals, as amended by Professional
pedagogical aspect of national examinations, the need for all parties to Regulation Commission (PRC) Resolution No. 332, series of 1994, only
fully ventilate their respective positions, and the view that government permitted access to the petitioner’s answer sheet (which she had been
transactions can only be improved by public scrutiny, we remand these shown previously), and that reconsideration of her examination result was
cases to the trial court for further proceedings. only proper under the grounds stated therein:

Factual Antecedents Sec. 36 An examinee shall be allowed to have access or to go over his/her
test papers or answer sheets on a date not later than thirty (30) days from
the official release of the results of the examination. Within ten (10) days
Petitioner took the accountancy licensure examinations (the Certified
from such date, he/she may file his/her request for reconsideration of
Public Accountant [CPA] Board Exams) conducted by the Board of
ratings. Reconsideration of rating shall be effected only on grounds of
Accountancy (the Board) in October 1997.1 The examination results were
mechanical error in the grading of his/her testpapers or answer sheets, or
released on October 29, 1997; out of 6,481 examinees, only 1,171
malfeasance.6lawph!l
Second, Acting Chairman Domondon clarified that the Board was precluded for lack of merit on the following grounds: (1) petitioner failed to exhaust
from releasing the Examination Papers (other than petitioner’s answer administrative remedies; (2) the petition stated no cause of action
sheet) by Section 20, Article IV of PRC Resolution No. 338, series of 1994, because there was no ministerial duty to release the information
which provides: demanded; and (3) the constitutional right to information on matters of
public concern is subject to limitations provided by law, including Section
Sec. 20. Illegal, Immoral, Dishonorable, Unprofessional Acts – The 20, Article IV, of PRC Resolution No. 338, series of 1994. 13
hereunder acts shall constitute prejudicial, illegal, grossly immoral,
dishonorable, or unprofessional conduct: On March 3, 1998, petitioner filed an Amended Petition (which was
admitted by the RTC), where she included the following allegation in the
A. Providing, getting, receiving, holding, using or reproducing questions body of her petition:

xxxx The allegations in this amended petition are meant only to plead a cause
of action for access to the documents requested, not for re-correction
which petitioner shall assert in the proper forum depending on, among
3. that have been given in the examination except if the test bank for the
others, whether she finds sufficient error in the documents to warrant such
subject has on deposit at least two thousand (2,000) questions. 7
or any other relief. None of the allegations in this amended petition,
including those in the following paragraphs, is made to assert a cause of
After a further exchange of correspondence, 8 the Board informed petitioner action for re-correction.14
that an investigation was conducted into her exam and there was no
mechanical error found in the grading of her test papers. 9
If only to underscore the fact that she was not asking for a re-checking of
her exam, the following prayer for relief was deleted from the Amended
Proceedings before the Regional Trial Court Petition: "and, if warranted, to issue to her a certificate of registration as a
CPA."
Undeterred, on January 12, 1998, petitioner filed a Petition for Mandamus
with Damages against the Board of Accountancy and its members 10 before On June 23, 1998, respondents filed a Manifestation and Motion to Dismiss
the Regional Trial Court (RTC) of Manila. The case was raffled to Branch Application for Writ of Preliminary Mandatory Injunction, on the ground
33, and docketed as Civil Case No. 98-86881. The Petition included a that petitioner had taken and passed the May 1998 CPA Licensure
prayer for the issuance of a preliminary mandatory injunction ordering the Examination and had taken her oath as a CPA.15 Petitioner filed her
Board of Accountancy and its members (the respondents) to furnish Opposition on July 8, 1998.16 Subsequently, on October 29, 1998,
petitioner with copies of the Examination Papers. Petitioner also prayed respondents filed their Answer with Counterclaim to the amended petition.
that final judgment be issued ordering respondents to furnish petitioner They reiterated their original allegations and further alleged that there was
with all documents and other materials as would enable her to determine no cause of action because at the time the Amended Petition was
whether respondents fairly administered the examinations and correctly admitted, they had ceased to be members of the Board of Accountancy
graded petitioner’s performance therein, and, if warranted, to issue to her and they were not in possession of the documents sought by the
a certificate of registration as a CPA.11 petitioner.17

On February 5, 1998, respondents filed their Opposition to the Application Ruling of the Regional Trial Court
for a Writ of Preliminary Mandatory Injunction, and argued, inter alia, that
petitioner was not entitled to the relief sought, that the respondents did
In an Order dated October 16, 1998, the trial court granted respondent’s
not have the duty to furnish petitioner with copies of the Examination
Motion to Dismiss Petitioner’s Application for a Writ of Preliminary
Papers, and that petitioner had other plain, speedy, adequate remedy in
Mandatory Injunction (not the main case), ruling that the matter had
the ordinary course of law, namely, recourse to the PRC. 12 Respondents
become moot since petitioner passed the May CPA Licensure 1998
also filed their Answer with Compulsory Counterclaim in the main case,
Examination and had already taken her oath as a CPA. 18
which asked that the Petition for Mandamus with Damages be dismissed
Undaunted, petitioner sought and obtained leave to file a Second Amended the PRC to preserve and safeguard the documents and make them
Petition for Mandamus with Damages19where she finally impleaded the PRC available anytime the Court or petitioner needs them.
as respondent and included the following plea in her prayer:
WHEREFORE, the Order of this Court dated June 20, 2002 is reconsidered
WHEREFORE, petitioner respectfully prays that: and set aside. The Professional Regulation Commission is ordered to
preserve and safeguard the following documents:
xxxx
a) Questionnaire in each of the seven subjects comprising the
2. Judgment be issued – Accountancy Examination of October, 1997;

(a) commanding respondents to give petitioner all documents and other b) Petitioner’s Answer Sheets; and
materials as would enable her to determine whether respondents fairly
administered the same examinations and correctly graded petitioner’s c) Answer keys to the questionnaires.
performance therein and, if warranted, to make the appropriate revisions
on the results of her examination. (Emphasis ours) SO ORDERED.23

On June 21, 2002, the trial court dismissed the petition on the ground that Respondents filed a motion for reconsideration which was denied. 24
the petition had already become moot, since petitioner managed to pass
the 1998 CPA Board examinations.20 Petitioner sought
Proceedings before the Court of Appeals
reconsideration21 which was granted by the trial court in its Omnibus
Order22 dated November 11, 2002. The Omnibus Order provides in part:
The RTC Decisions led to the filing of three separate petitions
for certiorari before the Court of Appeals (CA):
On the motion for reconsideration filed by the petitioner, the Court is
inclined to reconsider its Order dismissing the petition. The Court agrees
with the petitioner that the passing of the petitioner in the subsequent CPA (a) CA-GR SP No. 76498, a petition filed by respondents
examination did not render the petition moot and academic because the Domondon, Gangan, and Josef on April 11, 2003;
relief "and if warranted, to issue to her a certificate of registration as
Certified Public Accountant" was deleted from the original petition. As (b) CA-GR SP No. 76546, a petition filed by respondent Ibe on
regard the issue of whether the petitioner has the constitutional right to April 30, 2003; and
have access to the questioned documents, the Court would want first the
parties to adduce evidence before it can resolve the issue so that it can (c) CA-GR SP No. 76545, a petition filed by the Board of
make a complete determination of the rights of the parties. Accountancy and PRC.

The Court would also want the Professional Regulation Commission to give It is the first two proceedings that are pending before us. In both cases,
its side of the case the moment it is impleaded as a respondent in the the CA set aside the RTC Decisions and ordered the dismissal of Civil Case
Second Amended Petition for Mandamus filed by the petitioner which this No. 98-8681.
Court is inclined to grant.

Ruling of the Court of Appeals


As to the Motion for Conservatory Measures filed by the petitioner, the
Court denies the same. It is clear that the PRC has in custody the
documents being requested by the petitioner. It has also an adequate In its December 11, 2006 Decision25 in CA-GR SP No. 76546, the CA ruled
facility to preserve and safeguard the documents. To be sure that the that the petition has become moot in view of petitioner’s eventual passing
questioned documents are preserved and safeguarded, the Court will order of the 1998 CPA Board Exam. In CA-GR SP No. 76498, the CA found, in a
Decision dated February 16, 2004,26 that (i) Section 20, Article IV of PRC
Resolution No. 338 constituted a valid limitation on petitioner’s right to other respondents (Medical Board of Examiners and the Professional
information and access to government documents; (ii) the Examination Regulation Commission) "to reconsider, recorrect and/or rectify the board
Documents were not of public concern, because petitioner merely sought ratings of the petitioners from their present failing grades to higher or
review of her failing marks; (iii) it was not the ministerial or mandatory passing marks." The function of reviewing and re-assessing the petitioners’
function of the respondents to review and reassess the answers to answers to the examination questions, in the light of the facts and
examination questions of a failing examinee; (iv) the case has become arguments presented by them x x x is a discretionary function of the
moot, since petitioner already passed the May 1998 CPA Board Medical Board, not a ministerial and mandatory one, hence, not within the
Examinations and took her oath as a CPA; and (v) petitioner failed to scope of the writ of mandamus. The obvious remedy of the petitioners
exhaust administrative remedies, because, having failed to secure the from the adverse judgment by the Medical Board of Examiners was an
desired outcome from the respondents, she did not elevate the matter to appeal to the Professional Regulation Commission itself, and thence to the
the PRC before seeking judicial intervention. 27 Court of Appeals; and since they did not apply for relief to the Commission
prior to their institution of the special civil action of mandamus in the
CA-GR SP No. 76498 and CA-GR SP No. 76546 were brought before us by Regional Trial Court, the omission was fatal to the action under the familiar
the petitioner and docketed as G.R. Nos. 165036 and 175705, doctrine requiring exhaustion of administrative remedies. Apart from the
respectively. The cases were then consolidated, in view of the similarity of obvious undesirability of a procedure which would allow Courts to
the factual antecedents and issues, and to avoid the possibility of substitute their judgment for that of Government boards in the
conflicting decisions by different divisions of this Court. 28 determination of successful examinees in any administered examination –
an area in which courts have no expertise – and the circumstance that the
law declares the Court of Appeals to be the appropriate review Court, the
Issues
Regional Trial Court was quite correct in refusing to take cognizance of an
action seeking reversal of the quasi-judicial action taken by the Medical
Before us, petitioner argues that she has a right to obtain copies of the Board of Examiners.32(Emphasis ours)
examination papers so she can determine for herself why and how she
failed and to ensure that the Board properly performed its duties. She
For a writ of mandamus to issue, the applicant must have a well-defined,
argues that the Constitution29 as well as the Code of Conduct and Ethical
clear, and certain legal right to the thing demanded. The corresponding
Standards for Public Officials and Employees30 support her right to demand
duty of the respondent to perform the required act must be equally
access to the Examination Papers. Furthermore, she claims that there was
clear.33 No such clarity exists here; neither does petitioner’s right to
no need to exhaust administrative remedies, since no recourse to the PRC
demand a revision of her examination results. And despite petitioner’s
was available, and only a pure question of law is involved in this case.
assertions that she has not made any demand for re-correction, the most
Finally, she claims that her demand for access to documents was not
cursory perusal of her Second Amended Petition and her prayer that the
rendered moot by her passing of the 1998 CPA Board Exams.
respondents "make the appropriate revisions on the results of her
examination" belies this claim.
Our Ruling
Like the claimants in Agustin, the remedy of petitioner from the refusal of
Propriety of Writ of Mandamus the Board to release the Examination Papers should have been through an
appeal to the PRC. Undoubtedly, petitioner had an adequate remedy from
At the very outset let us be clear of our ruling. Any claim for re-correction the Board’s refusal to provide her with copies of the Examination Papers.
or revision of her 1997 examination cannot be compelled by mandamus. Under Section 5(a) of Presidential Decree No. 223, 34 the PRC has the
This much was made evident by our ruling in Agustin-Ramos v. power to promulgate rules and regulations to implement policies for the
Sandoval,31 where we stated: regulation of the accounting profession. 35 In fact, it is one such regulation
(PRC Resolution No. 338) that is at issue in this case. In addition, under
After deliberating on the petition in relation to the other pleadings filed in Section 5(c), the PRC has the power to
the proceedings at bar, the Court resolved to DENY said petition for lack of
merit. The petition at bar prays for the setting aside of the Order of review, coordinate, integrate and approve the policies, resolutions, rules
respondent Judge dismissing petitioners’ mandamus action to compel the and regulations, orders or decisions promulgated by the various
Boards with respect to the profession or occupation under their In this jurisdiction, any citizen may challenge any attempt to obstruct the
jurisdictions including the results of their licensure examinations but their exercise of his or her right to information and may seek its enforcement by
decisions on administrative cases shall be final and executory unless mandamus.43 And since every citizen possesses the inherent right to be
appealed to the Commission within thirty (30) days from the date of informed by the mere fact of citizenship,44 we find that petitioner’s belated
promulgation thereof. passing of the CPA Board Exams does not automatically mean that her
interest in the Examination Papers has become mere superfluity.
Petitioner posits that no remedy was available because the PRC’s power to Undoubtedly, the constitutional question presented, in view of the
"review" and "approve" in Section 5(c) only refers to appeals in decisions likelihood that the issues in this case will be repeated, warrants review. 45
concerning administrative investigations36 and not to instances where
documents are being requested. Not only is this position myopic and self- The crux of this case is whether petitioner may compel access to the
serving, it is bereft of either statutory or jurisprudential basis. The PRC’s Examination Documents through mandamus. As always, our inquiry must
quasi-legislative and enforcement powers, encompassing its authority to begin with the Constitution. Section 7, Article III provides:
review and approve "policies, resolutions, rules and regulations, orders, or
decisions" cover more than administrative investigations conducted Sec.7. The right of the people to information on matters of public concern
pursuant to its quasi-judicial powers.37 More significantly, since the PRC shall be recognized. Access to official records, and to documents, and
itself issued the resolution questioned by the petitioner here, it was in the papers pertaining to official acts, transactions, or decisions, as well to
best position to resolve questions addressed to its area of expertise. government research data used as basis for policy development, shall be
Indeed, petitioner could have saved herself a great deal of time and effort afforded the citizen, subject to such limitations as may be provided by law.
had she given the PRC the opportunity to rectify any purported errors
committed by the Board.
Together with the guarantee of the right to information, Section 28, Article
II promotes full disclosure and transparency in government, viz:
One of the reasons for exhaustion of administrative remedies is our well-
entrenched doctrine on separation of powers, which enjoins upon the
Sec. 28. Subject to reasonable conditions prescribed by law, the State
Judiciary a becoming policy of non-interference with matters falling
adopts and implements a policy of full public disclosure of all its
primarily (albeit not exclusively) within the competence of other
transactions involving public interest.
departments.38 Courts, for reasons of law, comity and convenience, should
not entertain suits unless the available administrative remedies have first
been resorted to and the proper authorities have been given an Like all the constitutional guarantees, the right to information is not
appropriate opportunity to act and correct their alleged errors, if any, absolute. The people's right to information is limited to "matters of public
committed in the administrative forum. 39 concern," and is further "subject to such limitations as may be provided by
law." Similarly, the State's policy of full disclosure is limited to
"transactions involving public interest," and is "subject to reasonable
However, the principle of exhaustion of administrative remedies is subject
conditions prescribed by law". The Court has always grappled with the
to exceptions, among which is when only a question of law is
meanings of the terms "public interest" and "public concern." As observed
involved.40 This is because issues of law – such as whether petitioner has a
in Legaspi v. Civil Service Commission:46
constitutional right to demand access to the Examination Papers - cannot
be resolved with finality by the administrative officer. 41
In determining whether x x x a particular information is of public concern
there is no rigid test which can be applied. "Public concern" like "public
Issues of Mootness
interest" is a term that eludes exact definition. Both terms embrace a
broad spectrum of subjects which the public may want to know, either
We now turn to the question of whether the petition has become moot in because these directly affect their lives, or simply because such matters
view of petitioner’s having passed the 1998 CPA examination. An issue naturally arouse the interest of an ordinary citizen. In the final analysis, it
becomes moot and academic when it ceases to present a justiciable is for the courts to determine on a case by case basis whether the matter
controversy, so that a declaration on the issue would be of no practical use at issue is of interest or importance, as it relates to or affects the public.
or value.42
We have also recognized the need to preserve a measure of confidentiality
on some matters, such as national security, trade secrets and banking
transactions, criminal matters, and other confidential matters. 47

We are prepared to concede that national board examinations such as the


CPA Board Exams are matters of public concern. The populace in general,
and the examinees in particular, would understandably be interested in the
fair and competent administration of these exams in order to ensure that
only those qualified are admitted into the accounting profession. And as
with all matters pedagogical, these examinations could be not merely
quantitative means of assessment, but also means to further improve the
teaching and learning of the art and science of accounting.

On the other hand, we do realize that there may be valid reasons to limit
access to the Examination Papers in order to properly administer the
exam. More than the mere convenience of the examiner, it may well be
that there exist inherent difficulties in the preparation, generation,
encoding, administration, and checking of these multiple choice exams
that require that the questions and answers remain confidential for a
limited duration. However, the PRC is not a party to these proceedings.
They have not been given an opportunity to explain the reasons behind
their regulations or articulate the justification for keeping the Examination
Documents confidential. In view of the far-reaching implications of this
case, which may impact on every board examination administered by the
PRC, and in order that all relevant issues may be ventilated, we deem it
best to remand these cases to the RTC for further proceedings.

IN VIEW OF THE FOREGOING, the petitions are GRANTED. The


December 11, 2006 and February 16, 2004 Decisions of the Court of
Appeals in CA-GR SP No. 76546 and CA-GR SP No. 76498, respectively,
are hereby SET ASIDE. The November 11, 2002 and January 30, 2003
Orders of the Regional Trial Court of Manila, Branch 33, in Civil Case No.
98-86881 are AFFIRMED. The case is remanded to the Regional Trial
Court for further proceedings.

SO ORDERED.

Вам также может понравиться